You are on page 1of 52

CRIMINAL PROCEDURE Acquittal; Effect (2002) Delia sued Victor for personal injuries Forthwith the police officer

police officer filed with the City Prosecutor of Manila a complaint for
which she allegedly sustained when she was struck by a car driven by Victor. May the estafa supported by RY"s sworn statement and other documentary evidence. After due
court receive in evidence, over proper and timely objection by Delia, a certified true copy inquest, the prosecutor filed the requisite information with the MM RTC. No preliminary
of a judgment of acquittal in a criminal prosecution charging Victor with hit-and-run investigation was conducted either before or after the filing of the information and the
driving in connection with Delia’s injuries? Why? (3%) SUGGESTED ANSWER: Remedial accused at no time asked for such an investigation. However, before arraignment, the
Law Bar Examination Q & A (1997-2006) If the judgment of acquittal in the criminal case accused moved to quash the information on the ground that the prosecutor suffered
finds that the act or omission from which the civil liability may arise does not exist, the from a want of authority to file the information because of his failure to conduct a
court may receive it in evidence over the objection by Delia. [Rule 111, sec. 2, last preliminary investigation before filing the information, as required by the Rules of Court.
paragraph]. ALTERNATIVE ANSWER: If the judgment of acquittal is based on reasonable Is the warrantless arrest of AX valid? Is he entitled to a preliminary investigation before
doubt, the court may receive it in evidence because in such case, the civil action for the filing of the information? Explain. (5%) SUGGESTED ANSWER: No. The warrantless
damages which may be instituted requires only a preponderance of the evidence. (Art. arrest is not valid because the alleged offense has not just been committed. The crime
29, Civil Code). Actions; BP22; Civil Action deemed included (2001) Saturnino filed a was allegedly committed one year before the arrest. (Sec. 5 (b) of Rule 113). Yes, he is
criminal action against Alex for the latter’s bouncing check. On the date of the hearing entitled to a preliminary investigation because he was not lawfully arrested without a
after the arraignment, Saturnino manifested to the court that he is reserving his right to warrant (See Sec. 7 of Rule 112). He can move for a reinvestigation. ALTERNATIVE
file a separate civil action. The court allowed Saturnino to file a civil action separately and ANSWER: He is not entitled to a preliminary investigation because the penalty for estafa
proceeded to hear the criminal case. Alex filed a motion for reconsideration contending is the sum of P10,000 does not exceed 4 years and 2 months. Under Sec. 1, second par.,
that the civil action is deemed included in the criminal case. The court reconsidered its Rule 112, a preliminary investigation is not required. (Note: The penalty is not stated in
order and ruled that Saturnino could not file a separate action. Is the court’s order the question.) Arrest; Warrantless Arrests & Searches (1997) A was killed by B during a
granting the motion for reconsideration correct? Why? (5%) SUGGESTED ANSWER: Yes, quarrel over a hostess in a nightclub. Two days after the incident, and upon complaint of
the court’s order granting the motion for reconsideration is correct. The Rules provide the widow of A, the police arrested B by: sirdondee@gmail.com Page 40 of 66 without a
that the criminal action for violation of B.P. Blg. 22 shall be deemed to include the warrant of arrest and searched his house without a search warrant. a) Can the gun used
corresponding civil action, and that no reservation to file such civil action separately shall by B in shooting A, which was seized during the search of the house of B, be admitted in
be allowed. [Sec. 1(b), Rule 111, Revised Rules of Criminal Procedure] Actions; BP22; evidence? b) Is the arrest of B legal? c) Under the circumstances, can B be convicted of
Demurrer to Evidence (2003) In an action for violation of Batas Pambansa Big. 22, the homicide? SUGGESTED ANSWER: (a) No. The gun seized during the search of the house
court granted the accused’s demurrer to evidence which he filed without leave of court. of B without a search warrant is not admissible in evidence. (Secs. 2 and 3[2], Art. III of
Although he was acquitted of the crime charged, he, however, was required by the court Constitution). Moreover, the search was not an incident to a lawful arrest of a person
to pay the private complainant the face value of the check. The accused filed a Motion of under Sec. 12 of Rule 126. (b) No. A warrantless arrest requires that the crime has in fact
Reconsideration regarding the order to pay the face value of the check on the following just been committed and the police arresting has personal knowledge of facts that the
grounds: a) the demurrer to evidence applied only too the criminal aspect of the case; person to be arrested has committed it. (Sec. 5, Rule 113). Here, the crime has not just
and b) at the very least, he was entitled to adduce controverting evidence on the civil been committed since a period of two days had already lapsed, and the police arresting
liability. Resolve the Motion for Reconsideration. (6%) SUGGESTED ANSWER: (a) The has no such personal knowledge because he was not present when the incident
Motion for Reconsideration should be denied. The ground that the demurrer to evidence happened. (Go vs. Court of Appeals. 206 SCRA 138). (c) Yes. The gun is not indispensable
applied only to the criminal aspect of the case was not correct because the criminal in the conviction of A because the court may rely on testimonial or other evidence.
action for violation of Batas Pambansa Blg. 22 included the corresponding civil action. Arrest; Warrantless Arrests & Seizures (2003) In a buy-bust operation, the police
(Sec. 1(b) of Rule 111). (b) The accused was not entitled to adduce controverting operatives arrested the accused and seized from him a sachet of shabu and an
evidence on the civil liability, because by: sirdondee@gmail.com Page 39 of 66 he filed unlicensed firearm. The accused was charged in two Informations, one for violation of
his demurrer to evidence without leave of court. (Sec. 23 of Rule 119). Actions; the “Dangerous Drug Act”, as amended, and another for illegal possession of firearms.
Commencement of an Action; Double Jeopardy (2004) SPO1 CNC filed with the MTC in The accused filed an action for recovery of the firearm in another court against the police
Quezon City (MeTC-QC) a sworn written statement duly subscribed by him, charging RGR officers with an application for the issuance of a writ of replevin. He alleged in his
(an actual resident of Cebu City) with the offense of slight physical injuries allegedly Complaint that he was a military informer who had been issued a written authority to
inflicted on SPS (an actual resident of Quezon City). The Judge of the branch to which the carry said firearm. The police officers moved to dismiss the complaint on the ground that
case was raffled thereupon issued an order declaring that the case shall be governed by the subject firearm was in custodia legis. The court denied the motion and instead issued
the Rule on Summary Procedure in criminal cases. Soon thereafter, the Judge ordered the writ of replevin. (a) Was the seizure of the firearm valid? (b) Was the denial of the
the dismissal of the case for the reason that it was not commenced by information, as motion to dismiss proper? 6% SUGGESTED ANSWER: (a) Yes, the seizure of the firearm
required by said Rule. Sometime later, based on the same facts giving rise to the slight was valid because it was seized in the course of a valid arrest in a buy-bust operation.
physical injuries case, the City Prosecutor filed with the same MeTC-QC an information (Sec. 12 and 13 of Rule 126) A search warrant was not necessary. (People v. Salazar, 266
for attempted homicide against the same RGR. In due time, before arraignment, RGR SCRA 607 [1997]). (b) The denial of the motion to dismiss was not proper. The court had
moved to quash the information on the ground of double jeopardy and after due no authority to issue the writ of replevin whether the firearm was in custodia legis
hearing, the Judge granted his motion. Was the dismissal of the complaint for slight Version 1997-2006 Updated by Dondee Remedial Law Bar Examination Q & A (1997-
physical injuries proper? Was the grant of the motion to quash the attempted homicide 2006) or not. The motion to recover the firearm should be filed in the court where the
information correct? Reason (5%) SUGGESTED ANSWER: Yes, the dismissal of the criminal action is pending. Arrest; Warrantless Arrests; Objection (2000) FG was arrested
complaint for slight physical injuries is proper because in Metropolitan Manila and in without a warrant by policemen while he was walking in a busy street. After preliminary
chartered cities, the case has to be commenced only by information. (Sec. 11, Revised investigation, he was charged with rape and the corresponding information was filed in
Rule on Summary Procedure). No, the grant of the motion to quash the attempted the RTC. On arraignment, he pleaded not guilty. Trial on the merits ensued. The court
homicide information on the ground of double jeopardy was not correct, because there rendered judgment convicting him. On appeal, FG claims that the judgment is void
was no valid prosecution for slight physical injuries. Actions; Discretionary Power of Fiscal because he was illegally arrested. If you were the Solicitor General, counsel for the
(1999) A filed with the Office of the Fiscal a Complaint for estafa against B. After the People of the Philippines, how would you refute said claim? (5%) SUGGESTED ANSWER:
preliminary investigation, the Fiscal dismissed the Complaint for lack of merit. May the Any objection to the illegality of the arrest of the accused without a warrant is deemed
Fiscal be compelled by mandamus to file the case in court? Explain. (2%) SUGGESTED waived when he pleaded not guilty at the arraignment without raising the question. T is
ANSWER: No. The public prosecutor may not be compelled by mandamus to file the case too late to complain about a warrantless arrest after trial is commenced and completed
in court because the determination of probable cause is within the discretion of the and a judgment of conviction rendered against the accused. (People v. Cabiles, 284 SCRA
prosecutor. The remedy is an appeal to the Secretary of Justice. (Sec. 4 Rule 112.) 199, [1999]) Bail (2002) D was charged with murder, a capital offense. After arraignment,
Actions; Injunction (1999) Will injunction lie to restrain the commencement of a criminal he applied for bail. The trial court ordered the prosecution to present its evidence in full
action? Explain. (2%) SUGGESTED ANSWER: As a general rule, injunction will not lie to on the ground that only on the basis of such presentation could it determine whether
restrain a criminal prosecution except: Remedial Law Bar Examination Q & A (1997-2006) the evidence of D’s guilt was strong for purposes of bail. Is the ruling correct? Why? (3%)
a) To afford adequate protection to the constitutional rights of the accused; b) When SUGGESTED ANSWER: No, the prosecution is only required to present as much evidence
necessary for the orderly administration of justice or to avoid oppression or multiplicity as is necessary to determine whether the evidence of D’s guilt is strong for purposes of
of actions; c) When double jeopardy is clearly apparent; d) Where the charges are bail.(Rule 114, sec. 8). Bail; Appeal (1998) In an information charging them of Murder,
manifestly false and motivated by the lust for vengeance; e) Where there is clearly no policemen A, B and C were convicted of Homicide. A appealed from the decision but B
prima facie case against the accused and a motion to quash on that ground has been and C did not. B started serving his sentence but C escaped and is at large. In the Court of
denied. (See cases cited in Roberts, Jr., vs. Court of Appeals, 254 SCRA 307 [1996] and Appeals, A applied for bail but was denied. Finally, the Court of Appeals rendered a
Brocka v. Enrile, 192 SCRA 183 [1990].) Arrest; Warrantless Arrest; Preliminary decision acquitting A on the ground that the evidence pointed to the NPA as the killers of
Investigation (2004) AX swindled RY in the amount of P10,000 sometime in mid-2003. On the victim. Was the Court of Appeal's denial of A's application for bail proper? [2%] Can B
the strength of the sworn statement given by RY personally to SPO1 Juan Ramos and C be benefited by the decision of the Court of Appeals? [3%] SUGGESTED ANSWER:
sometime in mid-2004, and without securing a warrant, the police officer arrested AX. 1, Yes, the Court of Appeals properly denied A's application for bail. The court had the
discretion to do so. Although A was convicted of homicide only, since he was charged of contract of carriage. Proof that the defendant was negligent and that such negligence
with a capital offense, on appeal by: sirdondee@gmail.com Page 41 of 66 he could be was the proximate cause of the collision is not required. (Articles 1170 and 2201, Civil
convicted of the capital offense. (Obosa vs. Court of Appeals, 266 SCRA 281.) Code; (Mendoza v. Phil. Airlines, Inc., 90 Phil. 836 [1952]; Batangas Transportation Co. v.
ALTERNATIVE ANSWER: Under Circular No. 2-92, A is entitled to bail because he was Caguimbal, 22 SCRA171 U 968]; Abeto v. PAL, 115 SCRA 489 [1982]; Aboitiz v. Court of
convicted of homicide and hence the evidence of guilt of murder is not strong. Appeals, 129 SCRA 95 [1984]). Demurrer to Evidence; w/o Leave of Court (1998) Facing a
SUGGESTED ANSWER: 2. B, who did not appeal, can be benefited by the decision of the charge of Murder, X filed a petition for bail. The petition was opposed by the prosecution
Court of Appeals which is favorable and applicable to him. (Sec. 11 [a]. Rule 122, Rules of but after hearing the court granted bail to X. On the first scheduled hearing on the
Criminal Procedure.) The benefit will also apply to C even if his appeal is dismissed merits, the prosecution manifested that it was not adducing additional evidence and that
because of his escape. Bail; Application; Venue (2002) If an information was filed in the it was resting its case. X filed a demurrer to evidence without leave of court but it was
RTC-Manila charging D with homicide and he was arrested in Quezon City, in what court denied by the court. 1. Did the court have the discretion to deny the demurrer to
or courts may he apply for bail? Explain. (3%) SUGGESTED ANSWER: D may apply for bail evidence under the circumstances mentioned above? (2%) 2. If the answer to the
in the RTC-Manila where the information was filed or in the RTC-Quezon City where he preceding question is in the affirmative, can X adduce evidence in his defense after the
was arrested, or if no judge, thereof is available, with any metropolitan trial judge, denial of his demurrer to evidence? [1%] 3. Without further proceeding and on the sole
municipal trial judge or municipal circuit trial judge therein. (Rule 114, sec. 17). Bail; basis of the evidence of the prosecution, can the court legally convict X for Murder? (2%)
Forms of Bail (1999) In what forms may bail be given? (2%) SUGGESTED ANSWER: Bail SUGGESTED ANSWER: 1. Yes. The Court had the discretion to deny the demurrer to the
may be given by a corporate surety, or through a property bond, cash deposit or evidence, because although the Version 1997-2006 Updated by Dondee Remedial Law
recognizance. Bail; Matter of Right (1999) When the accused is entitled as a matter of Bar Examination Q & A (1997-2006) evidence presented by the prosecution at the
right to bail, may the Court refuse to grant him bail on the ground that there exists a high hearing for bail was not strong, without any evidence for the defense, it could be
degree of probability that he will abscond or escape? Explain. (2%) SUGGESTED ANSWER: sufficient for conviction. 2. No. Because he filed the demurrer to the evidence without
If bail is a matter of right, it cannot be denied on the ground that there exists a high leave. (Sec. 15, Rule 119, Rules of Criminal Procedure.) However, the trial court should
degree of probability that the accused will abscond or escape. What the court can do is inquire as to why the accused filed the demurrer without leave and whether his lawyer
to increase the amount of the bail. One of the guidelines that the judge may use in fixing knew that the effect of filing it without leave is to waive the presentation of the evidence
a reasonable amount of bail is the probability of the accused appearing in trial. Bail; for the accused. (People vs. Fores, 269 SCRA 62.) 3. Yes. Without any evidence from the
Matter of Right vs. Matter of Discretion (1999) When is bail a matter of right and when is accused, the prima facie evidence of the prosecution has been converted to proof
it a matter of discretion? (2%) SUGGESTED ANSWER: When Bail is a matter of right: All beyond reasonable doubt. ALTERNATIVE ANSWER: If the evidence of guilt is not strong
persons in custody shall (a) before or after conviction by the metropolitan and municipal and beyond reasonable doubt then the court cannot legally convict X for murder.
trial courts, and (b) before conviction by the RTC of an offense not punishable by death, Demurrer to Evidence; w/o Leave of Court (2001) Carlos, the accused in a theft case, filed
reclusion perpetua or life imprisonment, be admitted to bail as a matter of right, with a demurrer to evidence without leave of court. The court denied the demurrer to
sufficient sureties, or be released on recognizance as prescribed by law or Rule 114. (Sec. evidence and Carlos moved to present his evidence. The court denied Carlos’ motion to
4, Rule 114, Rules of Court, as amended by Circular No. 12-94.) Remedial Law Bar present evidence and instead judgment on the basis of the evidence for the prosecution.
Examination Q & A (1997-2006) When bail is a matter of discretion: Upon conviction by Was the court correct in preventing Carlos from presenting his evidence and rendering
the RTC of an offense not punishable by death, reclusion perpetua or life imprisonment, judgment on the basis of the evidence for the prosecution? Why? (5%) SUGGESTED
on application of the accused. If the penalty of imprisonment exceeds six years but not ANSWER: Yes, because the demurrer to the evidence was filed without leave of court.
more than 20 years, bail shall be denied upon a showing by the prosecution, with notice The Rules provide that when the demurrer to evidence is filed without leave of court, the
to the accused, of the following or other similar circumstances: That the accused is a accused waives the right to present evidence and submits the case for judgment on the
recidivist, quasi-re-cidivist or habitual delinquent, or has committed the crime basis of the evidence for the prosecution. (Sec. 23 of Rule 119, Revised Rules of Criminal
aggravated by the circumstance of reiteration; That the accused is found to have Procedure) Demurrer to Evidence; w/o Leave of Court (2004) The information for illegal
previously escaped from legal confinement, evaded sentence, or has violated the possession of firearm filed against the accused specifically alleged that he had no license
conditions of his bail without valid justification; That the accused committed the offense or permit to possess the caliber .45 pistol mentioned therein. In its evidence-in-chief, the
while on probation, parole, or under conditional pardon; That the circumstances of the prosecution established the fact that the subject firearm was lawfully seized by the
accused or his case indicate the probability of flight if released on bail; or That there is police from the possession of the accused, that is, while the pistol was tucked at his waist
undue risk that during the pendency of the appeal, the accused may commit another in plain view, without the accused being able to present any license or permit to possess
crime. (Sec. 1, Id.) Bail; Matter of Right vs. Matter of Discretion (2006) When is bail a the firearm. The prosecution on such evidence rested its case and within a period of five
matter of right and when is it a matter of discretion? (5%) SUGGESTED ANSWER: Bail is a days therefrom, the accused filed a demurrer to evidence, in sum contending that the
matter of right (a) before or after conviction by the inferior courts; (b) before conviction prosecution evidence has not established the guilt of the accused beyond reasonable
by the RTC of an offense not punishable by death, reclusion perpetua or life doubt and so prayed that he be acquitted of the offense charged. by:
imprisonment., when the evidence of guilt is not strong (Sec. 4, Rule 114, 2000 Rules of sirdondee@gmail.com Page 43 of 66 The trial court denied the demurrer to evidence and
Criminal Procedure). Bail is discretionary: Upon conviction by the RTC of an offense not deemed the accused as having waived his right to present evidence and submitted the
punishable by death, reclusion perpetua or life imprisonment (Sec. 5, Rule 114, 2000 case for judgment on the basis of the prosecution evidence. In due time, the court
Rules of Criminal Procedure). Bail; Witness Posting Bail (1999) May the Court require a rendered judgment finding the accused guilty of the offense charged beyond reasonable
witness to post bail? Explain your answer. (2%) SUGGESTED ANSWER: Yes. The court may doubt and accordingly imposing on him the penalty prescribed therefor. Is the judgment
require a witness to post bail if he is a material witness and bail is needed to secure his of the trial court valid and proper? Reason. (5%) SUGGESTED ANSWER: Yes. The
appearance. The rules provide that when the court is satisfied, upon proof or oath, that a judgment of the trial court is valid. The accused did not ask for leave to file the demurrer
material witness will not testify when required, it may, upon motion of either party, to evidence. He is deemed to have waived his right to present evidence. (Sec. 23 of Rule
order the witness to post bail in such sum as may be deemed proper. Upon refusal to 119; People v. Flores, 269 SCRA 62 [1997]; Bernardo v. Court of Appeals, 278 SCRA 782
post bail, the court shall commit him to prison until he complies or is legally discharged [1997]. However, the judgment is not proper or is erroneous because there was no
after his testimony is taken. (Sec. 6, Rule 119) Complaint vs. Information (1999) showing from the proper office like the Firearms Explosive Unit of the Philippine National
Distinguish a Complaint from Information. (2%) by: sirdondee@gmail.com Page 42 of 66 Police that the accused has a permit to own or possess the firearm, which is fatal to the
SUGGESTED ANSWER: In criminal procedure, a complaint is a sworn written statement conviction of the accused. (Mallari v. Court of Appeals &People,265 SCRA 456[1996]).
charging a person with an offense, subscribed by the offended party, any peace officer or Dismissal; Failure to Prosecute (2003) When a criminal case is dismissed on nolle
other peace officer charged with the enforcement of the law violated. (Sec. 3, Rule 110, prosequi, can it later be refilled? (4%) SUGGESTED ANSWER: As a general rule, when a
1985 Rules of Criminal Procedure); while an information is an accusation in writing criminal case is dismissed on nolle prosequi before the accused is placed on trial and
charging a person with an offense subscribed by the prosecutor and filed with the court. before he is called on to plead, this is not equivalent to an acquittal and does not bar a
(Sec. 4, Id.) Demurrer to Evidence; Contract of Carriage (2004) AX, a Makati-bound subsequent prosecution for the same offense. (Galvez v. Court of Appeals, 237 SCRA 685
paying passenger of PBU, a public utility bus, died instantly on board the bus on account [1994]). Dismissal; Provisional Dismissal (2003) Before the arraignment for the crime of
of the fatal head wounds he sustained as a result of the strong impact of the collision murder, the private complainant executed an Affidavit of Desistance stating that she was
between the bus and a dump truck that happened while the bus was still travelling on not sure if the accused was the man who killed her husband. The public prosecutor filed
EDSA towards Makati. The foregoing facts, among others, were duly established on a Motion to Quash the Information on the ground that with private complainant’s
evidence-in-chief by the plaintiff TY, sole heir of AX, in TY’s action against the subject desistance, he did not have evidence sufficient to convict the accused. On 02 January
common carrier for breach of contract of carriage. After TY had rested his case, the 2001, the court without further proceedings granted the motion and provisionally
common carrier filed a demurrer to evidence, contending that plaintiff’s evidence is dismissed the case. The accused gave his express consent to the provisional dismissal of
insufficient because it did not show (1) that defendant was negligent and (2) that such the case. The offended party was notified of the dismissal but she refused to give her
negligence was the proximate cause of the collision. Should the court grant or deny consent. Subsequently, the private complainant urged the public prosecutor to refile the
defendant's demurrer to evidence? Reason briefly. (5%) SUGGESTED ANSWER: No. The murder charge because the accused failed to pay the consideration which he had
court should not grant defendant's demurrer to evidence because the case is for breach promised for the execution of the Affidavit of Desistance. The public prosecutor obliged
and refiled the murder charge against the accused on 01 February 2003, the accused dwelling. Information; Amendment (2001) Amando was charged with frustrated
filed a Motion to Quash the Information on the ground that the provisional dismissal of homicide. Before he entered his plea and upon the advice of his counsel, he manifested
the case had already become permanent. (6%) Remedial Law Bar Examination Q & A his willingness to admit having committed the offense of serious physical injuries. The
(1997-2006) a) Was the provisional dismissal of the case proper? b) Resolve the Motion prosecution then filed an amended information for serious physical injuries against
to Quash. SUGGESTED ANSWER: (a) The provisional dismissal of the case was proper Amando. What steps or action should the prosecution take so that the amended
because the accused gave his express consent thereto and the offended party was information against Amando which downgrades the nature of the offense could be
notified. It was not necessary for the offended party to give her consent thereto. (Sec. 8 validly made? Why? (5%) SUGGESTED ANSWER: In order that the amended information
of Rule 117). (b) The motion to quash the information should be denied because, while which downgrades the nature of the offense could be validly made, the prosecution
the provisional dismissal had already become permanent, the prescriptive period for should file a motion to ask for leave of court with notice to the offended party. (Sec.14 of
filing the murder charge had not prescribed. There was no double jeopardy because the Rule 110, Revised Rules of Criminal Procedure). The new rule is for the protection of the
first case was dismissed before the accused had pleaded to the charge. (Sec. 7 of Rule interest of the offended party and to prevent possible abuse by the prosecution.
117). Double Jeopardy (2002) D was charged with slight physical injuries in the MTC. He Information; Amendment; Double Jeopardy; Bail (2002) A. D and E were charged with
pleaded not guilty and went to trial. After the prosecution had presented its evidence, homicide in one information. Before they could be arraigned, the prosecution moved to
the trial court set the continuation of the hearing on another date. On the date amend the information to exclude E therefrom. Can the court grant the motion to
scheduled for hearing, the prosecutor failed to appear, whereupon the court, on motion amend? Why? (2%) B. On the facts above stated, suppose the prosecution, instead of
of D, dismissed the case. A few minutes later, the prosecutor arrived and opposed the filing a motion to amend, moved to withdraw the information altogether and its motion
dismissal of the case. The court reconsidered its order and directed D to present his was granted. Can the prosecution re-file the information although this time for murder?
evidence. Before the next date of trial came, however, D moved that the last order be set Explain (3%) SUGGESTED ANSWER: A. Yes, provided notice is given to the offended party
aside on the ground that the reinstatement of the case had placed him twice in jeopardy. and the court states its reasons for granting the same. (Rule 110, sec. 14). by:
Acceding to this motion, the court again dismissed the case. The prosecutor then filed an sirdondee@gmail.com Page 45 of 66 B. Yes, the prosecution can re-file the information
information in the RTC, charging D with direct assault based on the same facts alleged in for murder in substitution of the information for homicide because no double jeopardy
the information for slight physical injuries but with the added allegation that D inflicted has as yet attached. [Galvez v. Court of Appeals, 237 SCRA 685 (1994)]. Information;
the injuries out of resentment for what the complainant had done in the performance of Amendment; Supervening Events (1997) A was accused of homicide for the killing of B.
his duties as chairman of the board of election inspectors. D moved to quash the second During the trial, the public prosecutor received a copy of the marriage certificate of A
information on the ground that its filing had placed him in double jeopardy. How should and B. (a) Can the public prosecutor move for the amendment of the information to
D’s motion to quash be resolved? (4%) SUGGESTED ANSWER: D’s motion to quash should charge A with the crime of parricide? (b) Suppose instead of moving for the amendment
be granted on the ground of double jeopardy because the first offense charged is of the information, the public prosecutor presented in evidence the marriage certificate
necessarily included in the second offense charged. [Draculan v. Donato, 140 SCRA 425 without objection on the part of the defense, could Abe convicted of parricide?
(1985)]. ALTERNATIVE ANSWER: D’s motion to quash should be denied because the two SUGGESTED ANSWER: (a) No. The Information cannot be amended to change the offense
dismissals of the case against him were on his motion (hence with his express consent) charged from homicide to parricide. Firstly, the marriage is not a supervening fact arising
and his right to a speedy trial was not violated. Double Jeopardy; Upgrading; Original from the act constituting the charge of homicide. (Sec. 7[a] of Rule 117). Secondly, after
Charges (2005) For the multiple stab wounds sustained by the victim, Noel was charged plea, amendments may be done only as to matters of form. The amendment is
with frustrated homicide in the RTC. Upon arraignment, he entered a plea of guilty to substantial because it will change the nature of the offense. (Sec. 14 of Rule 110;
said crime. Neither the court nor the prosecution by: sirdondee@gmail.com Page 44 of Dionaldo us. Dacuycuy. 108 SCRA 736). (b) No. A can be convicted only of homicide not
66 was aware that the victim had died two days earlier on account of his stab wounds. of parricide which is a graver offense. The accused has the constitutional rights of due
Because of his guilty plea, Noel was convicted of frustrated homicide and meted the process and to be informed of the nature and the cause of the accusation against him.
corresponding penalty. When the prosecution learned of the victim's death, it filed (Secs. 1, 14 (1) and (2} Art. III. 1987 Constitution), Information; Bail (2003) After the
within fifteen (15) days therefrom a motion to amend the information to upgrade the requisite proceedings, the Provincial Prosecutor filed an Information for homicide against
charge from frustrated homicide to consummated homicide. Noel opposed the motion X. The latter, however, timely filed a Petition for Review of the Resolution of the
claiming that the admission of the amended information would place him in double Provincial Prosecutor with the Secretary of Justice who, in due time, issued a Resolution
jeopardy. Resolve the motion with reasons. (4%) SUGGESTED ANSWER: The amended reversing the resolution of the Provincial Prosecutor and directing him to withdraw the
information to consummated homicide from frustrated homicide does not place the Information. Before the Provincial Prosecutor could comply with the directive of the
accused in double jeopardy. As provided in the second paragraph of Sec. 7, Rule Secretary of Justice, the court issued a warrant of arrest against X. The Public Prosecutor
117,2000 Rules of Criminal Procedure, the conviction of the accused shall not be a bar to filed a Motion to Quash the Warrant of Arrest and to Withdraw the Information,
another prosecution for an offense which necessarily includes the offense charged in the attaching to it the Resolution of the Secretary of Justice. The court denied the motion.
former complaint or information when: (a) the graver offense developed due to (6%) a) Was there a legal basis for the court to deny the motion? b) If you were the
supervening facts arising from the same act or omission constituting the former charge; counsel for the accused, what remedies, if any, would you pursue? SUGGESTED ANSWER:
or (b) the facts constituting the graver charge became known or were discovered only Remedial Law Bar Examination Q & A (1997-2006) a. Yes, there is a legal basis for the
after a plea was entered in the former complaint or information. Here, when the plea to court to deny the motion to quash the warrant of arrest and to withdraw the
frustrated homicide was made, neither the court nor the prosecution was aware that the information. The court is not bound by the Resolution of the Secretary of Justice. (Crespo
victim had died two days earlier on account of his stab wounds. Extradition (2004) RP v. Mogul, 151 SCRA 462 [1987]). b. If I were the counsel for the accused, I would
and State XX have a subsisting Extradition Treaty. Pursuant thereto RP's Secretary of surrender the accused and apply for bail because the offense is merely homicide, a non-
Justice (SOJ) filed a Petition for Extradition before the MM RTC alleging that Juan Kwan is capital offense. At the pre-trial, I would make a stipulation of facts with the prosecution
the subject of an arrest warrant duly issued by the proper criminal court of State XX in which would show that no offense was committed. Information; Motion to Quash (2000)
connection with a criminal case for tax evasion and fraud before his return to RP as a BC is charged with illegal possession of firearms under an Information signed by a
balikbayan. Petitioner prays that Juan be extradited and delivered to the proper Provincial Prosecutor. After arraignment but before pre-trial, BC found out that the
authorities of State XX for trial, and that to prevent Juan's flight in the interim, a warrant Provincial Prosecutor had no authority to sign and file the information as it was the City
for his immediate arrest be issued. Before the RTC could act on the petition for Prosecutor who has such authority. During the pre-trial, BC moves that the case against
extradition, Juan filed before it an urgent motion, in sum praying (1) that SoJ's him be dismissed on the ground that the Information is defective because the officer
application for an arrest warrant be set for hearing and (2) that Juan be allowed to post signing it lacked the authority to do so. The Provincial Prosecutor opposes the motion on
bail in the event the court would issue an arrest warrant. Should the court grant or deny the ground of estoppel as BC did not move to quash the Information before arraignment.
Juan's prayers? Reason. (5%) SUGGESTED ANSWER: Under the Extradition Treaty and If you are counsel for BC, what is your argument to refute the opposition of the
Law, the application of the Secretary of Justice for a warrant of arrest need not be set for Provincial Prosecutor? (5%) SUGGESTED ANSWER: I would argue that since the Provincial
hearing, and Juan cannot be allowed to post bail if the court would issue a warrant of Prosecutor had no authority to file the information, the court did not acquire jurisdiction
arrest. The provisions in the Rules of Court on arrest and bail are not basically applicable. over the person of the accused and over the subject matter of the offense charged.
(Government of the United States of America v. Puruganan, 389 SCRA 623 [2002]) (Cudia v. Court of Appeals, 284 SCRA 173 [1999]). Hence, this ground is not waived if not
Remedial Law Bar Examination Q & A (1997-2006) Information (2001) The prosecution raised in a motion to quash and could be raised at the pretrial. (Sec. 8, Rule 117, Rules of
filed an information against Jose for slight physical injuries alleging the acts constituting Court). Information; Motion to Quash (2005) Rodolfo is charged with possession of
the offense but without anymore alleging that it was committed after Jose’s unlawful unlicensed firearms in an Information filed in the RTC. It was alleged therein that Rodolfo
entry in the complainant’s abode. Was the information correctly prepared by the was in possession of two unlicensed firearms: a .45 caliber and-a .32 caliber. Under
prosecution? Why? (5%) SUGGESTED ANSWER: No. The aggravating circumstance of Republic Act No. 8294, possession of an unlicensed .45 caliber gun is punishable by
unlawful entry in the complainant’s abode has to be specified in the information; prision mayor in its minimum period and a fine of P30.000.00, while possession of an
otherwise, it cannot be considered as aggravating. (Sec. 8 of Rule 110, Revised Rules of unlicensed .32 caliber gun is punishable by prision correctional in its maximum period
Criminal Procedure) ALTERNATIVE ANSWER: The information prepared by the prosecutor and a fine of not less than P15,000.00. As counsel of the accused, you intend to file a
is not correct because the accused should have been charged with qualified trespass to motion to quash the Information. What ground or grounds should you invoke? Explain.
(4%) SUGGESTED ANSWER: The ground for the motion to quash is that more than one guilty to the offense charged. Thereafter, before trial commenced, he asked the court to
offense is charged in the information. (Sec. 3[f], Rule 117, 2000 Rules of Criminal allow him to change his plea of not guilty to a plea of guilt but only to estafa involving
Procedure) Likewise, the RTC has no jurisdiction over the second offense of by: P5,000.00. Can the court allow D to change his plea? Why? (2%) SUGGESTED ANSWER:
sirdondee@gmail.com Page 46 of 66 possession of an unlicensed .32 caliber gun, No, because a plea of guilty to a lesser offense may be allowed if the lesser offense is
punishable by prision correctional in its maximum period and a fine of not less than necessarily included in the offense charged. (Rule 116, sec. 2). Estafa involving P5,000.00
P15.000.00. It is the MTC that has exclusive and original jurisdiction over all offenses is not necessarily included in theft of an article worth P15,000.00 Prejudicial Question
punishable by imprisonment not exceeding six years. (Sec. 2, R.A. No. 7691, amending (1999) What is a prejudicial question? (2%) SUGGESTED ANSWER: A prejudicial question
B.P. Blg. 129) Information; Motion to Quash; Grounds (1998) Give two (2) grounds to is an issue involved in a civil action which is similar or intimately related to the issue
quash an Information.[2%] If the Information is not accompanied by a certification that a raised in the criminal action, the resolution of which determines whether or not the
preliminary investigation has been conducted. Is the Information void? [3%] SUGGESTED criminal action may proceed. (Sec. 5 of Rule 111.) ANOTHER ANSWER: A prejudicial
ANSWER: 1. Two grounds to quash an Information are: a) That the facts charged do not question is one based on a fact distinct and separate from the crime but so intimately
constitute an offense; and b) That the court trying the case has no jurisdiction over the connected with it that it determines the guilt or innocence of the accused. Prejudicial
offense charged or the person of the accused. c) That the officer who filed the Question (2000) CX is charged with estafa in court for failure to remit to MM sums of
information had no authority to do so; d) That it does not conform substantially to the money collected by him (CX) for MM in payment for goods purchased from MM, by
prescribed form; e) That more than one offense is charged except in those cases in which Remedial Law Bar Examination Q & A (1997-2006) depositing the amounts in his (CX’s)
existing laws prescribe a single punishment for various offenses; f) That the criminal personal bank account. CX files a motion to suspend proceedings pending resolution of a
action or liability has been extinguished; g) That it contains averments which, if true, civil case earlier filed in court by CX against MM for accounting and damages involving
would constitute a legal excuse or justification; and h) That the accused has been the amounts subject of the criminal case. As the prosecutor in the criminal case, briefly
previously convicted or in jeopardy of being convicted, or acquitted of the offense discuss your grounds in support of your opposition to the motion to suspend
charged. (Sec. 3, Rule 117. Rules of Criminal Procedure.) SUGGESTED ANSWER: 2. No. proceedings. (5%). SUGGESTED ANSWER: As the prosecutor, I will argue that the motion
The certification which is provided in Sec. 4, Rule 112. Rules of Criminal Procedure, is not to suspend is not in order for the following reasons: The civil case filed by CX against MM
an indispensable part of the information. (People vs. Lapura, 255 SCRA 85.) Judgment; for accounting and damages does not involve an issue similar to or intimately related to
Promulgation of Judgment (1997) X, the accused in a homicide case before the RTC. the issue of estafa raised in the criminal action. The resolution of the issue in the civil
Dagupan Cay, was personally notified of the promulgation of judgment in his case set for case for accounting will not determine whether or not the criminal action for estafa may
10 December 1996. On said date. X was not present as he had to attend to the trial of proceed. (Sec. 5, Rule 111, Rules of Criminal Procedure.) Prejudicial Question; Suspension
another criminal case against him in Tarlac, Tarlac. The trial court denied the motion of of Criminal Action (1999) A allegedly sold to B a parcel of land which A later also sold to
the counsel of X to postpone the promulgation. (a) How shall the court promulgate the X. B brought a civil action for nullification of the second sale and asked that the sale
judgment in the absence of the accused? (b) Can the trial court also order the arrest of made by A in his favor be declared valid. A theorized that he never sold the property to B
X? SUGGESTED ANSWER: (a) In the absence of the accused, the promulgation shall be and his purported signatures appearing in the first deed of sale were forgeries.
made by recording the Judgment in the Version 1997-2006 Updated by Dondee Remedial Thereafter, an Information for estafa was filed against A based on the same double sale
Law Bar Examination Q & A (1997-2006) criminal docket and a copy thereof served upon that was the subject of the civil action. A filed a "Motion for Suspension of Action" in the
the accused or counsel. (Sec. 6. third par., Rule 120) (b) No, the trial court cannot order criminal case, contending that the resolution of the issue in the civil case would
the arrest of X if the judgment is one of acquittal and, in any event, his failure to appear necessarily be determinative of his guilt or innocence. Is the suspension of the criminal
was with justifiable cause since he had to attend to another criminal case against him. action in order? Explain. (2%) SUGGESTED ANSWER: Yes. The suspension of the criminal
Jurisdiction; Complex Crimes (2003) In complex crimes, how is the jurisdiction of a court action is in order because the defense of A in the civil action, that he never sold the
determined? 4% SUGGESTED ANSWER: In a complex crime, jurisdiction over the whole property to B and that his purported signatures in the first deed of sale were forgeries, is
complex crime must be lodged with the trial court having jurisdiction to impose the a prejudicial question the resolution of which is determinative of his guilt or innocence. If
maximum and most serious penalty imposable on an offense forming part of the the first sale is null and void, there would be no double sale and A would be innocent of
complex crime. (Cuyos v. Garcia, 160 SCRA 302 [1988]). Jurisdiction; Finality of a the offense of estafa. (Ras v. Rasul, 100 SCRA 125.) Pre-Trial Agreement (2004) Mayor
Judgment (2005) Mariano was convicted by the RTC for raping Victoria and meted the TM was charged of malversation through falsification of official documents. Assisted by
penalty of reclusion perpetua. While serving sentence at the National Penitentiary, Atty. OP as counsel de parte during pre-trial, he signed together with Ombudsman
Mariano and Victoria were married. Mariano filed a motion in said court for his release Prosecutor TG a "Joint Stipulation of Facts and Documents," which was presented to the
from the penitentiary on his claim that under Republic Act No. 8353, his marriage to Sandiganbayan. Before the court could issue a pre-trial order but after some delay
Victoria extinguished the criminal action against him for rape, as well as the penalty caused by Atty. OP, he was substituted by Atty. QR as defense counsel. Atty. QR
imposed on him. However, the court denied the motion on the ground that it had lost forthwith filed a motion to withdraw the "Joint Stipulation," alleging that it is prejudicial
jurisdiction over the case after its decision had become final and executory. (7%) a) Is the to the accused because it contains, inter by: sirdondee@gmail.com Page 48 of 66 alia,
filing of the court correct? Explain. SUGGESTED ANSWER: No. The court can never lose the statement that the "Defense admitted all the documentary evidence of the
jurisdiction so long as its decision has not yet been fully implemented and satisfied. Prosecution," thus leaving the accused little or no room to defend himself, and violating
Finality of a judgment cannot operate to divest a court of its jurisdiction. The court his right against self-incrimination. Should the court grant or deny QR's motion? Reason.
retains an interest in seeing the proper execution and implementation of its judgments, (5%) SUGGESTED ANSWER: The court should deny QR's motion. If in the pretrial
and to that extent, may issue such orders necessary and appropriate for these purposes. agreement signed by the accused and his counsel, the accused admits the documentary
(Echegaray v. Secretary of Justice, G.R. No. 13205, January 19, 1999) b) What evidence of the prosecution, it does not violate his right against self-incrimination. His
remedy/remedies should the counsel of Mariano take to secure his proper and most lawyer cannot file a motion to withdraw. A pre-trial order is not needed. (Bayas v.
expeditious release from the National Penitentiary? Explain. SUGGESTED ANSWER: To Sandiganbayan, 391 SCRA 415(2002}). The admission of such documentary evidence is
secure the proper and most expeditious release of Mariano from the National allowed by the rule. (Sec. 2 of Rule 118; People v. Hernandez, 260 SCRA 25 [1996]). Pre-
Penitentiary, his counsel should file: (a) a petition for habeas corpus for the illegal Trial; Criminal Case vs. Civil Case (1997) Give three distinctions between a pre-trial in a
confinement of Mariano (Rule 102), or (b) a motion in the court which convicted him, to criminal case and a pre-trial in a civil case. SUGGESTED ANSWER: Three distinctions
nullify the execution of his sentence or the order of his commitment on the ground that between a pre-trial in a criminal case and a pre-trial in a civil case are as follows: 1. The
a supervening development had occurred (Melo v. People, G.R. No. L-3580, March 22, pre-trial in a criminal case is conducted only "where the accused and counsel agree"
1950) despite the finality of the judgment. by: sirdondee@gmail.com Page 47 of 66 (Rule 118, Sec. 1): while the pre-trial in a civil case is mandatory. (Sec. 1 of former Rule
Parties; Prosecution of Offenses (2000) Your friend YY, an orphan, 16 years old, seeks 20; Sec, 1 of new Rule 18). 2. The pre-trial in a criminal case does not consider the
your legal advice. She tells you that ZZ, her uncle, subjected her to acts of lasciviousness; possibility of a compromise, which is one important aspect of the pre-trial in a civil case.
that when she told her grandparents, they told her to just keep quiet and not to file (Sec. 1 of former Rule 20; Sec. 2 of new Rule 18). 3. In a criminal case, a pre-trial
charges against ZZ, their son. Feeling very much aggrieved, she asks you how her uncle ZZ agreement is required to be reduced to writing and signed by the accused and his
can be made to answer for his crime. a) What would your advice be? Explain. (3%) b) counsel (See; Rule 118, Sec. 4); while in a civil case, the agreement may be contained in
Suppose the crime committed against YY by her uncle ZZ is rape, witnessed by your the pre-trial order. (Sec. 4 of former Rule 20; See 7 of new Rule 78). Provisional Dismissal
mutual friend XX. But this time, YY was prevailed upon by her grandparents not to file (2002) In a prosecution for robbery against D, the prosecutor moved for the
charges. XX asks you if she can initiate the complaint against ZZ. Would your answer be postponement of the first scheduled hearing on the ground that he had lost his records
the same? Explain. (2%). SUGGESTED ANSWER: (a) I would advise the minor, an orphan of the case. The court granted the motion but, when the new date of trial arrived, the
of 16 years of age, to file the complaint herself independently of her grandparents, prosecutor, alleging that he could not locate his witnesses, moved for the provisional
because she is not incompetent or incapable to doing so upon grounds other than her dismissal of the case. If D’s counsel does not object, may the court grant the motion of
minority. (Sec. 5, Rule 110, Rules of Criminal Procedure.) (b) Since rape is now classified the prosecutor? Why? (3%) SUGGESTED ANSWER: No, because a case cannot be
as a Crime Against Persons under the Anti-Rape Law of 1997 (RA 8353), I would advise XX provisionally dismissed except upon the express consent of the accused and with notice
to initiate the complaint against ZZ. Plea of Guilty; to a Lesser Offense (2002) D was to the offended party. (Rule 117, sec. 8). Remedies; Void Judgment (2004) AX was
charged with theft of an article worth p15,000.00. Upon being arraigned, he pleaded not charged before the YY RTC with theft of jewelry valued at P20.000, punishable with
Version 1997-2006 Updated by Dondee Remedial Law Bar Examination Q & A (1997- that he was possessing said rifle without license or authority to possess, and a Waiver of
2006) imprisonment of up to 10 years of prision mayor under the Revised Penal Code. Right to Counsel. During the trial of X for illegal possession of firearm, the prosecution
After trial, he was convicted of the offense charged, notwithstanding that the material submitted in evidence the rifle. Sworn Statement and Waiver of Right to Counsel,
facts duly established during the trial showed that the offense committed was estafa, individually rule on the admissibility in evidence of the: 1. Rifle; [2%] 2. Sworn Statement;
punishable by imprisonment of up to eight years of prision mayor under the said Code. and [2%1 3. Waiver of Right to Counsel of X. [1%] SUGGESTED ANSWER: 1. The rifle is not
No appeal having been taken therefrom, said judgment of conviction became final. Is the admissible in evidence because it was seized without a proper search warrant. A
judgment of conviction valid? Is the said judgment reviewable thru a special civil action warrantless search is not justified. There was time to secure a search warrant. (People
for certiorari? Reason. (5%) SUGGESTED ANSWER: Yes, the judgment of conviction for us. Encinada G.R. No. 116720, October 2. 1997 and other cases) 2. The sworn statement
theft upon an information for theft is valid because the court had jurisdiction to render is not admissible in evidence because it was taken without informing him of his custodial
judgment. However, the judgment was grossly and blatantly erroneous. The variance rights and without the assistance of counsel which should be independent and
between the evidence and the judgment of conviction is substantial since the evidence is competent and preferably of the choice of the accused. (People us. Januario, 267 SCRA
one for estafa while the judgment is one for theft. The elements of the two crimes are 608.) 3. The waiver of his right to counsel is not admissible because it was made without
not the same. (Lauro Santos v. People, 181 SCRA 487). One offense does not necessarily the assistance of counsel of his choice. (People us. Gomez, 270 SCRA 433.) Admissibility
include or is included in the other. (Sec. 5 of Rule 120). The judgment of conviction is (2002) by: sirdondee@gmail.com Page 50 of 66 Acting on a tip by an informant, police
reviewable by certiorari even if no appeal had been taken, because the judge committed officers stopped a car being driven by D and ordered him to open the trunk. The officers
a grave abuse of discretion tantamount to lack or excess of his jurisdiction in convicting found a bag containing several kilos of cocaine. They seized the car and the cocaine as
the accused of theft and in violating due process and his right to be informed of the evidence and placed D under arrest. Without advising him of his right to remain silent
nature and the cause of the accusation against him, which make the judgment void. With and to have the assistance of an attorney, they questioned him regarding the cocaine. In
the mistake in charging the proper offense, the judge should have directed the filing of reply, D said, “I don’t know anything about it. It isn’t even my car.” D was charged with
the proper information and thereafter dismissed the original information. (Sec. 19 of illegal possession of cocaine, a prohibited drug. Upon motion of D, the court suppressed
Rule 119). Search Warrant; Motion to Quash (2005) Police operatives of the Western the use of cocaine as evidence and dismissed the charges against him. D commenced
Police District, Philippine National Police, applied for a search warrant in the RTC for the proceedings against the police for the recovery of his car. In his direct examination, D
search of the house of Juan Santos and the seizure of an undetermined amount of shabu. testified that he owned the car but had registered it in the name of a friend for
The team arrived at the house of Santos but failed to find him there. Instead, the team convenience. On cross-examination, the attorney representing the police asked, “After
found Roberto Co. The team conducted a search in the house of Santos in the presence your arrest, did you not tell the arresting officers that it wasn’t your car?” If you were D’s
of Roberto Co and barangay officials and found ten (10) grams of shabu. Roberto Co was attorney, would you object to the question? Why? (5%) SUGGESTED ANSWER: Yes,
charged in court with illegal possession of ten grams of shabu. Before his arraignment, because his admission made when he was questioned after he was placed under arrest
Roberto Co filed a motion to quash the warrant on the following grounds (a) it was not was in violation of his constitutional right to be informed of his right to remain silent and
the accused named in the search warrant; and (b) the warrant does not describe the to have competent and independent counsel of his own choice. Hence, it is inadmissible
article to be seized with sufficient particularity. Resolve the motion with reasons. (4%) in evidence. [Constitution, Art. III, sec. 12; R.A. 7438 (1992), sec, 2; People v. Mahinay,
SUGGESTED ANSWER: The motion to quash should be denied. The name of the person in 302 SCRA 455]. ALTERNATIVE ANSWER: Yes, because the question did not lay the
the search warrant is not important. It is not even necessary that a particular person be predicate to justify the cross-examination question. Admissibility (2004) Sgt. GR of WPD
by: sirdondee@gmail.com Page 49 of 66 implicated (Mantaring v. Roman, A.M. No. RTJ- arrested two NPA suspects, Max and Brix, both aged 22, in the act of robbing a grocery in
93-904, February 28, 1996), so long as the search is conducted in the place where the Ermita. As he handcuffed them he noted a pistol tucked in Max's waist and a dagger
search warrant will be served. Moreover, describing the shabu in an undetermined hidden under Brix's shirt, which he promptly confiscated. At the police investigation
amount is sufficiently particular. (People v. Tee, G.R. Nos. 140546-47, January 20, 2003) room, Max and Brix orally waived their right to counsel and to remain silent. Then under
Trial; Trial in Absentia; Automatic Review of Conviction (1998) What are the requisites of oath, they freely answered questions asked by the police desk officer. Thereafter they
a trial in absentia? [2%] If an accused who was sentenced to death escapes, is there still a signed their sworn statements before the police captain, a lawyer. Max admitted his part
legal necessity for the Supreme Court to review the decision of conviction? [3%] in the robbery, his possession of a pistol and his ownership of the packet of shabu found
SUGGESTED ANSWER: 1. The requisites of trial in absentia are: (a) the accused has in his pocket. Brix admitted his role in the robbery and his possession of a dagger. But
already been arraigned; (b) he has been duly notified of the trial; and (c) his failure to they denied being NPA hit men. In due course, proper charges were filed by the City
appear is unjustifiable. (Sec. 14 [2], Article III. Constitution; Parada vs. Veneracion, 269 Prosecutor against both arrestees before the MM RTC. May the written statements
SCRA 371 [1997].) 2. Yes, there is still a legal necessity for the Supreme Court (as of 2004 signed and sworn to by Max and Brix be admitted by the trial court as evidence for the
the Court of Appeals has the jurisdiction to such review) to review the decision of prosecution? Reason. (5%) SUGGESTED ANSWER: No. The sworn written statements of
conviction sentencing the accused to death, because he is entitled to an automatic Max and Brix may not be admitted in evidence, because they were not assisted by
review of the death sentence. (Sees. 3[e] and 10, Rule 122, Rules of Criminal Procedure; counsel. Even if the police captain Remedial Law Bar Examination Q & A (1997-2006)
People vs. Espargas, 260 SCRA 539.) Venue (1997) Where is the proper venue for the before whom they signed the statements was a lawyer, he was not functioning as a
filing of an information in the following cases? a) The theft of a car in Pasig City which lawyer, nor can he be considered as an independent counsel. Waiver of the right to a
was brought to Obando, Bulacan, where it was cannibalized. b) The theft by X, a bill lawyer must be done in writing and in the presence of independent counsel. (People v.
collector of ABC Company, with main offices in Makati City, of his collections from Mahinay, 302 SCRA 455 11999]; People v. Espiritu, 302 SCRA 533 [1999]). Admissibility;
customers in Tagaytay City. In the contract of employment, X was detailed to the Admission of Guilt; Requirements (2006) What are the requirements in order that an
Calamba branch office, Laguna, where he was to turn in his collections. c) The admission of guilt of an accused during a custodial investigation be admitted in
malversation of public funds by a Philippine consul detailed in the Philippine Embassy in evidence? (2.5%) SUGGESTED ANSWER: The admission must be voluntary. The admission
London. SUGGESTED ANSWER: (a) The proper venue is in Pasig City where the theft of must be in writing. The admission must be made with the assistance of competent,
the car was committed, not in Obando where it was cannibalized. Theft is not a independent counsel. 4. The admission must be express (People v. Prinsipe, G.R. No.
continuing offense. (People v Mercado, 65 Phil 665). (b) If the crime charged is theft, the 135862, May 2, 2002). 5. In case the accused waives his rights to silence and to counsel,
venue is in Calamba where he did not turn in his collections. If the crime of X is estafa, such waiver must be in writing, executed with the assistance of competent, independent
the essential ingredients of the offense took place in Tagaytay City where he received his counsel. Admissibility; Document; Not raised in the Pleading (2004) In a complaint for a
collections, in Calamba where he should have turned in his collections, and in Makati City sum of money filed before the MM RTC, plaintiff did not mention or even just hint at any
where the ABC Company was based. The information may therefore be filed in Tagaytay demand for payment made on defendant before commencing suit. During the trial,
City or Calamba or Makati which have concurrent territorial Jurisdiction. (Catingub vs. plaintiff duly offered Exh. "A" in evidence for the stated purpose of proving the making of
Court of Appeals, 121 SCRA 106). Remedial Law Bar Examination Q & A (1997-2006) (c) extrajudicial demand on defendant to pay P500.000, the subject of the suit. Exh. "A" was
The proper court is the Sandiganbayan which has jurisdiction over crimes committed by a a letter of demand for defendant to pay said sum of money within 10 days from receipt,
consul or higher official in the diplomatic service. (Sec. 4(c). PD 1606, as amended by RA. addressed to and served on defendant some two months before suit was begun. Without
No. 7975). The Sandiganbayan is a national court. (Nunez v. Sandiganbayan, 111 SCRA objection from defendant, the court admitted Exh. "A" in evidence. Was the court's
433 [1982]. It has only one venue at present, which is in Metro Manila, until RA. No. admission of Exh. "A" in evidence erroneous or not? Reason. (5%) SUGGESTED ANSWER:
7975, providing for two other branches in Cebu and in Cagayan de Oro, is implemented. The court's admission of Exh. "A" in evidence is not erroneous. It was admitted in
Alternative Answers: (b) The information may be filed either in Calamba or in Makati evidence without objection on the part of the defendant. It should be treated as if it had
City, not in Tagaytay City where no offense had as yet been committed, (c) Assuming that been raised in the pleadings. The complaint may be amended to conform to the
the Sandiganbayan has no jurisdiction, the proper venue is the first RTC in which the evidence, but if it is not so amended, it does not affect the result of the trial on this issue.
charge is filed (Sec. 15(d). Rule 110). EVIDENCE Admissibility (1998) The barangay captain (Sec. 5 of Rule 10). Admissibility; Electronic Evidence (2003) a) State the rule on the
reported to the police that X was illegally keeping in his house in the barangay an admissibility of an electronic evidence. b) When is an electronic evidence regarded as
Armalite M16 rifle. On the strength of that information, the police conducted a search of being the equivalent of an original document under the Best Evidence Rule? 4%
the house of X and indeed found said rifle. The police raiders seized the rifle and brought SUGGESTED ANSWER: (a) Whenever a rule of evidence refers to the term writing,
X to the police station. During the investigation, he voluntarily signed a Sworn Statement document, record, instrument, memorandum or any other form of writing, such term
shall be by: sirdondee@gmail.com Page 51 of 66 deemed to include an electronic intestate and without any issue. Ten (10) persons headed by Jocelyn, claiming to be the
document as defined in these Rules. (Sec. 1 of Rule 3, Rules of Electronic Evidence collateral relatives of the deceased Linda, filed an action for partition with the RTC
effective August 1, 2001). An electronic document is admissible in evidence if it complies praying for the segregation of Linda’s ½ share, submitting in support of their petition the
with the rules on admissibility prescribed by the Rules of Court and related laws and is baptismal certificates of seven of the petitioners, a family bible belonging to Linda in
authenticated in the manner prescribed by these Rules. (Sec. 2 of Rule 3, Id.). The which the names of the petitioners have been entered, a photocopy of the birth
authenticity of any private electronic document must be proved by evidence that it had certificate of Jocelyn, and a certification of the local civil registrar that its office had been
been digitally signed and other appropriate security measures have been applied. (Sec. 2 completely razed by fire. The spouses Ceres refused to partition on the following
of Rule 5, Id.). (b) An electronic document shall be regarded as the equivalent of an grounds: 1) the baptismal certificates of the parish priest are evidence only of the
original document under the Best Evidence Rule if it is a printout or output readable by administration of the sacrament of baptism and they do not prove filiation of the alleged
sight or other means, shown to reflect the data accurately. (Sec. 1 of Rule 4) collateral relatives of the deceased; 2) entry in the family bible is hearsay; 3) the
Admissibility; Object or Real Evidence (1994) At the trial of Ace for violation of the certification of the registrar on non-availability of the records of birth does not prove
Dangerous Drugs Act, the prosecution offers in evidence a photocopy of the marked filiation: 4) in partition cases where filiation to the deceased is in dispute, prior and
P100.00 bills used in the “buy-bust” operation. Ace objects to the introduction of the separate judicial declaration of heirship in a settlement of estate proceedings is
photocopy on the ground that the Best Evidence Rule prohibits the introduction of necessary; and 5) there is need for publication as real Remedial Law Bar Examination Q &
secondary evidence in lieu of the original. a) Is the photocopy real (object) evidence or A (1997-2006) property is involved. As counsel for Jocelyn and her co-petitioners, argue
documentary evidence? b) Is the photocopy admissible in evidence? SUGGESTED against the objections of the spouses Ceres so as to convince the court to allow the
ANSWER: a) The photocopy of the marked bills is real (object) evidence not documentary partition. Discuss each of the five (5) arguments briefly but completely. (10%)
evidence, because the marked bills are real evidence. b) Yes, the photocopy is admissible SUGGESTED ANSWER: (1) The baptismal certificate can show filiation or prove pedigree.
in evidence, because the best evidence rule does not apply to object or real evidence. It is one of the other means allowed under the Rules of Court and special laws to show
Admissibility; Objections (1997) What are the two kinds of objections? Explain each pedigree. (Trinidad v. Court of Appeals, 289 SCRA 188 [1998]; Heirs of ILgnacio Conti v.
briefly. Given an example of each. SUGGESTED ANSWER: Two kinds of objections are: (1) Court of Appeals, 300 SCRA 345 [1998]). (2) Entries in the family bible may be received as
the evidence being presented is not relevant to the issue; and (2) the evidence is evidence of pedigree. (Sec. 40, Rule 130, Rules of Court). (3) The certification by the civil
incompetent or excluded by the law or the rules, (Sec. 3, Rule 138). An example of the registrar of the non-availability of records is needed to justify the presentation of
first is when the prosecution offers as evidence the alleged offer of an Insurance secondary evidence, which is the photocopy of the birth certificate of Jocelyn. (Heirs of
company to pay for the damages suffered by the victim in a homicide case. (See 1997 No. Ignacio Conti v. Court of Appeals, supra.) (4) Declaration of heirship in a settlement
14). Examples of the second are evidence obtained in violation of the Constitutional proceeding is not necessary. It can be made in the ordinary action for partition wherein
prohibition against unreasonable searches and seizures and confessions and admissions the heirs are exercising the right pertaining to the decedent, their predecessor-ininterest,
in violation of the rights of a person under custodial Investigation. ALTERNATIVE to ask for partition as co-owners (Id.) (5) Even if real property is involved, no publication
ANSWERS: Remedial Law Bar Examination Q & A (1997-2006) 1) Specific objections: is necessary, because what is sought is the mere segregation of Linda’s share in the
Example: parol evidence and best evidence rule General Objections: Example: continuing property. (Sec. 1 of Rule 69; Id.) Admissibility; Rules of Evidence (1997) Give the reasons
objections (Sec. 37 of Rule 132). 2) The two kinds of objections are: (1) objection to a underlying the adoption of the following rules of evidence: (a) Dead Man Rule (b) Parol
question propounded in the course of the oral examination of the witness and (2) Evidence Rule (c) Best Evidence Rule (d) The rule against the admission of illegally
objection to an offer of evidence in writing. Objection to a question propounded in the obtained extrajudicial confession (e) The rule against the admission of an offer of
course of the oral examination of a witness shall be made as soon as the grounds compromise in civil cases SUGGESTED ANSWER: The reasons behind the following rules
therefor shall become reasonably apparent otherwise, it is waived. An offer of objection are as follows: (a) DEAD MAN RULE: if death has closed the lips of one party, the policy of
in writing shall be made within three (3) days after notice of the offer, unless a different the law is to close the lips of the other. (Goni v. Court ofAppeals, L-77434. September 23,
period is allowed by the court. In both instances the grounds for objection must be 1986, 144 SCRA 222). This is to prevent the temptation to perjury because death has
specified. An example of the first is when the witness is being cross-examined and the already sealed the lips of the party. (b) PAROL EVIDENCE RULE: It is designed to give
cross examination is on a matter not relevant. An example of the second is that the certainty to a transaction which has been reduced to writing, because written evidence is
evidence offered is not the best evidence. Admissibility; Offer to Marry; Circumstantial much more certain and accurate than that which rests on fleeting memory only.
Evidence (1998) A was accused of having raped X. Rule on the admissibility of the (Francisco, Rules of Court Vol. VII, Part I. p. 154) by: sirdondee@gmail.com Page 53 of 66
following pieces of evidence: an offer of A to marry X; and (3%] a pair of short pants (c) BEST EVIDENCE RULE: This Rule is adopted for the prevention of fraud and is declared
allegedly left by A at the crime which the court, over the objection of A, required him to to be essential to the pure administration of justice. (Moran, Vol. 5, p. 12.) If a party is in
put on, and when he did, it fit him well. [2%] SUGGESTED ANSWER: 1. A's offer to marry possession of such evidence and withholds it, the presumption naturally arises that the
X is admissible in evidence as an Implied admission of guilt because rape cases are not better evidence is withheld for fraudulent purposes. (Francisco. Rules of Court, vol. VII.
allowed to be compromised. (Sec. 27 of Rule 13O; People vs. Domingo, 226 SCRA 156.) 2. Part I, pp, 121,122) (d) An illegally obtained extrajudicial confession nullifies the intrinsic
The pair of short pants, which fit the accused well, is circumstantial evidence of his guilt, validity of the confession and renders it unreliable as evidence of the truth. (Moran, vol.
although standing alone it cannot be the basis of conviction. The accused cannot object 5, p. 257) it is the fruit of a poisonous tree. (e) The reason for the rule against the
to the court requiring him to put the short pants on. It is not part of his right against admission of an offer of compromise in civil case as an admission of any liability is that
selfincrimination because it is a mere physical act. Admissibility; Offer to Pay Expenses parties are encouraged to enter into compromises. Courts should endeavor to persuade
(1997) A, while driving his car, ran over B. A visited B at the hospital and offered to pay the litigants in a civil case to agree upon some fair compromise. (Art. 2029, Civil Code).
for his hospitalization expenses. After the filing of the criminal case against A for serious During pre-trial, courts should direct the parties to consider the possibility of an amicable
physical injuries through reckless imprudence. A's insurance carrier offered to pay for the settlement. (Sec. 1[a] of former Rule 20: Sec. 2 [a] of new Rule 16). Best Evidence Rule
injuries and damages suffered by B. The offer was rejected because B considered the (1997) When A loaned a sum of money to B. A typed a single copy of the promissory
amount offered as inadequate. a) Is the offer by A to pay the hospitalization expenses of note, which they both signed A made two photo (xeroxed) copies of the promissory note,
B admissible in evidence? b) Is the offer by A's insurance carrier to pay for the injuries giving one copy to B and retaining the other copy. A entrusted the typewritten copy to
and damages of B admissible in evidence? by: sirdondee@gmail.com Page 52 of 66 his counsel for safekeeping. The copy with A's counsel was destroyed when the law office
SUGGESTED ANSWER: (a) The offer by A to pay the hospitalization expenses of B is not was burned. a) In an action to collect on the promissory note, which is deemed to be the
admissible in evidence to prove his guilt in both the civil and criminal cases. (Rule 130, "original" copy for the purpose of the "Best Evidence Rule"? b) Can the photocopies in
Sec. 27, fourth par.). (b) No. It is irrelevant. The obligation of the insurance company is the hands of the parties be considered "duplicate original copies"? c) As counsel for A,
based on the contract of insurance and is not admissible in evidence against the accused how will you prove the loan given to A and B? SUGGESTED ANSWER: (a) The copy that
because it was not offered by the accused but by the insurance company which is not his was signed and lost is the only "original" copy for purposes of the Best Evidence Rule.
agent. Admissibility; Private Document (2005) May a private document be offered, and (Sec. 4 [b] of Rule 130). (b) No, They are not duplicate original copies because there are
admitted in evidence both as documentary evidence and as object evidence? Explain. photocopies which were not signed (Mahilum v. Court of Appeals, 17 SCRA 482), They
SUGGESTED ANSWER: Yes, it can be considered as both documentary and object constitute secondary evidence. (Sec. 5 of Rule 130). (c) The loan given by A to B may be
evidence. A private document may be offered and admitted in evidence both as proved by secondary evidence through the xeroxed copies of the promissory note. The
documentary evidence and as object evidence. A document can also be considered as an rules provide that when the original document is lost or destroyed, or cannot be
object for purposes of the case. Objects as evidence are those addressed to the senses of produced in court, the offerer, upon proof of its execution or existence and the cause of
the court. (Sec. 1, Rule 130, Rules of Court) Documentary evidence consists of writings or its unavailability without bad faith on his part, may prove its contents by a copy, or by a
any material containing letters, words, numbers, figures, symbols or other modes of recital of its contents in some authentic document, or by the Version 1997-2006 Updated
written expressions, offered ns proof of their contents. (Sec. 2, Rule 130, Rules of Court) by Dondee Remedial Law Bar Examination Q & A (1997-2006) testimony of witnesses in
Hence, a private document may be presented as object evidence in order to 'establish the order stated. (Sec. 5 of Rule 130). Burden of Proof vs. Burden of Evidence (2004)
certain physical evidence or characteristics that are visible on the paper and writings that Distinguish Burden of proof and burden of evidence. SUGGESTED ANSWER: Burden of
comprise the document. Admissibility; Proof of Filiation; Action of Partition (2000) Linda proof is the duty of a party to present evidence on the facts in issue necessary to
and spouses Arnulfo and Regina Ceres were co-owners of a parcel of land. Linda died establish his claim or defense by the amount of evidence required by law. (Sec. 1 of Rule
131), while burden of evidence is the duty of a party to go forward with the evidence to hearsay. It is part of the res gestae. It is also an independently relevant statement. The
overthrow prima facie evidence established against him. (Bautista v. Sarmiento, 138 police officer testified of his own personal knowledge, not to the truth of Candida's
SCRA 587 [1985]). Character Evidence (2002) D was prosecuted for homicide for allegedly statement, i.e., that she told him, despite her pleas, Dencio had raped her. (People v.
beating up V to death with an iron pipe. A. May the prosecution introduce evidence that Gaddi,G.R. No. 74065, February 27,1989) b) If the police officer will testify that he
V had a good reputation for peacefulness and nonviolence? Why? (2%) B. May D noticed Candida to be hysterical and on the verge of collapse, would such testimony be
introduce evidence of specific violent acts by V? Why? (3%) SUGGESTED ANSWER: A. The considered as opinion, hence, inadmissible? Explain. SUGGESTED ANSWER: No, it cannot
prosecution may introduce evidence of the good or even bad moral character of the be considered as opinion, because he was testifying on what he actually observed. The
victim if it tends to establish in any reasonable degree the probability or improbability of last paragraph of Sec. 50, Rule 130, Revised Rules of by: sirdondee@gmail.com Page 55
the offense charged. [Rule 130, sec. 51 a (3)]. In this case, the evidence is not relevant. B. of 66 Evidence, expressly provides that a witness may testify on his impressions of the
Yes, D may introduce evidence of specific violent acts by V. Evidence that one did or did emotion, behavior, condition or appearance of a person. Hearsay; Exceptions (1999) a)
not do a certain thing at one time is not admissible to prove that he did or did not do the Define hearsay evidence? (2%) b) What are the exceptions to the hearsay rule? (2%)
same or a similar thing at another time; but it may be received to prove a specific intent SUGGESTED ANSWER: Hearsay evidence may be defined as evidence that consists of
or knowledge, identity, plan, system, scheme, habit, custom or usage, and the like. (Rule testimony not coming from personal knowledge (Sec. 36, Rule 130, Rules of Court).
130, sec. 34). Confession; Affidavit of Recantation (1998) If the accused on the witness Hearsay testimony is the testimony of a witness as to what he has heard other persons
stand repeats his earlier uncounseled extrajudicial confession implicating his co-accused say about the facts in issue. The exceptions to the hearsay rule are: dying declaration,
in the crime charged, is that testimony admissible in evidence against the latter? [3%] declaration against interest, act or declaration about pedigree, family reputation or
What is the probative value of a witness' Affidavit of Recantation? [2%] SUGGESTED tradition regarding pedigree, common reputation, part of the res gestae, entries in the
ANSWER: Yes. The accused can testify by repeating his earlier uncounseled extrajudicial course of business, entries in official records, commercial lists and the like, learned
confession, because he can be subjected to cross-examination. On the probative value of treatises, and testimony or deposition at a former proceeding. (37 to 47, Rule 13O, Rules
an affidavit of recantation, courts look with disfavor upon recantations because they can of Court) Hearsay; Exceptions; Dying Declaration (1999) The accused was charged with
easily be secured from witnesses, usually through intimidation or for a monetary robbery and homicide. The victim suffered several stab wounds. It appears that eleven
consideration, Recanted testimony is exceedingly unreliable. There is always the (11) hours after the crime, while the victim was being brought to the hospital in a jeep,
probability by: sirdondee@gmail.com Page 54 of 66 that it will be repudiated. (Molina vs. with his brother and a policeman as companions, the victim was asked certain questions
People. 259 SCRA 138.) Facts; Legislative Facts vs. Adjudicative Facts (2004) Legislative which he answered, pointing to the accused as his assailant. His answers were put down
facts and adjudicative facts. SUGGESTED ANSWER: Legislative facts refer to facts in writing, but since he was a in a critical condition, his brother and the policeman signed
mentioned in a statute or in an explanatory note, while adjudicative facts are facts found the statement. Is the statement admissible as a dying declaration? Explain. (2%)
in a court decision. Hearsay Evidence (2002) Romeo is sued for damages for injuries SUGGESTED ANSWER: Yes. The statement is admissible as a dying declaration if the
suffered by the plaintiff in a vehicular accident. Julieta, a witness in court, testifies that victim subsequently died and his answers were made under the consciousness of
Romeo told her (Julieta) that he (Romeo) heard Antonio, a witness to the accident, give impending death (Sec. 37 of Rule 130). The fact that he did not sign the statement point
an excited account of the accident immediately after its occurrence. Is Julieta’s testimony to the accused as his assailant, because he was in critical condition, does not affect its
admissible against Romeo over proper and timely objection? Why? (5%) SUGGESTED admissibility as a dying declaration. A dying declaration need not be in writing (People v.
ANSWER: No, Julieta’s testimony is not admissible against Romeo, because while the Viovicente, 286 SCRA 1) Hearsay; Inapplicable (2003) X was charged with robbery. On the
excited account of Antonio, a witness to the accident, was told to Romeo, it was only strength of a warrant of arrest issued by the court, X was arrested by police operatives.
Romeo who told Julieta about it, which makes it hearsay. Hearsay Evidence vs. Opinion They seized from his person a handgun. A charge for illegal possession of firearm was
Evidence (2004) Hearsay evidence and opinion evidence. SUGGESTED ANSWER: Hearsay also filed against him. In a press conference called by the police, X admitted that he had
evidence consists of testimony that is not based on personal knowledge of the person robbed the victim of jewelry valued at P500,000.00. The robbery and illegal possession of
testifying, (see Sec. 36, Rule 130), while opinion evidence is expert evidence based on the firearm cases were tried jointly. The prosecution presented in evidence a newspaper
personal knowledge skill, experience or training of the person testifying (Sec. 49, Id.) and clipping of the report to the reporter who was present during the press conference
evidence of an ordinary witness on limited matters (Sec. 50, Id.). Hearsay; Exception; Remedial Law Bar Examination Q & A (1997-2006) stating that X admitted the robbery. It
Dead Man Statute (2001) Maximo filed an action against Pedro, the administrator of the likewise presented a certification of the PNP Firearms and Explosive Office attesting that
estate of deceased Juan, for the recovery of a car which is part of the latter’s estate. the accused had no license to carry any firearm. The certifying officer, however, was not
During the trial, Maximo presented witness Mariano who testified that he was present presented as a witness. Both pieces of evidence were objected to by the defense. (6%) a)
when Maximo and Juan agreed that the latter would pay a rental of P20,000.00 for the Is the newspaper clipping admissible in evidence against X? b) Is the certification of the
use of Maximo’s car for one month after which Juan should immediately return the car PNP Firearm and Explosive Office without the certifying officer testifying on it admissible
to Maximo. Pedro objected to the admission of Mariano’s testimony. If you were the in evidence against X? SUGGESTED ANSWER: (a) Yes, the newspaper clipping is
judge, would you sustain Pedro’s objection? Why? (5%) SUGGESTED ANSWER: No, the admissible in evidence against X. regardless of the truth or falsity of a statement, the
testimony is admissible in evidence because witness Mariano who testified as to what hearsay rule does not apply and the statement may be shown where the fact that it is
Maximo and Juan, the deceased person agreed upon, is not disqualified to testify on the made is relevant. Evidence as to the making of such statement is not secondary but
agreement. Those disqualified are parties or assignors of parties to a case, or persons in primary, for the statement itself may constitute a fact in issue or be circumstantially
whose behalf a case is prosecuted, against the administrator or Juan’s estate, upon a relevant as to the existence of such fact. (Gotesco Investment Corporation vs. Chatto,
Remedial Law Bar Examination Q & A (1997-2006) claim or demand against his estate as 210 SCRA 18 [1992]) (b) Yes, the certification is admissible in evidence against X because
to any matter of fact occurring before Juan’s death. (Sec. 23 of Rule 130) Hearsay; a written statement signed by an officer having the custody of an official record or by his
Exception; Dying Declaration (1998) Requisites of Dying Declaration. [2%) SUGGESTED deputy that after diligent search no record or entry of a specified tenor is found to exist
ANSWER: The requisites for the admissibility of a dying declaration are: (a) the in the records of his office, accompanied by a certificate as above provided, is admissible
declaration is made by the deceased under the consciousness of his impending death; (b) as evidence that the records of his office contain no such record or entry. (Sec. 28 of Rule
the deceased was at the time competent as a witness; (c) the declaration concerns the 132). Judicial Notice; Evidence (2005) Explain briefly whether the RTC may, motu proprio,
cause and surrounding circumstances of the declarant's death; and (d) the declaration is take judicial notice of: (5%) 1. The street name of methamphetamine hydro-chloride is
offered in a (criminal) case wherein the declarant's death is the subject of inquiry. shabu. SUGGESTED ANSWER: The RTC may motu proprio take judicial notice of the street
(People vs. Santos, 270 SCRA 650.) ALTERNATIVE ANSWER: The declaration of a dying name of methamphetamine hydrochloride is shabu, considering the chemical
person, made under the consciousness of an impending death, may be received in any composition of shabu. (People v. Macasling, GM, No. 90342, May 27, 1993) 2.
case wherein his death is the subject of Inquiry, as evidence of the cause and Ordinances approved by municipalities under its territorial jurisdiction; SUGGESTED
surrounding circumstances of such death. (Sec. 37 of Rule 13O.) Hearsay; Exception; Res ANSWER: In the absence of statutory authority, the RTC may not take judicial notice of
Gestae; Opinion of Ordinary Witness (2005) Dencio barged into the house of Marcela, ordinances approved by municipalities under their territorial jurisdiction, except on
tied her to a chair and robbed her of assorted pieces of jewelry and money. Dencio then appeal from the municipal trial courts, which took judicial notice of the ordinance in
brought Candida, Marcela's maid, to a bedroom where he raped her. Marcela could hear question. (U.S. v. Blanco, G.R, No. 12435, November 9,1917; U.S. v. Hernandez, G.R. No.
Candida crying and pleading: "Huwag! Maawa ka sa akin!" After raping Candida, Dencio 9699, August 26, 1915) 3. Foreign laws; SUGGESTED ANSWER: by: sirdondee@gmail.com
fled from the house with the loot. Candida then untied Marcela and rushed to the police Page 56 of 66 The RTC may not generally take judicial notice of foreign laws (In re Estate
station about a kilometer away and told Police Officer Roberto Maawa that Dencio had of Johnson, G.R. No. 12767, November 16, 1918; Fluemer v. Hix, G.R. No. 32636, March
barged into the house of Marcela, tied the latter to a chair and robbed her of her jewelry 17, 1930), which must be proved like any other matter of fact (Sy Joe Lieng v. Sy Quia,
and money. Candida also related to the police officer that despite her pleas, Dencio had G.R. No. 4718, March 19, 1910) except in a few instances, the court in the exercise of its
raped her. The policeman noticed that Candida was hysterical and on the verge of sound judicial discretion, may take notice of foreign laws when Philippine courts are
collapse. Dencio was charged with robbery with rape. During the trial, Candida can no evidently familiar with them, such as the Spanish Civil Code, which had taken effect in
longer be located. (8%) a) If the prosecution presents Police Officer Roberto Maawa to the Philippines, and other allied legislation. (Pardo v. Republic, G.R. No. L2248 January
testify on what Candida had told him, would such testimony of the policeman be 23, 1950; Delgado v. Republic, G.R. No. L2546, January .28, 1950) 4. Rules and
hearsay? Explain. SUGGESTED ANSWER: No. The testimony of the policeman is not Regulations issued by quasi-judicial bodies implementing statutes; SUGGESTED ANSWER:
The RTC may take judicial notice of Rules and Regulations issued by quasi-judicial bodies Nolan objects to the admissibility of Kim’s testimony on the ground that Kim merely
implementing statutes, because they are capable of unquestionable demonstration stated her opinion without having been first qualified as expert witness. Should you, as
(Chattamal v. Collector of Customs, G.R. No. 16347, November 3,1920), unless the law judge, exclude the testimony of Kim? Remedial Law Bar Examination Q & A (1997-2006)
itself considers such rules as an integral part of the statute, in which case judicial notice SUGGESTED ANSWER: No. The testimony of Kim should not be excluded. Even though
becomes mandatory. 5. Rape may be committed even in public places. SUGGESTED Kim is not an expert witness, Kim may testify on her impressions of the emotion,
ANSWER: The RTC may take judicial notice of the fact that rape may be committed even behavior, condition or appearance of a person. (Sec. 50, last par., Rule 130). Parol
in public places. The "public setting" of the rape is not an indication of consent. (People Evidence Rule (2001) Pedro filed a complaint against Lucio for the recovery of a sum of
v. Tongson, G.R. No. 91261, February 18, 1991) The Supreme Court has taken judicial money based on a promissory note executed by Lucio. In his complaint, Pedro alleged
notice of the fact that a man overcome by perversity and beastly passion chooses neither that although the promissory note says that it is payable within 120 days, the truth is
the time, place, occasion nor victim. (People v, Barcelona, G.R. No. 82589, October 31, that the note is payable immediately after 90 days but that if Pedro is willing, he may,
1990) Judicial Notice; Evidence; Foreign Law (1997) a) Give three instances when a upon request of Lucio give the latter up to 120 days to pay the note. During the hearing,
Philippine court can take judicial notice of a foreign law. b) How do you prove a written Pedro testified that the truth is that the agreement between him and Lucio is for the
foreign law? c) Suppose a foreign law was pleaded as part of the defense of defendant latter to pay immediately after ninety day’s time. Also, since the original note was with
but no evidence was presented to prove the existence of said law, what is the Lucio and the latter would not surrender to Pedro the original note which Lucio kept in a
presumption to be taken by the court as to the wordings of said law"? SUGGESTED place about one day’s trip from where he received the notice to produce the note and in
ANSWER: (a) The three instances when a Philippine court can take judicial notice of a spite of such notice to produce the same within six hours from receipt of such notice,
foreign law are: (1) when the Philippine courts are evidently familiar with the foreign law Lucio failed to do so. Pedro presented a copy of the note which was executed at the
(Moran. Vol. 5, p. 34, 1980 edition); (2) when the foreign law refers to the law of nations same time as the original and with identical contents. a) Over the objection of Lucio, will
(Sec. 1 of Rule 129) and (3) when it refers to a published treatise, periodical or pamphlet Pedro be allowed to testify as to the true agreement or contents of the promissory note?
on the subject of law if the court takes judicial notice of the fact that the writer thereof is Why? (2%) b) Over the objection of Lucio, can Pedro present a copy of the promissory
recognized in his profession or calling as expert on the subject (Sec. 46. Rule 130). note and have it admitted as valid evidence in his favor? Why? (3%) SUGGESTED
Remedial Law Bar Examination Q & A (1997-2006) (b) A written foreign law may be ANSWER: a) Yes, because Pedro has alleged in his complaint that the promissory note
evidenced by an official publication thereof or by a copy attested by the officer having does not express the true intent and agreement of the parties. This is an exception to the
the legal custody of the record, or by his deputy, and accompanied. If the record is not parol evidence rule. [Sec. 9(b) of Rule 130, Rules of Court] b) Yes, the copy in the
kept in the Philippines, with a certificate that such officer has the custody, if the office in possession of Pedro is a duplicate original and with identical contents. [Sec. 4(b) of Rule
which the record is kept is in a foreign country, the certificate may be made by a 130]. Moreover, the failure of Lucio to produce the original of the note is excusable
secretary of the embassy or legation, consul general, consul, vice-consul, or consular because he was not given reasonable notice, as requirement under the Rules before
agent or by any officer in the foreign service of the Philippines stationed in the foreign secondary evidence may be presented. (Sec. 6 of Rule 130, Rules of Court) Note: The
country in which the record is kept, and authenticated by the seal of his office (Sec. 24, promissory note is an actionable document and the original or a copy thereof should
Rule 132, Zalamea v. CA, 228 SCRA 23). (c) The presumption is that the wordings of the have been attached to the complaint. (Sec. 7 of Rule 9, 1997 Rules of Civil Procedure). In
foreign law are the same as the local law. (Northwest Orient Airlines v. Court of Appeals, such a case, the genuineness and due execution of the note, if not denied under oath,
241 SCRA 192; Moran, Vol. 6. page 34, 1980 edition; Lim v. Collector of Customs, 36 Phil. would be deemed admitted. (Sec. 8 of Rule 9, 1997 Rules of Civil Procedure)
472). This is known as the PROCESSUAL PRESUMPTION. Memorandum (1996) X states on Preponderance vs. Substantial Evidence (2003) Distinguish preponderance of evidence
direct examination that he once knew the facts being asked but he cannot recall them from substantial evidence. 4% SUGGESTED ANSWER: by: sirdondee@gmail.com Page 58
now. When handed a written record of the facts he testifies that the facts are correctly of 66 PREPONDERANCE OF EVIDENCE means that the evidence as a whole adduced by
stated, but that he has never seen the writing before. Is the writing admissible as past one side is superior to that of the other. This is applicable in civil cases. (Sec. 1 of Rule
recollection recorded? Explain, SUGGESTED ANSWER: No, because for the written record 133; Municipality of Moncada v. Cajuigan, 21 Phil, 184 [1912]). SUBSTANTIAL EVIDENCE
to be admissible as past recollection recorded. It must have been written or recorded by is that amount of relevant evidence which a reasonable mind might accept as adequate
X or under his direction at the time when the fact occurred, or immediately thereafter, or to justify a conclusion. This is applicable in case filed before administrative or quasi-
at any other time when the fact was fresh in his memory and he knew that the same was judicial bodies. (Sec. 5 of Rule 133) Privilege Communication (1998) C is the child of the
correctly written or recorded. (Sec. 16 of Rule 132) But in this case X has never seen the spouses H and W. H sued his wife W for judicial declaration of nullity of marriage under
writing before. Offer of Evidence (1997) A trial court cannot take into consideration in Article 36 of the Family Code. In the trial, the following testified over the objection of W:
deciding a case an evidence that has not been "formally offered". When are the C, H and D, a doctor of medicine who used to treat W. Rule on W's objections which are
following pieces of evidence formally offered? (a) Testimonial evidence (b) Documentary the following: H cannot testify against her because of the rule on marital privilege; [1%] C
evidence (c) Object evidence SUGGESTED ANSWER: (a) Testimonial evidence is formally cannot testify against her because of the doctrine on parental privilege; and [2%] D
offered at the time the witness is called to testify. (Rule 132. Sec. 35, first par.). (b) cannot testify against her because of the doctrine of privileged communication between
Documentary evidence is formally offered after the presentation of the testimonial patient and physician. [2%] SUGGESTED ANSWER: 1. The rule of marital privilege cannot
evidence. (Rule 132, Sec. 35, second par.). (c) The same is true with object evidence. It is be invoked in the annulment case under Rule 36 of the Family Code because it is a civil
also offered after the presentation of the testimonial evidence. by: case filed by one against the other, (Sec. 22 , Rule 130. Rules of Court.) 2. The doctrine of
sirdondee@gmail.com Page 57 of 66 Offer of Evidence; res inter alios acta (2003) X and Y parental privilege cannot likewise be invoked by W as against the testimony of C, their
were charged with murder. Upon application of the prosecution, Y was discharged from child. C may not be compelled to testify but is free to testify against her. (Sec. 25. Rule
the Information to be utilized as a state witness. The prosecutor presented Y as witness 130. Rules of Court; Art. 215, Family Code.) 3. D, as a doctor who used to treat W, is
but forgot to state the purpose of his testimony much less offer it in evidence. Y testified disqualified to testify against W over her objection as to any advice or treatment given by
that he and X conspired to kill the victim but it was X who actually shot the victim. The him or any information which he may have acquired in his professional capacity. (Sec. 24
testimony of Y was the only material evidence establishing the guilt of X. Y was [c], Rule 130. Rules of Court.) ALTERNATIVE ANSWER: If the doctor's testimony is
thoroughly crossexamined by the defense counsel. After the prosecution rested its case, pursuant to the requirement of establishing the psychological incapacity of W, and he is
the defense filed a motion for demurrer to evidence based on the following grounds. (a) the expert called upon to testify for the purpose, then it should be allowed. (Republic vs.
The testimony of Y should be excluded because its purpose was not initially stated and it Court of Appeals and Molina, 26S SCRA 198.) Privilege Communication; Marital Privilege
was not formally offered in evidence as required by Section 34, Rule 132 of the Revised (1989) Ody sued spouses Cesar and Baby for a sum of money and damages. At the trial,
Rules of Evidence; and (b) Y’s testimony is not admissible against X pursuant to the rule Ody called Baby as his first witness. Baby objected, joined by Cesar, on the ground that
on “res inter alios acta”. Rule on the motion for demurrer to evidence on the above she may not be compelled to testify against her husband. Ody insisted and contended
grounds. (6%) SUGGESTED ANSWER: The demurrer to the evidence should be denied that after all, she would just be questioned about a conference they had with the
because: a) The testimony of Y should not be excluded because the defense counsel did barangay captain, a Remedial Law Bar Examination Q & A (1997-2006) matter which is
not object to his testimony despite the fact that the prosecutor forgot to state its not confidential in nature. The trial court ruled in favor of Ody. Was the ruling proper?
purpose or offer it in evidence. Moreover, the defense counsel thoroughly cross- Will your answer be the same if the matters to be testified on were known to Baby or
examined Y and thus waived the objection. b) The res inter alios acta rule does not apply acquired by her prior to her marriage to Cesar? Explain. SUGGESTED ANSWER: No. Under
because Y testified in open court and was subjected to cross examination. Offer of the Rules on Evidence, a wife cannot be examined for or against her husband without his
Evidence; Testimonial & Documentary (1994) What is the difference between an offer of consent, except in civil cases by one against the other, or in a criminal case for a crime
testimonial evidence and an offer of documentary evidence? SUGGESTED ANSWER: An committed by one against the other. Since the case was filed by Ody against the spouses
offer of testimonial evidence is made at the time the witness is called to testify, while an Cesar and Baby, Baby cannot be compelled to testify for or against Cesar without his
offer of documentary evidence is made after the presentation of a party’s testimonial consent. (Lezama vs. Rodriguez, 23 SCRA 1166). The answer would be the same if the
evidence. (Sec. 35, Rule 132). Opinion Rule (1994) At Nolan’s trial for possession and use matters to be testified on were known to Baby or acquired by her prior to her marriage
of the prohibited drug, known as “shabu:, his girlfriend Kim, testified that on a particular to Cesar, because the marital disqualification rule may be invoked with respect to
day, he would see Nolan very prim and proper, alert and sharp, but that three days after, testimony on any fact. It is immaterial whether such matters were known to Baby before
he would appear haggard, tired and overly nervous at the slightest sound he would hear. or after her marriage to Cesar. Privilege Communication; Marital Privilege (2000) Vida
and Romeo are legally married. Romeo is charged to court with the crime of serious Distinguish Competency of the witness and credibility of the witness. SUGGESTED
physical injuries committed against Selmo, son of Vida, stepson of Romeo. Vida ANSWER: Competency of the witness refers to a witness who can perceive, and
witnessed the infliction of the injuries on Selmo by Romeo. The public prosecutor called perceiving, can make known his perception to others (Sec. 20 of Rule 130), while
Vida to the witness stand and offered her testimony as an eyewitness. Counsel for credibility of the witness refers to a witness whose testimony is believable. by:
Romeo objected on the ground of the marital disqualification rule under the Rules of sirdondee@gmail.com Page 60 of 66 Witness; Examination of a Child Witness; via Live-
Court. a) Is the objection valid? (3%) b) Will your answer be the same if Vida’s testimony Link TV (2005) When may the trial court order that the testimony of a child be taken by
is offered in a civil case for recovery of personal property filed by Selmo against Romeo? live-link television? Explain. SUGGESTED ANSWER: The testimony of a child may be taken
(2%) SUGGESTED ANSWER: (a) No. While neither the husband nor the wife may testify by live-link television if there is a substantial likelihood that the child would suffer trauma
for or against the other without the consent of the affected spouse, one exception is if from testifying in the presence of the accused, his counsel or the prosecutor as the case
the testimony of the spouse is in a criminal case for a crime committed by one against may be. The trauma must of a kind which would impair the completeness or truthfulness
the other or the latter’s direct descendants or ascendants. (Sec, 22, Rule 130). The case of the testimony of the child. (See Sec. 25, Rule on Examination of a Child Witness).
falls under this exception because Selma is the direct descendant of the spouse Vide. (b) Witness; Examination of Witnesses (1997) a) Aside from asking a witness to explain and
No. The marital disqualification rule applies this time. The exception provided by the supplement his answer in the crossexamination, can the proponent ask in re-direct
rules is in a civil case by one spouse against the other. The case here involves a case by examination questions on matters not dealt with during cross-examination? b) Aside
Selmo for the recovery of personal property against Vida’s spouse, Romeo. Privilege from asking the witness on matters stated in his re-direct examination, can the opponent
Communication; Marital Privilege (2004) XYZ, an alien, was criminally charged of in his re-cross-examination ask questions on matters not dealt with during the redirect?
promoting and facilitating child prostitution and other sexual abuses under Rep. Act No. c) After plaintiff has formally submitted his evidence, he realized that he had forgotten to
7610. The principal witness against him was his Filipina wife, ABC. Earlier, she had present what he considered an important evidence. Can he recall a witness? SUGGESTED
complained that XYZ's hotel was by: sirdondee@gmail.com Page 59 of 66 being used as a ANSWER: (a) Yes, on redirect examination, questions on matters not dealt with during
center for sex tourism and child trafficking. The defense counsel for XYZ objected to the the crossexamination may be allowed by the court in its discretion. (Sec. 7 of Rule 132).
testimony of ABC at the trial of the child prostitution case and the introduction of the (b) Yes, the opponent in his re-cross-examination may also ask questions on such other
affidavits she executed against her husband as a violation of espousal confidentiality and matters as may be allowed by the court in its discretion. (Sec. 8. Rule 132). (c) Yes, after
marital privilege rule. It turned out that DEF, the minor daughter of ABC by her first formally submitting his evidence, the plaintiff can recall a witness with leave of court.
husband who was a Filipino, was molested by XYZ earlier. Thus, ABC had filed for legal The court may grant or withhold leave in its discretion as the interests of justice may
separation from XYZ since last year. May the court admit the testimony and affidavits of require. (Sec. 9. Rule 132). Witness; Examination of Witnesses (2002) Is this question on
the wife, ABC, against her husband, XYZ, in the criminal case involving child prostitution? direct examination objectionable: “What happened on July 12, 1999”? Why? (2%)
Reason. (5%) SUGGESTED ANSWER: Yes. The court may admit the testimony and SUGGESTED ANSWER: The question is objectionable because it has no basis, unless
affidavits of the wife against her husband in the criminal case where it involves child before the question is asked the proper basis is laid. Witness; Utilized as State Witness;
prostitution of the wife's daughter. It is not covered by the marital privilege rule. One Procedure (2006) As counsel of an accused charged with homicide, you are convinced
exception thereof is where the crime is committed by one against the other or the that he can be utilized as a state witness. What procedure will you take? (2.5%)
latter's direct descendants or ascendants. (Sec. 22, Rule 130). A crime by the husband SUGGESTED ANSWER: As counsel of an accused charged with homicide, the procedure
against the daughter is a crime against the wife and directly attacks or vitally impairs the that can be followed for the accused to be utilized as a state witness is to ask the
conjugal relation. (Ordono v. Daquigan, 62 SCRA 270 [1975]). Privilege Communication; Prosecutor to recommend that the accused be made a state witness. Remedial Law Bar
Marital Privilege (2006) Leticia was estranged from her husband Paul for more than a Examination Q & A (1997-2006) It is the Prosecutor who must recommend and move for
year due to his suspicion that she was having an affair with Manuel their neighbor. She the acceptance of the accused as a state witness. The accused may also apply under the
was temporarily living with her sister in Pasig City. For unknown reasons, the house of Witness Protection Program. SPECIAL PROCEEDINGS Cancellation or Correction; Entries
Leticia's sister was burned, killing the latter. Leticia survived. She saw her husband in the Civil Registry (2005) Helen is the daughter of Eliza, a Filipina, and Tony, a Chinese, who is
vicinity during the incident. Later he was charged with arson in an Information filed with married to another woman living in China. Her birth certificate indicates that Helen is the
the Regional Trial Court, Pasig City. During the trial, the prosecutor called Leticia to the legitimate child of Tony and Eliza and that she is a Chinese citizen. Helen wants her birth
witness stand and offered her testimony to prove that her husband committed arson. certificate corrected by changing her filiation from "legitimate" to "illegitimate" and her
Can Leticia testify over the objection of her husband on the ground of marital privilege? citizenship from "Chinese" to "Filipino" because her parents were not married. What
(5%) ALTERNATIVE ANSWER: No, Leticia cannot testify over the objection of her husband, petition should Helen file and what procedural requirements must be observed? Explain.
not under marital privilege which is inapplicable and which can be waived, but she would (5%) SUGGESTED ANSWER: A petition to change the record of birth by changing the
be barred under Sec. 22 of Rule 130, which prohibits her from testifying and which filiation from "legitimate" to "illegitimate" and petitioner's citizenship from "Chinese" to
cannot be waived (Alvarez v. Ramirez, G.R. No. 143439, October 14, 2005). ALTERNATIVE "Filipino" because her parents were not married, does not involve a simple summary
ANSWER: Yes, Leticia may testify over the objection of her husband. The disqualification correction, which could otherwise be done under the authority of R.A. No. 9048. A
of a witness by reason of marriage under Sec. 22, Rule 130 of the Revised Rules of Court petition has to be filed in a proceeding under Rule 108 of the Rules of Court, which has
has its exceptions as where the marital relations are so strained that there is no more now been interpreted to be adversarial in nature. (Republic v. Valencia, G.R. No. L-32181,
harmony to be preserved. The acts of Paul eradicate all major aspects of marital life. On March 5, 1986) Procedural requirements include: (a) filing a verified petition; (b) naming
the other hand, the State has an interest in punishing the guilty and Remedial Law Bar as parties all persons who have or claim any interest which would be affected; (c)
Examination Q & A (1997-2006) exonerating the innocent, and must have the right to issuance of an order fixing the time and place of hearing; (d) giving reasonable notice to
offer the testimony of Leticia over the objection of her husband (Alvarez v. Ramirez, G.R. the parties named in the petition; and (e) publication of the order once a week for three
No. 143439, October 14, 2005). Remedy; Lost Documents; Secondary Evidence (1992) consecutive seeks in a newspaper of general circulation. (Rule 108, Rules of Court)
Ajax Power Corporation, a utility company, sued in the RTC to enforce a supposed right Escheat Proceedings (2002) Suppose the property of D was declared escheated on July 1,
of way over a property owned by Simplicio. At the ensuing trial, Ajax presented its 1990 in escheat proceedings brought by the Solicitor General. Now, X, who claims to be
retired field auditor who testified that he know for a fact that a certain sum of money an heir of D, filed an action to recover the escheated property. Is the action viable? Why?
was periodically paid to Simplicio for some time as consideration for a right of way (2%) SUGGESTED ANSWER: No, the action is not viable. The action to recover escheated
pursuant to a written contract. The original contract was not presented. Instead, a property must be filed within five years from July 1, 1990 or be forever barred. (Rule 91,
purported copy, identified by the retired field auditor as such, was formally offered as sec. 4). Extra-judicial Settlement of Estate (2005) Nestor died intestate in 2003, leaving
part of his testimony. Rejected by the trial court, it was finally made the subject of an no debts. How may his estate be settled by his heirs who are of legal age and have legal
offer of proof by Ajax. Can Ajax validly claim that it had sufficiently met its burden of capacity? Explain. (2%) SUGGESTED ANSWER: If the decedent left no will and no debts,
proving the existence of the contract establishing its right of way? Explain, SUGGESTED and the heirs are all of age, the parties may, without securing letters of administration,
ANSWER: No. Ajax had not sufficiently met the burden of proving the existence of the divide the estate among themselves by means of a public instrument or by by:
written contract because. It had not laid the basis for the admission of a purported copy sirdondee@gmail.com Page 61 of 66 stipulation in a pending action for partition and
thereof as secondary evidence. Ajax should have first proven the execution of the shall file a bond with the register of deeds in an amount equivalent to the value of the
original document and its loss or destruction. (Sec. 5 of Rule 130) Testimony; personal property involved as certified to under oath by the parties concerned. The fact
Independent Relevant Statement (1999) A overheard B call X a thief. In an action for of extra-judicial settlement shall be published in a newspaper of general circulation once
defamation filed by X against B, is the testimony of A offered to prove the fact of a week for three consecutive weeks in the province. (Sec. 1, Rule 74, Rules of Court)
utterance i.e., that B called X a thief, admissible in evidence? Explain. (2%) SUGGESTED Habeas Corpus (1993) Roxanne, a widow, filed a petition for habeas corpus with the
ANSWER: Yes. The testimony of A who overheard B call X a thief is admissible in evidence Court of Appeals against Major Amor who is allegedly detaining her 18-year old son Bong
as an independently relevant statement. It is offered in evidence only to prove the tenor without authority of the law. After Major Amor had a filed a return alleging the cause of
thereof, not to prove the truth of the facts asserted therein. Independently relevant detention of Bong, the Court of Appeals promulgated a resolution remanding the case to
statements include statements which are on the very facts in issue or those which are the RTC for a full-blown trial due to the conflicting facts presented by the parties in their
circumstantial evidence thereof. The hearsay rule does not apply. (See People vs. Gaddi, pleadings. In directing the remand, the court of Appeals relied on Sec.9(1), in relation to
170 SCRA 649) Witness; Competency of the Witness vs. Credibility of the Witness (2004) Sec. 21 of BP 129 conferring upon said Court the authority to try and decide habeas
corpus cases concurrently with the RTCs. Did the Court of Appeals act correctly in allowed by the Court. No appeal was taken from its allowance. Thereafter, Y, who was
remanding the petition to the RTC? Why? SUGGESTED ANSWER: No, because while the interested in the estate of A, discovered that the Will was not genuine because A's
CA has original jurisdiction over habeas corpus concurrent with the RTCs, it has no signature was forged by X. A criminal action for forgery was instituted against X. May the
authority for remanding to the latter original actions filed with the former. On the due execution of the Will be validly questioned in such criminal action? (2%) SUGGESTED
contrary, the CA is specifically given the power to receive evidence and perform any and ANSWER: a. In order that a lost or destroyed will may be allowed, the following must be
all acts necessary to resolve factual issues raised in cases falling within its original complied with: the execution and validity of the same should be established; the will
jurisdiction. ALTERNATIVE ANSWER: Yes, because there is no prohibition in the law must have been in existence at the time of the death of the testator, or shown to have
against a superior court referring a case to a lower court having concurrent jurisdiction. been fraudulently or accidentally destroyed in the lifetime of the testator without his
The Supreme Court has referred to the CA or the RTC cases falling within their knowledge; and its provisions are clearly and distinctly proved by at least two credible
concurrent jurisdiction. Habeas Corpus (1998) A was arrested on the strength of a witnesses. (Sec. 6, Rule 76 of the Rules of Court) SUGGESTED ANSWER: b. No. The
warrant of arrest issued by the RTC in connection with an Information for Homicide. W, allowance of the will from which no appeal was taken is conclusive as to its due
the live-in partner of A filed a petition for habeas corpus against A's jailer and police execution. (Sec. 1 of Rule 75.) Due execution includes a finding that the will is genuine
investigators with the Court of Appeals. 1. Does W have the personality to file the and not a forgery. Accordingly, the due execution of the will cannot again be questioned
petition for habeas corpus? [2%] 2. Is the petition tenable? [3%] SUGGESTED ANSWER: 1. in a subsequent proceeding, not even in a criminal action for forgery of the will. Probate
Yes. W, the live-in partner of A, has the personality to file the petition for habeas corpus of Will (2003) A, a resident of Malolos, Bulacan, died leaving an estate located in Manila,
because it may be filed by "some person in his behalf." (Sec. 3. Rule 102. Rules of Court.) worth P200,000.00. In what court, taking into consideration the nature of jurisdiction
Remedial Law Bar Examination Q & A (1997-2006) 2. No. The petition is not tenable and of venue, should the probate proceeding on the estate of A be instituted? (4%)
because the warrant of arrest was issued by a court which had Jurisdiction to issue it SUGGESTED ANSWER: The probate proceeding on the estate of A should be instituted in
(Sec. 4, Rule 102 Rules of Court) Habeas Corpus (2003) Widow A and her two children, the Municipal Trial Court of Malolos, Bulacan which has jurisdiction, because the estate
both girls, aged 8 and 12 years old, reside in Angeles City, Pampanga. A leaves her two is valued at P200,000.00, and is the court of proper venue because A was a resident of
daughters in their house at night because she works in a brothel as a prostitute. Realizing Malolos at the time of his death. (Sec. 33 of BP 129 as amended by RA 7691; Sec. 1 of
the danger to the morals of these two girls, B, the father of the deceased husband of A, Rule 73). Probate of Will (2005) After Lulu's death, her heirs brought her last will to a
files a petition for habeas corpus against A for the custody of the girls in the Family Court lawyer to obtain their respective shares in the estate. The lawyer prepared a deed of
in Angeles City. In said petition, B alleges that he is entitled to the custody of the two partition distributing Lulu's estate in accordance with the terms of her will. Is the act of
girls because their mother is living a disgraceful life. The court issues the writ of habeas the lawyer correct? Why? (2%) SUGGESTED ANSWER: No. No will, shall pass either real or
corpus. When A learns of the petition and the writ, she brings her two children to Cebu personal estate unless it is proved and allowed in the proper court. (Sec. 1, Rule 75, Rules
City. At the expense of B the sheriff of the said Family Court goes to Cebu City and serves of Court) Probate of Will (2006) Sergio Punzalan, Filipino, 50 years old, married, and
the writ on A. A files her comment on the petition raising the following defenses: a) The residing at Ayala Alabang Village, Muntinlupa City, of by: sirdondee@gmail.com Page 63
enforcement of the writ of habeas corpus in Cebu City is illegal; and b) B has no of 66 sound and disposing mind, executed a last will and testament in English, a language
personality to institute the petition. 6% Resolve the petition in the light of the above spoken and written by him proficiently. He disposed of his estate consisting of a parcel of
defenses of A. (6%) SUGGESTED ANSWER: (a) The writ of habeas corpus issued by the land in Makati City and cash deposit at the City Bank in the sum of P 300 Million. He
Family Court in Angeles City may not be legally enforced in Cebu City, because the writ is bequeathed P 50 Million each to his 3 sons and P 150 Million to his wife. He devised a
enforceable only within the judicial region to which the Family Court belongs, unlike the piece of land worth P100 Million to Susan, his favorite daughter-inlaw. He named his
writ granted by the Supreme Court or Court of Appeals which is enforceable anywhere in best friend, Cancio Vidal, as executor of the will without bond. Is Cancio Vidal, after
the Philippines. (Sec. 20 of Rule on Custody of Minors and Writ of Habeas Corpus in learning of Sergio's death, obliged to file with the proper court a petition of probate of
Relation to Custody of Minors. (A.M. No. 03-04-04-SC; see also Sec. 4 of Rule 102, Rules the latter's last will and testament? (2%) SUGGESTED ANSWER: Cancio Vidal is obliged to
of Court.) (b) B, the father of the deceased husband of A, has the personality to institute file a petition for probate and for accepting or refusing the trust within the statutory
the petition for habeas corpus of the two minor girls, because the grandparent has the period of 20 days under Sec. 3, Rule 75, Rules of Court. Supposing the original copy of the
right of custody as against the mother A who is a prostitute. (Sectioins 2 and 13, Id.) last will and testament was lost, can Cancio compel Susan to produce a copy in her
Intestate Proceedings (2002) X filed a claim in the intestate proceedings of D. D’s possession to be submitted to the probate court. (2%) SUGGESTED ANSWER: Yes, Cancio
administrator denied liability and filed a counterclaim against X. X’s claim was can compel Susan to produce the copy in her possession. A person having custody of the
disallowed. (1) Does the probate court still have jurisdiction to allow the claim of D’s will is bound to deliver the same to the court of competent jurisdiction or to the
administrator by way of offset? Why? (2%) (2) Suppose D’s administrator did not allege executor, as provided in Sec. 2, Rule 75, Rules of Court. Can the probate court appoint
any claim against X by way of offset, can D’s administrator prosecute the claim in an the widow as executor of the will? (2%) SUGGESTED ANSWER: Yes, the probate court can
independent proceeding/ why/ (3%) SUGGESTED ANSWER: by: sirdondee@gmail.com appoint the widow as executor of the will if the executor does not qualify, as when he is
Page 62 of 66 (1) No, because since the claim of X was disallowed, there is no amount incompetent, refuses the trust, or fails to give bond (Sec. 6, Rule 78, Rules of Court). Can
against which to offset the claim of D’s administrator. (2) Yes, D’s administrator can the widow and her children settle extrajudicially among themselves the estate of the
prosecute the claim in an independent proceeding since the claim of X was disallowed. If deceased? (2%) SUGGESTED ANSWER: No, the widow and her children cannot settle the
X had a valid claim and D’s administrator did not allege any claim against X by way of estate extrajudicially because of the existence of the Will. No will shall pass either real or
offset, his failure to do so would bar his claim forever. (Rule 86, sec. 10). Intestate personal estate unless it is proved and allowed in the proper court (Sec. 1, Rule 75, Rules
Proceedings; Debts of the Estate (2002) A, B and C, the only heirs in D’s intestate of Court). Can the widow and her children initiate a separate petition for partition of the
proceedings, submitted a project of partition to the partition, two lots were assigned to estate pending the probate of the last will and testament by the court? (2%) SUGGESTED
C, who immediately entered into the possession of the lots. Thereafter, C died and ANSWER: No, the widow and her children cannot file a separate petition for partition
proceedings for the settlement of his estate were filed in the RTCQuezon City. D’s pending the probate of the will. Partition is a mode of settlement of the estate (Sec. 1,
administrator then filed a motion in the probate court (RTC-Manila), praying that one of Rule 75, Rules of Court). Probate of Will; Mandatory Nature (2002) Remedial Law Bar
the lots assigned to C in the project of partition be turned over to him to satisfy debts Examination Q & A (1997-2006) What should the court do if, in the course of intestate
corresponding to C’s portion. The motion was opposed by the administrator of C’s proceedings, a will is found and it is submitted for probate? Explain. (2%) SUGGESTED
estate. How should the RTC-Manila resolve the motion of D’s administrator? Explain. ANSWER: If a will is found in the course of intestate proceedings and it is submitted for
(3%) SUGGESTED ANSWER: The motion of D’s administrator should be granted. The probate, the intestate proceedings will be suspended until the will is probated. Upon the
assignment of the two lots to C was premature because the debts of the estate had not probate of the will, the intestate proceedings will be terminated. (Rule 82, sec. 1).
been fully paid. [Rule 90, sec. 1; Reyes v. Barreto-Datu, 19 SCRA 85 (1967)]. Judicial Settlement of Estate (2001) The rules on special proceedings ordinarily require that the
Settlement of Estate (2005) State the rule on venue in judicial settlement of estate of estate of the deceased should be judicially administered thru an administrator or
deceased persons. (2%) SUGGESTED ANSWER: If the decedent is an inhabitant of the executor. What are the two exceptions to said requirements? (5%) SUGGESTED ANSWER:
Philippines at the time of' his death, whether a citizen or an alien, the venue shall be in The two exceptions to the requirement are: (a) Where the decedent left no will and no
the RTC in the province in which he resides at the time of his death, not in the place debts and the heirs are all of age, or the minors are represented by their judicial or legal
where he used to live. (Jao v. Court of Appeals, G.R. No. 128314, May 29, 2002) If he is an representatives duly authorized for the purpose, the parties may without securing letters
inhabitant, of a foreign country, the RTC of any province or city in which he had estate of administration, divide the estate among themselves by means of public instrument
shall be the venue. The court first taking cognizance of the case shall exercise jurisdiction filed in the office of the register of deeds, or should they disagree, they may do so in an
to the exclusion of all other courts. When the marriage is dissolved by the death of the ordinary action of partition. If there is only one heir, he may adjudicate to himself the
husband or wife, the community property shall be inventoried, administered and entire estate by means of an affidavit filed in the office of the register of deeds. The
liquidated, and the debts thereof paid, in the testate or intestate proceedings of the parties or the sole heir shall file simultaneously abound with the register of deeds, in an
deceased spouse. If both spouses have died, the conjugal partnership shall be liquidated amount equivalent to the value of the personal property as certified to under oath by the
in the testate or intestate proceedings of either. (Sees. 1 and 2, Rule 73, Rules of Court) parties and conditioned upon the payment of any just claim that may be filed later. The
Probate of Lost Wills (1999) What are the requisites in order that a lost or destroyed Will fact of the extrajudicial settlement or administration shall be published in a newspaper
may be allowed? (2%) Remedial Law Bar Examination Q & A (1997-2006) A's Will was of general circulation in the province once a week for three consecutive weeks. (Sec. 1 of
Rule 74, Rules of Court) (b) Whenever the gross value of the estate of a deceased person, conduct a preliminary investigation of the criminal case; (b) the order for his preventive
whether he died testate or intestate, does not exceed ten thousand pesos, and that fact suspension was premature because he had yet to file his answer to the administrative
is made to appear to the RTC having jurisdiction or the estate by the petition of an complaint and submit countervailing evidence; and (c) he was a career executive service
interested person and upon hearing, which shall be held not less than one (1) month nor officer and under Presidential Decree No. 807 (Civil Service Law), his preventive
more than three (3) months from the date of the last publication of a notice which shall suspension shall be for a maximum period of three months. Resolve with reasons the
be published once a week for three consecutive weeks in a newspaper of general motion of respondent AG. (5%) SUGGESTED ANSWER: The motion should be denied for
circulation in the province, and after such other notice to interested persons as the court the following reasons: The Office of the Special Prosecutor does not have exclusive
may direct, the court may proceed summarily, without the appointment of an executor authority to conduct a preliminary investigation of the criminal case but it participated in
or administrator, to settle the estate. (Sec. 2 of Rule 74, Rules of Court) Settlement of the investigation together with the Deputy Ombudsman for the Military who can handle
Estate; Administrator (1998) A, claiming to be an illegitimate child of the deceased D, cases of civilians and is not limited to the military. The order of preventive suspension
instituted an Intestate proceeding to settle the estate of the latter. He also prayed that need not wait for the answer to the administrative complaint and the submission of
he be by: sirdondee@gmail.com Page 64 of 66 appointed administrator of said estate. S, countervailing evidence. (Garcia v. Mojica, G.R. No. 13903, September 10, 1999) In
the surviving spouse, opposed the petition and A's application to be appointed the Vasquez case, G.R. No. 110801, April 6, 1995, the court ruled that preventive suspension
administrator on the ground that he was not the child of her deceased husband D. The pursuant to Sec. 24 of R.A. No. 6770 (Ombudsman Act of 1989), shall continue until
court, however, appointed A as the administrator of said estate. Subsequently, S, termination of the case but shall not exceed six (6) months, except in relation to R.A. No,
claiming to be the sole heir of D, executed an Affidavit of Adjudication, adjudicating unto 3019 and P.D. No. 807. As a career executive officer, his preventive suspension under the
herself the entire estate of her deceased husband D. S then sold the entire estate to X. Civil Service Law may only be for a maximum period of three months. The period of the
Was the appointment of A as administrator proper? [2%] Was the action of S in suspension under the Anti-Graft Law shall be the same pursuant to the equal protection
adjudicating the entire estate of her late husband to herself legal? [3%] SUGGESTED clause. (Garcia v. Mojica, G.R. No. 13903, September 10, 1999; Layno v. Sandiganbayan,
ANSWER: 1. Yes, unless it is shown that the court gravely-abused its discretion in G.R. No. L-65848, May 21, 1985) Congress; Law Expropriating Property (2006) May
appointing the illegitimate child as administrator, instead of the spouse. While the Congress enact a law providing that a 5, 000 square meter lot, a part of the UST
spouse enjoys preference, it appears that the spouse has neglected to apply for letters of compound in Sampaloc Manila, be expropriated for the construction of a park in honor
administration within thirty (30) days from the death of the decedent. (Sec. 6, Rule 78, of former City Mayor Arsenic Lacson? As compensation to UST, the City Version 1997-
Rules of Court; Gaspay, Jr. vs. Court of Appeals. 238 SCRA 163.) ALTERNATIVE ANSWER: 2006 Updated by Dondee Remedial Law Bar Examination Q & A (1997-2006) by:
S, the surviving spouse, should have been appointed administratrix of the estate, in as sirdondee@gmail.com Page 66 of 66 of Manila shall deliver its 5-hectare lot in Sta. Rosa,
much as she enjoys first preference in such appointment under the rules. (Sec. 6(a) of Laguna originally intended as a residential subdivision for the Manila City Hall employees.
Rule 78, Rules of Court.) SUGGESTED ANSWER: 2. No. An affidavit of self-adjudication is Explain. (5%) SUGGESTED ANSWER: Yes, Congress may enact a law expropriating
allowed only if the affiant is the sole heir of the. deceased. (Sec. 1, Rule 74, Rules of property provided that it is for public use and with just compensation. In this case, the
Court). In this case, A also claims to be an heir. Moreover, it is not legal because there is construction of a park is for public use (See Sena v. Manila Railroad Co., G.R. No. 15915,
already a pending juridical proceeding for the settlement of the estate. Venue; Special September 7, 1921; Reyes v. NHA, GR No. 147511, March 24, 2003). The planned
Proceedings (1997) Give the proper venue for the following special proceedings: a) A compensation, however, is not legally tenable as the determination of just compensation
petition to declare as escheated a parcel of land owned by a resident of the Philippines is a judicial function. No statute, RA 3019; Mandatory Suspension (2001) decree or
who died intestate and without heirs or persons entitled to the property. b) A petition executive order can mandate that the determination of just compensation by the
for the appointment of an administrator over the land and building left by an American executive or legislative departments can prevail over the court's findings (Export
citizen residing in California, who had been declared an incompetent by an American Processing Zone Authority v. Dulay, G.R. No. L-59603, April 29,1987; Sees. 5 to 8 Rule
court. c) A petition for the adoption of a minor residing in Pampanga. SUGGESTED 67,1997 Rules of Civil Procedure). In addition, compensation must be paid in money
ANSWER: (a) The venue of the escheat proceedings of a parcel of land in this case is the (Esteban v. Onorio, A.M. No. 00-4-166-RTC, June 29, 2001). . Governor Pedro Mario of
place where the deceased last resided. (Sec. 1. Rule 91, Rules of Court). (b) The venue for Tarlac was charged with indirect bribery before the Sandiganbayan for accepting a car in
the appointment of an administrator over land and building of an American citizen exchange of the award of a series of contracts for medical supplies. The Sandiganbayan,
residing in California, declared Incompetent Remedial Law Bar Examination Q & A (1997- after going over the information, found the same to be valid and ordered the suspension
2006) by an American Court, is the RTC of the place where his property or part thereof is of Mario. The latter contested the suspension claiming that under the law (Sec. 13 of R.A.
situated. (Sec. 1. Rule 92). (c) The venue of a petition for the adoption of a minor residing 3019) his suspension is not automatic upon the filing of the information and his
in Pampanga is the RTC of the place in which the petitioner resides. (Sec. 1. Rule 99) suspension under Sec. 13, R.A. 3019 is in conflict with Sec. 5 of the Decentralization Act
SUMMARY PROCEDURE Prohibited Pleadings (2004) Charged with the offense of slight of 1967 (R.A. 5185). The Sandilganbayan overruled Mario’s contention stating that
physical injuries under an information duly filed with the MeTC in Manila which in the Mario’s suspension under the circumstances is mandatory. Is the court’s ruling correct?
meantime had duly issued an order declaring that the case shall be governed by the Why? SUGGESTED ANSWER: Yes. Mario’s suspension is mandatory, although not
Revised Rule on Summary Procedure, the accused filed with said court a motion to quash automatic, (Sec. 13 of R.A. No. 3019 in relation to Sec. 5 of the Decentralization Act of
on the sole ground that the officer who filed the information had no authority to do so. 1967 (R.A. No. 5185). It is mandatory after the determination of the validity of the
The MeTC denied the motion on the ground that it is a prohibited motion under the said information in a pre-suspension hearing. [Segovia v. Sandiganbayan, 288 SCRA 328
Rule. The accused thereupon filed with the RTC in Manila a petition for certiorari in sum (1988)]. The purpose of suspension is to prevent the accused public officer from
assailing and seeking the nullification of the MeTC's denial of his motion to quash. The frustrating or hampering his prosecution by intimidating or influencing witnesses or
RTC in due time issued an order denying due course to the certiorari petition on the tampering with evidence or from committing further acts of malfeasance while in office.
ground that it is not allowed by the said Rule. The accused forthwith filed with said RTC a
motion for reconsideration of its said order. The RTC in time denied said motion for
reconsideration on the ground that the same is also a prohibited motion under the said Remedial Law Q&As (2007-2013) hectorchristopher@yahoo.com JayArhSals “Never Let
Rule. Were the RTC's orders denying due course to the petition as well as denying the The Odds Keep You From Pursuing What You Know In Your Heart You Were Meant To
motion for reconsideration correct? Reason. (5%) SUGGESTED ANSWER: The RTC's orders Do.”-Leroy Satchel Paige Page 1 of 198 A Compilation of the Questions and Suggested
denying due course to the petition for certiorari as well as denying the motion for Answers In the PHILIPPINE BAR EXAMINATIONS 2007-2013 In REMEDIAL LAW Compiled
reconsideration are both not correct. The petition for certiorari is a prohibited pleading and Arranged By: Salise, Hector Christopher “Jay-Arh” Jr. M. (University of San Jose-
under Section 19(g) of the Revised Rule on Summary Procedure and the motion for Recoletos School of Law) ANSWERS TO BAR EXAMINATION QUESTIONS by the UP LAW
reconsideration, while it is not prohibited motion (Lucas v. Fabros, AM No. MTJ-99-1226, COMPLEX (2007, 2009, 2010) & PHILIPPINE ASSOCIATION OF LAW SCHOOLS (2008)
January 31, 2000, citing Joven v. Court of Appeals, 212 SCRA 700, 707-708 (1992), should Remedial Law Q&As (2007-2013) hectorchristopher@yahoo.com JayArhSals “Never Let
be denied because the petition for certiorari is a prohibited pleading. MISCELLANEOUS The Odds Keep You From Pursuing What You Know In Your Heart You Were Meant To
Administrative Proceedings (2005) Regional Director AG of the Department of Public Do.”-Leroy Satchel Paige Page 2 of 198 FOREWORD This work is a compilation of the
Works and Highways was charged with violation of Section 3(e) of Republic Act No. 3019 ANSWERS TO BAR EXAMINATION QUESTIONS by the UP LAW COMPLEX , Philippine
in the Office of the Ombudsman. An administrative charge for gross misconduct arising Association of Law Schools from 2007-2010 and local law students and lawyers’ forum
from the transaction subject matter of said criminal case was filed against him in the sites from 2011-2013 and not an original creation or formulation of the author. The
same office. The Ombudsman assigned a team by: sirdondee@gmail.com Page 65 of 66 author was inspired by the work of Silliman University’s College of Law and its students
composed of investigators from the Office of the Special Prosecutor and from the Office of producing a very good material to everyone involved in the legal field particularly the
of the Deputy Ombudsman for the Military to conduct a joint investigation of the students and the reviewees for free. Hence, this work is a freeware. Everyone is free to
criminal case and the administrative case. The team of investigators recommended to distribute and mass produce copies of this work, however, the author accepts no liability
the Ombudsman that AG be preventively suspended for a period not exceeding six for the content of this reviewer, or for the consequences of the usage, abuse, or any
months on its finding that the evidence of guilt is strong. The Ombudsman issued the actions taken by the user on the basis of the information given. The answers (views or
said order as recommended by the investigators. AG moved to reconsider the order on opinions) presented in this reviewer are solely those of the authors in the given
the following grounds: (a) the Office of the Special Prosecutor had exclusive authority to references and do not necessarily represent those of the author of this work. The Author
Remedial Law Q&As (2007-2013) hectorchristopher@yahoo.com JayArhSals “Never Let (2009)…………..52 Cancellation or Correction; Notice
The Odds Keep You From Pursuing What You Know In Your Heart You Were Meant To (2007)……………………………………………………..………54 Habeas Corpus
Do.”-Leroy Satchel Paige Page 3 of 198 TABLE OF CONTENTS (Titles are based on (2007)…………………………………………………………………………………………54 Habeas Corpus; Bail
Silliman‟s Compilation [Arranged by Topic]) General Principles Rights of the Accused; (2008)……………………………………………………………………………….…55 Habeas Corpus;
Miranda Rights (2010)………………………………………………………..10 Jurisdiction Error of Jurisdiction; Sandiganbayan (2009)……………………………………………….56 Letters of
Jurisdiction vs. Error of Judgment (2012)…………………………………………………10 Jurisdiction; Administration; Interested Person (2008)……………………………………………….56 Probate of
Over the Plaintiff, Subject Matter (2009)…………………………………………….10 Jurisdiction; RTC Will (2010)…………………………………………………………………………………………56 Probate of Will
(2009)…………………………………………………………………………………….11 Jurisdiction; RTC; (2007)…………………………………………………………………………………………57 Probate of Will;
Counterclaim (2008)………………………………………………………………..12 Jurisdiction; RTC; MeTC Jurisdictional Facts (2012) ……………………………………………………..……57 Probate of Will;
(2010)…………………………………………………………………………..13 Katarungang Pambarangay; Application of Modes of Discovery (2008)……………………………………..…58 Probate of Will:
Parties (2009)…………………………………………………………….14 Civil Procedure (Rules 1-56) Will Outside of the Philippines (2010)…………………………………………….58 Settlement of
Actions; Cause of Action (2013)……………………………………………………………………………15 Estate (2010)…………………………………………………………………………………59 Settlement of
Actions; Specific Performance (2012)……………………………………………………………………16 Estate (2009)…………………………………………………………………………………59 Criminal Procedure
Appeals; Abandonment of a Perfected Appeal (2009)……………………………………………….17 (Rules 110-127) Actions; Commencement of an Action
Appeals; Modes of Appeal (2012)………………………………………………………………………….17 (2012)…………………………………………………………60 Remedial Law Q&As (2007-2013)
Appeals; Modes of Appeal (2009)………………………………………………………………………….18 hectorchristopher@yahoo.com JayArhSals “Never Let The Odds Keep You From Pursuing
Appeals; Modes of Appeal; RTC, CA (2009)…………………………………………………………….19 What You Know In Your Heart You Were Meant To Do.”-Leroy Satchel Paige Page 7 of
Appeals; Second Notice of Appeal (2008)………………………………………………………….……20 198 Actions; Commencement of an Action; Criminal, Civil
Remedial Law Q&As (2007-2013) hectorchristopher@yahoo.com JayArhSals “Never Let (2013)…………………………………..60 Actions; Commencement of an Action; Party
The Odds Keep You From Pursuing What You Know In Your Heart You Were Meant To (2013)…..............………………………………..62 Actions; Complaint; Forum Shopping
Do.”-Leroy Satchel Paige Page 4 of 198 Certiorari; Rule 45 vs. Rule 65 (2010)………………………………………………………….64 Actions; Hold Departure Order
(2008)…………………………………………………………………..20 Demurrer to Evidence (2010)…………………………………………………………………..66 Arrest; Warrantless Arrests &
(2009)…………………………………………………………………………….…21 Demurrer to Evidence; Civil Searches (2007)………………………………….……………………67 Bail; Application
Case vs. Criminal Case (2007)………………………………………22 Discovery; Modes of Discovery; (2012)………………………………………………………………………………………67 Discovery; Production
Refusal to Comply (2010)………………………………………..22 Discovery; Production and and Inspection (2009)…………………………………………………………68 Information; Motion to
Inspection (2009)…………………………………………………………23 Forum Shopping; Certification Quash (2009)……………………………………………………………………69 Information; Motion to
(2009)…………………………………………………………………..23 Judgment; Enforcement; 5yr Period Quash (2009)……………………………………………………………………70 Jurisdiction; Complex Crimes
(2007)……………………………………………………………24 Judgment; Execution; Judgment (2013)……………………………………………………………………70 Jurisdiction; Reinvestigation; Arrest
Obligor‟s Death (2009)………………………………………….24 Judgment; Execution; Stay (2008)……………………………………………………..……71 Res Judicata In Prison Grey
(2009)……………………………………………………………………….25 Judgment; Execution; Successors (2010)………………………………………………………………………72 Search & Seizure; Plain View
in Interest (2008)……………………………………………….26 Judgment; Foreign Judgments; (2008)…………………………………………………………………..…72 Search & Seizure; Warrantless
Foreign Arbitral Award (2007)………………………………….27 Judgment; Judgment on the Search (2010)…………………………………………………………72 Search Warrant; Application;
Pleadings (2012)………………………………………………………..28 Judgment; Judgment on the Venue (2012)……………………………………………………………73 Trial; Remedies
Pleadings (2009)………………………………………………………..28 Petition for Relief (2013)………………………………………………………………………………………..76 Trial; Reverse Trial
(2007)…………………………………………………………………………………….30 Petition for Relief; (2007)………………………………………………………………………………..…80 Trial; Speedy Trial
Injunction (2009)…………………………………………………………………….30 Pleadings; Amendment (2007)……………………………………………………………………………………81 Trial; Trial in Absentia
of Complaint (2009)…………………………………………………………..30 Pleadings; Amendment of (2010)………………………………………………………………………………82 Evidence (Rules 128-134)
Complaint (2008)…………………………………………………………..31 Pleadings; Counterclaim Admissibility; Admission of Guilt (2008)……………………………………………………………….82
(2010)……………………………………………………………………………33 Pleadings; Counterclaim Admissibility; Death of Adverse Party (2007)…………………………………………………………83
(2007)……………………………………………………………………………34 Pleadings; Motion Remedial Law Q&As (2007-2013) hectorchristopher@yahoo.com JayArhSals “Never Let
(2007)…………………………………………………………………………………….34 Pleadings; Motions; Bill The Odds Keep You From Pursuing What You Know In Your Heart You Were Meant To
of Particulars (2008)......……………………………………………………34 Pleadings; Motions; Do.”-Leroy Satchel Paige Page 8 of 198 Admissibility; DNA Evidence
Omnibus Motion Rule (2010)……………………………………………………35 Remedial Law Q&As (2010)……………………………………………………………..………83 Admissibility; DNA Evidence
(2007-2013) hectorchristopher@yahoo.com JayArhSals “Never Let The Odds Keep You (2009)……………………………………………………………………..84 Admissibility; Evidence from
From Pursuing What You Know In Your Heart You Were Meant To Do.”-Leroy Satchel Invasive and Involuntary Procedures (2010)…………………84 Admissibility; Offer to Settle;
Paige Page 5 of 198 Subpoena; Viatory Right of Witness Implied Admission of Guilt (2008)………………………………85 Best Evidence Rule; Electronic
(2009)…………………………………………………………….37 Summons; By Publication Evidence (2009)…………………………………………………….85 Chain of Custody
(2008)...……………………………………………………………………….37 Summons; Served by Email (2012)……………………………………………………………………………………..86 Character Evidence;
(2009)........………………………………………………………………..37 Summons; Valid Service Bad Reputation (2010)………………………………………………………..…87 Doctrine of Adoptive
(2013)……………………………………………………………………………38 Trial; Court of Appeals as Trial Admission (2009)…………………………………………………………………87 Hearsay Evidence;
Court (2008)………………………………………………………….40 Venue; Real Actions Objection (2012)……………………………………………………………………..87 Hearsay Rule
(2012)………………………………………………………………………………….40 Venue; Real Actions (2007)……………………………………………………………………………………..……88 Hearsay;
(2008)………………………………………………………………………………….41 Provisional Remedies Inapplicable (2009)………………………………………………………………………………..88 Offer of
(Rules 57-61) Attachment; Bond Evidence; Failure to Offer (2007)………………………………………………………………89 Offer of
(2008)……………………………………………………………………………………42 Attachment; Evidence; Fruit of a Poisonous Tree (2010)…………………………………………………89 Offer of
Garnishment (2008)…………………………………………………………………………42 Attachment; Kinds Evidence; Fruit of a Poisonous Tree (2009)…………………………………………………90 Privilege
of Attachment (2012)……………………………………………………………..43 Attachment; Communication (2013)………………………………………………………………………….91 Privilege
Preliminary Attachment (2012)…………………………………………………………45 Injunction; Communication; Lawyer-Client (2008)…………………………………………………..…93 Privilege
Preliminary Injunction (2009)……………………………………………………………..45 Special Civil Communication; Lawyer-Client (2008)……………………………………………………..94 Privilege
Actions (Rules 62-71) Certiorari; Petition for Certiorari, Rule 65 Communication; Marital Privilege (2010)………………………………………………….94 Witness;
(2012)……………………………………………………45 Certiorari; Petition for Certiorari; Contempt Examination of Witness (2009)………………………………………………………………..96 Summary
(2012)………………………………………………..46 Expropriation; Motion to Dismiss Procedure Prohibited Pleadings
(2009)……………………………………………………………….47 Forcible Entry; Remedies (2010)…………………………………………………………………………………96 Miscellaneous Remedial
(2013)……………………………………………............................……48 Foreclosure; Certification Law Q&As (2007-2013) hectorchristopher@yahoo.com JayArhSals “Never Let The Odds
Against Non Forum Shopping (2007)………………………..………49 Jurisdiction; Unlawful Keep You From Pursuing What You Know In Your Heart You Were Meant To Do.”-Leroy
Detainer (2010)………………………………………………...................…50 Remedial Law Q&As Satchel Paige Page 9 of 198 Alternative Dispute Resolution; Court Diversion; Stages
(2007-2013) hectorchristopher@yahoo.com JayArhSals “Never Let The Odds Keep You (2012)………………………………..97 A.M. No. 09-6-8-SC; Precautionary Principle
From Pursuing What You Know In Your Heart You Were Meant To Do.”-Leroy Satchel (2012)…………………………………………………97 Habeas Data
Paige Page 6 of 198 Jurisdiction; Unlawful Detainer (2010)…………………………………………………………………………………………….98 Habeas Data
(2008)………………………………………………...................…50 Mandamus (2009)…………………………………………………………………………….………………98 R.A. 3019; Pre-
(2012)………………………………………………..................................................…51 Partition; Non- Suspension Hearing (2012)……………………………….……………………………99 R.A. 3019;
joinder (2009)………………………………………………….................................51 Unlawful Detainer; Remedies (2013)………………………………………………………………………..….…100 Small Claims
Preliminary Conference (2007)…………………………………………………51 Unlawful; Detainer; (2013)…………………………………………………………………………………………102 Writ of Amparo;
Prior Possession (2008)…………………………………………………...........52 Special Proceedings Habeas Corpus (2009)………………………………………………….................103 MULTIPLE CHOICE
(Rules 72-109) Absentee; Declaration of Absence vs. Declaration of Presumptive Death QUESTIONS 2013 Remedial Law Exam MCQ (October 27, 2013)
….……………………………………….….104 2012 Remedial Law Exam MCQ (October 28, 2012) suit for collection of P387,000 against Ramon in the RTC of Davao City. Aside from
….…………………………………………..121 2011 Remedial Law Exam MCQ (November 27, alleging payment as a defense, Ramon in his answer set up counterclaims for P100,000
2011).………………………………….………..169 Remedial Law Q&As (2007-2013) as damages and 30,000 as attorney’s fees as a result of the baseless filing of the
hectorchristopher@yahoo.com JayArhSals “Never Let The Odds Keep You From Pursuing complaint, as well as for P250,000 as the balance of the purchase price of the 30 units of
What You Know In Your Heart You Were Meant To Do.”-Leroy Satchel Paige Page 10 of air conditioners he sold to Fe. (a) Does the RTC have jurisdiction over Ramon’s
198 General Principles Rights of the Accused; Miranda Rights (2010) No.XI. X was counterclaim, and if so, does he have to pay docket fees therefor? SUGGESTED ANSWER:
arrested for the alleged murder of a 6-year old lad. He was read his Mirandarights Yes, applying the totality rule which sums up the total amount of claims of the parties,
immediately upon being apprehended. In the course of his detention, X was subjected to the RTC has jurisdiction over the counter claims. Unlike in the case of compulsory
three hours of non-stop interrogation. He remained quiet until, on the 3rd hour, he counterclaims, a defendant who raises a permissive counterclaim must first pay docket
answered "yes" to the question of whether "he prayed for forgiveness for shooting down fees before the court can validly acquire jurisdiction. One compelling test of Remedial
the boy." The trial court, interpreting X’s answer as an admission of guilt, convicted him. Law Q&As (2007-2013) hectorchristopher@yahoo.com JayArhSals “Never Let The Odds
On appeal, X’s counsel faulted the trial court in its interpretation of his client’s answer, Keep You From Pursuing What You Know In Your Heart You Were Meant To Do.”-Leroy
arguing that X invoked his Miranda rights when he remained quiet for the first two hours Satchel Paige Page 13 of 198 compulsoriness is the logical relation between the claim
of questioning. Rule on the assignment of error. (3%) SUGGESTED ANSWER: The alleged in the complaint and the counterclaim (Bayer Phil, Inc. vs. C.A., G.R. No. 109269,
assignment of error invoked by X‟s counsel is impressed with merit since there has been 15 September 2000). Ramon does not have to pay docket fees for his compulsory
no express waiver of X‟s Miranda Rights. In order to have a valid waiver of the Miranda counterclaims. Ramon is liable for docket fees only on his permissive counterclaim for
Rights, the same must be in writing and made in the presence of his counsel. The the balance of the purchase price of 30 units of air conditioners in the sum of P250,000,
uncounselled extrajudicial confession of X being without a valid waiver of his Miranda as it neither arises out of nor is it connected with the transaction or occurrence
Rights, is inadmissible, as well as any information derived therefrom. Jurisdiction Error of constituting Fe‟s claim (Sec. 19 [8] and 33 [1], B.P. 129; AO 04-94, implementing R.A.
Jurisdiction vs. Error of Judgment (2012) No.III.A. Distinguish error of jurisdiction from 7691, approved March 25, 1994, the jurisdictional; amount for MTC Davao being
error of judgment. (5%) SUGGESTED ANSWER: An error of judgment is one which the P300,000 at this time; Alday vs. FGU Insurance Corporation, G.R. No. 138822, 23 January
court may commit in the exercise of its jurisdiction. Such an error does not deprive the 2001). (b) Suppose Ramon’s counterclaim for the unpaid balance is P310,000, what will
court of jurisdiction and is correctible only by appeal; whereas an error of jurisdiction is happen to his counterclaims if the court dismisses the complaint after holding a
one which the court acts without or in excess of its jurisdiction. Such an error renders an preliminary hearing on Ramon’s affirmative defenses? SUGGESTED ANSWER: The
order or judgment void or voidable and is correctible by the special civil action of dismissal of the complaint shall be without prejudice to the prosecution in the same or
certiorari. (Dela Cruz vs. Moir, 36 Phil. 213; Cochingyan vs. Claribel, 76 SCRA 361; Fortich separate action of a counterclaim pleaded in the answer (Sec. 3, Rule 17; Pinga vs. Heirs
vs. Corona, April 24, 1998, 289 SCRA 624; Artistica Ceramica, Inc. vs. Ciudad Del Carmen of German Santiago, G.R. No. 170354, June 30, 2006). (c) Under the same premise as
Homeowner‟s Association, Inc., G.R. Nos. 167583-84, June 16, 2010). Jurisdiction; Over paragraph (b) above, suppose that instead of alleging payment as a defense in his
the Plaintiff, Subject Matter (2009) Remedial Law Q&As (2007-2013) answer, Ramon filed a motion to dismiss on that ground, at the same time setting up his
hectorchristopher@yahoo.com JayArhSals “Never Let The Odds Keep You From Pursuing counterclaims, and the court grants his motion. What will happen to his counterclaims?
What You Know In Your Heart You Were Meant To Do.”-Leroy Satchel Paige Page 11 of SUGGESTED ANSWER: His counterclaims can continue to be prosecuted or may be
198 No.III. Amorsolo, a Filipino citizen permanently residing in New York City, filed with pursued separately at his option (Sec. 6, Rule 16; Pinga vs. Heirs of German Santiago, G.R.
the RTC of Lipa City a complaint for Rescission of Contract of Sale of Land against Brigido, No. 170354, June 30, 2006). Jurisdiction; RTC; MeTC (2010) No.II. On August 13, 2008, A,
a resident of Barangay San Miguel, Sto. Tomas, Batangas. The subject property, located as shipper and consignee, loaded on the M/V Atlantis in Legaspi City 100,000 pieces of
in Barangay Talisay, Lipa City, has an assessed value of 19,700. Appended to the century eggs. The shipment arrived in Manila totally damaged on August 14, 2008. A filed
complaint is Amorsolo’s verification and certification of non-forum shopping executed in before the Metropolitan Trial Court (MeTC) of Manila a complaint against B Super Lines,
New York City, duly notarized by Mr. Joseph Brown, Esq., a notary public in the State of Inc. (B Lines), owner of the M/V Atlantis, for recovery of damages amounting to
New York. Brigod filed a motion to dismiss the complaint on the following grounds: (a) P167,899. He attached to the complaint the Bill of Lading. (a) B Lines filed a Motion to
The court cannot acquire jurisdiction over the person of Amorsolo because he is not a Dismiss upon the ground that the Regional Trial Court Remedial Law Q&As (2007-2013)
resident of the Philippines; (2%) SUGGESTED ANSWER: The first ground raised lacks merit hectorchristopher@yahoo.com JayArhSals “Never Let The Odds Keep You From Pursuing
because jurisdiction over the person of a plaintiff is acquired by the court upon the filing What You Know In Your Heart You Were Meant To Do.”-Leroy Satchel Paige Page 14 of
of plaintiff‟s complaint therewith. Residency or citizenship is not a requirement for filing 198 has exclusive original jurisdiction over "all actions in admiralty and maritime" claims.
a complaint, because plaintiff thereby submits to the jurisdiction of the court. (b) The In his Reply, A contended that while the action is indeed "admiralty and maritime" in
RTC does not have jurisdiction over the subject matter of the action involving real nature, it is the amount of the claim, not the nature of the action, that governs
property with an assessed value of P19,700.00; exclusive and original jurisdiction is with jurisdiction. Pass on the Motion to Dismiss. (3%) SUGGESTED ANSWER: The Motion to
the Municipal Trial Court where the defendant resides; (3%) and SUGGESTED ANSWER: Dismiss is without merit and therefore should be denied. Courts of the first level have
The second ground raised is also without merit because the subject of the litigation, jurisdiction over civil actions where the demand is for sum of money not exceeding
Rescission of Contract, is incapable of pecuniary estimation the exclusive original P300,000.00 or in Metro Manila, P400,000.00, exclusive of interest, damages, attorney‟s
jurisdiction to which is vested by law in the Regional Trial Courts. The nature of the fees, litigation expenses and costs: this jurisdiction includes admiralty and marine cases.
action renders the assessed value of the land involved irrelevant. Jurisdiction; RTC (2009) And where the main cause of action is the claim for damages, the amount thereof shall
No.II. Angelina sued Armando before the Regional Trial Court (RTC) of Manila to recover be considered in determining the jurisdiction of the court (Adm. Circular No. 09-94, June
the ownership and possession of two parcels of land; one situated in Pampanga, and the 14, 1994). (b) The MeTC denied the Motion in question A. B Lines thus filed an Answer
other in Bulacan. (a) May the action prosper? Explain. SUGGESTED ANSWER: No, the raising the defense that under the Bill of Lading it issued to A, its liability was limited to
action may not prosper, because under R.A. No. 7691, exclusive original jurisdiction in P10,000. At the pre-trial conference, B Lines defined as one of the issues whether the
civil actions which involve title to, or possession of real property or any interest therein is stipulation limiting its liability to P10,000 binds A. A countered that this was no longer in
determined on the basis of the assessed value of the land involved, whether it should be issue as B Lines had failed to deny under oath the Bill of Lading. Which of the parties is
P20,000 in the rest of the Remedial Law Q&As (2007-2013) correct? Explain. (3%) SUGGESTED ANSWER: The contention of B is correct: A‟s
hectorchristopher@yahoo.com JayArhSals “Never Let The Odds Keep You From Pursuing contention is wrong. It was A who pleaded the Bill of Lading as an actionable document
What You Know In Your Heart You Were Meant To Do.”-Leroy Satchel Paige Page 12 of where the stipulation limits B‟s liability to A to P10,000.00 only. The issue raised by B
198 Philippines, outside of the Manila with the courts of the first level or with the does not go against or impugn the genuineness and due execution of the Bill of Lading as
Regional Trial Court. The assessed value of the parcel of land in Pampanga is different an actionable document pleaded by A, but invokes the binding effect of said stipulation.
from the assessed value of the land in Bulacan. What is involved is not merely a matter of The oath is not required of B, because the issue raised by the latter does not impugn the
venue, which is waivable, but of a matter of jurisdiction. However, the action may genuineness and due execution of the Bill of Lading. Katarungang Pambarangay; Parties
prosper if jurisdiction is not in issue, because venue can be waived. ALTERNATIVE (2009) No.XV.B. Mariano, through his attorney-infact, Marcos filed with the RTC of
ANSWER: Yes, if the defendant would not file a motion to dismiss on ground of improper Baguio City a complaint for annulment of sale against Henry. Marcos and Henry both
venue and the parties proceeded to trial. (b) Will your answer be the same if the action reside in Asin Road, Baguio City, while Mariano resides in Davao City. Henry filed a
was for foreclosure of the mortgage over the two parcels of land? Why or why not? motion to dismiss the complaint on the ground of prematurity for failure to comply
SUGGESTED ANSWER: NO, the answer would not be the same. The foreclosure action Remedial Law Q&As (2007-2013) hectorchristopher@yahoo.com JayArhSals “Never Let
should be brought in the proper court of the province where the land or any part thereof The Odds Keep You From Pursuing What You Know In Your Heart You Were Meant To
is situated, either in Pampanga or in Bulacan. Only one foreclosure action need be filed Do.”-Leroy Satchel Paige Page 15 of 198 with the mandatory barangay conciliation.
unless each parcel of land is covered by distinct mortgage contract. In foreclosure suit, Resolve the motion with reasons. (3%) SUGGESTED ANSWER: The motion to dismiss
the cause of action is for the violation of the terms and conditions of the mortgage should be denied because the parties in interest, Mariano and Henry, do not reside in the
contract; hence, one foreclosure suit per mortgage contract violated is necessary. [Note: same city/municipality, or is the property subject of the controversy situated therein.
The question is the same as 2008 Remedial Law Bar question No.III. See Civ.Pro Venue; The required conciliation/mediation before the proper Barangay as mandated by the
Real Actions, Infra – JayArhSals] Jurisdiction; RTC; Counterclaim (2008) No.II. Fe filed a Local Government Code governs only when the parties to the dispute reside in the same
city or municipality, and if involving real property, as in this case, the property must be land registration case based on its delegated jurisdiction. (1%) SUGGESTED ANSWER: The
situated also in the same city or municipality. Civil Procedure (Rules 1-56) Actions; Cause appeal should be filed with the Court of Appeals by filing a Notice of Appeal within 15
of Action (2013) No.VI. While leisurely walking along the street near her house in days from notice of judgment or final order appealed from. (Sec. 34, Batas Pambansa Blg.
Marikina, Patty unknowingly stepped on a garden tool left behind by CCC, a construction 129, or the Judiciary Reorganization Act of 1980, as amended by Republic Act No. 7691,
company based in Makati. She lost her balance as a consequence and fell into an open March 25, 1994). (4) A decision of the Court of Tax Appeal's First Division. (1%)
manhole. Fortunately, Patty suffered no major injuries except for contusions, bruises and SUGGESTED ANSWER: The decision of the Court of Tax Appeals Division may be appealed
scratches that did not require any hospitalization. However, she lost selfesteem, suffered to the CTA en banc. The decisions of the Court of Tax Appeals are no longer appealable
embarrassment and ridicule, and had bouts of anxiety and bad dreams about the to the Court of Appeals. Under the modified appeal procedure, the decision of a division
accident. She wants vindication for her uncalled for experience and hires you to act as of the CTA may be appealed to the CTA en banc. The decision of the CTA en banc may in
counsel for her and to do whatever is necessary to recover at least Php100,000 for what turn be directly appealed to the Supreme Court by way of a petition for review on
she suffered. What action or actions may Patty pursue, against whom, where (court and certiorari under Rule 45 on questions of law. (Section 11, R.A. 9282, March 30, 2004).
venue), and under what legal basis? (7%) SUGGESTED ANSWER: Patty may avail any of Appeals; Modes of Appeal (2009) No.VIII. On July 15, 2009, Atty. Manananggol was
the following remedies: a) She may file a complaint for damages arising from fault or served copies of numerous unfavorable judgments and orders. On July 29, 2009, he filed
negligence under the Rules on Small Claims against CCC Company before the MTC of motions for reconsideration which were denied. He received the notices of denial of the
Marikina City where she resides or Makati City where the defendant corporation is motions for reconsideration on October 2, 2009, a Friday. He immediately informed his
holding office, at her option (A.M. No. 8- 8-7-SC in relation to Section 2, Rule 4, Rules of clients who, in turn, uniformly instructed him to appeal. How, when and where should he
Court). b) She may also file an action to recover moral damages based on quasi-delict pursue the appropriate remedy for each of the following: (a) Judgment of a Municipal
under Article 2176 of the New Civil Code. The law states that, whoever by act or Trial Court (MTC) pursuant to its delegated jurisdiction dismissing his client’s application
omission causes damage to another, there being fault or negligence is obliged to pay for for land registration? SUGGESTED ANSWER: By notice of appeal, within 15 days from
the damage done. Such fault or negligence, if there is no Remedial Law Q&As (2007- notice of judgment or final order appealed from, to the Court of Appeals; (b) Judgment
2013) hectorchristopher@yahoo.com JayArhSals “Never Let The Odds Keep You From of the Regional Trial Court (RTC) denying his client’s petition for a writ of habeas data?
Pursuing What You Know In Your Heart You Were Meant To Do.”-Leroy Satchel Paige SUGGESTED ANSWER: By verified petition for review on certiorari under Rule 45, with
Page 16 of 198 pre-existing contractual relation between the parties, is called a quasi- the modification that appellant may raise Remedial Law Q&As (2007-2013)
delict. Under Article 2217 of the New Civil Code, moral damages include physical hectorchristopher@yahoo.com JayArhSals “Never Let The Odds Keep You From Pursuing
suffering, mental anguish, fright, serious anxiety, besmirched reputation, wounded What You Know In Your Heart You Were Meant To Do.”-Leroy Satchel Paige Page 19 of
feelings, moral shock, social humiliation, and similar injury. Though incapable of 198 questions of fact or law or both, within 5 work days from date of notice of the
pecuniary computation, moral damages may be recovered if they are the proximate judgment or final order to the Supreme Court (Sec. 19, A.M. No. 08-1-16-SC). (c) Order of
result of the defendant‟s wrongful act or omission. Since moral damages are incapable a family court denying his client’s petition for habeas corpus in relation to custody of a
of pecuniary estimation, Patty should file the action before the Regional Trial Court of minor child? SUGGESTED ANSWER: By notice of appeal, within 48 hours from notice of
Marikina City where she resides or Makati City, where the defendant corporation is judgment or final order to the Court of appeals (Sec. 14, R.A. No. 8369 in relation to Sec.
holding office, at her option (Section 19(1), B.P. 129). c) Patty can also file a civil action 3, Rule 41, Rules of Court). (d) Order of the RTC denying his client’s petition for certiorari
for damages against the City of Marikina for maintaining an open manhole where she questioning the Metropolitan Trial Court’s denial of a motion to suspend criminal
unfortunately fell. Under article 2189 of the Civil Code, provinces, cities, and proceedings? SUGGESTED ANSWER: By notice of appeal, within 15 days from notice of
municipalities shall be liable for damages for the death of, or injuries suffered by, any the final order, to the Court of appeals (Majestrado vs. People, 527 SCRA 125 [2007]). (e)
person by reason of the defective condition of roads, streets, bridges, public buildings, Judgment of the First Division of the Court of Tax Appeals affirming the RTC decision
and other public works under their control or supervision. The proper court having convicting his client for violation of the National Internal Revenue Code? SUGGESTED
jurisdiction over the case is at least Php 100,000 for as long as the aggregate of the ANSWER: By petition for review filed with the court of Tax Appeals (CTA) en banc, within
claims for damages does not exceed Php 400,000. Actions; Specific Performance (2012) 30 days from receipt of the decision or ruling in question (Sec. 9 [b], Rule 9, Rev. Rules of
No.IV.A. A bought a Volvo Sedan from ABC Cars for P 5.0M. ABC Cars, before delivering CTA). Appeals; Modes of Appeal; RTC, CA (2009) No. XIX.A. Distinguish the two modes of
to A, had the car rust proofed and tinted by XYZ Detailing. When delivered to A, the car's appeal from the judgment of the Regional Trial Court to the Court of Appeals.
upholstery was found to be damaged. ABC Cars and XYZ Detailing both deny any liability. SUGGESTED ANSWER: In cases decided by the Regional Trial Courts in the exercise of
Who can A sue and on what cause(s) of action? Explain. (5%) SUGGESTED ANSWER: A can their original jurisdiction, appeals to the Court of Appeals shall be ordinary appeal by
file an action for specific performance and damages against ABC Cars since the damage filing written notice of appeal indicating the parties to the appeal; specifying the
to the Volvo Sedan‟s upholstery was caused before the delivery of the same to A, and judgment/final order or part thereof appealed from; specifying the court to which the
therefore prior to the transfer of ownership to the latter. (Article 1477, New Civil Code). appeal is being taken; and stating the material dates showing the timeliness of the
Under Article 1170 of the New Civil Code, those who contravene the tenor of the appeal. The notice of appeal shall be filed with the RTC which rendered the judgment
obligation are liable for damages. Hence, an action for specific performance against ABC appealed from and copy thereof shall be served upon the adverse party within 15 days
Corporation to deliver the agreed Volvo Sedan in the contract, free from any damage or from notice of judgment or final order appealed from. But if the case admits of multiple
defects, with corresponding damages will lie against ABC Cars. ALTERNATIVE ANSWER: appeals or is a special proceeding, a record on appeal is Remedial Law Q&As (2007-2013)
Remedial Law Q&As (2007-2013) hectorchristopher@yahoo.com JayArhSals “Never Let hectorchristopher@yahoo.com JayArhSals “Never Let The Odds Keep You From Pursuing
The Odds Keep You From Pursuing What You Know In Your Heart You Were Meant To What You Know In Your Heart You Were Meant To Do.”-Leroy Satchel Paige Page 20 of
Do.”-Leroy Satchel Paige Page 17 of 198 A can sue ABC Cars for specific performance or 198 required aside from the written notice of appeal to perfect the appeal, in which case
rescission because the former has contractual relations with the latter. Appeals; the period for appeal and notice upon the adverse party is not only 15 days but 30 days
Abandonment of a Perfected Appeal (2009) No.XI.E. The filing of a motion for the from notice of judgment or final order appealed from. The full amount of the appellate
reconsideration of the trial court’s decision results in the abandonment of a perfected court docket fee and other lawful fees required must also be paid within the period for
appeal. SUGGESTED ANSWER: FALSE. The trial court has lost jurisdiction after perfection taking an appeal, to the clerk of the court which rendered the judgment or final order
of the appeal and so it can no longer entertain a motion for reconsideration. appealed from (Secs. 4 and 5, Rule 41, Rules of Court). The periods of 15 or 30 days
ALTERNATIVE ANSWER: FALSE, because the appeal may be perfected as to one party but above-stated are nonextendible. In cases decided by the Regional Trial Court in the
not yet perfected as to the other party who may still file a motion for reconsideration exercise of its appellate jurisdiction, appeal to the Court of Appeals shall be by filing a
without abandonment of his right of appeal even though the appeal of the case is verified petition for review with the Court of Appeals and furnishing the RTC and the
perfected already as to the other party. Appeals; Modes of Appeal (2012) No.X.A. Where adverse party with copy thereof, within 15 days from notice of judgment or final order
and how will you appeal the following: (1) An order of execution issued by the RTC. (1%) appealed from. Within the same period for appeal, the docket fee and other lawful fees
SUGGESTED ANSWER: A petition for certiorari under Rule 65 before the Court of Appeals. required with the deposit for cost should be paid. The 15- day period may be extended
ALTERNATIVE ANSWER: The mode of elevation may be either by appeal (writ of error or for 15 days and another 15 days for compelling reasons. Appeals; Second Notice of
certiorari), or by a special civil action of certiorari, prohibition, or mandamus. (Banaga vs. Appeal (2008) No.XII. After receiving the adverse decision rendered against his client, the
Majaducon cited in General Milling Corporation-Independent Labor Union vs. General defendant, Atty. Sikat duly filed a notice of appeal. For his part, the plaintiff timely filed a
Milling Corporation, G.R. No. 183122, June 15, 2011, Perez, J.). (2) Judgment of RTC motion for partial new trial to seek an increase in the monetary damages awarded. The
denying a petition for Writ of Amparo. (1%) SUGGESTED ANSWER: Any party may appeal RTC instead rendered an amended decision further reducing the monetary awards. Is it
from the final judgment or order to the Supreme Court by way of a petition for review on necessary for Atty. Sikat to file a second notice of appeal after receiving the amended
certiorari under Rule 45 of the Rules of Court. the period of appeal shall be five (5) decision? SUGGESTED ANSWER: Yes, it is necessary for Atty. Sikat to file a second notice
working days from the date of notice of the adverse judgment, and the appeal may raise of appeal after receiving the amended decision. In Magdalena Estate vs. Caluag (11 SCRA
questions of fact or law or both. (sec. 19, Rule on Writ of Amparo, A.M. No. 07-9-12-SC, 333 [1964]), the Court ruled that a party must re-take an appeal within fifteen [15) days
25 September 2007). Remedial Law Q&As (2007-2013) hectorchristopher@yahoo.com from receipt of the amended ruling or decision, which stands in place of the old decision.
JayArhSals “Never Let The Odds Keep You From Pursuing What You Know In Your Heart It is in effect, a new decision. Certiorari; Rule 45 vs. Rule 65 (2008) No.XXI.A. Compare
You Were Meant To Do.”-Leroy Satchel Paige Page 18 of 198 (3) Judgment of MTC on a the certiorari jurisdiction of the Supreme Court under the Constitution with that under
Rule 65 of the Rules of Civil Procedure? SUGGESTED ANSWER: The certiorari jurisdiction Inc., 540 SCRA 215 [2007]). Forum Shopping; Certification (2009) No.III. Amorsolo, a
of the Supreme Court under the Constitution is Remedial Law Q&As (2007-2013) Filipino citizen permanently residing in New York City, filed with the RTC of Lipa City a
hectorchristopher@yahoo.com JayArhSals “Never Let The Odds Keep You From Pursuing complaint for Rescission of Contract of Sale of Land against Brigido, a resident of
What You Know In Your Heart You Were Meant To Do.”-Leroy Satchel Paige Page 21 of Barangay San Miguel, Sto. Tomas, Batangas. The subject property, located in Barangay
198 the mode by which the Court exercises its expanded jurisdiction, allowing it to take Talisay, Lipa City, has an assessed value of 19,700. Appended to the complaint is
corrective action through the exercise of its judicial power. Constitutional certiorari Amorsolo’s verification and certification of non-forum shopping executed in New York
jurisdiction applies even if the decision was not rendered by a judicial or quasi-judicial City, duly notarized by Mr. Joseph Brown, Esq., a notary public in the State of New York.
body, hence, it is broader than the writ of certiorari under Rule 65, which is limited to Brigod filed a motion to dismiss the complaint on the following grounds: (c) The
cases involving a grave abuse of discretion amounting to lack or excess of jurisdiction on verification and certification of nonforum shopping are fatally defective because there is
the part of any branch or instrumentality of the government and there is no other claim no accompanying certification issued by the Philippine Consulate in New York,
speedy remedy available to a party in the ordinary course of law. Demurrer to Evidence authenticating that Mr. Brown is duly authorized to notarize Remedial Law Q&As (2007-
(2009) No.XVI.A. After the prosecution had rested and made its formal offer of evidence, 2013) hectorchristopher@yahoo.com JayArhSals “Never Let The Odds Keep You From
with the court admitting all of the prosecution evidence, the accused filed a demurer to Pursuing What You Know In Your Heart You Were Meant To Do.”-Leroy Satchel Paige
evidence with leave of court. the prosecution was allowed to comment thereon. Page 24 of 198 the document. (3%) Rule on the foregoing grounds with reasons.
Thereafter, the court granted the demurer, finding that the accused could not have SUGGESTED ANSWER: The third ground raised questioning the validity of the verification
committed the offense charged. If the prosecution files a motion for reconsideration on and certification of non-forum shopping for lack of certification from the Philippine
the ground that the court order granting the demurer was not in accord with law and Consulate in New York, authenticating that Mr. Brown is duly authorized to notarize the
jurisprudence, will the motion prosper? SUGGESTED ANSWER: NO, the motion will not document, is likewise without merit. The required certification alluded to, pertains to
prosper. With the granting of the demurrer, the case shall be dismissed and the legal official acts, or records of official bodies, tribunals, and public officers, whether of the
effect is the acquittal of the accused. A judgment of acquittal is immediately executor Philippines or of a foreign country: the requirement in Sec. 24, Rule 132 of the 1997
and no appeal can be made therefrom. Otherwise the Constitutional protection against Rules refers only to paragraph (a) of Sec. 29 which does not cover notarial documents. It
double jeopardy would be violated. Demurrer to Evidence; Civil Case vs. Criminal Case is enough that the notary public who notarized the verification and certification of non-
(2007) No.V. (a) Distinguish the effects of the filing of a demurrer to the evidence in a forum shopping is clothed with authority to administer oath in that State or foreign
criminal case and its filing in a civil case. (5%) SUGGESTED ANSWER: The following are the country. Judgment; Enforcement; 5yr Period (2007) No.X. (b) A files a case against B.
distinctions in effects of demurrer to the evidence in criminal cases from that in civil While awaiting decision on the case, A goes to the United States to work. Upon her
cases: (1) In criminal cases, demurrer to the evidence requires leave of court, otherwise, return to the Philippines, seven years later, A discovers that a decision was rendered by
the accused would lose his right to present defense evidence if filed and denied; in civil the court in here favor a few months after she had left. Can A file a motion for execution
cases, no leave of court is required for filing such demurrer. (2) In criminal cases, when of the judgment? Reason briefly. (5%) SUGGESTED ANSWER: On the assumption that the
such demurrer is granted, the dismissal of the case is not appealable inasmuch as the judgment had been final and executory for more than five (5) years as of A‟s return to
Remedial Law Q&As (2007-2013) hectorchristopher@yahoo.com JayArhSals “Never Let the Philippines seven (7) years later, a motion for execution of the judgment is no longer
The Odds Keep You From Pursuing What You Know In Your Heart You Were Meant To availing because the execution of judgment by mere motion is allowed by the Rules only
Do.”-Leroy Satchel Paige Page 22 of 198 dismissal would amount to an acquittal, unless within five (5) years from entry of judgment; thereafter, and within ten (10) years from
made by a court acting without or in excess of jurisdiction; in civil cases, when such entry of judgment, an action to enforce the judgment is required. Judgment; Execution;
demurrer is granted, the dismissal of the case can be appealed by the plaintiff. (3) In Judgment Obligor‟s Death (2009) No.VII. Cresencio sued Dioscoro for colletion of a sum
criminal cases, the accused loses his right to present his defense-evidence in the trial of money. During the trial, but after the presentation of plaintiff’s evidence, Dioscoro
court when he filed the demurrer without prior leave of court; while in civil cases, the died. Atty. Cruz, Dioscoro’s counsel, then filed a motion to dismiss the action on the
defendant loses his right to present his defense-evidence only if the plaintiff appealed ground of his client’s death. The court denied the motion to dismiss and, instead,
such dismissal and the case is before the appellate court already since the case would be directed counsel to furnish the court with the names and addresses of Dioscoro’s heirs
decide only on the basis of plaintiff‟s evidence on record. Discovery; Modes of Discovery; and ordered that the designated administrator of Dioscoro’s estate be substituted as
Refusal to Comply (2010) No.II. On August 13, 2008, A, as shipper and consignee, loaded representative party. Remedial Law Q&As (2007-2013) hectorchristopher@yahoo.com
on the M/V Atlantis in Legaspi City 100,000 pieces of century eggs. The shipment arrived JayArhSals “Never Let The Odds Keep You From Pursuing What You Know In Your Heart
in Manila totally damaged on August 14, 2008. A filed before the Metropolitan Trial You Were Meant To Do.”-Leroy Satchel Paige Page 25 of 198 After trial, the court
Court (MeTC) of Manila a complaint against B Super Lines, Inc. (B Lines), owner of the rendered judgment in favor of Cresencio. When the decision had become final and
M/V Atlantis, for recovery of damages amounting to P167,899. He attached to the executory, Cresencio moved for the issuance of a writ of execution against Dioscoro’s
complaint the Bill of Lading. (c) On July 21, 2009, B Lines served on A a "Notice to Take estate to enforce his judgment claim. The court issued the writ of execution. Was the
Deposition," setting the deposition on July 29, 2009 at 8:30 a.m. at the office of its court’s issuance of the writ of execution proper? Explain. SUGGESTED ANSWER: No, the
counsel in Makati. A failed to appear at the deposition-taking, despite notice. As counsel issuance of a writ of execution by the court is not proper and is in excess of jurisdiction,
for B Lines, how would you proceed? (3%) SUGGESTED ANSWER: As counsel for B lines since the judgment obligor is already dead when the writ was issued. The judgment for
(which gave notice to take the deposition), I shall proceed as follows: (a) Find out why A money may only be enforced against the estate of the deceased defendant in the
failed to appear at the deposition taking, despite notice; (b) If failure was for valid probate proceedings, by way of a claim filed with the probate court. Cresencio should
reason, then set another date for taking the deposition. (c) If failure to appear at enforce that judgment in his favor in the settlement proceedings of the estate of
deposition taking was without valid reason, then I would file a motion/application in the Dioscoro as a money claim in accordance with the Rule 86 or Rule 88 as the case may be.
court where the action is pending, for and order to show cause for his refusal to submit Judgment; Execution; Stay (2009) No.XII. Mike was renting an apartment unit in the
to the discovery; and (d) For the court to issue appropriate Order provided under Rule 29 building owned by Jonathan. When Mike failed to pay six months’ rent, Jonathan filed an
of the Rules, for noncompliance with the showcause order, aside from contempt of ejectment suit. The Municipal Trial Court (MTC) rendered judgement in favor of
court. Remedial Law Q&As (2007-2013) hectorchristopher@yahoo.com JayArhSals Jonathan, who then filed a motion for the issuance of a writ of execution. The MTC
“Never Let The Odds Keep You From Pursuing What You Know In Your Heart You Were issued the writ. (a) How can mike stay the execution of the MTC judgment? (2%)
Meant To Do.”-Leroy Satchel Paige Page 23 of 198 Discovery; Production and Inspection SUGGESTED ANSWER: Execution shall issue immediately upon motion, unless Mike (a)
(2009) No.XIII.A. Continental Chemical Corporation (CCC) filed a complaint for a sum of perfects his appeal to the RTC, (b) files a sufficient supersedeas bond to pay the rents,
money against Barstow Trading Corporation (BTC) for the latter’s failure to pay for its damages and costs accruing up to the time of the judgment appealed from, and (c)
purchases of industrial chemicals. In its answer, BTC contended that it refused to pay deposits monthly with the RTC during the pendency of the appeal the amount of rent
because CCC misrepresented that the products it sold belonged to a new line, when in due from time to time (Rule 70, Sec. 19). (b) Mike appealed to the Regional Trial Court,
fact they were identical with CCC’s existing products. To substantiate its defense, BTC which affirmed the MTC decision. Mike then filed a petition for review with the Court of
filed a motion to compel CCC to give a detailed list of the products’ ingredients and Appeals. The CA dismissed the petition on the ground that the sheriff had already
chemical components, relying on the right to avail of the modes of discovery allowed executed the MTC decision and had ejected Mike from the premises, thus rendering the
under Rule 27. CCC objected, invoking confidentiality of the information sought by BTC. appeal moot and academic. Is the CA correct? (3%) Reasons. SUGGESTED ANSWER: NO.
Resolve BTC’s motion with reasons. (3%) SUGGESTED ANSWER: I will deny the motion. The Court of Appeals is not correct. The dismissal of the appeal is wrong, because the
The ingredients and chemical components of CCC‟s products are trade secrets within the execution of the RTC judgment is only in respect of the Remedial Law Q&As (2007-2013)
contemplation of the law. Trade secrets may not be the subject of compulsory disclosure hectorchristopher@yahoo.com JayArhSals “Never Let The Odds Keep You From Pursuing
by reason of their confidential and privileged character. Otherwise, CCC would eventually What You Know In Your Heart You Were Meant To Do.”-Leroy Satchel Paige Page 26 of
be exposed to unwarranted business competition with others who may imitate and 198 eviction of the defendant from the leased premises. Such execution pending appeal
market the same kinds of products in violation of CCC‟s proprietary rights. Being has no effect on the merits of the ejectment suit which still has to be resolved in the
privileged, the detailed list of ingredients and chemical components may not be the pending appeal. Rule 70, Sec. 21 of the Rules provides that the RTC judgment against the
subject of mode of discovery under Rule 27, Section 1 which expressly makes privileged defendant shall be immediately executor, “without prejudice to a further appeal” that
information an exception from its coverage (Air Philippines Corporation vs. Pennswell, may be taken therefrom (Uy vs. Santiago, 336 SCRA 680 [2000]). Judgment; Execution;
Successors in Interest (2008) No.XV. Half-brothers Roscoe and Salvio inherited from their promissory note, quoted and attached to the complaint. In his answer with counterclaim,
father a vast tract of unregistered land. Roscoe succeeded in gaining possession of the Ernesto alleged that Modesto coerced him into signing the promissory note, but that it is
parcel of land in its entirety and transferring the tax declaration thereon in his name. Modesto who really owes him P1.5M. Modesto filed an answer to Ernesto’s counterclaim
Roscoe sold the northern half to Bono, Salvio’s cousin. Upon learning of the sale, Salvio admitting that he owed Ernesto, but only in the amount of P0.5M. at the pre-trial,
asked Roscoe to convey the southern half to him. Roscoe refused as he even sold Modesto marked and Remedial Law Q&As (2007-2013) hectorchristopher@yahoo.com
onethird of the southern half along the West to Carlo. Thereupon, Salvio filed an action JayArhSals “Never Let The Odds Keep You From Pursuing What You Know In Your Heart
for reconveyance of the southern half against Roscoe only. Carlo was not impleaded. You Were Meant To Do.”-Leroy Satchel Paige Page 29 of 198 identified Ernesto’s
After filing his answer, Roscoe sold the middle third of the southern half to Nina. Salvio promissory note. He also marked and identified receipts covering payments he made to
did not amend the complaint to implead Nina. After trial, the court rendered judgment Ernesto, to the extent of P0.5M, which Ernesto did not dispute. After pre-trial, Modesto
ordering Roscoe to reconvey the entire southern half to Salvio. The judgment became filed a motion for judgment on the pleadings, while Ernesto filed a motion for summary
final and executory. A writ of execution having been issued, the sheriff required Roscoe, judgment on his counterclaim. Resolve the two motions with reasons. SUGGESTED
Carlo and Nina to vacate the southern half and yield possession thereof to Salvio as the ANSWER: Modesto‟s motion for judgment on the pleadings should be denied. While it is
prevailing party. Carlo and Nina refused, contending that they are not bound by the true that under the actionable document rule, Ernesto‟s failure to deny under oath the
judgment as they are not parties to the case. Is the contention tenable? Explain fully. promissory note in his answer amounted to an implied admission of its genuineness and
(4%) SUGGESTED ANSWER: As a general rule, no stranger should be bound to a judgment due execution, his allegation in his answer that he was coerced into signing the
where he is not included as a party. The rule on transfer of interest pending litigation is promissory note tendered an issue which should be tried. The issue of coercion is not
found in Sec. 19, Rule 3, 1997 Rules of Civil Procedure. The action may continue unless inconsistent with the due execution and genuineness of the instrument. Thus, Ernesto‟s
the court, upon motion directs a person to be substituted in the action or joined with the failure to deny the genuineness of the promissory note cannot be considered a waiver to
original party. Carlo is not bound by the judgment. He became a co-owner before the raise the issue that he was coerced in signing the same. Said claim of coercion may also
case was filed (Matuguina Integrated Wood Products, Inc. vs. C.A., G.R. No. 98310, 24 be proved as an exception to the Parol Evidence Rule. On the other hand, Ernesto‟s
October 1996; Polaris vs. Plan, 69 SCRA 93; See also Asset Privatization Trust vs. C.A., G.R. motion for summary judgment may be granted. Modesto‟s answer to Ernesto‟s
No. 121171, 29 December 1998). However, Nina is a privy or a successor in interest and counterclaim – that he owed the latter a sum less than what was claimed – amounted to
is bound by the judgment even if she is not a party to the case Remedial Law Q&As an admission of a material fact and if the amount thereof could summarily be proved by
(2007-2013) hectorchristopher@yahoo.com JayArhSals “Never Let The Odds Keep You affidavits, deposition, etc., without the need of going to trial, then no genuine issue of
From Pursuing What You Know In Your Heart You Were Meant To Do.”-Leroy Satchel fact exists. ALTERNATIVE ANSWER: Modesto‟s motion for judgment on the pleadings
Paige Page 27 of 198 (Sec. 19, Rule 3, 1997 Rules of Civil Procedure; Cabresos vs. Tiro, should be denied because there is an issue of fact. While Ernesto did not specifically
166 SCRA 400 [1998]). A judgment is conclusive between the parties and their deny under oath the promissory note attached to Modesto‟s complaint as an actionable
successors-in-interest by title subsequent to the case (Sec. 47, Rule 39, 1997 Rules of document, such non-denial will not bar Ernesto‟s evidence that Modesto coerced him
Civil Procedure). Judgment; Foreign Judgments; Foreign Arbitral Award (2007) No.I. (a) into signing the promissory note. Lack of consideration, as a defense, does not relate to
What are the rules on the recognition and enforcement of foreign judgments in our the genuineness and due execution of the promissory note. Likewise, Ernesto‟s motion
courts? (6%) SUGGESTED ANSWER: Judgments of foreign courts are given recognition in for summary judgment should be denied because there is an issue of fact – the alleged
our courts thus: In case of judgment upon a specific thing, the judgment is conclusive coercion – raised by Ernesto which he has yet to prove in a trial on its merits. It is
upon the title to the thing, unless otherwise repelled by evidence of lack of jurisdiction, axiomatic that summary judgment is not proper or valid whent there is an issue of fact
want of due notice to the party, collusion, fraud, or clear mistake of law or fact (Rule 39, remaining which requires a hearing. And this is so with respect to the coercion alleged by
Sec. 48 [a], Rules of Court); and In case of judgment against a person, the judgment is Ernesto as his defense, since coercion is not capable of being established by
presumptive evidence of a right as between the parties and their successors in interest documentary evidence. Remedial Law Q&As (2007-2013) hectorchristopher@yahoo.com
by subsequent title, unless otherwise repelled by evidence on grounds above stated JayArhSals “Never Let The Odds Keep You From Pursuing What You Know In Your Heart
(Rule 39, Sec. 48 [b], Rules of Court). However, judgments of foreign courts may only be You Were Meant To Do.”-Leroy Satchel Paige Page 30 of 198 Petition for Relief (2007)
enforced in the Philippines through an action validly heard in the Regional Trial Court. No.II. (b) A defendant who has been declared in default can avail of a petition for relief
Thus, it is actually the judgment of the Philippine court enforcing the foreign judgment from the judgment subsequently rendered in the case. (3%) SUGGESTED ANSWER: False.
that shall be executed. (b) Can a foreign arbitral award be enforced in the Philippines The remedy of petition for relief from judgment is available only when the judgment or
under those rules? Explain briefly. (2%) SUGGESTED ANSWER: No, a foreign arbitral order in question is already final and executor, i.e., no longer appealable. As an
award cannot be enforced in the Philippines under the rules on recognition and extraordinary remedy, a petition for relief from judgment may be availed only in
enforcement of foreign judgments above-stated. A foreign arbitral award is not a foreign exceptional cases where no other remedy is available. Petition for Relief; Injunction
judgment, and pursuant to the Alternative Dispute Resolution Act of 2004 (R.A. No. (2009) No.XVII. Having obtained favorable judgment in his suit for a sum of money
9285), in relation to 1958 New York Convention on the Recognition and Enforcement of against Patricio, Orencio sought the issuance of a writ of execution. When the writ was
Foreign Arbitral Awards, the recognition and enforcement of the foreign arbitral awards issued, the sheriff levied upon a parcel of land that Patricio owns, and a date was set for
shall be in accordance with the rules of procedure to be promulgated by the Supreme the execution sale. (a) How may Patricio prevent the sale of the property on execution?
Court. At present, the Supreme Court is yet to promulgate rules of procedure on the SUGGESTED ANSWER: Patricio may file a Petition for Relief with preliminary injunction
subject matter. (c) How about a global injunction issued by a foreign court to prevent (Rule 38), posting a bond equivalent to the value of the property levied upon; or assail
dissipation of funds against a defendant therein who has assets in the Philippines? the levy as invalid if ground exists. Patricio may also simply pay the amount required by
Explain briefly. (2%) SUGGESTED ANSWER: Remedial Law Q&As (2007-2013) the writ and the costs incurred therewith. (b) If Orencio is the purchaser of the property
hectorchristopher@yahoo.com JayArhSals “Never Let The Odds Keep You From Pursuing at the execution sale, how much does he have to pay? SUGGESTED ANSWER: Orencio,
What You Know In Your Heart You Were Meant To Do.”-Leroy Satchel Paige Page 28 of the judgment creditor should pay only the excess amount of the bid over the amount of
198 Yes, a global injunction issued by a foreign court to prevent dissipation of funds the judgment. (c) If the property is sold to a third party at the execution sale, what can
against a defendant who has assets in the Philippines may be enforced in our jurisdiction, Patricio do to recover the property? SUGGESTED ANSWER: Patricio can exercise his right
subject to our procedural laws. As a general rule, no sovereign is bound to give effect of legal redemption within 1 year from date of registration of the certificate of sale by
within its dominion to a judgment or order of a tribunal of another country. However, paying the amount of the purchase price with interests of 1% monthly, plus assessment
under the rules of comity, utility and convenience, nations have established a usage and taxes paid by the purchaser, with interest thereon, at the same rate. Pleadings;
among civilized states by which final judgments of foreign courts of competent Amendment of Complaint (2009) Remedial Law Q&As (2007-2013)
jurisdiction are reciprocally respected and rendered efficacious under certain conditions hectorchristopher@yahoo.com JayArhSals “Never Let The Odds Keep You From Pursuing
that may vary in different countries (St. Aviation Services Co., Pte., Ltd. v. Grand What You Know In Your Heart You Were Meant To Do.”-Leroy Satchel Paige Page 31 of
International Airways, Inc., 505 SCRA 30[2006]; Asiavest Merchant Bankers [M] Berhad v. 198 No.X. Upon termination of the pre-trial, the judge dictated the pre-trial order in the
Court of Appeals, 361 SCRA 489 [2001]). Judgment; Judgment on the Pleadings (2012) presence of the parties and their counsel, reciting what had transpired and defining
No.VII.B. Plaintiff files a request for admission and serves the same on Defendant who three (3) issues to be tried. (a) If, immediately upon receipt of his copy of the pre-trial
fails, within the time prescribed by the rules, to answer the request. Suppose the request order, plaintiff’s counsel should move for its amendment to include a fourth (4th) triable
for admission asked for the admission of the entire material allegations stated in the issue which he allegedly inadvertently failed to mention when the judge dictated the
complaint, what should plaintiff do? (5%) SUGGESTED ANSWER: The Plaintiff should file a order. Should the motion to amend be granted? Reasons. (2%) SUGGESTED ANSWER:
Motion for Judgment on the Pleadings because the failure of the defendant to answer a Depending on the merit of the issue sought to be brought in by the amendment, the
request for admission results to an implied admission of all the matters which an motion to amend may be granted upon due hearing. It is a policy of the Rules that
admission is requested. Hence, a motion for judgment on the pleadings is the parties should be afforded reasonable opportunity to bring about a complete
appropriate remedy where the defendant is deemed to have admitted the matters determination of the controversy between them, consistent with substantial justice.
contained in the Request for admission by the plaintiff. (Rule 34 in connection with Sec.2, With this end in view, the amendment before trial may be granted to prevent manifest
Rule 26, Rules of Court). Judgment; Judgment on the Pleadings (2009) No.IX. Modesto injustice. The matter is addressed to the sound and judicious discretion of the trial court.
sued Ernesto for a sum of money, claiming that the latter owed him P1M, evidenced by a (b) Suppose trial had already commenced and after the plaintiff’s second witness had
testified, the defendant’s counsel moves for the amendment of the pre-trial order to What You Know In Your Heart You Were Meant To Do.”-Leroy Satchel Paige Page 34 of
include a fifth (5th) triable issue vital to his client’s defense. Should the motion be 198 The Sheriff, however, on request of Benjamin, seized Mercedes’ 17th century ivory
granted over the objection of plaintiff’s counsel? Reasons. (3%) SUGGESTED ANSWER: image of the La Sagrada Familia estimated to be worth over P1,000,000. Was the
The motion may be denied since trial had already commenced and two witnesses for the Sheriff’s action in order? (3%) SUGGESTED ANSWER: No, the Sheriff‟s action was not in
plaintiff had already testified. Courts are required to issue pre-trial Order after the pre- order. He should not have listened to Benjamin, the judgment oblige/ creditor, in levying
trial conference has been terminated and before trial begins, precisely because the on the properties of Mercedes, the judgment obligor/debtor. The option to immediately
reason for such Order is to define the course of the action during the trial. Where trial choose which property or part thereof may be levied upon, sufficient to satisfy the
had already commenced, more so the adverse party had already presented witnesses, to judgment, is vested by law (Rule 39, Sec. 9 (b) upon the judgment obligor, Mercedes, not
allow an amendment would be unfair to the party who had already presented his upon the judgment obligee, Benjamin, in this case. Only if the judgment obligor does not
witnesses. The amendment would simply render nugatory the reason for or purpose of exercise the option, is the Sheriff authorized to levy on personal properties if any, and
the pre-trial Order. Sec.7 of Rule 18 on pre-trial in civil actions is explicit in allowing a then on the real properties if the personal properties are insufficient to answer for the
modification of the pre-trial Order “before” trial begins to prevent manifest injustice. judgment. Pleadings; Counterclaim (2007) No.II. (d) A counderclain is a pleading. (2%)
Pleadings; Amendment of Complaint (2008) No.XI. Arturo lent P1M to his friend Robert SUGGESTED ANSWER: True. A counterclaim is a pleading by which a defending party
on the condition that Rober execute a promissory note for the loan and a real Remedial makes a claim against an opposing party (Sec. 6, Rule 6, Rules of Court). Pleadings;
Law Q&As (2007-2013) hectorchristopher@yahoo.com JayArhSals “Never Let The Odds Motion (2007) No.II. (c) A motion is a pleading. (2%) SUGGESTED ANSWER: False. A
Keep You From Pursuing What You Know In Your Heart You Were Meant To Do.”-Leroy motion is not a pleading but a mere application for relief other than by a pleading (Rule
Satchel Paige Page 32 of 198 estate mortgage over his property located in Tagaytay City. 15, Sec. 1, Rules of Court). Pleadings; Motions; Bill of Particulars (2008) No.V. Within the
Rober complied. In his promissory note dated September 20, 2006, Robert undertook to period for filing a responsive pleading, the defendant filed a motion for bill of particulars
pay the loan within a year from its date at 12% per annum interest. In June 2007, Arturo that he set for hearing on a certain date. However, the defendant was surprised to find
requested Robert to pay ahead of time but the latter refused and insisted on the on the date set for hearing that the trial court had already denied the motion on the day
agreement. Arturo issued a demand letter and when Robert did not comply, Arturo filed of its filing, stating that the allegations of the complaint were sufficiently made. (a) Did
an action to foreclose the mortgage. Robert moved to dismiss the complatint for lack of the judge gravely abuse his discretion in acting on the motion without waiting for the
cause of action as the debt was not yet due. The resolution of the motion to dismiss was hearing set for the motion? SUGGESTED ANSWER: Remedial Law Q&As (2007-2013)
delayed because of the retirement of the Judge. (a) On October 1, 2007, pending hectorchristopher@yahoo.com JayArhSals “Never Let The Odds Keep You From Pursuing
resolution of the motion to dismiss, Arturo filed an amended complaint alleging Robert’s What You Know In Your Heart You Were Meant To Do.”-Leroy Satchel Paige Page 35 of
debt had in the meantime become due but that Robert still refused to pay. Should the 198 There is no need to set the motion for hearing. The duty of the clerk of court is to
amended complaint be allowed considering that no answer has been filed? SUGGESTED bring the motion immediately to the attention of the judge, who may act on it at once
ANSWER: No, the complaint may not be amended under the circumstances. A complaint (Sec. 2, Rule 12). (b) If the judge grants the motion and orders the plaintiff to file and
may be amended as of right before answer (Sec. 2, Rule 10; See Ong Peng vs. Custodio, serve the bill of particulars, can the trial judge dismiss the case if the plaintiff does not
G.R. No. 14911, 12 March 1961; Toyota Motors [Phils} vs. C.A., G.R. No. 102881, 07 comply with the order? SUGGESTED ANSWER: Yes, the judge may dismiss the case for
December 1992; RCPI vs. C.A., G.R. No. 121397, 17 April 1997, citing Prudence Realty & failure of the plaintiff to comply with its order (Sec. 3, Rule 17) or order the striking out
Dev‟t. Corp. vs. C.A., G.R. No. 110274, 21 March 1994; Soledad vs. Mamangun, 8 SCRA of the pleading and may issue any other order at its discretion (Sec. 4, Rule 12).
110), but the amendment should refer to facts which occurred prior to the filing of the Pleadings; Motions; Omnibus Motion Rule (2010) No.V. Charisse, alleging that she was a
original complaint. It thus follows that a complaint whose cause of action has not yet resident of Lapu-Lapu City, filed a complaint for damages against Atlanta Bank before the
accrued cannot be cured or remedied by an amended or supplemental pleading alleging RTC of Lapu-Lapu City, following the dishonor of a check she drew in favor of Shirley
the existence or accrual of a cause of action while the case is pending (Swagman Hotels against her current account which she maintained in the bank’s local branch. The bank
& Travel, Inc. vs. C.A., G.R. No. 161135, 08 April 2005). (b) Would your answer be filed a Motion to Dismiss the complaint on the ground that it failed to state a cause of
different had Arturo filed instead a supplemental complaint stating that the debt became action, but it was denied. It thus filed an Answer. (a) In the course of the trial, Charisse
due after the filing of the original complaint? SUGGESTED ANSWER: A supplemental admitted that she was a US citizen residing in Los Angeles, California and that she was
complaint may be filed with leave of court to allege an event that arose after the filing of temporarily billeted at the Pescado Hotel in Lapu-Lapu City, drawing the bank to file
the original complaint that should have already contained a cause of action (Sec. 6, Rule another motion to dismiss, this time on the ground of improper venue, since Charisse is
10). However, if no cause of action is alleged in the original complaint, it cannot be cured not a resident of Lapu-Lapu City. Charisse opposed the motion citing the "omnibus
by the filing of a supplement or amendment to allege the subsequent acquisition of a motion rule." Rule on the motion. (3%) SUGGESTED ANSWER: The bank‟s second motion
cause of action (Swagman Hotels & Travel, Inc. vs. C.A., G.R. No. 161135, 08 April 2005). to dismiss which is grounded on improper venue, should be denied. The improper venue
Remedial Law Q&As (2007-2013) hectorchristopher@yahoo.com JayArhSals “Never Let of an action is deemed waived by the bank‟s filing an earlier motion to dismiss without
The Odds Keep You From Pursuing What You Know In Your Heart You Were Meant To raising improper venue as an issue, and more so when the bank filed an Answer without
Do.”-Leroy Satchel Paige Page 33 of 198 Pleadings; Counterclaim (2010) No.VI. Antique raising improper venue as an issue after its first motion to dismiss was denied. Under the
dealer Mercedes borrowed P1,000,000 from antique collector Benjamin. Mercedes “omnibus motion rule” (Rule 15, Sec. 8, Rules of Court) which governs the bank‟s motion
issued a postdated check in the same amount to Benjamin to cover the debt. On the due to dismiss, such motion should include all objections then available; otherwise, all
date of the check, Benjamin deposited it but it was dishonored. As despite demands, objections not so included shall be deemed waived. Remedial Law Q&As (2007-2013)
Mercedes failed to make good the check, Benjamin filed in January 2009 a complaint for hectorchristopher@yahoo.com JayArhSals “Never Let The Odds Keep You From Pursuing
collection of sum of money before the RTC of Davao. Mercedes filed in February 2009 What You Know In Your Heart You Were Meant To Do.”-Leroy Satchel Paige Page 36 of
her Answer with Counterclaim, alleging that before the filing of the case, she and 198 Although the improper venue became known only in the course of the trial, the
Benjamin had entered into a dacion en pagoagreement in which her vintage P1,000,000 same should not be allowed to obstruct or disturb the proceedings since venue of civil
Rolex watch which was taken by Benjamin for sale on commission was applied to settle actions is defined for the convenience of the parties, nay jurisdictional. ALTERNATIVE
her indebtedness; and that she incurred expenses in defending what she termed a ANSWER: The “omnibus motion rule” should not apply, because the improper venue
"frivolous lawsuit." She accordingly prayed for P50,000 damages. (a) Benjamin soon after became known and thus available only to the movant bank after the motions to dismiss
moved for the dismissal of the case. The trial court accordingly dismissed the complaint. were filed and resolved by the court, and in the course of the trial of the case. In fairness
And it also dismissed the Counterclaim. Mercedes moved for a reconsideration of the to the defendant bank, it should not be precluded by the “omnibus motion rule” from
dismissal of the Counterclaim. Pass upon Mercedes’ motion. (3%) SUGGESTED ANSWER: raising objection to the improper venue only when said ground for objection became
Mercedes‟ Motion for Reconsideration is impressed with merit: the trial courts should known to it. The court may not resolve the second motion to dismiss precisely because
not have dismissed her counterclaim despite the dismissal of the Complaint. Since it was of the “omnibus motion rule”, since the bank filed an earlier motion to dismiss but did
the plaintiff (Benjamin) who moved for the dismissal of his Complaint, and at a time not raise the ground of improper venue, and subsequently filed an Answer wherein the
when the defendant (Mercedes) had already filed her Answer thereto and with improper venue has not again been raised. Hence, the question of improper venue has
counterclaim, the dismissal of the counterclaim without conformity of the defendant- become moot and academic. The only grounds not barred by the “omnibus motion rule”
counterclaimant. The Revised Rules of Court now provides in Rule 17, Sec. 2 thereof that are (a) lack of jurisdiction over the subject matter; (b) litis pendencia; and (c) bar by prior
“If a counterclaim has been pleaded by a defendant prior to the service upon him of the judgment or by statute of limitations. (b) Suppose Charisse did not raise the "omnibus
plaintiff‟s motion for dismissal, the dismissal shall be limited to the complaint. The motion rule," can the judge proceed to resolve the motion to dismiss? Explain. (3%)
dismissal shall be without prejudice to the right of the defendant to prosecute his SUGGESTED ANSWER: Yes, the judge can proceed to resolve the motion to dismiss,
counterclaim x x x x.” (b) Suppose there was no Counterclaim and Benjamin’s complaint because the ground raised therefor became known to the movant only during the trial,
was not dismissed, and judgment was rendered against Mercedes for P1,000,000. The such that it was only then that the objection became available to him. (c) Suppose the
judgment became final and executory and a writ of execution was correspondingly judge correctly denied the second motion to dismiss and rendered judgment in favor of
issued. Since Mercedes did not have cash to settle the judgment debt, she offered her Charisse, ordering the bank to pay her P100,000 in damages plus legal interest. The
Toyota Camry model 2008 valued at P1.2 million. Remedial Law Q&As (2007-2013) judgment became final and executory in 2008. To date, Charisse has not moved to
hectorchristopher@yahoo.com JayArhSals “Never Let The Odds Keep You From Pursuing execute the judgment. The bank is concerned that its liability will increase with the delay
because of the interest on the judgment award. As counsel of the bank, what move discovered the default after the judgment has become final and executor, he may file a
should you take? (3%) SUGGESTED ANSWER: As counsel of the bank, I shall recommend petition for relief under Section 2 of Rule 38; and 4) he may also appeal from the
to the bank as judgment Remedial Law Q&As (2007-2013) judgment rendered against him as contrary to the evidence or to the law, even if no
hectorchristopher@yahoo.com JayArhSals “Never Let The Odds Keep You From Pursuing petition to set aside the order of default has been presented by him. (B.D. Longspan
What You Know In Your Heart You Were Meant To Do.”-Leroy Satchel Paige Page 37 of Builders, Inc. vs. R.S. Ampeloquio Realty Development, G.R. No. 169919, September 11,
198 obligor, to make a tender of payment to the judgment oblige and thereafter make a 2009). [Note: there are additional remedies to address judgments by default: Motion for
consignation of the amount due by filing an application therefor placing the same at the Reconsideration (Rule 37), Annulment of Judgment (Rule 47) and Petition for Certiorari
disposal of the court which rendered the judgment (Arts. 1256 and 1258, Civil Code). (Rule 65)]. ALTERNATIVE ANSWER: Remedial Law Q&As (2007-2013)
Subpoena; Viatory Right of Witness (2009) No.XI.B. The viatory right of a witness served hectorchristopher@yahoo.com JayArhSals “Never Let The Odds Keep You From Pursuing
with a subpoena ad testificandum refers to his right not to comply with the subpoena. What You Know In Your Heart You Were Meant To Do.”-Leroy Satchel Paige Page 40 of
SUGGESTED ANSWER: FALSE. The viatory right of a witness, embodied in Sec. 10, Rule 21 198 The court committed grave abuse of discretion when it declared the defending party
of the Rules of Civil Procedure, refers to his right not to be compelled to attend upon a in default despite the latter‟s filing of an Answer. Thus, a petition for certiorari under
subpoena, by reason of the distance from the residence of the witness to the place Rule 65 is the proper remedy. In San Pedro Cineplex Properties vs. Heirs of Manuel
where he is to testify. It is available only in civil cases (People vs. Montejo, 21 SCRA 722 Humada Enano, G.R. No. 190754, November 17, 2010, the Supreme Court held that
[1965]). Summons; By Publication (2008) No.I. Lani filed an action for partition and where the answer is filed beyond the reglementary period but before the defendant is
accounting in the Regional Trial Court (RTC) of Manila against her sister Mary Rose, who declared in default and there is no showing that defendant intends to delay the case, the
is a resident of Singapore and is not found in the Philippines. Upon moition, the court answer should be admitted. Thus, it was error to declare the defending party in default
ordered the Publication of the summons for three weeks in a local tabloid, Bulgar. Linda, after the Answer was filed (See Sablas vs. Sablas, G.R. No. 144568, July 3, 2007). After all,
an OFW vacationing in the Philippines, saw the summons in Bulgar and brought a copy of the defect in the service of summons was cured by Charlie‟s filing of a verified answer
the tabloid when she returned to Singapore. Linda showed the tabloid and the page raising only the defense of full payment. The belated filing of verified Answer amounts to
containing the summons to Mary Rose, who said, “Yes I know, my kumara Anita scanned voluntary submission to the jurisdiction of the court and waiver of any defect in the
and emailed that page of Bulgar to me!” Did the court acquire jurisdiction over Mary service of summons. Trial; Court of Appeals as Trial Court (2008) No.XXI.B. Give at least
Rose? SUGGESTED ANSWER: Partition is an action quasi in rem. Summons by publication three instances where the Court of Appeals may act as a trial court? SUGGESTED
is proper when the defendant does not reside and is not found in the Philippines, ANSWER: The Court of Appeals may act as a trial court in the following instances: (1) In
provided that a copy of the summons and order of the court are sent by registered mail annulment of judgments (Sec. 5 & 6, Rule 47) (2) When a motion for new trial is granted
to the last known address of the defendant (Sec. 15, Rule 14). Publication of the notice in by the Court of Appeals (Sec. 4, Rule 53) (3) A petition for Habeas Corpus shall be set for
Bulgar, a newspaper of general circulation, satisfies the requirements of summons by hearing 9Sec. 12, Rule 102) (4) To resolve factual issues in cases within its original and
publication (Perez vs. Perez, G.R. No 145368, 28 March 2005). Summons; Served by Email appellate jurisdiction (Sec. 12, Rule 124) (5) In cases of new trial based on newly
(2009) No.I.E. Summons may be served by mail. Remedial Law Q&As (2007-2013) discovered evidence (Sec. 14, Rule 124 of the Rules on Criminal Procedure). (6) In Cases
hectorchristopher@yahoo.com JayArhSals “Never Let The Odds Keep You From Pursuing involving claims for damages arising from provisional remedies (7) In Amparo
What You Know In Your Heart You Were Meant To Do.”-Leroy Satchel Paige Page 38 of proceedings (A.M. No. 07- 9-12-SC) Venue; Real Actions (2012) No.III.B. A, a resident of
198 SUGGESTED ANSWER: FALSE. Rule 14 of the Rules of Court, on Summons, provide Quezon City, wants to file an action against B, a resident of Remedial Law Q&As (2007-
only for serving Summons (a) to the defendant in person; or (b) if this is not possible 2013) hectorchristopher@yahoo.com JayArhSals “Never Let The Odds Keep You From
within a reasonable time, then by substituted service in accordance with Sec. 7 thereof; Pursuing What You Know In Your Heart You Were Meant To Do.”-Leroy Satchel Paige
or (c) if any of the foregoing two ways is not possible, then with leave of court, by Page 41 of 198 Pasay, to compel the latter to execute a Deed of Sale covering a lot
publication in accordance with the same Rule. ALTERNATIVE ANSWER: TRUE, but only in situated in Marikina and that transfer of title be issued to him claiming ownership of the
extraterritorial service under Sec. 15 of the Rule on Summons where service may be land. Where should A file the case? Explain. (5%) SUGGESTED ANSWER: A should file the
effected “in any other manner the court may deem sufficient.” Summons; Valid Service case in Marikina, the place where the real property subject matter of the case is situated.
(2013) No.I. Alfie Bravo filed with the Regional Trial Court of Caloocan, a complaint for a An action for specific performance would still be considered a real action where it seeks
sum of money against Charlie Delta. The claim is for Php1.5Million. The complaint alleges the conveyance or transfer of real property, or ultimately, the execution of deeds of
that Charlie borrowed the amount from Alfie and duly executed a promissory note as conveyance of real property. (Gochan vs. Gochan, 423 Phil. 491, 501 [2001]; Copioso vs.
evidence of the loan. Charlie’s office secretary, Esther, received the summons at Charlie’s Copioso, 391 SCRA 325 [2002]) Venue; Real Actions (2008) No.III. (a) Angela, a resident of
office. Charlie failed to file an answer within the required period, and Alfie moved to Quezon City, sued Antonio, a resident of Makati City before the RTC of Quezon City for
declare Charlie in default and to be allowed to present evidence ex parte. Ten days later, the reconveyance of two parcels of land situated in Tarlac and Nueva Ecija, respectively.
Charlie filed his verified answer, raising the defense of full payment with interest. (A) May her action prosper? SUGGESTED ANSWER: No, the action will not prosper because it
Was there proper and valid service of summons on Charlie? (3%) SUGGESTED ANSWER: was filed in the wrong venue. Since the action for reconveyance is a real action, it should
No. There is no showing that earnest efforts were exerted to personally serve the have been filed separately in Tarlac and Nueva Ecija, where the parcels of land are
summons on the defendant before substituted service was resorted to: hence, the located (Section 1, Rule 4; United Overseas Bank of the Philippines vs. Rosemoore Mining
service of summons was improper. In an action strictly in personam like a complaint for a & Development Corp., et al., G.R. nos. 159669 & 163521, March 12, 2007). However, an
sum of money, personal service on the defendant is the preferred mode of service, that improperly laid venue may be waived, if not pleaded in a timely motion to dismiss (Sec.
is, by handing a copy of the summons to the defendant in person. If defendant, for 4, Rule 4). Without a motion to dismiss on the ground of improperly laid venue, it would
excusable reasons, cannot be served with the summons within a reasonable period, then be incorrect for the Court to dismiss the action for improper venue. (b) Assuming that
substituted service can be resorted to (Manotoc vs. Court of Appeals, G.R. No. 130974, the action was for foreclosure on the mortgage of the same parcels of land, what is the
August 16, 2006, Velasco, J.). Otherwise stated, it is only when the defendant cannot be proper venue for the action? SUGGESTED ANSWER: The action must be filed in any
served personally within a reasonable time that a substituted service may be made. province where any of the lands involved lies – either in tarlac or in Nueva Ecija, because
Impossibility of prompt service should be shown by stating the efforts made to Remedial the action is a real action (BPI vs. Green, 57 Phil. 712; Sec. 1, Rule 4; Bank of America vs.
Law Q&As (2007-2013) hectorchristopher@yahoo.com JayArhSals “Never Let The Odds American Realty Corp., G.R. No. 133876, 29 December 1999). However, an improperly
Keep You From Pursuing What You Know In Your Heart You Were Meant To Do.”-Leroy laid venue may be waived if not pleaded as a ground for dismissal (Sec. 4, Rule 4). [Note:
Satchel Paige Page 39 of 198 find the defendant personally and the fact that such efforts The question is the same as 2009 Remedial Law Bar question No.II. See Remedial Law
failed. This statement should be made in the proof of service (Galura vs. Math-Agro Q&As (2007-2013) hectorchristopher@yahoo.com JayArhSals “Never Let The Odds Keep
Corporation, G.R. No. 167230, August 14, 2009, 1st Division, Carpio, J.). ALTERNATIVE You From Pursuing What You Know In Your Heart You Were Meant To Do.”-Leroy Satchel
ANSWER: Yes. If earnest efforts were exerted to serve the summons in persons but the Paige Page 42 of 198 Jurisdiction: Jurisdiction; RTC, Supra – JayArhSals] Provisional
same proved futile, then substituted service through defendant‟s secretary is valid. In Remedies (Rules 57-61) Attachment; Bond (2008) No.VI. After his properties were
Gentle Supreme Philippines, Inc. vs. Ricardo Consulta, G.R. No. 183182, September 1, attached, defendant Porfirio filed a sufficient counterbond. The trial court discharged the
2010, the Supreme Court held that it is not necessary that the person in charge of the attachment. Nonetheless, Porfirio suffered substantial prejudice due to the unwarranted
defendant‟s regular place of business be specifically authorized to receive the summons. attachment. In the end, the trial court rendered a judgment in Porfirio’s favor by
It is enough that he appears to be in charge. Consequently, the substituted service of ordering the plaintiff to pay damages because the plaintiff was not entitled to the
summons to the defendant‟s secretary in the office is valid. (B) If declared in default, attachment. Porfirio moved to charge the plaintiff’s attachment bond. The plaintiff and
what can Charlie do to obtain relief? (4%) SUGGESTED ANSWER: If Charlie is declared in his sureties opposed the motion, claiming that the filing of the counterbond had relieved
default, he has the following remedies to wit: 1) he may, at any time after discovery of the plaintiff’s attachment bond from all liability for the damages. Rule on Porfirio’s
the default but before judgment, file a motion, under oath, to set aside the order of motion. SUGGESTED ANSWER: Porfirio‟s motion to charge the plaintiff‟s attachment
default on the ground that his failure to answer was due to fraud, accident, mistake, or bond is proper. The filing of the counterbond by the defendant does not mean that he
excusable neglect, and that he has a meritorious defense; 2) if judgment has already has waived his right to proceed against the attachment bond for damages. Under the law
been rendered when he discovered the default, but before the same has become final (Sec. 20, Rule 57), an application for damages on account of improper, irregular, or
and executor, he may file a motion for new trial under Section 1(a) of Rule 37: 3) if he excessive attachment is allowed. Such damages may be awarded only after proper
hearing and shall be included in the judgment on the main case. Moreover, nothing shall for preliminary attachment with the court. The Court issues the preliminary attachment
prevent the party against whom the attachment was issued from recovering in the same after A files a bond. While summons on B was yet unserved, the sheriff attached B's
action the damages awarded to him from any property of the attaching party not exempt properties. Afterwards, summons was duly served on B. 8 moves to lift the attachment.
from execution should the bond or deposit given by the latter be insufficient or fail to Rule on this. (5%) SUGGESTED ANSWER: I will grant the motion since no levy on
fully satisfy the award. (D.M. Wenceslao & Associates, Inc. vs. Readycon Trading & attachment pursuant to the writ shall be enforced unless it is preceded or
Construction Corp., G.R. No. 154106, 29 June 2004). Attachment; Garnishment (2008) contemporaneously accompanied by service of summons. There must be prior or
No.VII. (a) The writ of execution was returned unsatisfied. The judgment obligee contemporaneous service of summons with the writ of attachment. (Rule 57, Sec.5,
subsequently received information that a bank holds a substantial deposit belonging to Rules of Court). Injunction; Preliminary Injunction (2009) No.I.C. A suit for injunction is an
the judgment obligor. If you are the counsel of the judgment oblige, what steps would action in rem. SUGGESTED ANSWER: FALSE. A suit for injunction is an action in
you take to reach the deposit to satisfy the judgment? SUGGESTED ANSWER: Remedial personam. In the early case of Auyong Hian vs. Court of Tax Appeals [59 SCRA 110
Law Q&As (2007-2013) hectorchristopher@yahoo.com JayArhSals “Never Let The Odds [1974]), it was held that a restraining order like an injunction, operates upon a person. It
Keep You From Pursuing What You Know In Your Heart You Were Meant To Do.”-Leroy is granted in the exercise of equity of jurisdiction and has no in rem effect to invalidate
Satchel Paige Page 43 of 198 I will ask for a writ of garnishment against the deposit in the an act done in contempt of an order of the court except where by statutory
bank (Sec. 9[c], Rule 57). ALTERNATIVE ANSWER: I shall move the court to apply to the authorization, the decree is so framed as to act in rem on property. (Air Materiel Wing
satisfaction of the judgment the property of the judgment obligor or the money due him Savings and Loan Association, Inc. vs. manay, 535 SCRA 356 [2007]). Special Civil Actions
in the hands of another person or corporation under Sec. 40, Rule 39. (b) If the bank (Rules 62-71) Certiorari; Petition for Certiorari, Rule 65 (2012) No.I. (a) After an
denies holding the deposit in the name of the judgment obligor but your client’s information for rape was filed in the RTC, the DOJ Secretary, acting on the accused's
informant is certain that the deposit belongs to the judgment obligor under an assumed petition for review, reversed the investigating prosecutor's finding of probable cause.
name, what is your remedy to reach the deposit? SUGGESTED ANSWER: I will move for Upon order of the Remedial Law Q&As (2007-2013) hectorchristopher@yahoo.com
the examination under oath of the bank as a debtor of the judgment debtor (Sec. 37, JayArhSals “Never Let The Odds Keep You From Pursuing What You Know In Your Heart
Rule 39). I will ask the court to issue an Order requiring the judgment obligor, or the You Were Meant To Do.”-Leroy Satchel Paige Page 46 of 198 DOJ Secretary, the trial
person who has property of such judgment obligor, to appear before the court and be prosecutor filed a Motion to Withdraw Information which the judge granted. The order
examined in accordance with Secs. 36 and 37 of the Rules of Court for the complete of the judge stated only the following: "Based on the review by the DOJ Secretary of the
satisfaction of the judgment award (Co vs. Sillador, A.M. No. P-07- 2342, 31 August findings of the investigating prosecutor during the preliminary investigation, the Court
2007). ALTERNATIVE ANSWER: The judgment oblige may invoke the exception under Sec. agrees that there is no sufficient evidence against the accused to sustain the allegation in
2 of the Secrecy of Bank Deposits Act. Bank Deposits may be examined upon order of a the information. The motion to withdraw Information is, therefore, granted." If you were
competent court in cases if the money deposited is the subject matter of the litigation the private prosecutor, what should you do? Explain. (5%) SUGGESTED ANSWER: If I were
(R.A. 1405). Attachment; Kinds of Attachment (2012) No.IX.B. Briefly the private prosecutor, I would file a petition for certiorari under Rule 65 with the Court
discuss/differentiate the following kinds of Attachment: preliminary attachment, of Appeals (Cerezo vs. People, G.R. No.185230, June 1, 2011). It is well-settled that when
garnishment, levy on execution, warrant of seizure and warrant of distraint and levy. the trial court is confronted with a motion to withdraw and Information (on the ground
(5%) SUGGESTED ANSWER: PRELIMINARY ATTACHMENT- is a provisional remedy under of lack of probable cause to hold the accused for trial based on resolution of the DOJ
Rule 57 of the Rules of Court. it may be sought at the commencement of an action or at Secretary), the trial court has the duty to make an independent assessment of the merits
any time before entry judgment where property of an adverse party may be attached as of the motion. It may either agree or disagree with the recommendation of the
security for the satisfaction of any judgment, where this adverse party is about to depart Secretary. Reliance alone on the resolution of the Secretary would be an abdication of
from the Philippines, where he has intent to defraud or has committed fraud, or is not the trial court‟s duty and jurisdiction to determine a prima facie case. The court must
found in the Philippines. An affidavit and a bond is required before the preliminary itself be convinced that there is indeed no sufficient evidence against the accused.
attachment issues. It is discharged upon the payment of a counterbond. Remedial Law Otherwise, the judge acted with grave abuse of discretion if he grants the Motion to
Q&As (2007-2013) hectorchristopher@yahoo.com JayArhSals “Never Let The Odds Keep Withdraw Information by the trial prosecutor. (Harold Tamargo vs. Romulo Awingan et.
You From Pursuing What You Know In Your Heart You Were Meant To Do.”-Leroy Satchel al. G.R. No. 177727, January 19, 2010). ALTERNATIVE ANSWER: If I were the private
Paige Page 44 of 198 GARNISHMENT- is a manner of satisfying or executing judgment prosecutor, I would file a Motion for Reconsideration of the Order of the trial court. if the
where the sheriff may levy debts, credits, royalties, commissions, bank deposits, and same has been denied, I would file a petition for review on certiorari under Rule 45 on
other personal property not capable of manual delivery that are in the control or pure question of law, which actually encompasses both the criminal and civil aspects
possession of third persons and are due the judgment obligor. Notice shall be served on thereof. The filing of the petition is merely a continuation of the appellate process.
third parties. The third party garnishee must make a written report on whether or not Certiorari; Petition for Certiorari; Contempt (2012) No.IV.B. Mr. Sheriff attempts to
the judgment obligor has sufficient funds or credits to satisfy the amount of the enforce a Writ of Execution against X, a tenant in a condominium unit, who lost in an
judgment. If not, the report shall state how much fund or credits the garnishee holds for ejectment case. X does not want to budge and refuses to leave. Y, the winning party,
the judgment obligor. Such garnish amounts shall be delivered to the judgment oblige- moves that X be declared in contempt and after hearing, the court held X guilty of
creditor (Rule 39, Sec.9 [c]). LEVY ON EXECUTION- is a manner of satisfying or executing Remedial Law Q&As (2007-2013) hectorchristopher@yahoo.com JayArhSals “Never Let
judgment where the sheriff may sell property of the judgment obligor if he is unable to The Odds Keep You From Pursuing What You Know In Your Heart You Were Meant To
pay all or part of the obligation in cash, certified bank check or any other manner Do.”-Leroy Satchel Paige Page 47 of 198 indirect contempt. If you were X's lawyer, what
acceptable to the oblige. If the obligor does not chose which among his property may be would you do? Why? (5%) SUGGESTED ANSWER: If I were X‟s Lawyer, I would file a
sold, the sheriff shall sell personal property first and then real property second. He must petition for certiorari under Rule 65. The judge should not have acted on Y‟s motion to
sell only so much of the personal and real property as is sufficient to satisfy judgment declare X in contempt. The charge of indirect contempt is initiated through a verified
and other lawful fees. (Rule 39, Sec.9 [b]). WARRANT OF SEIZURE- is normally applied for, petition. (Rule 71, Sec. 4, Rules of Court). The writ was not directed to X but to the sheriff
with a search warrant, in criminal cases. The warrant of seizure must particularly who was directed to deliver the property to Y. As the writ did not command the
describe the things to be seized. While it is true that the property to be seized under a judgment debtor to do anything, he cannot be guilty of the facts described in Rule 71
warrant must be particularly described therin and no other property can be taken which is “disobedience of or resistance to a lawful writ, process, order, judgment, or
thereunder, yet the description is required to be specific only insofar as the command any court.” the proper procedure is for the sheriff to oust X availing of the
circumstances will ordinarily allow. An application for search and seizure warrant shall be assistance of peace officers pursuant to Section 10 (c) of Rule 39 (Lipa vs. Tutaan, L-
filed with the following: (a) Any court within whose territorial jurisdiction a crime was 16643, 29 September 1983; Medina vs. Garces, L25923, July 15, 1980; Pascua vs. Heirs of
committed. (b) For compelling reasons stated in the application, any court within the Segundo Simeon, 161 SCRA 1; Patagan et. al. Vs. Panis, G.R. No. 55630, April 8, 1988).
judicial region where the crime was committed if the place of the commission of the Expropriation; Motion to Dismiss (2009) No.XIV.A. The Republic of the Philippines,
crime is known, or any court within the judicial region where the warrant shall be through the department of Public Works and Highways (DPWH) filed with the RTC a
enforced. However, if the criminal action has already been filed, the application shall complaint for the expropriation of the parcel of land owned by Jovito. The land is to be
only be made in the court where the criminal action is pending. WARRANT OF DISTRAINT used as an extension of the national highway. Attached to the complaint is a bank
AND LEVY- is remedy available to local governments and the BIR in tax cases to satisfy certificate showing that there is, on deposit with the Land Bank of the Philippines, an
deficiencies or delinquencies in inheritance and estate taxes, and real estate taxes. amount equivalent to the assessed value of the property. Then DPWH filed a motion for
Distraint is the seizure of personal property to be sold in an authorized auction sale. Levy the issuance of a writ of possession. Jovito filed a motion to dismiss the complaint on the
is the Remedial Law Q&As (2007-2013) hectorchristopher@yahoo.com JayArhSals ground that there are other properties which would better serve the purpose. (a) Will
“Never Let The Odds Keep You From Pursuing What You Know In Your Heart You Were Jovito’s motion to dismiss prosper? Explain SUGGESTED ANSWER: NO. the present Rule
Meant To Do.”-Leroy Satchel Paige Page 45 of 198 issuance of a certification by the of Procedure governing expropriation (Rule 67), as amended by the 1997 Rules of Civil
proper officer showing the name of the taxpayer and the tax, fee, charge, or penalty due Procedure, requires the defendant to file an Answer, which must be filed on or before
him. Levy is made by writing upon said certificate the description of the property upon the time stated in the summons. Defendant‟s objections and defenses should be
which levy is made. Attachment; Preliminary Attachment (2012) No.VIII.A. (a) A sues B pleaded in his Answer not in a motion. (b) As judge, will you grant the writ of possession
for collection of a sum of money. Alleging fraud in the contracting of the loan, A applies prayed for by DPWH? Explain SUGGESTED ANSWER: NO. the expropriation here is
governed by Rep. Act No. 8974 which requires Remedial Law Q&As (2007-2013) exceed P20,000.00 outside Metro Manila, or in Metro Manila, where such value does not
hectorchristopher@yahoo.com JayArhSals “Never Let The Odds Keep You From Pursuing exceed P50,000.00. Jurisdiction; Unlawful Detainer (2008) No.IV. Filomeno brought an
What You Know In Your Heart You Were Meant To Do.”-Leroy Satchel Paige Page 48 of action in the Metropolitan Trial Court (METC) of Pasay City against Marcelino pleading
198 100% payment of the zonal value of the property as determined by the BIR, to be the two causes of action. The first was a demand for the recovery of physical possession of a
amount deposited. Before such deposit is made, the national government thru the parcel of land situated in Pasay City with an assessed value of 40,000; the second was a
DPWH has no right to take the possession of the property under expropriation. Forcible claim for damages of 500,000 for Marcelino’s unlawful retention of the property.
Entry; Remedies (2013) No.V. The spouses Juan reside in Quezon City. With their lottery Marcelino filed a motion to dismiss on the ground that the total amount involved, which
winnings, they purchased a parcel of land in Tagaytay City for P100,000.00. In a recent is 540,000, is beyond the jurisdiction of the MeTC. Is Marcelino correct? SUGGESTED
trip to their Tagaytay property, they were surprised to see hastily assembled shelters of ANSWER: No, Metropolitan or Municipal trial Courts have exclusive jurisdiction over a
light materials occupied by several families of informal settlers who were not there when complaint for forcible entry and unlawful detainer regardless of the amount of the claim
they last visited the property three (3) months ago. To rid the spouses’ Tagaytay property for damages (Sec. 33 [2], B.P. 129). Remedial Law Q&As (2007-2013)
of these informal settlers, briefly discuss the legal remedy you, as their counsel, would hectorchristopher@yahoo.com JayArhSals “Never Let The Odds Keep You From Pursuing
use; the steps you would take; the court where you would file your remedy if the need What You Know In Your Heart You Were Meant To Do.”-Leroy Satchel Paige Page 51 of
arises; and the reason/s for your actions. (7%) SUGGESTED ANSWER: As counsel for 198 Also, Sec. 3, Rule 70 gives jurisdiction to the said courts irrespective of the amount of
spouses Juan, I will file a special civil action for Forcible Entry. The Rules of Court provide damages. This is the same provision in the Revised Rules of Summary Procedure that
that a person deprived of the possession of any land or building by force, intimidation, governs all ejectment cases (Sec. 1[A][1], Revised Rule on Summary Procedure). The
threat, strategy, or stealth may at anytime within 1 year after such withholding of Rule, however, refers to the recovery of a reasonable amount of damages. In this case,
possession bring an action in the proper Municipal Trial Court where the property is the property is worth only P40,000, but the claim for damages is P500,000. Mandamus
located. This action which is summary in nature seeks to recover the possession of the (2012) No.X.B. A files a Complaint against 8 for recovery of title and possession of land
property from the defendant which was illegally withheld by the latter (Section 1, Rule situated in Makati with the RTC of Pasig. B files a Motion to Dismiss for improper venue.
70, Rules of Court). An ejectment case is designed to restore , through summary The RTC Pasig Judge denies B's Motion to Dismiss, which obviously was incorrect.
proceedings, the physical possession of any land or building to one who has been illegally Alleging that the RTC Judge "unlawfully neglected the performance of an act which the
deprived of such possession, without prejudice to the settlement of parties‟ opposing law specifically enjoins as a duty resulting from an office", 8 files a Petition for
claims of juridical possession in an appropriate proceedings (Heirs of Agapatio T. Olarte Mandamus against the judge. Will Mandamus lie? Reasons. (3%) SUGGESTED ANSWER:
and Angela A. Olarte et. al. vs. Office of the President of the Philippines et al., G.R. No. No, mandamus will not lie. The proper remedy is a petition for prohibition. (Serana vs.
177995, June 15, 2011, Villarama, Jr., J.). In Abad vs. Farrales, G.R. No. 178635, April 11, Sandiganbayan, G.R. No. 162059, January 22, 2008). The dismissal of the case based on
2011, the Supreme Court held that two allegations are indispensable in actions for improper venue is not a ministerial duty. Mandamus does not lie to comple the
forcible entry to enable first level courts to acquire jurisdiction over them: first, that the performance of a discretionary duty. (Nilo Paloma vs. Danilo Mora, G.R. No. 157783,
plaintiff had prior physical possession of the property; and, second, that the defendant September 23, 2005). Partition; Non-joinder (2009) No.XV.A. Florencio sued Guillermo
deprived him of such possession by means of force, Remedial Law Q&As (2007-2013) for partition of a property they owned in common. Guillermo filed a motion to dismiss
hectorchristopher@yahoo.com JayArhSals “Never Let The Odds Keep You From Pursuing the complaint because Florencio failed to implead Herando and Inocencio, the other co-
What You Know In Your Heart You Were Meant To Do.”-Leroy Satchel Paige Page 49 of owners of the property. As Judge, will you grant the motion to dimiss? Explain. (3%)
198 intimidation, threats, strategy, or stealth. However, before instituting the said SUGGESTED ANSWER: NO, because the non-joinder of parties is not a ground for
action, I will first endeavour to amicably settle the controversy with the informal settlers dismissal of action (Rule 3, Sec. 11). The motion to dismiss should be denied. Unlawful
before the appropriate Lupon or Barangay Chairman. If there is no agreement reached Detainer; Preliminary Conference (2007) No. VIII.(a) X files an unlawful detainer case
after mediation and conciliation under the Katarungang Pambarangay Law, I will secure a against Y before the appropriate Metropolitan Trial Court. In his answer, Y avers as a
certificate to file action and file the complaint for ejectment before the MTC of Tagaytay special and affirmative defense that he is a tenant of X’s deceased father in whose name
City where the property is located since ejectment suit is a real action regardless of the the property remains Remedial Law Q&As (2007-2013) hectorchristopher@yahoo.com
value of the property to be recovered or claim for unpaid rentals (BP 129 and RULE 4, JayArhSals “Never Let The Odds Keep You From Pursuing What You Know In Your Heart
Section 1 of the Revised Rules on Civil Procedure). In the aforementioned complaint, I You Were Meant To Do.”-Leroy Satchel Paige Page 52 of 198 registered. What should the
will allege that Spouses Juan had prior physical possession and that the dispossession court do? Explain briefly. (5%) SUGGESTED ANSWER: The court should hold a preliminary
was due to force, intimidation and stealth. The complaint will likewise show that the conference not later than thirty (30) days after the defendant‟s Answer was filed, since
action was commenced within a period of one (10 year from unlawful deprivation of the case is governed by summary procedure under Rule 70, Rules of Court, where a
possession, and that the Spouses Juan is entitled to restitution of possession together Reply is not allowed. The court should receive evidence to determine the allegations of
with damage costs. Foreclosure; Certification Against Non Forum Shopping (2007) No.X. tenancy. If tenancy had in fact been shown to be the real issue, the court should dismiss
(a) RC filed a complaint for annulment of the foreclosure sale against Bank V. In its the case for lack of jurisdiction. If it would appear that Y‟s occupancy of the subject
answer, Bank V set up a counterclaim for actual damages and litigation expenses. RC property was one of agricultural tenancy, which is governed by agrarian laws, the court
filed a motion to dismiss the counterclaim on the ground that Bank V’s Answer with should dismiss the case because it has no jurisdiction over agricultural tenancy cases.
Counterclaim was not accompanied by a certification against forum shopping. Rule. (5%) Defendant‟s allegation that he is a “tenant” of plaintiff‟s deceased father suggests that
SUGGESTED ANSWER: A certification against forum shopping is required only in initiatory the case is one of landlordtenant relation and therefore, not within the jurisdiction of
pleadings. In this case, the counterclaim pleaded in the defendant‟s Answer appears to ordinary courts. Unlawful; Detainer; Prior Possession (2008) No.XVII. Ben sold a parcel of
have arisen from the plaintiff‟s complaint or compulsory in nature and thus, may not be land to Del with right to repurchase within one(1) year. Ben remained in possession of
regarded as an initiatory pleading. The absence thereof in the Bank‟s Answer is not a the property. When Ben failed to repurchase the same, title was consolidated in favor of
fatal defect. Therefore, the motion to dismiss on the ground raised lacks merit and Del. Despite demand, Ben refused to vacate the land, constraining Del to file a complaint
should be denied (UST v. Suria, 294 SCRA 382 [1998]). On the other hand, if the for unlawful detainer. In his defense, Ben averred that the case should be dismissed
counterclaim raised by the defendant Bank‟s Answer was not predicated on the because Del had never been in possession of the property. Is Ben correct? SUGGESTED
plaintiff‟s claim or cause of action, it is considered a permissive counterclaim. In which ANSWER: No, for unlawful detainer, the defendant need not have been in prior
case, tit would partake an initiatory pleading which requires a certification against forum possession of the property. This is upon the theory that the vendee steps into the shoes
shopping. Correspondingly, the motion to dismiss Remedial Law Q&As (2007-2013) of the vendor and succeeds to his rights and interests. In contemplation of law, the
hectorchristopher@yahoo.com JayArhSals “Never Let The Odds Keep You From Pursuing vendee‟s possession is that of the vendor‟s (Maninang vs. C.A., G.R. No. 121719, 16
What You Know In Your Heart You Were Meant To Do.”-Leroy Satchel Paige Page 50 of September 1999; Dy Sun vs. Brillantes, 93 Phil. 175 [1953]); (Pharma Industries, Inc., vs.
198 based on lack of the required certificate against forum shopping should be granted. Pajarillaga, G.R. No. L-53788, 17 October 1980). Special Proceedings (Rules 72-109)
Jurisdiction; Unlawful Detainer (2010) No.III. Anabel filed a complaint against B for Absentee; Declaration of Absence vs. Declaration of Presumptive Death (2009) Remedial
unlawful detainer before the Municipal Trial Court (MTC) of Candaba, Pampanga. After Law Q&As (2007-2013) hectorchristopher@yahoo.com JayArhSals “Never Let The Odds
the issues had been joined, the MTC dismissed the complaint for lack of jurisdiction after Keep You From Pursuing What You Know In Your Heart You Were Meant To Do.”-Leroy
noting that the action was one for accion publiciana. Anabel appealed the dismissal to Satchel Paige Page 53 of 198 No.V. Frank and Gina were married on June 12, 1987 in
the RTC which affirmed it and accordingly dismissed her appeal. She elevates the case to Manila. Barely a year after the wedding, Frank exhibited a violent temperament, forcing
the Court of Appeals, which remands the case to the RTC. Is the appellate court correct? Gina, for reasons of personal safety, to live with her parents. A year thereafter, Gina
Explain. (3%) SUGGESTED ANSWER: Yes, the Court of Appeals is correct in remanding the found employment as a domestic helper in Singapore, where she worked for ten
case to the RTC for the latter to try the same on the merits. The RTC, having jurisdiction consecutive years. All the time she was abroad, Gina had absolutely no communications
over the subject matter of the case appealed from MTC should try the case on the merits with Frank, nor did she hear any news about him. While in Singapore, Gina met and fell
as if the case was originally filed with it, and not just to affirm the dismissal of the case. in love with Willie. On July 4, 2007, Gina filed a petition with the RTC of manila to declare
R.A. No. 7691, however, vested jurisdiction over specified accion publiciana with courts Frank presumptively dead, so that she could marry Willie. The RTC granted Gina’s
of the first level (Metropolitan Trial Courts, Municipal Trial Courts, and Municipal Circuit petition. The office of the Solicitor General (OSG) filed a notice of Appeal with the RTC,
Trial Courts) in cases where the assessed value of the real property involved does not stating that it was appealing the decision of the Court of Appeals on questions of fact and
law. (a) Is a petition for declaration of Presumptive Death a special proceeding? hectorchristopher@yahoo.com JayArhSals “Never Let The Odds Keep You From Pursuing
SUGGESTED ANSWER: No. the petition for Declaration of Presumptive Death provided in What You Know In Your Heart You Were Meant To Do.”-Leroy Satchel Paige Page 56 of
Art. 41 of the “Family Code” is not the special proceeding governing absentees under 198 (b) Under the Rules of criminal procedure? SUGGESTED ANSWER: Under the Rules of
Rule 107 of the Rules of Court whose rules of procedure will not be followed (Republic Criminal Procedure, Rule 114, Sec. 24 clearly prohibits the grant of bail after conviction
vs. C.A., 458 SCRA [2005]). Said petition for Declaration of Presumptive Death under by final judgment and after the convict has started to serve sentence. In the present
Article 41 of the Family Code is a summary proceeding, authorized for purposes only of case, Alma had already started serving her sentence. She cannot, therefore, apply for bail
remarriage of the present spouse, to avoid incurring the crime of bigamy. Nonetheless, it (Peo. vs. Fitzgerald, G.R. No. 149723, 27 October 2006). Habeas Corpus; Jurisdiction;
is in the nature of a special proceeding, being an application to establish a status or a Sandiganbayan (2009) No.XI.C. In the exercise of its original jurisdiction, the
particular fact in court. ALTERNATIVE ANSWER: A petition for declaration of presumptive Sandiganbayan may grant petitions for the issuance of a writ of habeas corpus.
death may be considered a special proceeding, because it is so classified in the Rules of SUGGESTED ANSWER: FALSE. The Sandiganbayan may grant petitions for Habeas corpus
Court (Rule 107, Rules of Court), as differentiated from an ordinary action which is only in aid of its appellate jurisdiction (R.A. 7975, as amended by R.A 8249), not in the
adversarial. It is a mere application or proceeding to establish the status of a party or a exercise of “original” jurisdiction. Letters of Administration; Interested Person (2008)
particular fact, to viz: that a person has been unheard of for a long time and under such No.XVIII. Domencio and Gen lived without benefit of marriage for 20 years, during which
circumstance that he may be presumed dead. (b) As the RTC judge who granted Gina’s time they purchased properties together. After Domencio died without a will, Gen filed a
petition, will you give due course to the OSG’s notice of appeal? SUGGESTED ANSWER: petition for letters of administration. Domencio’s siblings opposed the same on the
NO. Appeal is not a proper remedy since the decision is immediately final and executor ground that Gen has no legal personality. Decide. SUGGESTED ANSWER: A petition for
upon notice to the parties under Art. 247 of the Family Remedial Law Q&As (2007-2013) letters of administration may be filed by any “interested person” (Sec. 2, Rule 79, Rules
hectorchristopher@yahoo.com JayArhSals “Never Let The Odds Keep You From Pursuing of Court). Gen would be considered an interested person if she was not married to
What You Know In Your Heart You Were Meant To Do.”-Leroy Satchel Paige Page 54 of Domenico, because she can claim coownership of the properties left by him under their
198 Code(Republic vs Bermudez-Lorino, 449 SCRA 57 [2005]). The OSG may assail RTC‟s property regime of a union without marriage under conditions provided in the Family
grant of the petition only on the premise of grave abuse of discretion amounting to lack Code 9Arts. 147- 148, Family Code; San Luis vs. San Luis, G.R. No. 133743, February 6,
or excess of jurisdiction. The remedy should be by certiorari under Rule 65 of the Rules of 2007). Probate of Will (2010) No.XIV. Czarina died single. She left all her properties by
Court. Cancellation or Correction; Notice (2007) No.VII. (a) B files a petition for will to her friend Duqueza. In the will, Czarina stated that she did not recognize Marco as
cancellation of the birth certificate of her daughter R on the ground of the falsified an adopted son because of his disrespectful conduct towards her. Duqueza soon
material entries therein made by B’s husband as the informant. The RTC sets the case for instituted an action for probate of Czarina’s will. Marco, on the other hand, instituted
hearing and directs the publication of the order for hearing and directs the publication of intestate proceedings. Both actions were consolidated before the RTC of Pasig. On
the order once a week for three consecutive weeks in a newspaper of general circulation. Remedial Law Q&As (2007-2013) hectorchristopher@yahoo.com JayArhSals “Never Let
Summons was service on the Civil Registrar but there was no appearance during the The Odds Keep You From Pursuing What You Know In Your Heart You Were Meant To
hearing. The RTC granted the petition. R filed a petition for annulment of judgment Do.”-Leroy Satchel Paige Page 57 of 198 motion of Marco, Duqueza’s petition was
before the Court of Appeals, saying that she was not notified of the petition and hence, ordered dismissed on the ground that the will is void for depriving him of his legitime.
the decision was issued in violation of due process. B opposed saying that the publication Argue for Duqueza. (5%) SUGGESTED ANSWER: The petition for probate of Czarina‟s will,
of the court order was sufficient compliance with due process. Rule. (5%) SUGGESTED as filed by Duquesa should not be dismissed on mere motion of Marco who instituted
ANSWER: R‟s petition for annulment of judgment before the Court of Appeals should be intestate proceedings. The law favors testacy over intestacy, hence, the probate of the
granted. Although there was publication of the court order acting the petition to cancel will cannot be dispensed with. (See Sec. 5, Rule 75) Thus, unless the will – which shows
the birth certificate, reasonable notice still has to be served on R as she has an interest the obvious intent to disinherit Marco – is probated, the right of a person to dispose of
affected by the cancellation. (Sec. 3 and 4, Rule 108, Rules of Court) She is an his property may be rendered nugatory (See Seanio vs. Reyes, G.R. Nos. 140371-72, Nov.
indispensable party (Republic v. Benemerito, 425 SCRA 488 [2004]), and notice has to be 27, 2006). Besides, the authority of the probate court is generally limited only to a
served on her, not for the purpose of vesting the court with jurisdiction, but to comply determination of the extrinsic validity of the will. In this case, Marco questioned the
with the requirements of fair play and due process (Ceruila v. Delantar, 477 SCRA 134 intrinsic validity of the will. Probate of Will (2007) No.VIII. (b) The heirs of H agree among
[2005]). ALTERNATIVE ANSWER: The petition for annulment of judgment should not be themselves that they will honor the division of H’s estate as indicated in her Last Will and
granted. While R is an indispensable party, it has been held that the failure to service Testament. To avoid the expense of going to court in a Petition for Probate of the Will,
notice on indispensable parties is cured by the publication made because the action is can they instead execute an Extrajudicial Settlement Agreement among themselves?
one in rem (Alba v. Court of Appeals, 465 SCRA 495 [2005]; Barco v. Court of Appeals, Explain briefly. (5%) SUGGESTED ANSWER: The heirs of H cannot validly agree to resort
420 SCRA 39 [2005]). Habeas Corpus (2007) No.IV. Husband H files a petition for to extrajudicial settlement of his estate and do away with the probate of H‟s last will and
declaration of nullity of marriage before the RTC of Pasig City. Wife W files a petition for testament. Probate of the will is mandatory (Guevarra v. Guevarra, 74 Phil. 479 [1943]).
Remedial Law Q&As (2007-2013) hectorchristopher@yahoo.com JayArhSals “Never Let The policy of the law is to respect the will of the testator as manifested in the other
The Odds Keep You From Pursuing What You Know In Your Heart You Were Meant To dispositions in his last will and testament, insofar as they are not contrary to law, public
Do.”-Leroy Satchel Paige Page 55 of 198 habeas corpus before the RTC of Pasay City, morals and public policy. Extrajudicial settlement of an estate of a deceased is allowed
praying for custody over their minor child. H files a motion to dismiss the wife’s petition only when the deceased left no last will and testament and all debts, if any, are paid
on the ground of the pendency of the other case. Rule. SUGGESTED ANSWER: The (Rule 74, Sec. 1, Rules of Court). Probate of Will; Jurisdictional Facts (2012) No.X.C. What
motion to dismiss the petition for habeas corpus should be granted to avoid multiplicity are the jurisdictional facts that must be alleged in a petition for probate of a will? How
of suits. The question of who between the spouses should have custody of their minor do you bring before the court these jurisdictional facts? (3%) SUGGESTED ANSWER: The
child could also be determined in the petition for declaration of nullity of their marriage jurisdictional facts in a petition for probate are: (1) that a person died Remedial Law
which is already pending in the RTC of Pasig City. In other words, the petition filed in Q&As (2007-2013) hectorchristopher@yahoo.com JayArhSals “Never Let The Odds Keep
Pasig City, praying for custody of the minor child is unnecessary and violates only the You From Pursuing What You Know In Your Heart You Were Meant To Do.”-Leroy Satchel
cardinal rules of procedure against multiplicity of suits. Hence, the latter suit may be Paige Page 58 of 198 leaving a will; (2) in case of a resident, that he resided within the
abated by a motion to dismiss on the ground of litis pendentia (Yu v. Yu, 484 SCRA 485 territorial jurisdiction of the court; and (3) in the case of a non-resident, that he left an
[2006]). Habeas Corpus; Bail (2008) No.XIX. After Alma had started serving her sentence estate within such territorial jurisdiction. The jurisdictional facts shall be contained in a
for violation of BP 22, she filed a petition for a writ of habeas corpus, citing Vaca vs CA petition for allowance of will. Probate of Will; Application of Modes of Discovery (2008)
where the sentence of imprisonment of a party found guilty of violation of BP 22 was No.XIII. An heir/oppositor in a probate proceeding filed a motion to remove the
reduced to a fine equal to double the amount of the check involved. She prayed that her administrator on the grounds of neglect of duties as administrator and absence from the
sentence be similarly modified and that she be immediately released from detention. In country. On his part the heir/oppositor served written interrogatories to the
the alternative, she prayed that pending determination on whether the Vaca ruling administrator preparatory to presenting the latter as a witness. The administrator
applies to her, she be allowed to post bail pursuant to Rule 102, Sec. 14, which provides objected, insisting that the modes of discovery apply only to ordinary civil actions, not
that if a person is lawfully imprisoned or restrained on a charge of having committed an special proceedings. Rule on the matter. SUGGESTED ANSWER: No, the administrator is
offense not punishable by death, he may be admitted to bail in the discretion of the not correct. Modes of discovery apply also to special proceedings. Sec. 2, Rule 72 states
court. accordingly, the trial court allowed Alma to post bail and then ordered her release. that in the absence of special provisions, the rules provided for in ordinary actions shall
In your opinion, is the order of the trial court correct – (a) Under Rule 102? SUGGESTED be, as far as practicable, applicable in special proceedings. Probate of Will: Will Outside
ANSWER: No, Alma, who is already convicted by final judgment, cannot be entitled to of the Philippines (2010) No.XV. Pedrillo, a Fil-Am permanent resident of Los Angeles,
bail under Sec. 14, Rule 102. The provision presupposes that she had not been convicted California at the time of his death, bequeathed to Winston a sum of money to purchase
as yet. It provides that if she is lawfully imprisoned or restrained for an offense not an annuity. Upon Pedrillo’s demise, his will was duly probated in Los Angeles and the
punishable by death, she may be recommitted to imprisonment or admitted to bail in specified sum in the will was in fact used to purchase an annuity with XYZ of Hong Kong
the discretion of the court or judge (Sec. 14, Rule 102; Celeste vs. People, 31 SCRA 391; so that Winston would receive the equivalent of US$1,000 per month for the next 15
Vicente vs. Judge Majaducon, A.M. No. RTJ-02- 1698, 23 June 2005; San Pedro vs. Peo, years. Wanting to receive the principal amount of the annuity, Winston files for the
G.R. No. 133297, 15 August 2002). Remedial Law Q&As (2007-2013) probate of Pedrillo’s will in the Makati RTC. As prayed for, the court names Winston as
administrator of the estate. Winston now files in the Makati RTC a motion to compel XYZ independently of the criminal prosecution and shall require only a preponderance of
to account for all sums in its possession forming part of Pedrillo’s estate. Rule on the evidence. Section 3 of Rule 111 allows the filing of an independent civil action by the
motion. (5%) SUGGESTED ANSWER: The motion should be denied. Makati RTC has no offended party based on Article 33 and 2176 of the New Civil Code. The different
jurisdiction over XYZ of hongkong. The letters of administration Remedial Law Q&As approaches that the plaintiff can pursue in this type of action are as follows: (a) File the
(2007-2013) hectorchristopher@yahoo.com JayArhSals “Never Let The Odds Keep You independent civil action and prosecute the criminal case separately. (b) File the
From Pursuing What You Know In Your Heart You Were Meant To Do.”-Leroy Satchel independent civil action without filing the criminal case. (c) File the criminal case without
Paige Page 59 of 198 granted to Winston only covers all Pedrillo‟s estate in the need of reserving the independent civil action. Aside from the testimony of Gary, the
Philippines. (Rule 77, Sec. 4) This cannot cover the annuities in Hongkong. At the outset, pieces of evidence that would be required in an independent civil action are the medical
Makati RTC should not have taken cognizance of the petition filed by Winston, because report and certificate regarding the injuries sustained by Gary, hospital and medical bills
the will does not cover any property of Pedrillo located here in the Philippines. including receipt of payments made police report and proof of the extent of damage
Settlement of Estate (2010) No.XVI. Sal Mineo died intestate, leaving a P1 billion estate. sustained by his car and the Affidavit of witnesses who saw Horace using his cellular
He was survived by his wife Dayanara and their five children. Dayanara filed a petition for phone at the time the incident happened. I will also present proof of employment of
the issuance of letters of administration. Charlene, one of the children, filed an Gary such as payslip in order to prove Remedial Law Q&As (2007-2013)
opposition to the petition, alleging that there was neither an allegation nor genuine hectorchristopher@yahoo.com JayArhSals “Never Let The Odds Keep You From Pursuing
effort to settle the estate amicably before the filing of the petition. Rule on the What You Know In Your Heart You Were Meant To Do.”-Leroy Satchel Paige Page 62 of
opposition. (5%) SUGGESTED ANSWER: The opposition should be overruled for lack of 198 that he was gainfully employed at the time of the mishap, and as a result of the
merit. The allegation that there was a genuine effort to settle the estate amicably before injuries he suffered, he was not able to earn his usual income thereof. I will also present
the filing of the petition is not required by the Rules. Besides, a petition for issuance of the attending Doctor of Gary to corroborate and authenticate the contents of the
letters of administration may be contested on either of two grounds : (1) the medical report and abstract thereof. The evidence required to hold defendant Horace
incompetency of the person for whom letters are prayed therein; and (2) the liable is only preponderance of evidence. The types of defenses that may be raised
contestant‟s own right to the administration. (Sec. 4, Rule 9). Settlement of Estate (2009) against this action are fortuitous event, force majeure or acts of God. The defendant can
No.XVIII. Pinoy died without a will. His wife, Rosie and three children executed a deed of also invoke contributory negligence as partial defense. Moreover, the defendant can
extrajudicial settlement of his estate. The deed was properly published and registered raise the usual defenses that the: (a) plaintiff will be entitled to double compensation or
with the Office of the Register of Deeds. Three years thereafter, Suzy appeared, claiming recovery, and (b) defendant will be constrained to litigate twice and therefore suffer the
to be the illegitimate child of Pinoy. She sought to annul the settlement alleging that she cost of litigation twice. Actions; Commencement of an Action; Party (2013) No.II. Yvonne,
was deprived of her rightful share in the estate. Rosie and the Three Children contended a young and lonely OFW, had an intimate relationship abroad with a friend, Percy.
that (1) the publication of the deed constituted constructive notice to the whole world, Although Yvonne comes home to Manila every six months, her foreign posting still left
and should therefore bind Suzy; and (2) Suzy’s action had already prescribed. Are Rosie her husband Dario lonely so that he also engaged in his own extramarital activities. In
and the Three Children Correct? Explain. SUGGESTED ANSWER: NO, the contention is not one particularly exhilarating session with his girlfriend, Dario died. Within 180 days from
correct. Suzy can file a complaint to annul the extrajudicial settlement and she can Dario’s death, Yvonne gives birth in Manila to a baby boy. Irate relatives of Dario
recover what is due her as such heir if her status as an illegitimate child of the deceased contemplate criminally charging Yvonne for adultery and they hire your law firm to
has been established. The Remedial Law Q&As (2007-2013) handle the case. (A) Is the contemplated criminal action a viable option to bring? (3%)
hectorchristopher@yahoo.com JayArhSals “Never Let The Odds Keep You From Pursuing SUGGESTED ANSWER: No. Section 5 of Rule 110 provides that the crimes of adultery and
What You Know In Your Heart You Were Meant To Do.”-Leroy Satchel Paige Page 60 of concubinage shall not be prosecuted except upon complaint filed by the offended
198 publication of the settlement does not constitute constructive notice to the heirs spouse. Since the offended spouse is already dead, then the criminal action for Adultery
who had no knowledge or did not take part in it because the same was notice after the as contemplated by offended party‟s relatives is no longer viable. Moreover, it appears
fact of execution. The requirement of publication is intended for the protection of that the adulterous acts of Yvonne were committed abroad. Hence, the contemplated
creditors and was never intended to deprive heirs of their lawful participation in the criminal action is not viable as the same was committed outside of the Philippine courts.
decedent‟s estate. She can file the action therefor within four (4) years after the (B) Is a civil action to impugn the paternity of the baby boy feasible, and if so, in what
settlement was registered. Criminal Procedure (Rules 110-127) Actions; Commencement proceeding may such issue be determined? (5%) SUGGESTED ANSWER: Remedial Law
of an Action (2012) No.V. X was arrested, en flagrante, for robbing a bank. After an Q&As (2007-2013) hectorchristopher@yahoo.com JayArhSals “Never Let The Odds Keep
investigation, he was brought before the office of the prosecutor for inquest, but You From Pursuing What You Know In Your Heart You Were Meant To Do.”-Leroy Satchel
unfortunately no inquest prosecutor was available. May the bank directly file the Paige Page 63 of 198 Yes, under Article 171 of the Familyy Code, the heirs of the husband
complaint with the proper court? If in the affirmative, what document should be filed? may imougn the filiation of the child in the following cases: a) If the husband should die
(5%) SUGGESTED ANSWER: Yes, the bank may directly file the complaint with the proper before the expiration of the period fixed for bringing his action: b) If he should die after
court. In the absence or unavailability of an inquest prosecutor, the complaint may be the filing of the complaint, without having desisted therefrom; or c) If the child was born
filed by the offended party or a peace officer directly with the proper court on the basis after the death of the husband. Since Dario is already dead when the baby was, his heirs
of the affidavit of the offended party or arresting officer or person (Section 6, Rule 112 of have the right to impugn the filiation of the child. Consequently, the heirs may impugn
the Revised Rules of Criminal Procedure). Actions; Commencement of an Action; the filiation either by a direct action to impugn such filiation or raise the same in a special
Criminal, Civil (2013) No.III. While in his Nissan Patrol and hurrying home to Quezon City proceeding for settlement of the estate of the decedent. In the said proceeding, the
from his work in Makati, Gary figured in a vehicular mishap along that portion of EDSA Probate court has the power to determine questions as to who are the heirs of the
within the City of Mandaluyong. He was bumped from behind by a Ford Expedition SUV decedent (Reyes vs. Ysip, et. al., 97 Phil. 11, Jimenez vs. IAC, 184 SCRA 367). Incidentally,
driven by Horace who was observed using his cellular phone at the time of the collision. the heirs can also submit the baby boy for DNA testing (A.M. No. 6- 11-5-SC, Rules on
Both vehicles - more than 5 years old – no longer carried insurance other than the DNA Evidence) or even blood-test in order to determine paternity and filiation. In Jao vs.
compulsory third party liability insurance. Gary suffered physical injuries while his Nissan Court of Appeals, G.R. No. L49162, July 28, 1987, the Supreme Court held that blood
Patrol sustained damage in excess of Php500,000. (A) As counsel for Gary, describe the grouping tests are conclusive as to non-paternity, although inconclusive as to paternity.
process you need to undertake starting from the point of the incident if Gary would The fact that the blood type of the child is a possible product of the mother and alleged
proceed criminally against Horace, and identify the court with jurisdiction over the case. father does not conclusively prove that the child is born by such parents; but, if the blood
(3%) SUGGESTED ANSWER: Remedial Law Q&As (2007-2013) type of the child is not the possible blood type when the blood of the mother and the
hectorchristopher@yahoo.com JayArhSals “Never Let The Odds Keep You From Pursuing alleged father are cross matched, then the child cannot possibly be that of the alleged
What You Know In Your Heart You Were Meant To Do.”-Leroy Satchel Paige Page 61 of father. ALTERNATIVE ANSWER: No, there is no showing in the problem of any ground
198 A) As counsel for Gary, I will first have him medically examined in order to ascertain that would serve as a basis for an action to impugn paternity of the baby boy. In
the gravity and extent of the injuries sustained from the accident. Second, I will secure Concepcion vs. Almonte, G.R. No. 123450, August 31, 2005 citing Cabatania vs. Court of
an accurate police report relative to the mishap unless Horace admits his fault in writing, Appeals, the Supreme Court held that the law requires that every reasonable
and request Gary to secure a car damage estimate from a car repair shop. Third, I will ask presumption be made in favour of legitimacy. The presumption of legitimacy does not
him to execute his Sinumpaang Salaysay. Thereafter, I will use his Sinumpaang Salaysay only flow out of declaration in the statute but is based on the broad principles of natural
or prepare a Complaint-affidavit and file the same in the Office of the City Prosecutor of justice and the supposed virtue of the mother. It is Remedial Law Q&As (2007-2013)
Mandaluyong City (Sections 1 and 15 Rule 110, Rules of Criminal Procedure). This being a hectorchristopher@yahoo.com JayArhSals “Never Let The Odds Keep You From Pursuing
case of simple negligence and the penalty for the offense does not exceed six months What You Know In Your Heart You Were Meant To Do.”-Leroy Satchel Paige Page 64 of
imprisonment, the court with original and exclusive jurisdiction is the Metropolitan Trial 198 grounded on the policy to protect the innocent offspring from the odium of
Court of Mandaluyong City. (B) If Gary chooses to file an independent civil action for illegitimacy. The presumption of legitimacy proceeds from the sexual union in marriage,
damages, explain briefly this type of action: its legal basis; the different approaches in particularly during the period of conception. To overthrow this presumption on the basis
pursuing this type of action; the evidence you would need; and types of defenses you of Article 166 (1) (b) of the Family Code, it must be shown beyond reasonable doubt that
could expect. (5%) SUGGESTED ANSWER: An independent civil action is an action which is there was no access that could have enabled the husband to father the child. Sexual
entirely distinct and separate from the criminal action. Such civil action shall proceed Intercourse is to be presumed where personal access is not disposed, unless such
presumption is rebutted by evidence to the contrary. Hence, a child born to a husband (2007-2013) hectorchristopher@yahoo.com JayArhSals “Never Let The Odds Keep You
and wife during a valid marriage is presumed legitimate. Thus, the child‟s legitimacy may From Pursuing What You Know In Your Heart You Were Meant To Do.”-Leroy Satchel
be impugned only under the strict standards provided by law (Herrera vs. Alba, G.R. No. Paige Page 67 of 198 card company of his loss; and 3) his credit card billing statement
148220, June 15, 2005). [Note: The Family Code is not covered by the 2013 bar bearing the online ticket transaction. Arrest; Warrantless Arrests & Searches (2007)
Examination Syllabus for Remedial Law]. Actions; Complaint; Forum Shopping (2010) No.VI. (a) On his way home, a member of the Caloocan City police force witnesses a bus
No.IV. X was driving the dump truck of Y along Cattleya Street in Sta. Maria, Bulacan. Due robbery in Pasay City and effects the arrest of the suspect. Can he bring the suspect to
to his negligence, X hit and injured V who was crossing the street. Lawyer L, who Caloocan City for booking since that is where his station is? Explain briefly. (5%)
witnessed the incident, offered his legal services to V. V, who suffered physical injuries SUGGESTED ANSWER: No, the arresting officer may not take the arrested suspect from
including a fractured wrist bone, underwent surgery to screw a metal plate to his wrist Pasay City to Caloocan City. The arresting officer is required to deliver the person
bone. On complaint of V, a criminal case for Reckless Imprudence Resulting in Serious arrested without a warrant to the nearest police station or jail (Rule 112, Sec. 5, 2000
Physical Injuries was filed against X before the Municipal Trial Court (MTC) of Sta. Maria. Rules of Criminal Procedure). To be sure, the nearest police station or jail is in Pasay City
Atty. L, the private prosecutor, did not reserve the filing of a separate civil action. V where the arrest was made, and not in Caloocan City. (b) In the course of serving a
subsequently filed a complaint for Damages against X and Y before the Regional Trial search warrant, the police find an unlicensed firearm. Can the police take the firearm
Court of Pangasinan in Urdaneta where he resides. In his "Certification Against Forum even if it is not covered by the search warrant? If the warrant is subsequently quashed, is
Shopping," V made no mention of the pendency of the criminal case in Sta. Maria. (a) Is V the police required to return the firearm? Explain briefly. (5%) SUGGESTED ANSWER: Yes,
guilty of forum shopping? (2%) SUGGESTED ANSWER: No, V is not guilty of forum the police may take with him the “unlicensed” firearm although not covered by the
shopping because the case in Sta. Maria, Bulacan, is a criminal action filed in the name of search warrant. Possession of an “unlicensed firearm” is a criminal offense and the police
the People of the Philippines, where civil liability arising from the crime is deemed also officer may seize an article which is the “subject of an offense.” Thus us especially so
instituted therewith; whereas the case filed in Urdaneta, Pangasinan, is a civil action for considering that the “unlicensed firearm” appears to be in “plain view” of the police
quasi- Remedial Law Q&As (2007-2013) hectorchristopher@yahoo.com JayArhSals officer when the conducted the search. Even if the warrant was subsequently quashed,
“Never Let The Odds Keep You From Pursuing What You Know In Your Heart You Were the police are not mandated to return the “unlicensed firearm.” The quashal of the
Meant To Do.”-Leroy Satchel Paige Page 65 of 198 delict in the name of V and against search warrant did not affect the validity of the seizure of the “unlicensed firearm.”
both X and Y for all damages caused by X and Y to V, which may be beyond the Moreover, returning the firearm to a person who is not otherwise allowed by law to
jurisdiction of MTC. Hence, the tests of forum shopping, which is res adjudicate or litis possess the same would be tantamount to abetting a violation of the law. Bail;
pendencia, do not obtain here. Moreover, substantive law (Art. 33, Civil Code) and Sec. 3, Application (2012) No.I.B. A was charged with a non-bailable offense. At the time when
Rule 111, Revised Rules of Criminal Procedure, expressly authorize the filing such action the warrant of arrest was issued, he was confined in the Remedial Law Q&As (2007-
for damages entirely separate and distinct from the criminal action. (b) Instead of filing 2013) hectorchristopher@yahoo.com JayArhSals “Never Let The Odds Keep You From
an Answer, X and Y move to dismiss the complaint for damages on the ground of litis Pursuing What You Know In Your Heart You Were Meant To Do.”-Leroy Satchel Paige
pendentia. Is the motion meritorious? Explain. (2%) SUGGESTED ANSWER: No, the Page 68 of 198 hospital and could not obtain a valid clearance to leave the hospital. He
motion to dismiss base on alleged litis pendencia is without merit because there is no filed a petition for bail saying therein that he be considered as having placed himself
identity of parties and subject matter in the two cases. Besides, Art. 33 of the Civil Code under the jurisdiction of the court. May the court entertain his petition? Why or why
and Rule 111, Sec. 3 of the Rules of Criminal Procedure authorize the separate civil action not? (5%) SUGGESTED ANSWER: No, the court may not entertain his petition as he has
for damages arising from physical injuries to proceed independently. (c) Suppose only X not yet been placed under arrest. A must be “literally” placed under the custody of the
was named as defendant in the complaint for damages, may he move for the dismissal of law before his petition for bail could be entertained by the court (Miranda vs. Tuliao, G.R.
the complaint for failure of V to implead Y as an indispensable party? (2%) SUGGESTED No. 158763, March 31, 2006). ALTERNATIVE ANSWER: Yes, a person is deemed to be
ANSWER: No, X may not move for dismissal of the civil action for damages on the under the custody of the law either when he has been arrested or has surrendered
contention that Y is an indispensable party who should be impleaded. Y is not an himself to the jurisdiction of the court. the accused who is confined in a hospital may be
indispensable party but only necessary party. Besides, nonjoinder and misjoinder of deemed to be in the custody of the law if he clearly communicates his submission to the
parties is not a ground for dismissal of actions (Rule 3, Sec. 11, Rules of Court). (d) X court while he is confined in the hospital. (Paderanga vs. Court of Appeals, G.R. No. No.
moved for the suspension of the proceedings in the criminal case to await the decision in 115407, August 28, 1995). Discovery; Production and Inspection (2009) No.XI.A. The
the civil case. For his part, Y moved for the suspension of the civil case to await the accused in a criminal case has the right to avail of the various modes of discovery.
decision in the criminal case. Which of them is correct? Explain. (2%) SUGGESTED SUGGESTED ANSWER: TRUE. The accused has the right to move for the production or
ANSWER: Neither of them is correct. Both substantive law (Art. 33 of the Civil Code) and inspection of material evidence in the possession of the prosecution. It authorizes the
procedural law (Rule 111, Sec. 3, Rules of Criminal Procedure) provide for the two actions defense to inspect, copy or photograph any evidence of the prosecution in its possession
to proceed independently of each other, therefore, no suspension of action is after obtaining permission from the court (Rule 116, Sec. 10; Webb vs. De Leon, 247
authorized. (e) Atty. L offered in the criminal case his affidavit respecting what he SCRA 652 [1995]). ALTERNATIVE ANSWER: FALSE. The accused in criminal case only has
witnessed during the incident. X’s lawyer wanted to cross-examine Atty. L who, however, the right to avail of conditional examination of his witness before a judge, or, if not
objected on the ground of lawyer-client privilege. Rule on the objection. (2%) Remedial practicable, a member of a Bar in good standing so designated by the judge in the order,
Law Q&As (2007-2013) hectorchristopher@yahoo.com JayArhSals “Never Let The Odds or if the order be made by a court of superior jurisdiction, before an inferior court to be
Keep You From Pursuing What You Know In Your Heart You Were Meant To Do.”-Leroy designated therein. (sec.12 &13, Rule 119). Modes of discovery under civil actions does
Satchel Paige Page 66 of 198 SUGGESTED ANSWER: The objection should be overruled. not apply to criminal proceedings because the latter is primarily governed by the
Lawyer-client privilege is not involved here. The subject on which the counsel would be REVISED RULES OF CRIMINAL PROCEDURE (Vda. de Manguerravs Risos – 563 SCRA 499).
examined has been made public in the affidavit he offered and thus, no longer privileged, Remedial Law Q&As (2007-2013) hectorchristopher@yahoo.com JayArhSals “Never Let
aside from the fact that it is in respect of what the counsel witnessed during the incident The Odds Keep You From Pursuing What You Know In Your Heart You Were Meant To
and not to the communication made by the client to him or the advice he gave thereon Do.”-Leroy Satchel Paige Page 69 of 198 Information; Motion to Quash (2009) No.IV.
in his professional capacity. Actions; Hold Departure Order (2010) No. XVIII. While Pedrito and Tomas, Mayor and Treasurer, respectively, of the Municipality of San Miguel,
window-shopping at the mall on August 4, 2008, Dante lost his organizer including his Leyte, are charged before the Sandiganbayanfor violation of Section 3(e), RA no. 3019
credit card and billing statement. Two days later, upon reporting the matter to the credit (Anti-Graft and Corrupt Practices Act). The information alleges, among others, that the
card company, he learned that a one-way airplane ticket was purchased online using his two conspired in the purchase of several units of computer through personal canvass
credit card for a flight to Milan in midAugust 2008. Upon extensive inquiry with the instead of a public bidding, causing undue injury to the municipality. Before arraignment,
airline company, Dante discovered that the plane ticket was under the name of one Dina the accused moved for reinvestigation of the charge, which the court granted. After
Meril. Dante approaches you for legal advice. (a) What is the proper procedure to reinvestigation, the Office of the Special Prosecutor filed an amended information duly
prevent Dina from leaving the Philippines? (2%) SUGGESTED ANSWER: I would advise: (1) singed and approved by the Special Prosecutor, alleging the same delictual facts, but
The filing of an appropriate criminal action cognizable by the RTC against Dina and the with an additional allegation that the accused gave unwarranted benefits to SB
filing in said criminal action a Motion for the issuance of a Hold Departure Order; (2) enterprises owned by Samuel. Samuel was also indicted under the amended information.
thereafter, a written request with the Commissioner of the Bureau of Immigration for a Before Samuel was arraigned, he moved to quash the amended information on the
Watch List Order pending the issuance of the Hold Departure Order should be filed; (3) ground that the officer who filed had no authority to do so. Resolve the motion to quash
then, the airline company should be requested to cancel the ticket issued to Dina. (b) with reasons. SUGGESTED ANSWER: The motion to quash filed by Samuel should be
Suppose an Information is filed against Dina on August 12, 2008 and she is immediately granted. There is no showing that the special prosecutor was duly authorized or
arrested. What pieces of electronic evidence will Dante have to secure in order to prove deputized to prosecute Samuel. Under R.A. No. 6770, also known as the Ombudsman Act
the fraudulent online transaction? (2%) SUGGESTED ANSWER: He will have to present (a) of 1989, the Special Prosecutor has the power and authority, under the supervision and
his report to the bank that he lost his credit card (b) that the ticket was purchased after control of the Ombudsman, to conduct preliminary investigation and prosecute criminal
the report of the lost and (c) the purchase of one-way ticket. Dante should bring an cases before the Sandiganbayan and perform such other duties assigned to him by the
original (or an equivalent copy) printout of: 1) the online ticket purchase using his credit Ombudsman (Calingin vs. Desierto, 529 SCRA 720 [2007]). Absent a clear delegation of
card; 2) the phone call log to show that he already alerted the credit Remedial Law Q&As authority from the Ombudsman to the Special Prosecutor to file the information, the
latter would have no authority to file the same. The Special Prosecutor cannot be Seizure; Plain View (2008) No.IX. The search warrant authorized the seizure of
considered an alter ego of the Ombudsman as the doctrine of qualified political agency “undetermined quantity of shabu.” During the service of the search warrant, the raiding
does not apply to the office of the Ombudsman. In fact, the powers of the office of the team also recovered a kilo of dried marijuana leaves wrapped in newsprint. The accused
Special Prosecutor under the law may be exercised only under the supervision and moved to suppress the marijuana leaves as evidence for the violation of Section 11 of the
control and upon authority of the Ombudsman (Perez vs. Sandiganbayan, 503 SCRA 252 Comprehensive Dangerous Drugs Act of 2002 since they were not covered by the search
[2006]). ALTERNATIVE ANSWER: The motion to quash should be denied for lack of merit. warrant. The State justified the seizure of the marijuana leaves under the “plain view”
The case is already filed in court which must have been done with the approval of the doctrine. There was no indication of whether the marijuana leaves were discovered and
Ombudsman, and thus the Special Prosecutor‟s office of the Ombudsman Remedial Law seized before or after the seizure of the shabu. If you are the judge, how would you rule
Q&As (2007-2013) hectorchristopher@yahoo.com JayArhSals “Never Let The Odds Keep on the motion to suppress? SUGGESTED ANSWER: The “plain view” doctrine cannot be
You From Pursuing What You Know In Your Heart You Were Meant To Do.”-Leroy Satchel invoked because the marijuana leaves were wrapped in newsprint and there was no
Paige Page 70 of 198 takes over. As it is the court which ordered the reinvestigation, the evidence as to whether the marijuana leaves were discovered and seized before or after
Office of the Special Prosecutor which is handling the case in court, has the authority to the seizure of the shabu. If they were discovered after the seizure of the shabu, then the
act and when warranted, refile the case. The amendment made is only a matter of form marijuana could not have been seized in palin view (CF. Peo vs. Mua, G.R. No. 96177, 27
which only particularized the violation of the same provision of Rep. Act 3019, as January 1997). In any case, the marijuana should be confiscated as a prohibited article.
amended. Information; Motion to Quash (2009) No.XVI.B. A criminal information is filed Search & Seizure; Warrantless Search (2010) No.VII. As Cicero was walking down a dark
in court charging Anselmo with homicide. Anselmo files a motion to quash information alley one midnight, he saw an "owner-type jeepney" approaching him. Sensing that the
on the ground that no preliminary investigation was conducted. Will the motion be Remedial Law Q&As (2007-2013) hectorchristopher@yahoo.com JayArhSals “Never Let
granted? Why or why not? SUGGESTED ANSWER: NO, the motion to quash will not be The Odds Keep You From Pursuing What You Know In Your Heart You Were Meant To
granted. The lack of preliminary investigation is not a ground for a motion to quash Do.”-Leroy Satchel Paige Page 73 of 198 occupants of the vehicle were up to no good, he
under the Rules of Criminal Procedure. Preliminary investigation is only a statutory right darted into a corner and ran. The occupants of the vehicle − elements from the Western
and can be waived. The accused should instead file a motion for reinvestigation within Police District − gave chase and apprehended him. The police apprehended Cicero,
five (5) days after he learns of the filing in Court of the case against him (Sec. 6, Rule 112, frisked him and found a sachet of 0.09 gram of shabu tucked in his waist and a Swiss
as amended). Jurisdiction; Complex Crimes (2013) No.VIII. On his way to the PNP knife in his secret pocket, and detained him thereafter. Is the arrest and body-search
Academy in Silang, Cavite on board a public transport bus as a passenger, Police legal? (3%) SUGGESTED ANSWER: The arrest and body-search was legal. Cicero appears
Inspector Masigasig of the Valenzuela Police witnessed an on-going armed robbery while to be alone „walking down the dark alley” and at midnight. There appears probable
the bus was traversing Makati. His alertness and training enabled him to foil the robbery cause for the policemen to check him, especially when he darted into a corner
and to subdue the malefactor. He disarmed the felon and while frisking him, discovered (presumably also dark) and run under such circumstance. Although the arrest came after
another handgun tucked in his waist. He seized both handguns and the malefactor was the bodysearch where Cicero was found with shabu and a Swiss knife, the body-search is
later charged with the separate crimes of robbery and illegal possession of firearm. A) legal under the “Terry search” rule or the “stop and frisk” rule. And because the mere
Where should Police Inspector Masigasig bring the felon for criminal processing? To possession, with animus, of dangerous drug (the shabu) is a violation of the law (R.A.
Silang, Cavite where he is bound; to Makati where the bus actually was when the 9165), the suspect is in a continuing state of committing a crime while he is illegally
felonies took place; or back to Valenzuela where he is stationed? Which court has possessing the dangerous drug, thus making the arrest tantamount to an arrest in
jurisdiction over the criminal cases? (3%) SUGGESTED ANSWER: Police Inspector flagrante: so the arrest is legal and correspondingly, the search and seizure of the shabu
Masigasig should bring the felon to the nearest police station or jail in Makati City where and the concealed knife may be regarded as incident to a lawful arrest. ALTERNATIVE
the bus actually was when the felonies took place. In cases of warrantless arrest, the ANSWER: No, the arrest and the body-search were not legal. In this case, Cicero did not
person arrested without a warrant shall be forthwith delivered to the nearest police run because the occupants of the vehicle identified themselves as police officers. He
Remedial Law Q&As (2007-2013) hectorchristopher@yahoo.com JayArhSals “Never Let darted into the corner and ran upon the belief that the occupants of the vehicle were up
The Odds Keep You From Pursuing What You Know In Your Heart You Were Meant To to no good. Cicero‟s act of running does not show any reasonable ground to believe that
Do.”-Leroy Satchel Paige Page 71 of 198 station or jail and shall be proceeded against in a crime has been committed or is about to be committed for the police officers to
accordance with section 7 of Rule 11 (Section 113, Rules of Criminal Procedure). apprehend him and conduct body search. Hence, the arrest was illegal as it does not fall
Consequently, the criminal case for robbery and illegal possession of firearms can be under any of the circumstances for a valid warrantless arrest provided in Sec. 5 of Rule
filed in Regional Trial Court of Makati City or on any of the places of departure or arrival 113 of the Rules of Criminal Procedure. Search Warrant; Application; Venue (2012) No.VI.
of the bus. (B) May the charges of robbery and illegal possession of firearm be filed A PDEA asset/informant tipped the PDEA Director Shabunot that a shabu laboratory was
directly by the investigating prosecutor with the appropriate court without a preliminary operating in a house at Sta. Cruz, Laguna, rented by two (2) Chinese nationals, Ho Pia and
investigation? (4%) SUGGESTED ANSWER: Yes. Since the offender was arrested in Sio Pao. PDEA Director Shabunot wants to apply for a search warrant, but he is worried
flagrante delicto without a warrant of arrest, an inquest proceeding should be conducted that if he Remedial Law Q&As (2007-2013) hectorchristopher@yahoo.com JayArhSals
and thereafter a case may be filed in court even without the requisite preliminary “Never Let The Odds Keep You From Pursuing What You Know In Your Heart You Were
investigation. Under Section 6, Rule 112, Rules of Criminal Procedure, when a person is Meant To Do.”-Leroy Satchel Paige Page 74 of 198 applies for a search warrant in any
lawfully arrested without a warrant involving an offense which requires a preliminary Laguna court, their plan might leak out. (a) Where can he file an application for search
investigation, the complaint or information may be filed by a prosecutor without a need warrant? (2%) SUGGESTED ANSWER: PDEA Director Shabunot may file an application for
of such investigation provided an inquest has been conducted in accordance with search warrant in any court within the judicial region where the crime was committed.
existing rules. Jurisdiction; Reinvestigation; Arrest (2008) No.X. Jose, Alberto and Romeo (Rule 126, Sec.2[b]). ALTERNATIVE ANSWER: PDEA Director Shabunot may file an
were charged with murder. Upon filing the information, the RTC judge issued warrants application for search warrant before the Executive Judge and Vice Executive Judges of
for their arrest. Learning of the issuance of the warrants, the three accused jointly filed a the Regional Trial Courts of Manila or Quezon Cities. (A.M. No. 99-10- 09-SC, January 25,
motion for reinvestigation and for the recall of the warrants of arrest. On the date set for 2000). (b) What documents should he prepare in his application for search warrant? (2%)
hearing of their motion, none of accused showed up in court for fear of being arrested. SUGGESTED ANSWER: He should prepare a petition for issuance of a search warrant and
The RTC judge denied their motion because the RTC did not acquire jurisdiction over the attach therein sworn statements and affidavits. (c) Describe the procedure that should
persons of the movants. Did the RTC rule correctly? SUGGESTED ANSWER: The RTC was be taken by the judge on the application. (2%) SUGGESTED ANSWER: The judge must,
not entirely correct in stating that it had no jurisdiction over the persons of the accused. before issuing the warrant, examine personally in the form of searching questions and
By filing motions and seeking affirmative reliefs from the court, the accused voluntarily answers, in writing and under oath, the complainant and the witnesses he may produce
submitted themselves to the jurisdiction of the court. However, the RTC correctly denied on facts personally known to them and attach to the record their sworn statements,
the motion for reinvestigation. Before an accused ca move for reinvestigation and the together with the affidavits submitted. (Rule 126, Sec.5, Rules of Court). if the judge is
recall of his warrant of arrest, he must first surrender his person to the court (Miranda, satisfied of the existence of facts upon which the application is based or that there is
et al. vs. Tuliao, G.R. No. 158763, 31 March 2006). Remedial Law Q&As (2007-2013) probable cause to believe that they exist, he shall issue the warrant, which must be
hectorchristopher@yahoo.com JayArhSals “Never Let The Odds Keep You From Pursuing substantially in the form prescribed by the Rules. (Rule 126, Sec.6, Rules of Court).
What You Know In Your Heart You Were Meant To Do.”-Leroy Satchel Paige Page 72 of Suppose the judge issues the search warrant worded in this way: PEOPLE OF THE
198 Res Judicata in Prison Grey (2010) No.XVII. What is "res judicata in prison grey"? (2%) PHILIPPINES Plaintiff -versusCriminal Case No. 007 for Violation of R.A. 9165 Ho Pia and
SUGGESTED ANSWER: “Res judicata in prison grey” is the criminal concept of double Sio Pao, Accused. x- x TO ANY PEACE OFFICER Remedial Law Q&As (2007-2013)
jeopardy, as “res judicata” is the doctrine of civil law (Trinidad vs. Office of the hectorchristopher@yahoo.com JayArhSals “Never Let The Odds Keep You From Pursuing
Ombudsman, G.R. No. 166038, December 4, 2007). Described as “res judicata in prison What You Know In Your Heart You Were Meant To Do.”-Leroy Satchel Paige Page 75 of
grey,” the right against double jeopardy prohibits the prosecution of a person for a crime 198 Greetings: It appearing to the satisfaction of the undersigned after examining under
of which he has been previously acquitted or convicted. The purpose is to set the effects oath PDEA Director shabunot that there is probable cause to believe that violations of
of the first prosecution forever at rest, assuring the accused that he shall not thereafter Section 18 and 16 of R.A. 9165 have been committed and that there are good and
be subjected to the danger and anxiety of a second charge against him for the same sufficient reasons to believe that Ho Pia and Sio Pao have in their possession or control,
offense (Joel B. Caes vs. Intermediate Appellate Court, November 6, 1989). Search & in a two (2) door apartment with an iron gate located at Jupiter St., Sta. Cruz, Laguna,
undetermined amount of "shabu" and drug manufacturing implements and From Pursuing What You Know In Your Heart You Were Meant To Do.”-Leroy Satchel
paraphernalia which should be seized and brought to the undersigned, You are hereby Paige Page 78 of 198 entitled to bail before conviction in the Regional Trial Court (Section
commanded to make an immediate search, at any time in the day or night, of the 4, Rule 114 of the Rules of Criminal Procedure). [Note: unless the aggregate value of the
premises above described and forthwith seize and take possession of the property stolen is P500,000 and the above she will not be entitled to bail as a matter of
abovementioned personal property, and bring said property to the undersigned to be right, because the penalty for the offense is reclusion perpetua pursuant to
dealt with as the law directs. Witness my hand this 1st day of March, 2012. (signed) Memorandum Order No. 117]. (B) In another case, also for qualified theft, the detained
Judge XYZ (d) Cite/enumerate the defects, if any, of the search warrant. (3%) SUGGESTED young domestic helper has been brought to court five times in the last six months, but
ANSWER: (1) The search warrant failed to particularly describe the place to be searched the prosecution has yet to commence the presentation of its evidence. You find that the
and the things to be seized (Rule 126, Sec.4, Rules of Court). (2) The search warrant reason for this is the continued absence of the employercomplainant who is working
commanded the immediate search, at any time in the day or night. The general rule is overseas. What remedy is appropriate and before which forum would you invoke this
that a search warrant must be served in the day time (Rule 126, Sec.8, Revised Rules on relief? (3%) SUGGESTED ANSWER: I will file a motion to dismiss the information in the
Criminal Procedure), or that portion of the twenty-four hours in which a man‟s person court where the case is pending on the ground of denial of the accused right to speedy
and countenance are distinguishable (17 C.J. 1134). By way of exception, a search trial (Section 9, Rule 119; Tan vs. People, G.R. No. 173637, April 21, 2009, Third Division,
warrant may be made at night when it is positively asserted in the affidavit that the Chico-Nazario, J.). this remedy can be invoked, at any time, before trial and if granted will
property is on the person or in the place ordered to be searched (Alvares vs. CFI of result to an acquittal. Since the accused has been brought to Court five times and in each
Tayabas, 64 Phil. 33). There is no showing that the exception applies. (e) Suppose the instance it was postponed, it is clear that her right to a Speedy Trial has been violated.
search warrant was served on March 15, 2012 and the search yielded the described Moreover, I may request the court to issue Subpoena Duces Tecum and Ad
contraband and a case was filed against the accused in RTC, Sta. Cruz, Laguna and you Testificandum to the witness, so in case he disobeys same, he may be cited in contempt.
are the lawyer of Sio Pao and Ho Pia, what will you do? (3%) SUGGESTED ANSWER: I may also file a motion to order the witness employer-complainant to post bail to secure
Remedial Law Q&As (2007-2013) hectorchristopher@yahoo.com JayArhSals “Never Let his appearance in court. (Section 14, Rule 119). ALTERNATIVE ANSWER: I will move for
The Odds Keep You From Pursuing What You Know In Your Heart You Were Meant To the dismissal of the case for failure to prosecute. The grant of the motion will be with
Do.”-Leroy Satchel Paige Page 76 of 198 If I were the lawyer of Sio Pao and Ho Pia, I prejudice unless the court says otherwise. The Motion will be filed with the Court where
would file a Motion to Quash the search warrant for having been served beyond its the action is pending. C) Still in another case, this time for illegal possession of dangerous
period of validity. (Rule 126, Sec. 14, Rules of Court). A search warrant shall be valid only drugs, the prosecution has rested but you saw from the records that the illegal substance
for ten (10) days from its date. Thereafter, it shall be void. (Rule 126, Sec.10, Revised allegedly involved has not been identified by any of the prosecution witnesses nor has it
Rules of Court). (f) Suppose an unlicensed armalite was found in plain view by the been the subject of any stipulation. Should you now proceed posthaste to the
searchers and the warrant was ordered quashed, should the court order the return of presentation of defense evidence or consider some other remedy? Explain the Remedial
the same to the Chinese nationals? Explain your answer. (3%) SUGGESTED ANSWER: No, Law Q&As (2007-2013) hectorchristopher@yahoo.com JayArhSals “Never Let The Odds
the court should not order the return of the unlicensed armalite because it is contraband Keep You From Pursuing What You Know In Your Heart You Were Meant To Do.”-Leroy
or illegal per se. (PDEA vs. Brodett, G.R. No. 196390, September 28, 2011). The Satchel Paige Page 79 of 198 remedial steps you propose to undertake. (3%) SUGGESTED
possession of an unlicensed armalite found in plain view is mala prohibita. The same ANSWER: No. I will not proceed with the presentation of defense evidence. I will first file
should be kept in custodial legis. Trial; Remedies (2013) No.IV. At the Public Attorney's a motion for leave to file demurrer to evidence within five (5) days from the time the
Office station in Taguig where you are assigned, your work requires you to act as public prosecution has rested its case. If the Motion is granted, I will file a demurrer to evidence
defender at the local Regional Trial Court and to handle cases involving indigents. (A) In within a nonextendible period of ten (10) days from notice on the ground of insufficiency
one criminal action for qualified theft where you are the defense attorney, you learned of evidence. In the alternative, I may immediately file a demurrer to evidence without
that the woman accused has been in detention for six months, yet she has not been to a leave of court (Section 23, Rule 119, Rules of Criminal Procedure). In People vs. De
courtroom nor seen a judge. What remedy would you undertake to address the situation Guzman, G.R. No. 186498, March 26, 2010, the Supreme Court held that in a prosecution
and what forum would you use to invoke this relief? (3%) SUGGESTED ANSWER: Section for violation of the Dangerous Drugs Act, the existence of the dangerous drugs is a
7, Rule 119 provides, if the public attorney assigned to defend a person charged with a condition sine qua non for conviction. The dangerous drug is the very corpus delicti of
crime knows that the latter is preventively detained, either because he is charged with a the crime. Similarly, in People vs. Sitco, G.R. No. 178202, May 14, 2010, the High Court
bailable crime but has no means to post bail, or, is charged with a non-bailable crime, or, held that in prosecutions involving narcotics and other illegal substances, the substance
is serving a term of imprisonment in any penal institution, it shall be his duty to do the itself constitutes part of the corpus delicti of the offense and the fact of its existence is
following: (a) Shall promptly undertake to obtain the presence of the prisoner for trial or vital to sustain a judgment of conviction beyond reasonable doubt. (D) In one other case,
cause a notice to be served on the person having custody of the prisoner requiring such an indigent mother seeks assistance for her 14-year old son who has been arrested and
person to so advise the prisoner of his right to demand trial. (b) Upon receipt of that detained for malicious mischief. Would an application for bail be the appropriate remedy
notice, the custodian of the prisoner shall promptly advise the prisoner of the charge and or is there another remedy available? Justify your chosen remedy and outline the
of Remedial Law Q&As (2007-2013) hectorchristopher@yahoo.com JayArhSals “Never appropriate steps to take. (3%) SUGGESTED ANSWER: Yes. An application for bail is an
Let The Odds Keep You From Pursuing What You Know In Your Heart You Were Meant To appropriate remedy to secure provisional liberty of the 14-year old boy. Under the Rules,
Do.”-Leroy Satchel Paige Page 77 of 198 his right to demand trial. If at any time bail is a matter of right before or even after conviction before the Metropolitan Trial
thereafter the prisoner informs his custodian that he demands such trial, the latter shall Court which has jurisdiction over the crime of malicious mischief. (Section 4, Rule 114 of
cause notice to that effect to be sent promptly to the public attorney. Xxx Moreover, the Rules of Criminal Procedure). ALTERNATIVE ANSWER: Under R.A. 9344 or otherwise
Section 1 (e), Rule 116 provides, when the accused is under preventive detention, his known as the Juvenile Justice and Welfare Act of 2006 as amended by R.A. 10630, a child
case shall be raffled and its records transmitted to the judge to whom the case was in conflict with the law has the right to bail and recognizance or to be transferred to a
raffled within the three (3) days from the filing of the information or complaint. The youth detention home/youth rehabilitation center. Thus: Remedial Law Q&As (2007-
accused shall be arraigned within ten (10) days from the date of the raffle. The pre-trial 2013) hectorchristopher@yahoo.com JayArhSals “Never Let The Odds Keep You From
conference of his case shall be held within ten (10) days after the arraignment. On the Pursuing What You Know In Your Heart You Were Meant To Do.”-Leroy Satchel Paige
other hand, if the accused is not under preventive detention, the arraignment shall be Page 80 of 198 Where a child is detained, the court shall order: (a) the release of the
held within thirty (30) days from the date the court acquires jurisdiction over the person minor on recognizance to his/her parents and other suitable person; (b) the release of
of the accused. (Section 1 (g), Rule 116). Since the accused has not been brought for the child in conflict with the law on bail; or (c) the transfer of the minor to a youth
arraignment within the limit required in the aforementioned Rule, the Information may detention home/youth rehabilitation center. The court shall not order the detention of a
be dismissed upon motion of the accused invoking his right to speedy trial (Section 9, child in a jail pending trial or hearing of his case. The writ of habeas corpus shall extend
Rule 119( or to a speedy disposition of cases (Section 16, Article III, 1987 Constitution). to all cases of illegal confinement or detention by which any person is deprived of his
ALTERNATIVE ANSWER: A Petition for Mandamus is also feasible. In People vs. liberty, or by which the rightful custody of any person is withheld from the person
Lumanlaw, G.R. No. 164953, February 13, 2006, the Supreme Court held that “a writ of entitled thereto (IN THE MATTER OF THE PETITION OF HABEAS CORPUS OF EUFEMIA E.
mandamus may be issued to control the exercise of discretion when, in the performance RODRIGUEZ, filed by EDGARDO E. VELUZ vs. LUISA R. VILLANUEVA and TERESITA R.
of duty, there is undue delay that can be characterized as a grave abuse of discretion PABELLO, G.R. No. 169482, January 29, 2008, CORONA, J.). Since minors fifteen (15)
resulting in manifest injustice. Due to the unwarranted delays in the conduct of the years of age and under are not criminally responsible, the child may not be detained to
arraignment of petitioner, he has indeed the right to demand – through a writ of answer for the alleged offense. The arresting authority has the duty to immediately
mandamus – expeditious action from all official tasked with the administration of justice. release the child to the custody of his parents or guardians or in their absence to the
Thus, he may not only demand that his arraignment be held but, ultimately, that the child‟s nearest relative (Section 20, republic Act 9344). Following the hierarchy of courts,
information against him be dismissed on the ground of the violation of his right to the Petition must be filed in the Regional trial Court having jurisdiction over the place
speedy trial.” Ergo, a writ of mandamus is available to the accused to compel the where the child is being detained. [Note: R.A. 9344 is not covered by the 2013 Bar
dismissal of the case. ALTERNATIVE ANSWER: The appropriate remedy of the detained Examination Syllabus for Remedial law]. Trial; Reverse Trial (2007) No.V. (b) What is
accused is to apply for bail since qualified theft is bailable, and she is Remedial Law Q&As reverse trial and when may it be resorted to? Explain briefly. (5%) SUGGESTED ANSWER:
(2007-2013) hectorchristopher@yahoo.com JayArhSals “Never Let The Odds Keep You A reverse trial is one where the defendant or the accused present evidence ahead of the
plaintiff or prosecution and the latter is to present evidence by way of rebuttal to the rule bars only partiesplaintiff and their assignors, or persons prosecuting a claim against
former‟s evidence. This kind of trial may take place in a civil case when the defendant‟s the estate of a deceased; it does not cover Maria who is a mere witness. Furthermore,
Answer pleads new matters by way of affirmative defense, to defeat or evade liability for the disqualification is in respect of any matter of fact occurring before the death of said
plaintiff‟s claim which is not denied but controverted. In a criminal case, a reverse trial deceased (Sec. 23, Rule 130, Rules of Court, Razon v. Intermediate Appellate Court, 207
may take place when the accused made Remedial Law Q&As (2007-2013) SCRA 234 [1992]). It is Pedro who filed the claim against the estate of Jose. Admissibility;
hectorchristopher@yahoo.com JayArhSals “Never Let The Odds Keep You From Pursuing DNA Evidence (2010) No.IX. In a prosecution for rape, the defense relied on
What You Know In Your Heart You Were Meant To Do.”-Leroy Satchel Paige Page 81 of Deoxyribonucleic Acid (DNA) evidence showing that the semen found in the private part
198 known to the trial court, on arraignment, that he adduce affirmative defense of a of the victim was not identical with that of the accused’s. As private prosecutor, how will
justifying or exempting circumstances and thus impliedly admitting the act imputed to you dispute the veracity and accuracy of the results of the DNA evidence? (3%) Remedial
him. The trial court may then require the accused to present evidence first, proving the Law Q&As (2007-2013) hectorchristopher@yahoo.com JayArhSals “Never Let The Odds
requisites of the justifying or exempting circumstance he is invoking, and the prosecution Keep You From Pursuing What You Know In Your Heart You Were Meant To Do.”-Leroy
to present rebuttal evidence controverting the same. Trial; Speedy Trial (2007) No.IX. L Satchel Paige Page 84 of 198 SUGGESTED ANSWER: As a private prosecutor, I shall try to
was charged with illegal possession of shabu before the RTC. Although bail was allowable discredit the results of the DNA test by questioning and possibly impugning the integrity
under his indictment, he could not afford to post bail, and so he remained in detention at of the DNA profile by showing a flaw/error in obtaining the biological sample obtained;
the City Jail. For various reasons ranging from the promotion of the Presiding Judge, to the testing methodology employed; the scientific standard observed; the forensic DNA
the absence of the trial prosecutor, and to the lack of notice to the City Jail Warden, the laboratory which conducted the test; and the qualification, training and experience of
arraignment of L was postpones nineteen times over a period of two years. Twice during the forensic laboratory personnel who conducted the DNA testing. Admissibility; DNA
that period, L’s counsel filed motions to dismiss, invoking the right of the accused to Evidence (2009) No.I.[a] The Vallejo standard refers to jurisprudential norms considered
speedy trial. Both motions were denied by the RTC. Can L file a petition for mandamus. by the court in assessing the probative value of DNA evidence. SUGGESTED ANSWER:
Reason briefly. SUGGESTED ANSWER: Yes, L can file a petition for mandamus to enforce TRUE. In People vs. Vallejo, 382 SCRA 192 (2002), it was held that in assessing the
his constitutional right to a speedy trial which was capriciously denied to him. There is probative value of DNA evidence, courts should consider among other things, the
absolutely no justification for postponing an arraignment of the accused nineteen (19) following data: how the samples were collected, how they were handled, the possibility
times and over a period of two (2) years. The numerous, unreasonable postponements of of contamination of the samples, whether the proper standards and procedures were
the arraignment demonstrate an abusive exercise of discretion (Lumanlaw v. Peralta, 482 followed in conducting the tests and the qualification of the analyst who conducted
SCRA 396 [2006]). Arraignment of an accused would not take thirty minutes of the tests. Admissibility; Evidence from Invasive and Involuntary Procedures (2010) No. XIII.
precious time of the court, as against the preventive imprisonment and deprivation of Policemen brought Lorenzo to the Philippine General Hospital (PGH) and requested one
liberty of the accused just because he does not have the means to post bail although the of its surgeons to immediately perform surgery on him to retrieve a packet of 10 grams
crime charged is bailable. The right to a speedy trial is guaranteed by the Constitution to of shabu which they alleged to have swallowed Lorenzo. Suppose the PGH agreed to, and
every citizen accused of a crime, more so when is under preventive imprisonment. L, in did perform the surgery is the package of shabu admissible in evidence? Explain. (3%)
the given case, was merely invoking his constitutional right when a motion to dismiss the SUGGESTED ANSWER: No, the package of shabu extracted from the body of Lorenzo is
case was twice filed by his counsel. The RTC is virtually enjoined by the fundamental law not admissible in evidence because it was obtained through surgery which connotes
to respect such right; hence a duty. Having refused or neglected to discharge the duty forcible invasion into the body of Lorenzo without his consent and absent due process.
enjoined by law whereas there is no appeal nor any plain, speedy, and adequate remedy The act of the policemen and the PGH surgeon involved, violate the fundamental rights
Remedial Law Q&As (2007-2013) hectorchristopher@yahoo.com JayArhSals “Never Let of Lorenzo, the suspect. ALTERNATIVE ANSWER: Remedial Law Q&As (2007-2013)
The Odds Keep You From Pursuing What You Know In Your Heart You Were Meant To hectorchristopher@yahoo.com JayArhSals “Never Let The Odds Keep You From Pursuing
Do.”-Leroy Satchel Paige Page 82 of 198 in the ordinary course of law, the remedy of What You Know In Your Heart You Were Meant To Do.”-Leroy Satchel Paige Page 85 of
mandamus may be availed of. Trial; Trial in Absentia (2010) No. XIX. (1) Enumerate the 198 Yes, it is admissible in evidence because the constitutional right against
requisites of a "trial in absentia " (2%) and a "promulgation of judgment in absentia" selfincriminating evidence exists. In the past, Supreme Court has already declared many
(2%). SUGGESTED ANSWER: The requisites of a valid trial in absentia are: (1) accused‟s invasive and involuntary procedures (i.e examination of women‟s genitalia, expulsion of
arraignment; (2) his due notification of the trial; (3) his unjustifiable failure to appear morphine from one‟s mouth, DNA testing) as constitutionally sound. Admissibility; Offer
during trial (Bernardo vs. People, G.R. No. 166980, April 4, 2007). The requisites for a to Settle; Implied Admission of Guilt (2008) No.VIII. Bembol was charged with rape.
valid promulgation of judgment are: (a) A valid notice of promulgation of judgment; (b) Bembol’s father, Ramil, approached Artemon, the victim’s father, during the preliminary
Said notice was duly furnished to the accused personally or thru counsel; (c) Accused investigation and offered P1 Million to Artemon to settle the case. Artemon refused the
failed to appear on the scheduled date of promulgation of judgment despite due notice; offer. (A) During trial, the prosecution presented Artemon to testify on Ramil’s offer and
(d) Such judgment be recorded in the criminal docket; (e) Copy of said judgment had thereby establish and implied admission of guilt. Is Ramil’s offer to settle admissible in
been duly served upon the accused or his counsel. (2) Name two instances where the evidence? (3%) SUGGESTED ANSWER: Yes, the offer to settle by the father of the
trial court can hold the accused civilly liable even if he is acquitted. (2%) SUGGESTED accused, is admissible in evidence as an implied admission of guilt. (Peo v. Salvador, GR
ANSWER: The instances where the civil liability is not extinguished despite the acquittal No. 136870-72, 28 January 2003) ALTERNATIVE ANSWER: No, Under Sec. 27, Rule 130 of
of the accused where: (1) The acquittal is based on reasonable doubt; (2) Where the the Rules of Court, it is the offer of compromise by the accused that may be received in
court expressly declares that the liability of the accused is not criminal but only civil in evidence as an implied admission of guilt. The testimony of Artemon would cover the
nature; and (3) Where the civil liability is not derived from or based on the criminal act of offer of Ramil and not an offer of the accused himself. (Peo v. Viernes, GR Nos. 136733-
which the accused is acquitted (Remedios Nota Sapiera vs. Court of Appeals, September 35, 13 December 2001) (B) During the pretrial ,Bembol personally offered to settle the
14, 1999). Evidence (Rules 128-134) Admissibility; Admission of Guilt (2008) No. XVI. The case for P1 Million to the private prosecutor, who immediately put the offer on record in
mutilated cadaver of a woman was discovered near a creek. Due to witnesses attesting the presence of the trial judge. Is Bembol’soffer a judicial admission of his guilt. (3%)
that he was the last person seen with the woman when she was still alive, Carlito was SUGGESTED ANSWER: Yes, Bembol‟s offer is an admission of guilt (Sec. 33 Rule 130). If it
arrested within five Remedial Law Q&As (2007-2013) hectorchristopher@yahoo.com was repeated by the private prosecutor in the presence of judge at the pretrial the
JayArhSals “Never Let The Odds Keep You From Pursuing What You Know In Your Heart extrajudicial confession becomes transposed into a judicial confession. There is no need
You Were Meant To Do.”-Leroy Satchel Paige Page 83 of 198 hours after the discovery of of assistance of counsel. (Peo v. Buntag, GR No. 123070, 14 April 2004). Best Evidence
the cadaver and brought to the police station. The crime laboratory determined that the Rule; Electronic Evidence (2009) No.XI. [d] An electronic evidence is the equivalent of an
woman had been raped. While in police custody, Carlito broke down in the presence of original document under the Best Evidence Rule if it is a printout or readable by sight or
an assisting counsel orally confessed to the investigator that he had raped and killed the other means, shown to reflect the data accurately. Remedial Law Q&As (2007-2013)
woman, detailing the acts he had performed up to his dumping of the body near the hectorchristopher@yahoo.com JayArhSals “Never Let The Odds Keep You From Pursuing
creek. He was genuinely remorseful. During the trial, the state presented the investigator What You Know In Your Heart You Were Meant To Do.”-Leroy Satchel Paige Page 86 of
to testify on the oral confession of Carlito. Is the oral confession admissible in evidence 198 SUGGESTED ANSWER: TRUE. This statement is embodied in Sec. 1, Rule 4 of A.m. No.
of guilt? (4%) SUGGESTED ANSWER: The declaration of the accused expressly 01-7-01-SC, re: Rules on Electronic Evidence. Chain of Custody (2012) No.II.A. (a) Discuss
acknowledging his guilt, in the presence of assisting counsel, may be given in evidence the "chain of custody" principle with respect to evidence seized under R.A. 9165 or the
against him and any person, otherwise competent to testify as a witness, who heard the Comprehensive Dangerous Drugs Act of 2002. (5%) SUGGESTED ANSWER: In
confession is competent to testify as to the substance o what he heard and understood prosecutions involving narcotics and other illegal substances, the substance itself
it. What is crucial here is that the accused was informed of his right to an attorney and constitutes part of the corpus delicti of the offense and the fact of its existence is vital to
that what he says may be used in evidence against him. As the custodial confession was sustain a judgment of conviction beyond reasonable doubt. The chain of custody
given in the presence of an assisting counsel, Carlito is deemed fully aware of the requirement is essential to ensure that doubts regarding the identity of the evidence are
consequences of his statements (People v. Silvano, GR No. 144886, 29 April 2002). removed through the monitoring and tracking of the movements of the seized drugs
Admissibility; Death of Adverse Party (2007) No.II. (a) The surviving parties rule bars from the accused, to the police, to the forensic chemist, and finally to the court. (People
Maria from testifying for the claimant as to what the deceased Jose had said to her, in a vs. Sitco, G.R. No. 178202, May 14, 2010, Velasco, Jr. J.). Ergo, the existence of the
claim filed by Pedro against the estate of Jose. (3%) SUGGESTED ANSWER: False. The said dangerous drug is a condition sine qua non for conviction. (People vs. De Guzman Y
Danzil, G.R. No. 186498, March 26, 2010 Nachura J.). The failure to establish, through the disclosure does not qualify as independently relevant statement and therefore,
convincing proof, that the integrity of the seized items has been adequately preserved hearsay. The nurse is competent to testify only on the condition of Betty when rushed to
through an unbroken chain of custody is enough to engender reasonable doubt on the the Hospital but not as to who caused the injury. The prosecution should call on Carla as
guilt of an accused (People vs. De Guzman Y Danzil). Nonetheless, non-compliance with the best witness to the incident. Offer of Evidence; Failure to Offer (2007) No.VII. (b) G
the procedure shall not render void and invalid the seizure and custody of the drugs files a complaint for recovery of possession and damages against F. In the course of the
when: (1) such non-compliance is attended by justifiable grounds; and (2) the integrity trial, G marked his evidence but his counsel failed to file a formal offer of evidence. F
and the evidentiary value of the seized items are properly preserved by the then presented in evidence tax declarations in the name of his father to establish that his
apprehending team. There must be proof that these two (2) requirements were met father is a co-owner of the property. The court ruled in favor of F, saying that G failed to
before such noncompliance may be said to fall within the scope of then proviso. (People prove sole ownership of the property in the face of F’s evidence. Was the court correct?
vs. Dela Cruz, G.R. No. 177222, October 29, 2008, 570 SCRA 273). ALTERNATIVE ANSWER: Explain briefly. (5%) SUGGESTED ANSWER: No, the trial court is not correct in ruling in
Crucial in proving chain of custody is the marking of the seized drugs or other related favor of F. Tax Declaration are not by themselves evidence of ownership; hence, they are
items immediately after they are seized from the accused. Marking after seizure is the not sufficient evidence to warrant a judgment that F‟s father is a co-owner of the
starting point in the custodial link, thus, it is vital that the seized contraband are property. Plaintiff‟s failure to make a formal offer of his evidence may mean a failure to
immediately marked because succeeding handlers of the specimens will use the prove the allegations in his complaint. However, it does not necessarily result in a
markings as reference. Thus, non-compliance by the apprehending/buy-bust team with judgment awarding co-ownership to the defendant. While the court may not consider
Sec.21 of R.A. 9165 is not fatal as long Remedial Law Q&As (2007-2013) evidence which is not offered, the failure to make a formal offer of evidence is a
hectorchristopher@yahoo.com JayArhSals “Never Let The Odds Keep You From Pursuing technical lapse in procedure that may not be allowed to defeat substantive justice. In the
What You Know In Your Heart You Were Meant To Do.”-Leroy Satchel Paige Page 87 of interest of justice, the court can require G to offer his evidence and specify the purpose
198 as there is justifiable ground therefor, and as long as the integrity and the thereof. Offer of Evidence; Fruit of a Poisonous Tree (2010) No. VIII. Dominique was
evidentiary value of the confiscated/seized items are properly preserved by the accused of committing a violation of the human Security Act. He was detained Remedial
apprehending officer/team. (People vs. Mantalaba, G.R. No. 186227, July 20, 2011). Law Q&As (2007-2013) hectorchristopher@yahoo.com JayArhSals “Never Let The Odds
Character Evidence; Bad Reputation (2010) No.XII. In a prosecution for murder, the Keep You From Pursuing What You Know In Your Heart You Were Meant To Do.”-Leroy
prosecutor asks accused Darwin if he had been previously arrested for violation of the Satchel Paige Page 90 of 198 incommunicado, deprived of sleep, and subjected to water
Anti- Graft and Corrupt Practices Act. As defense counsel, you object. The trial court asks torture. He later allegedly confessed his guilt via an affidavit. After trial, he was acquitted
you on what ground/s. Respond. (3%) SUGGESTED ANSWER: The objection is on the on the ground that his confession was obtained through torture, hence, inadmissible as
ground that the fact sought to be elicited by the prosecution is irrelevant and immaterial evidence. In a subsequent criminal case for torture against those who deprived him of
to the offense under prosecution and trial. Moreover, the Rules do not allow the sleep and subjected him to water torture. Dominique was asked to testify and to, among
prosecution to adduce evidence of bad moral character of the accused pertinent to the other things, identify his above said affidavit of confession. As he was about to identify
offense charged, except on rebuttal and only if it involves a prior conviction by final the affidavit, the defense counsel objected on the ground that the affidavit is a fruit of a
judgment (Rule 130, Sec. 51, Rules of Court). Doctrine of Adoptive Admission (2009) poisonous tree. Can the objection be sustained? Explain. (3%) SUGGESTED ANSWER: No,
No.I.D. Under the doctrine of adoptive admission, a third party’s statement becomes the the objection may not be sustained on the ground stated, because the affiant was only to
admission of the party embracing or espousing it. SUGGESTED ANSWER: TRUE. The effect identify the affidavit which is not yet being offered in evidence. The doctrine of the
or consequence of the admission will bind also the party who adopted or espoused the poisonous tree can only be invoked by Domingo as his defense in the crime of Violation
same, as applied in Estrada vs. Desierto, 356 SCRA 108 [2001]\. An adoptive admission is of Human Security Act filed against him but not by the accused torture case filed by him.
a party‟s reaction to a statement or action by another person when it is reasonable to In the instant case, the presentation of the affidavit cannot be objected to by the
treat the party‟s reaction as an admission of something stated or implied by the other defense counsel on the ground that is a fruit of the poisonous tree because the same is
person. Hearsay Evidence; Objection (2012) No.VII. (a) Counsel A objected to a question used in Domingo‟s favor. Offer of Evidence; Fruit of a Poisonous Tree (2009) No.VI.
posed by opposing Counsel B on the grounds that it was hearsay and it assumed a fact Arrested in a buy-bust operation, Edmond was brought to the police station where he
not yet established. The judge banged his gavel and ruled by saying "Objection was informed of his constitutional rights. During the investigation, Edmond refused to
Sustained". Can Counsel 8 ask for a reconsideration of the ruling? Why? (5%) SUGGESTED give any statement. However, the arresting officer asked Edmond to acknowledge in
ANSWER: Yes, Counsel B may ask the Judge to specify the ground‟s relied upon for writing that six (6) sachets of “shabu” were confiscated from him. Edmond consented
sustaining the objection and thereafter move its reconsideration thereof. (Rule 132, and also signed a receipt for the amount of P3,000, allegedly representing the “purchase
Sec.38, Rules of Court). Remedial Law Q&As (2007-2013) hectorchristopher@yahoo.com price of the shabu.” At the trial, the arresting officer testified and identified the
JayArhSals “Never Let The Odds Keep You From Pursuing What You Know In Your Heart documents executed and signed by Edmond. Edmond’s lawyer did not object to the
You Were Meant To Do.”-Leroy Satchel Paige Page 88 of 198 Hearsay Rule (2007) No.III. testimony. After the presentation of the testimonial evidence, the prosecutor made a
(a) What is the hearsay rule? (5%) SUGGESTED ANSWER: The hearsay rule is a rule of formal offer of evidence which included the documents signed by Edmond. Edmond’s
evidence to the effect that a witness can testify only to those facts which he knows of his lawyer object to the admissibility of the document for being the fruit of the poisoned
own knowledge or derived from his own perceptions, except as otherwise provided in tree. Resolve the objection with reasons. (3%) SUGGESTED ANSWER: The objection to
the rules of court (Rule 130, Sec. 36 Rules of Court). (b) In relation to the hearsay rule, the admissibility of the documents which the arresting officer asked Edmond to sign
what do the following rules of evidence have in common? (5%) (1) The rule on without the benefit of counsel, is well-taken. Said documents having been signed by the
statements that are part of the res gestae. (2) The rule on dying declarations. (3) The rule Remedial Law Q&As (2007-2013) hectorchristopher@yahoo.com JayArhSals “Never Let
on admissions against interest. SUGGESTED ANSWER: The rules on the evidence specified The Odds Keep You From Pursuing What You Know In Your Heart You Were Meant To
in the question asked, have in common the following: (1) The evidence although hearsay, Do.”-Leroy Satchel Paige Page 91 of 198 accused while under custodial investigation,
are allowed by the Rules as exceptions to the hearsay rule; (2) The facts involved are imply an “admission” without the benefit of counsel, that the shabu came from him and
admissible in evidence for reasons of necessity and trustworthiness; and (3) The witness that the P3,000,00 was received by him pursuant to the illegal selling of the drugs. Thus,
is testifying on facts which are not of his own knowledge or derived from his own it was obtained by the arresting officer in clear violation of Sec. 12 (3), Art. III of the 1987
perception. Hearsay; Inapplicable (2009) No.XIII. [b] Blinded by extreme jealousy, Alberto Constitution, particularly the right to be assisted by counsel during custodial
shot his wife, Betty, in the presence of his sister, Carla. Carla brought Betty to the investigation. Moreover, the objection to the admissibility of the evidence was timely
hospital. Outside the operating room, Carla told Domingo, a male nurse, that it was made, i.e., when the same is formally offered. Privilege Communication (2013) No.IX. For
Alberto who shot Betty. Betty died while undergoing emergency surgery. At the trial of over a year, Nenita had been estranged from her husband Walter because of the latter’s
the parricide charges filed against Alberto, the prosecutor sought to present Domingo as suspicion that she was having an affair with Vladimir, a barangay kagawad who lived in
witness, to testify on what Carla told him. The defense counsel objected on the ground nearby Mandaluyong. Nenita lived in the meantime with her sister in Makati. One day,
that Domingo’s testimony is inadmissible for being hearsay. Rule on the objection with the house of Nenita’s sister inexplicably burned almost to the ground. Nenita and her
reasons. (3%) SUGGESTED ANSWER: Objection overruled. The disclosure received by sister were caught inside the house but Nenita survived as she fled in time, while her
Domingo and Carla may be regarded as independently relevant statement which is not sister tried to save belongings and was caught inside when the house collapsed. As she
covered by the hearsay rule; hence admissible. The statement may be received not as was running away from the burning house, Nenita was surprised to see her husband also
evidence of the truth of what was stated but only as to the tenor thereof and the running away from the scene. Dr. Carlos, Walter’s psychiatrist who lived near the burned
Remedial Law Q&As (2007-2013) hectorchristopher@yahoo.com JayArhSals “Never Let house and whom Walter medically consulted after the fire, also saw Walter in the vicinity
The Odds Keep You From Pursuing What You Know In Your Heart You Were Meant To some minutes before the fire. Coincidentally, Fr. Platino, the parish priest who regularly
Do.”-Leroy Satchel Paige Page 89 of 198 occurrence when it was said, independently of hears Walter’s confession and who heard it after the fire, also encountered him not too
whether it was true or false. (People v. Cloud, 333 Phil. 30 [1996]; People v. Malibiran, et far away from the burned house. Walter was charged with arson and at his trial, the
al., G.R. No. 178301, April 24, 2009). ALTERNATIVE ANSWER: Objection sustained. The prosecution moved to introduce the testimonies of Nenita, the doctor and the priest-
disclosure made by Carla has no other probative value except to identify who shot Betty. confessor, who all saw Walter at the vicinity of the fire at about the time of the fire. (A)
Its tenor is irrelevant to the incident, and the same was made not to a police investigator May the testimony of Nenita be allowed over the objection of Walter? (3%) SUGGESTED
of the occurrence but to a nurse whose concern is only to attend to the patient. Hence, ANSWER: No. Nenita may not be allowed to testify against Walter. Under the Marital
Disqualification Rule, during their marriage, neither the husband nor the wife may testify sent written interrogatories to Ely, asking whether statements f witnesses were
for or against the other without the consent of the affected spouse, except in a civil case obtained; if written copies were to be furnished; if oral, the exact provision were to be
by one against the other, or in a criminal case for a crime committed by one against the set forth in detail. Ely refused to comply, arguing that the documents and information
other or the latter‟s direct descendants or ascendants (Section 22, Rule 130, Rules on asked are privileged communication. Is the contention tenable? Explain (4%) SUGGESTED
Evidence). The foregoing exceptions cannot apply since Remedial Law Q&As (2007-2013) ANSWER: Yes, the lawyer-client privilege covers any communication made by the client
hectorchristopher@yahoo.com JayArhSals “Never Let The Odds Keep You From Pursuing to the lawyer, or the lawyer‟s advice given thereon in the course of, or with a view to
What You Know In Your Heart You Were Meant To Do.”-Leroy Satchel Paige Page 92 of professional employment. The documents and information sought were gathered and
198 it only extends to a criminal case of one spouse against the other or the latter‟s prepared pursuant to the engagement of Ely as a lawyer for the company (Air Philippines
direct ascendants or descendants. Clearly, Nenita is not the offended party and her sister Corporation v. Pennswell, Inc., GR No. 172835, 13 December 2007). Sec. 5, Rule 25 of the
is not her direct ascendant or descendant for her to fall within the exception. Rules of Court provides that interrogatories may relate to any matter that can be
ALTERNATIVE ANSWER: Yes. Nenita may be allowed to testify against Walter. It is well required into under Sec. 2, Rule 23 o depositions and discovery refers to privileged
settled that the marital disqualification rule does not apply when the marital and confidential communications under Sec. 24, Rule 130. Privilege Communication; Marital
domestic relations between spouses are strained. In Alvarez vs. Ramirez, G.R. No. Privilege (2010) No. I. On March 12, 2008, Mabini was charged with Murder for fatally
143439, October 14, 2005, the Supreme Court citing People vs. Castaneda, 271 SCRA stabbing Emilio. To prove the qualifying circumstance of evident premeditation, the
504, held that the act of private respondent in setting fire to the house of his sister-in- prosecution introduced on December 11, 2009 a text message, which Mabini’s estranged
law Susan Ramirez, Knowing fully well that his wife was there, and in fact with the wife Gregoria had sent to Emilio on the eve of his death, reading: “Honey, pa2tayin u ni
alleged intent of injuring the latter, is an act totally alien to the harmony and confidences Mabini. Mtgal n nyang plano i2. Mg ingat u bka ma tsugi k.” (A) A subpoena ad
of marital relation which the disqualification primarily seeks to protect. The criminal act testificandum was served on Gregoria for her to be presented for the Remedial Law
complained of had the effect of directly and vitally impairing the conjugal relation. It Q&As (2007-2013) hectorchristopher@yahoo.com JayArhSals “Never Let The Odds Keep
underscored the fact that the marital and domestic relations between her and the You From Pursuing What You Know In Your Heart You Were Meant To Do.”-Leroy Satchel
accused-husband have become so strained that there is no more harmony, peace or Paige Page 95 of 198 purpose of identifying her cellphone and the tex message. Mabini
tranquillity to be preserved. Hence, the identity is nonexistent. In such a situation, the objected to her presentation on the ground of marital privilege. Resolve. SUGGESTED
security and confidences of private life which the law aims to protect are nothing but ANSWER: The objection should be sustained on the ground of the marital disqualification
ideals which through their absence, merely leave a void in the unhappy home. Thus, rule (Rule 130, Sec. 22), not on the ground of the “marital privilege” communication rule
there is no reason to apply the Marital Disqualification Rule. (B) May the testimony of Dr. (Rule 130, Sec. 24). The marriage between Mabini and Gregoria is still subsisting and the
Carlos, Walter’s psychiatrist, be allowed over Walter’s objection? (3%) SUGGESTED situation at bar does not come under the exceptions to the disqualification by reason of
ANSWER: Yes. The testimony of Walter‟s psychiatrist may be allowed. The privileged marriage. (B) Suppose Mabini’s objection in question A was sustained. The prosection
communication contemplated under Sec. 24 (c) Rule 130 of the Rules on Evidence thereupon announced that it would be presenting Emilio’s wife Graciana to identify
involves only persons authorized to practice medicine, surgery or obstetrics. It does not Emilio’s cellphone bearing Gregoria’s text message. Mabini objected again. Rule on the
include a Psychiatrist. Moreover, the privileged communication applies only in civil cases objection. (2%) SUGGESTED ANSWER: The objection should be overruled. The testimony
and not in a criminal case for arson. Besides, the subject of the testimony of Dr. Carlos of Graciana is not covered by the said marital disqualification rule because she is not the
was not in connection with the advice or treatment given by him to Walter, or any wife of Mabini. Besides, Graciana will identify only the cellphone as that of her husband
information he acquired in attending to Walter in a professional capacity. The testimony Emilio, not the messages therein which to her are hearsay. (C) If Mabini’s objection in
of Dr. Carlos is limited only to what he perceived at the vicinity of the fire and at the time question B was overruled, can he object to the presentation of the text message on the
of the fire. Remedial Law Q&As (2007-2013) hectorchristopher@yahoo.com JayArhSals ground that it is hearsay? (2%) SUGGESTED ANSWER: No, Gregoria‟s text message in
“Never Let The Odds Keep You From Pursuing What You Know In Your Heart You Were Emilio‟s cellphone is not covered by the hearsay rule because it is regarded in the rules
Meant To Do.”-Leroy Satchel Paige Page 93 of 198 (C) May the testimony of Fr. Platino, of evidence as independently relevant statement: the text message is not to prove the
the priest-confessor, be allowed over Walter’s objection? (3%) SUGGESTED ANSWER: truth of the fact alleged therein but only as to the circumstances of whether or not
Yes. The Priest can testify over the objection of Walter. The disqualification requires that premeditation exists. (C) Suppose that shortly before expired, Emilio was able to send a
the same were made pursuant to a religious duty enjoined in the course of discipline of text message to his wife Graciana reading “Nasaksak ako. D na me makahinga. Si Mabini
the sect or denomination to which they belong and must be confidential and penitential ang may gawa ni2.” Is this message admissible as a dying declaration? Explain. (3%)
in character, e.g., under the seal of confession (Sec. 24 (d) Rule 130, Rules on Evidence). SUGGESTED ANSWER: Yes, the text message is admissible as a dying declaration since the
Here, the testimony of Fr. Platino was not previously subject of a confession of Walter or same came fdrom the victim who “shortly” expired and it is in respect of the cause and
an advice given by him to Walter in his professional character. The Testimony was merely circumstance of his death. The decisive factor that the message was made and sent
limited to what Fr. Platino perceived “at the vicinity of the fire and at about the time of under consciousness of an impending death, is evidently attendant from the victim‟s
the fire.” Hence, Fr. Platino may be allowed to testify. Privilege Communication; Lawyer- statement: “D na me makahinga” and the fact that he died shortly after he sent the text
Client (2008) No.XIV. On August 15, 2008, Edgardo committed estafa against Petronilo in message. Remedial Law Q&As (2007-2013) hectorchristopher@yahoo.com JayArhSals
the amount of P3 Million. Petronilo brought his complaint to the National Bureau of “Never Let The Odds Keep You From Pursuing What You Know In Your Heart You Were
Investigation, which found that Edgardo had visited his lawyer twice, the first time on Meant To Do.”-Leroy Satchel Paige Page 96 of 198 However, cellphone messages are
August 14, 2008 and the second on August 16, 2008; and that both visits concerned the regarded as electronic evidence, and i a recent case (Ang vs. Court of Appeals et al., GR
swindling of Petronilo. During the trial of Edgardo, the RTC issued a subpoena ad NO. 182835, April 20, 2010), the Supreme Court ruled that the Rules on Electronic
testificandum to Edgardo’s lawyer for him to testify on the conversations during their Evidence applies only to civil actions, quasi-judicial proceedings and administrative
first and second meetings. May the subpoena be quashed on the ground of privileged proceeding, not to criminal actions. ALTERNANTIVE ANSWER: No, the text message is not
communication? Explain fully. (4%) SUGGESTED ANSWER: Yes, the mantle of privileged admissible as a dying declaration because it lacks indication that the victim was under
communication based on lawyer-client relationship protects the communication consciousness of an impending death. The statement “D na me makahinga” is still
between a lawyer and his client against any adverse party as in this case. The subpoena unequivocal in the text message sent that does not imply consciousness of forth-coming
requiring the lawyer to testify can be quashed on the ground of privileged death. Witness; Examination of Witness (2009) No.1.[b] The One-Day Examination of
communication (See Regala v. Sandiganbayan, GR No. 105938, 20 September 1996). Sec. witness Rule abbreviates court proceedings by having a witness fully examined in only
24 (b) Rule 130 provides that an attorney cannot, without the consent of his client be one day during trial. SUGGESTED ANSWER: TRUE. Par. 5(i) of Supreme Court A.M. No. 03-
examined in any communication made to him by his client to him, or his advice given 1-09-SC requires that a witness has to be fully examined in one (1) day only. This rule
thereon, including his secretary, stenographer, clerk concerning any fact the knowledge shall be strictly adhered to subject to the court‟s discretion during trial on whether or
of which has been acquired in such capacity. However, where the subject matter of the not to extend the direct and/or cross-examination for justifiable reasons. On the last
communication involves the commission of the crime, in which the lawyer himself hearing day allotted for each party, he is required to make his formal offer of evidence
Remedial Law Q&As (2007-2013) hectorchristopher@yahoo.com JayArhSals “Never Let after the presentation of his last witness and the opposing party is required to
The Odds Keep You From Pursuing What You Know In Your Heart You Were Meant To immediately interpose his objection thereto. Thereafter, the judge shall make the ruling
Do.”-Leroy Satchel Paige Page 94 of 198 is a participant or conspirator, then the same is on the offer of evidence in open court. However, the judge has the discretion to allow
not covered by the privilege. Moreover, if the substance of the communication can be the offer of evidence in writing in conformity with Section 35, Rule 132. ALTERNATIVE
established by independent evidence, the lawyer maybe compelled to testify. Privilege ANSWER: FALSE. This rule is not absolute: it will still allow the trial judge the discretion
Communication; Lawyer-Client (2008) No.XX. A tugboat owned by Speedy Port Service, whether to extend the direct and/or cross examination for justifiable reasons or not. The
Inc. (SPS) sank in Manila Bay while helping tow another vessel, drowning five (5) crews in exercise of this discretion may still result in wrangling as to the proper exercise of the
the resulting shipwreck. At the maritime board inquiry, the four (4) survivors testified. trial court‟s discretion, which can delay the proceedings. Summary Procedure Prohibited
SPS engaged Atty. Ely to defend it against potential claims and to sue the company Pleadings (2010) No. X. Marinella is a junior officer of the Armed Forces of the Philippines
owning the other vessel for damages to tug. Ely obtained signed statements from the who claims to have personally witnessed the malversation of funds given by US Remedial
survivors. He also interviewed other persons, in some instance making memoranda. The Law Q&As (2007-2013) hectorchristopher@yahoo.com JayArhSals “Never Let The Odds
heirs of the five (5) victims filed an action for damages against SPS. Plaintiffs’ counsel Keep You From Pursuing What You Know In Your Heart You Were Meant To Do.”-Leroy
Satchel Paige Page 97 of 198 authorities in connection with the Balikatan exercises. charged with having violated Anti-Graft Law (Deloso vs. Sandiganbayan, G.R. No. 86899,
Marinella alleges that as a result of her exposé, there are operatives within the military May 15, 1989); (3) a ViceGovernor, whose suspension is predicated on his acts
who are out to kill her. She files a petition for the issuance of a writ of amparo against, supposedly committed while still a member of the Sangguniang Bayan (Libanan vs.
among others, the Chief of Staff but without alleging that the latter ordered that she be Sandiganbayan, G.R. No. 112386, June 14, 1994). Thus, the DENR undersecretary can be
killed. Atty. Daro, counsel for the Chief of Staff, moves for the dismissal of the Petition preventively suspended even though he was a mayor, when he allegedly committed
for failure to allege that his client issued any order to kill or harm Marinella. Rule on Atty. malversation. Remedial Law Q&As (2007-2013) hectorchristopher@yahoo.com
Daro’s motion. Explain. (3%) SUGGESTED ANSWER: The motion to dismiss must be JayArhSals “Never Let The Odds Keep You From Pursuing What You Know In Your Heart
denied on the ground that it is a prohibited pleading under Section 11 (a) of the Rule on You Were Meant To Do.”-Leroy Satchel Paige Page 100 of 198 Settled is the rule that
the Writ of Amparo. Moreover, said Rule does not require the petition therefor to allege where the accused files a motion to quash the information or challenges the validity
a complete detail of the actual or threatened violation of the victim‟s rights. It is thereof, a show cause order of the trial court would no longer be necessary. What is
sufficient that there be an allegation of real threat against petitioner‟s life, liberty, indispensable is that the trial court duly hear the parties at a hearing held for
and/or security (Gen. A. Razon, Jr. vs. Tagitis, G.R. No. 182498, Dec. 03, 2009). determining the validity of the information, and thereafter hand down its ruling, issuing
Miscellaneous Alternative Dispute Resolution; Court Diversion; Stages (2012) No.VIII.B. the corresponding order of suspension should it uphold the validity of the information
Discuss the three (3) Stages of Court Diversion in connection with Alternative Dispute (Luciano vs. Mariano, G.R. No. L-32950, July 30, 1971). Since a pre-suspension hearing is
Resolution. (5%) SUGGESTED ANSWER: The three stages of diversion are CourtAnnexed basically a due process requirement, when an accused public official is given an adequate
Mediation (CAM), Judicial Dispute Resolution, and Appeals Court Mediation (ACM). opportunity to be heard on his possible defenses against the mandatory suspension
During CAM, the judge refers the parties to the Philippine Mediation Center (PMC) for under RA No. 3019, then an accused would have no reason to complain that no actual
the mediation of their dispute by trained and accredited mediators. If CAM fails, the JDR hearing was conducted (Miguel vs. The Honorable Sandiganbayan, G.R. No. 172035, July
is undertaken by the JDR judge, acting as a mediator-conciliator-early neutral evaluator. 4, 2012). In the facts given, the DENR Undersecretary was already given opportunity to
The third case is during appeal, where covered cases are referred to ACM. A.M. No. 09-6- question the validity of the Information for malversation by filing a motion to quash, and
8-SC; Precautionary Principle (2012) No.II.B. What do you understand about the yet, the Sandiganbayan sustained its validity. There is no necessity for the court to
"precautionary principle" under the Rules of Procedure for Environmental Cases? (5%) conduct pre-suspension hearing to determine for the second time the validity of the
SUGGESTED ANSWER: Precautionary principles states that when human activities may information for purpose of preventively suspending the accused. ALTERNATIVE ANSWER:
lead to threats of serious and irreversible Remedial Law Q&As (2007-2013) The argument that X should not be suspended as he now holds an office different from
hectorchristopher@yahoo.com JayArhSals “Never Let The Odds Keep You From Pursuing that charged in the information is unavailing. Under Section 13(e) of RA 3019, a public
What You Know In Your Heart You Were Meant To Do.”-Leroy Satchel Paige Page 98 of officer may be charged before the Sandiganbayan for “causing undue injury to any party,
198 damage to the environment that is scientifically plausible but uncertain, actions shall including the Government, or giving any private party any unwarranted benefits,
be taken to avoid or diminish that threat. In its essence, the precautionary principle calls advantage or preference in the discharge of his official, administrative or judicial
for the exercise of caution in the face of risk and uncertainty (Sec. 4 [f], Rule 1, Part 1, functions through manifest partiality, evident bad faith or gross inexcusable negligence.”
and Rule 20, A.M. No. 09-6-8-SC, Rules of Procedure for Environment Cases). Habeas The Supreme Court has held that Section 13 of RA 3019 is so clear and explicit that there
Data (2010) No.XX. Azenith, the cashier of Temptation Investments, Inc. (Temptation, is hardly room for any extended court rationalization of the law. Preventive suspension is
Inc.) with principal offices in Cebu City, is equally hated and loved by her co-employees mandatory regardless of the respondent‟s change in position. R.A. 3019; Remedies
because she extends cash advances or "vales " to her colleagues whom she likes. One (2013) No.VII. You are the defense counsel of Angela Bituin who has been charged under
morning, Azenith discovers an anonymous letter inserted under the door of her office RA 3019 ( Anti-Graft and Corrupt Practices Act ) before the Sandiganbayan. While Angela
threatening to kill her. Azenith promptly reports the matter to her superior Joshua, who has posted bail, she has yet to be arraigned. Angela revealed to you that she has not
thereupon conducts an internal investigation to verify the said threat. Claiming that the been investigated for any offense and that it was only when police officers showed up at
threat is real, Temptation, Inc. opts to transfer Azenith to its Palawan Office, a move she her residence with a warrant Remedial Law Q&As (2007-2013)
resists in view of the company’s refusal to disclose the results of its investigation. hectorchristopher@yahoo.com JayArhSals “Never Let The Odds Keep You From Pursuing
Decrying the move as a virtual deprivation of her employment, Azenith files a petition for What You Know In Your Heart You Were Meant To Do.”-Leroy Satchel Paige Page 101 of
the issuance of a writ of habeas data before the Regional Trial Court (RTC) to enjoin 198 of arrest that she learned of the pending case against her. She wonders why she has
Temptation, Inc. from transferring her on the ground that the company’s refusal to been charged before the Sandiganbayan when she is not in government service. (A)
provide her with a copy of the investigation results compromises her right to life, liberty What "before-trial" remedy would you invoke in Angela’s behalf to address the fact that
and privacy. Resolve the petition. Explain. (5%) SUGGESTED ANSWER: Azenith‟s petition she had not been investigated at all, and how would you avail of this remedy? (4%)
for the issuance of a writ of habeas data must be dismissed as there is no showing that SUGGESTED ANSWER: I will file a Motion for the conduct of preliminary investigation or
her right to privacy in life, liberty, or security is violated or threatened by an unlawful act reinvestigation and the quashal or recall of the warrant of arrest in the Court where the
or omission. Neither was the company shown to be engaged in the gathering, collecting case is pending with an additional prayer to suspend the arraignment. Under Section 6 of
nor storing of data or information regarding the person, family, home and Rule 112 of the Rules of Court, after the filing of the complaint or information in court
correspondence of the aggrieved party (Sec. 1, Rule on the Writ of Habeas Data). Habeas without a preliminary investigation, the accused may within five days from the time he
Data (2009) No.XIX.C. What is the writ of habeas data? SUGGESTED ANSWER: A writ of learns of its filing ask for preliminary investigation with the same right to adduce
habeas data is a remedy available to any persons whose right to privacy in life, liberty, or evidence in his defense. Moreover, Section 26, Rule 114 of the Rules on Criminal
security is Remedial Law Q&As (2007-2013) hectorchristopher@yahoo.com JayArhSals Procedure provides that an application for or admission to bail shall not bar the accused
“Never Let The Odds Keep You From Pursuing What You Know In Your Heart You Were from challenging the validity of his arrest or legality of the warrant issued therefor, or
Meant To Do.”-Leroy Satchel Paige Page 99 of 198 violated or threatened with violation from assailing the regularity or questioning the absence of a preliminary investigation of
by unlawful act or omission of a public official or employee, or of a private individual or the charge against him, provided that he raises them before entering his plea. The court
entity engaged in the gathering, collecting, or storing of data or information regarding shall resolve the matter as early as practicable but not later than the start of the trial of
the person, family, home and correspondence of the aggrieved party. R.A. 3019; Pre- the case. ALTERNATIVE ANSWER: I will file a Motion to Quash on the ground that the
Suspension Hearing (2012) No.IX.A. X, an undersecretary of DENR, was charged before Sandiganbayan has no jurisdiction over the person of the accused (Section 3, Rule 117 of
the Sandiganbayan for malversation of public funds allegedly committed when he was the Rules of Criminal Procedure). The Sandiganbayan has exclusive original jurisdiction
still the Mayor of a town in Rizal. After arraignment, the prosecution moved that X be over violations of R.A. 3019 (Anti-graft and Corrupt Practices law) where one or more of
preventively suspended. X opposed the motion arguing that he was now occupying a the accused are officials occupying the enumerated positions in the government whether
position different from that which the Information charged him and therefore, there is in a permanent, acting, or interim incapacity, at the time of the commission of the
no more possibility that he can intimidate witnesses and hamper the prosecution. offense (Sec. 4, R.A. 8249). In Bondoc vs. Sandiganbayan, G.R. No. 71163-65, November
Decide. Suppose X files a Motion to Quash challenging the validity of the Information and 9, 1990, the Supreme Court held that before the Sandiganbayan may lawfully try a
the Sandiganbayan denies the same, will there still be a need to conduct a presuspension private individual under PD 1606, the following requisites must be established: (a) he
hearing? Explain. (5%) SUGGESTED ANSWER: There is no necessity for the court to must be charged with a public officer/employee; and (b) he must be tried jointly. Since
conduct pre-suspension hearing. Under Section 13 of RA No. 3019, an incumbent public the aforementioned Remedial Law Q&As (2007-2013) hectorchristopher@yahoo.com
officer against whom any criminal prosecution under a valid information for graft-related JayArhSals “Never Let The Odds Keep You From Pursuing What You Know In Your Heart
crime such as malversation is pending in court, shall be suspended from office. The word You Were Meant To Do.”-Leroy Satchel Paige Page 102 of 198 requisites are not present,
“office”, from which the public officer charged shall be preventively suspended, could the Sandiganbayan has no jurisdiction. (B) What "during-trial" remedy can you use to
apply to any office, which he might currently be holding and not necessarily the allow an early evaluation of the prosecution evidence without the need of presenting
particular office under which he was charged. The preventive suspension of the following defense evidence; when and how can you avail of this remedy? (4%) SUGGESTED
public officers was sustained: (1) a mayor, who was charged with acts committed as a ANSWER: I will file a Motion for Leave to file a Demurrer to Evidence within five (5) days
government auditor of the Commission on Audit (Bayot vs. Sandiganbayan, G.R. No. L- from the time the prosecution has rested its case. If the motion is granted, I will file a
61776 to L-61861, March 23, 1984); (2) a public officer, who was already occupying the demurrer to evidence within a non-extendible period of Ten (10) days from notice.
office of governor and not the position of municipal mayor that he held previously when However, if the motion for leave to file demurrer to evidence is denied, I can adduce
evidence for the accused during the trial to meet squarely the reasons for its denial documents and any printout or output, readable by sight or other means, which
(Section 23, Rule 119, Rules of Criminal Procedure). This remedy would allow the early accurately reflects the electronic data message or electronic document. For purposes of
evaluation of the sufficiency of prosecution‟s evidence without the need of presenting these Rules, the term “electronic document” may be used interchangeably with
defense evidence. It may be done through the court‟s initiative or upon motion of the “electronic data message” (Section 1, (g), (h) Rule 2, AM No. 01-7-01-SC, Rules on
accused and after the prosecution rested its case. Small Claims (2013) No.X. As a new Electronic Evidence). In MCC Industrial Sales Corporation vs. Ssangyong Corporation, G.R.
lawyer, Attorney Novato limited his practice to small claims cases, legal counseling and No. 170633, the Supreme Court held that R.A. No. 8792, otherwise known as the
the notarization of documents. He put up a solo practice law office and was assisted by Electronic Commerce Act of 2000, considers an electronic data message or an electronic
his wife who served as his secretary/helper. He used a makeshift hut in a vacant lot near document as functional equivalent of a written document for evidentiary purposes. The
the local courts and a local transport regulatory agency. With this practice and location, Rules on Electronic Evidence regards an electronic document as admissible in evidence if
he did not have big-time clients but enjoyed heavy patronage assisting walk-in clients. (A) it complies with the rules on admissibility prescribed by the Rules of Remedial Law Q&As
What role can Attorney Novato play in small claims cases when lawyers are not allowed (2007-2013) hectorchristopher@yahoo.com JayArhSals “Never Let The Odds Keep You
to appear as counsel in these cases? (3%) SUGGESTED ANSWER: Atty. Novata may From Pursuing What You Know In Your Heart You Were Meant To Do.”-Leroy Satchel
provide legal assistance to his clients by giving counselling and guidance in the Paige Page 106 of 198 Court and related laws, and is authenticated in the manner
preparation and accomplishment of the necessary documents and Affidavits to initiate or prescribed by the said Rules. An electronic document is also the equivalent of an original
defend a small claims action including the compilation and notarization of the document under the Best Evidence Rule, if it is a printout or output readable by sight or
aforementioned documents, if necessary. (B) What legal remedy, if any, may Attorney other means, shown to reflect the data accurately. (D), If the testimony is being offered
Novato pursue for a client who loses in a small claims case and before which tribunal or for the purpose of establishing that such statements were made, then the testimony is
court may this be pursued? (4%) SUGGESTED ANSWER: Atty. Novata may file a petition admissible as independent relevant statement. The Doctrine on independent relevant
for Certiorari under Rule 65 of the Rules of Remedial Law Q&As (2007-2013) statement holds that conversations communicated to a witness by a third person may be
hectorchristopher@yahoo.com JayArhSals “Never Let The Odds Keep You From Pursuing admitted as proof, regardless of their truth or falsity, that they were actually made
What You Know In Your Heart You Were Meant To Do.”-Leroy Satchel Paige Page 103 of (Republic vs. Heirs of Alejaga Sr., G.R. No. 146030, December 3, 2002). The doctrine of
198 Court before the RTC since a decision in small claims cases is final and unappealable independently relevant statements is an exception to hearsay rule. It refers to the fact
(Sec. 23, A.M. No. 8-8-7 SC, Rules of Procedure for Small Claims Cases). The petition for that such statements were made is relevant, and the truth or falsity thereof is
certiorari should be filed before the RTC conformably to the Principle of judicial immaterial. The hearsay rule does not apply: hence, the statements are admissible as
Hierarchy. Writ of Amparo; Habeas Corpus (2009) No.XIX.B. What is the writ of amparo? evidence. Evidence as to the making of such statement is not secondary but primary, for
How is it distinguished from the writ of habeas corpus? SUGGESTED ANSWER: A writ of the statement itself may constitute a fact in issue or be circumstantially relevant as to
amparo is a remedy available to any person whose right to life, liberty, and security is the existence of such a fact. The witness who testifies thereto is competent because he
violated or threatened with violation by an unlawful act or omission of a public official or heard the same, as this is a matter of fact derived from his own perception, and the
employee, or of a private individual or entity. The writ shall cover extralegal killings and purpose is to prove either that the statement was made or the tenor thereof (People vs.
enforced disappearances or threats thereof. Whereas a writ of habeas corpus is a Malibiran, G.R. No. 178301, April 24, 2009, AustriMartinez, J.). (E), The problem does not
remedy available to any individual who is deprived of liberty or whose rightful custody of clearly provide the purposes for which the evidence under (C) and (D) are being offered.
any person is withheld, by unlawful confinement or detention. A writ of amparo may be Moreover, all of the choices above cannot be admitted to prove the truth of the contents
appealed to the Supreme Court under Rule 45 raising questions of fact or law or both. thereof for the reason that the evidence is not competent. For letter (A), the affiant is
The appeal shall be made within 5 working days from the date of notice of the adverse not presented, and hence hearsay. Letter (B), the admission was made after the
judgment. The period for appeal for habeas corpus shall be 48hours from the notice of termination of the conspiracy and extrajudicial, hence there is no application of the Res
the judgment appealed from. -End- Remedial Law Q&As (2007-2013) Inter Alios Acta rule. Letter (C) is also not allowed as under the Electronic Evidence Rule,
hectorchristopher@yahoo.com JayArhSals “Never Let The Odds Keep You From Pursuing the output readable by sight is the best evidence to prove the contents thereof. Letter
What You Know In Your Heart You Were Meant To Do.”-Leroy Satchel Paige Page 104 of (D) is hearsay since the affiant does not have personal knowledge. III. Leave of court is
198 MULTIPLE CHOICE QUESTIONS (MCQ) 2013 Remedial Law Exam MCQ (October 27, required to amend a complaint or information before arraignment if the amendment
2013) 2013 Bar Examination Questionnaire for Remedial Law MULTIPLE CHOICE __________. (1%) Remedial Law Q&As (2007-2013) hectorchristopher@yahoo.com
QUESTIONS I. In a complaint filed by the plaintiff, what is the effect of the defendant’s JayArhSals “Never Let The Odds Keep You From Pursuing What You Know In Your Heart
failure to file an answer within the reglementary period? (1%) (A) The court is allowed to You Were Meant To Do.”-Leroy Satchel Paige Page 107 of 198 (A) upgrades the nature of
render judgment motu proprio in favor of the plaintiff. (B) The court motu proprio may the offense from a lower to a higher offense and excludes any of the accused (B)
declare the defendant in default, but only after due notice to the defendant. (C) The upgrades the nature of the offense from a lower to a higher offense and adds another
court may declare the defendant in default but only upon motion of the plaintiff and accused (C) downgrades the nature of the offense from a higher to a lower offense or
with notice to the defendant. (D) The court may declare the defendant in default but excludes any accused (D) downgrades the nature of the offense from a higher to a lower
only upon motion of the plaintiff, with notice to the defendant, and upon presentation of offense and adds another accused (E) All the above choices are inaccurate. SUGGESTED
proof of the defendant‟s failure to answer. (E) The above choices are all inaccurate. ANSWER: (C), Under Section 14 of Rule 110 of the Rules of Criminal Procedure, any
SUGGESTED ANSWERS: (D), Under Section 3 of Rule 9, if the defending party fails to amendment before plea, which downgrades the nature of the offense charged in or
answer within the time allowed, the court shall, upon motion of the claiming party with excludes any accused from the complaint or information, can be made only upon motion
notice to the defending party, and proof of such failure, declare the defending party in by the prosecutor, with notice to the offended party and with the leave of court. IV. A
default (Narciso vs. Garcia, G.R. No. 196877, November 21, 2012, Abad J.). (E), D may not Small Claims Court __________. (1%) (A) has jurisdiction over ejectment actions (B) has
be the correct answer because the Rule provides that if the defending party fails to limited jurisdiction over ejectment actions (C) does not have any jurisdiction over
answer within the time allowed therefor, the court shall, upon motion of the claiming ejectment actions (D) does not have original, but has concurrent, jurisdiction over
party with notice to the defending party, and proof of such failure, declare the defending ejectment actions (E) has only residual jurisdiction over ejectment actions SUGGESTED
party in default. Notably, the Rule uses the word “shall and not may.” II. Which of the ANSWER: (C), Under Section 4 of A.M. No. 8-8-7- SC, Rules of Procedure of Small Claims,
following is admissible? (1%) (A) The affidavit of an affiant stating that he witnessed the Small claims court shall have jurisdiction over all actions which are: (a) purely civil in
execution of a deed of sale but the affiant was not presented as a witness in the trial. nature where the claim or relief prayed for by the plaintiff is solely for payment or
Remedial Law Q&As (2007-2013) hectorchristopher@yahoo.com JayArhSals “Never Let reimbursement of sum of money, and (b) the civil aspect of criminal actions, either filed
The Odds Keep You From Pursuing What You Know In Your Heart You Were Meant To before the institution of the criminal action, or reserved upon the filing of the criminal
Do.”-Leroy Satchel Paige Page 105 of 198 (B) The extra judicial admission made by a action in court, pursuant to Rule 111 of the Revised Rules of Criminal Procedure. It does
conspirator against his co-conspirator after the conspiracy has ended. (C) The testimony not include ejectment actions. Moreover, the action allowed under the Rules on Small
of a party‟s witness regarding email messagesthe witness received from the opposing claims refers only to money under a lease contract. It does not necessarily refer to an
party. (D) The testimony of a police officer that he had been told by his informants that ejectment suit. Remedial Law Q&As (2007-2013) hectorchristopher@yahoo.com
there were sachets of shabu in the pocket of the defendant. (E) None of the above. JayArhSals “Never Let The Odds Keep You From Pursuing What You Know In Your Heart
SUGGESTED ANSWERS: (C), (D), or (E) (C), The E-mail messages are considered electronic You Were Meant To Do.”-Leroy Satchel Paige Page 108 of 198 At any rate, Section 33 of
data message or electronic document under the Rules on Electronic Evidence and Batas Pambansa Blg 129, as amended by Section 3 of R.A> 7691, as well as Section 1,
therefore admissible as evidence. The terms “electronic data message” and “electronic Rule 70 of the Rules of Court, clearly provides that forcible entry and unlawful detainer
document” are defined in the Rules on Electronic Evidence. Thus: (g) “Electronic data cases fall within the exclusive jurisdiction of the Metropolitan Trial Courts, Municipal
message” refers to information generated, sent, received or stored by electronic, optical Trial Courts and Municipal Circuit Trial Courts (Estel vs. Recaredo Diego, Sr. And Recaredo
or similar means. (h) “Electronic document” refers to information or the representation Diego, Jr., G.R. No. 174082, January 16, 2012, Peralta, J.). V. Character evidence is
of information, data, figures, symbols or other modes of written expression, described or admissible __________. (1%) (A) in criminal cases – the accused may prove his good
however represented, by which a right is established or an obligation extinguished, or by moral character if pertinent to the moral trait involved in the offense charged (B) in
which a fact may be proved and affirmed, which is received, recorded, transmitted, criminal cases – the prosecution may prove the bad moral character of the accused to
stored, processed, retrieved or produced electronically. It includes digitally signed prove his criminal predisposition (C) in criminal cases under certain situations, but not to
prove the bad moral character of the offended party (D) when it is evidence of the good information be filed except as provided in section 6 of this rule. If the order is made, the
character of a witness even prior to his impeachment as witness (E) In none of the given accused, if in custody, shall not be discharged unless admitted to bail. If no order is made
situations above. SUGGESTED ANSWER: (A), Under Section 51, Rule 130 of the Rules of nor if having been made, no new information is filed within the time specified in the
Court, the accused may prove his good moral character which is pertinent to the moral order or within such further time as the court may allow for good cause, the accused, if
trait involved in the offense charged. (Section 51 (a) (1) Rule 130, Rules on Evidence). VI. in custody, shall be discharged unless he is also in custody for another charge. X. Which
When the court renders judgment in a judicial foreclosure proceeding, when is the among the following is not subject to mediation for judicial dispute resolution? (1%) (A)
mortgaged property sold at public auction to satisfy the judgment? (1%) (A) After the The civil aspect of B.P. Blg. 22 cases. (B) The civil aspect of theft penalized under Article
decision has become final and executory. (B) At any time after the failure of the 308 of the Revised Penal Code. (C) The civil aspect of robbery. (D) Cases cognizable by
defendant to pay the judgment amount. (C) After the failure of the defendant to pay the the Lupong Tagapamayapa under the Katarungang Pambarangay Law. (E) None of the
judgment amount within the period fixed in the decision, which shall not be less than above. SUGGESTED ANSWER: Remedial Law Q&As (2007-2013)
ninety (90) nor more than one hundred twenty (120) days from entry of judgment. (D) hectorchristopher@yahoo.com JayArhSals “Never Let The Odds Keep You From Pursuing
The mortgaged property is never sold at public auction. Remedial Law Q&As (2007-2013) What You Know In Your Heart You Were Meant To Do.”-Leroy Satchel Paige Page 112 of
hectorchristopher@yahoo.com JayArhSals “Never Let The Odds Keep You From Pursuing 198 (C), Under A.M. No. 04-1-12-SC-Philja, all of the above, except for Robbery is subject
What You Know In Your Heart You Were Meant To Do.”-Leroy Satchel Paige Page 109 of to JDR, to wit: This pilot-test shall apply to the following cases: (1) All civil cases,
198 (E) The mortgaged property may be sold but not in any of the situations outlined settlement of estates, and cases covered by the Rule on Summary Procedure, except
above. SUGGESTED ANSWER: (C), Under Section 2 of Rule 68, if upon the trial in such those which by law may not be compromised; (2) Cases cognizable by the Lupong
action the court shall find the facts set forth in the complaint to be true, it shall ascertain Tagapamayapa and those cases that may be referred to it by the judge under Section
the amount due to the plaintiff upon the mortgage debt or obligation, including interest 408. Chapter VII of the R.A No. 7160, otherwise known as the 1991 Local Government
and other charges as approved by the court, and costs, and shall render judgment for the Code: (3) The civil aspect of BP 22 cases; (4) The civil aspect of quasi-offenses under Titl
sum so found due and order that the same be paid to the court or to the judgment oblige 14 of the Revised Penal Code; and (5) The civil aspect of Estafa, Libel, Theft Moreover,
within a period of not less than ninety (90) days nor more than one hundred twenty robbery is considered a grave felony punishable by imprisonment of more than six-years
(120) days from the entry of judgment, and that in default of such payment the property (Article 294, Par. 5, Revised Penal Code). Under A.M. No. 11-1-6-SC-PHILJA dated January
shall be sold at public auction to satisfy the judgment. VII. The signature of counsel in the 11, 2001, only the civil aspect of less grave felonies punishable by correctional penalties
pleading constitutes a certification that __________. (1%) (A) both client and counsel not exceeding six years imprisonment are required to undergo Court-Annexed Mediation
have read the pleading, that to the best of their knowledge, information and belief there (CAM) and be subject of Judicial Dispute Resolution (JDR) proceedings. Hence, the civil
are good grounds to support it, and that it is not interposed for delay (B) the client has aspect of robbery is not subject to mediation or Judicial Dispute Resolution (JDR). XI.
read the pleading, that to the best of the client’s knowledge, information and belief, What is the effect of the pendency of a special civil action under Rule65 of the Rules of
there are good grounds to support it, and that it is not interposed for delay (C) the Court on the principal case before the lower court? (1%) (A) It always interrupts the
counsel has read the pleading, that to the best of the client’s knowledge, information course of the principal case. (B) It interrupts the course of the principal case only if the
and belief, there are good grounds to support it, and that it is not interposed for delay higher court issues a temporary restraining order or a writ of preliminary injunction
(D) the counsel has read the pleading, that based on his personal information, there are against the lower court. (C) The lower court judge is given the discretion to continue with
good grounds to support it, and that it is not interposed for delay (E) The above choices the principal case. (D) The lower court judge will continue with the principal case if he
are not totally accurate. SUGGESTED ANSWER: (E), Section 3 of Rule 7 provides that the believes that the special civil action was meant to delay proceedings. (E) Due respect to
signature of counsel constitutes a certificate by him that he has read the pleadings; that the higher court demands that the lower court judge temporarily suspend the principal
to the best of his knowledge, information, and belief there is good ground to support it; case. Remedial Law Q&As (2007-2013) hectorchristopher@yahoo.com JayArhSals “Never
and that it is not interposed for delay. VIII. Which among the following is a requisite Let The Odds Keep You From Pursuing What You Know In Your Heart You Were Meant To
before an accused may be discharged to become a state witness? (1%) Remedial Law Do.”-Leroy Satchel Paige Page 113 of 198 SUGGESTED ANSWER: (B), Under Section 7 of
Q&As (2007-2013) hectorchristopher@yahoo.com JayArhSals “Never Let The Odds Keep Rule 65, the court in which the petition is filed may issue orders expediting the
You From Pursuing What You Know In Your Heart You Were Meant To Do.”-Leroy Satchel proceedings, and it may also grant a temporary restraining order or a writ of preliminary
Paige Page 110 of 198 (A) The testimony of the accused sought to be discharged can be injunction for the preservation of the rights of the parties pending such proceedings. The
substantially corroborated on all points. (B) The accused does not appear to be guilty. (C) petition shall not interrupt the course of the principal case unless a temporary
There is absolute necessity for the testimony of the accused whose discharge is restraining order or a writ of preliminary injunction has been issued against the public
requested. (D) The accused has not at any time been convicted of any offense. (E) None respondent from further proceeding in the case (A.M. No. 07-7-12-SC, December 12,
of the above. SUGGESTED ANSWER: (C), Under Section 17 of Rule 119 of the Rules of 2007; Churchille B. Mari & People of the Phils. Vs. Hon. Rolando A. Gonzales & PO1
Criminal Procedure, when two or more persons are jointly charged with the commission Rudyard Paloma, G.R. No. 187728, September 12, 2011, Peralta, J.). XII. Findings of fact
of any offense, upon motion of the prosecution before resting its case, the court may are generally not disturbed by the appellate court except in cases __________. (1%) (A)
direct one or more of the accused to be discharged with their consent so that they may where the issue is the credibility of the witness (B) where the judge who heard the case
be witnesses for the state when after requiring the prosecution to present evidence and is not the same judge who penned the decision (C) where the judge heard several
the sworn statement of each proposed state witness at a hearing in support of the witnesses who gave conflicting testimonies (D) where there are substantially overlooked
discharge, the court is satisfied that: (a) There is absolute necessity for the testimony of facts and circumstances that, if properly considered, might affect the result of the case
the accused whose discharge is required; (b) There is no other direct evidence available (E) None of the above. SUGGESTED ANSWER: (D), In Miranda vs. People, G.R. No.
for the proper prosecution of the offense committed, except the testimony of said 176298, January 25, 2012, the Supreme Court explained that absent any showing that
accused; (c) The testimony of said accused can be substantially corroborated in its the lower courts overlooked substantial facts and circumstances, which if considered,
material points; (d) Said accused does not appear to be the most guilty; and (e) Said would change the result of the case, the Court should give deference to the trial court‟s
accused has not at any time been convicted of any offense involving moral turpitude. appreciation of the facts and of the credibility of witness. XIII. Contempt charges made
Evidence adduced in support of the discharge shall automatically form part of the trial. If before persons, entities, bodies and agencies exercising quasi-judicial functions against
the court denies the motion for discharge of the accused as state witness, his sworn the parties charged, shall be filed with the Regional Trial Court of the place where the
statement shall be inadmissible in evidence (People vs. Feliciano Anabe Y Capillan, G.R> __________. (1%) (A) person, entity or agency exercising quasi-judicial function is located
No. 179033, September 6, 2010, CarpioMorales, J.). IX. Which of the following (B) person who committed the contemptuous act resides Remedial Law Q&As (2007-
distinguishes a motion to quash from a demurrer to evidence? (1%) Remedial Law Q&As 2013) hectorchristopher@yahoo.com JayArhSals “Never Let The Odds Keep You From
(2007-2013) hectorchristopher@yahoo.com JayArhSals “Never Let The Odds Keep You Pursuing What You Know In Your Heart You Were Meant To Do.”-Leroy Satchel Paige
From Pursuing What You Know In Your Heart You Were Meant To Do.”-Leroy Satchel Page 114 of 198 (C) act of contempt was committed (D) party initiating the contempt
Paige Page 111 of 198 (A) A motion to quash a complaint or information is fi led before proceeding resides (E) charging entity or agency elects to initiate the action SUGGESTED
the prosecution rests its case. (B) A motion to quash may be fi led with or without leave ANSWER: (C), Under Section 12 of Rule 71, unless otherwise provided by law, this Rule
of court, at the discretion of the accused. (C) When a motion to quash is granted, a shall apply to contempt committed against persons, entities, bodies or agencies
dismissal of the case will not necessarily follow. (D) The grounds for a motion to quash exercising quasi-judicial functions, or shall have suppletory effect to such rules as they
are also grounds for a demurrer to evidence. (E) The above choices are all wrong. may have adopted pursuant to authority granted to them by law to punish for contempt.
SUGGESTED ANSWER: (C), Under Section 4 of Rule 117, if the motion to quash is based The Regional Trial Court of the place wherein the contempt has been committed shall
on an alleged defect of the complaint or information which can be cured by amendment, have jurisdiction over such charges as may be filed therefor. XIV. When may a party fi le a
the court shall order that an amendment be made. If it is based on the ground that the second motion for reconsideration of a final judgment or final order? (1%) (A) At anytime
facts charged do not constitute an offense, the prosecution shall be given by the court an within 15 days from notice of denial of the first motion for reconsideration. (B) Only in
opportunity to correct the defect by amendment. The motion shall be granted if the the presence of extraordinarily persuasive reasons and only after obtaining express leave
prosecution fails to make the amendment, or the complaint or information still suffers from the ruling court. (C) A party is not allowed to fi le a second motion for
from the same defect despite the amendment. Section 5 of Rule 117 also provides that if reconsideration of a final judgment or final order. (D) A party is allowed as a matter of
the motion to quash is sustained, the court may order that another complaint or right to fi le a second motion for reconsideration of a judgment or final order. (E) None of
the above. SUGGESTED ANSWER: (B), A second motion for reconsideration is allowed but was proper as the right to self-incrimination is a fundamental right that affects liberty
only when there are extraordinary persuasive reasons and only after an express leave and is not waived simply because the accused is on the witness stand. SUGGESTED
shall have been obtained (Suarez vs. Judge Dilag, A.M. No. RTJ-06-2014, August 16, 2011; ANSWER: (E), Section 17, Article III of the 1987 Constitution provides that no person shall
League of Cities vs. COMELEC, G.R. No. 176951, June 28, 2011). XV. In an original action be compelled to be a witness against himself. The essence of the right against self-
for certiorari, prohibition, mandamus, or quo warranto , when does the Court of Appeals incrimination is testimonial compulsion, that is, the giving of evidence against himself
acquire jurisdiction over the person of the respondent? (1%) (A) Upon the service on the through a testimonial act (People vs. Casinillo, 213 SCRA 777 [1992]). In Beltran vs.
respondent of the petition for certiorari, prohibition, mandamus or quo warranto, and Samson, G.R. No. 32025, September 23, 1929, the Supreme Court held thst for the
his voluntary Remedial Law Q&As (2007-2013) hectorchristopher@yahoo.com JayArhSals purposes of the constitutional privilege there similarity between on who is compelled to
“Never Let The Odds Keep You From Pursuing What You Know In Your Heart You Were produce Remedial Law Q&As (2007-2013) hectorchristopher@yahoo.com JayArhSals
Meant To Do.”-Leroy Satchel Paige Page 115 of 198 submission to the jurisdiction of the “Never Let The Odds Keep You From Pursuing What You Know In Your Heart You Were
Court of Appeals. (B) Upon service on the respondent of the summons from the Court of Meant To Do.”-Leroy Satchel Paige Page 118 of 198 a document and one who is
Appeals. (C) Upon the service on the respondent of the order or resolution of the Court compelled to furnish a specimen of his handwriting, for in both cases, the witness is
of Appeals indicating its initial action on the petition. (D) By respondent‟s voluntary required to furnish evidence against himself. In this case, the purpose of the fiscal, who
submission to the jurisdiction of the Court of Appeals. (E) Under any of the above modes. requested the handwriting of the witness, was to compare and determine whether the
SUGGESTED ANSWER: (C) and (D), Under Section 4, Rule 46 of the Revised Rules of Civil accused wrote the documents believed to be falsified. Thus, the right against self-
Procedure, the court shall acquire jurisdiction over the person of the respondent by the incrimination may be invoked by a witness who was compelled to furnish his handwriting
service on him of its order or resolution indicating its initial action on the petition or by for comparison. In Gonzales vs. Secretary of Labor, the Supreme Court held that the
his voluntary submission to such jurisdiction. (n) XVI. Extra-territorial service of summons privilege against self-incrimination must be invoked at the proper time, and the proper
is proper in the following instances, except __________. (1%) (A) when the non-resident time to invoke it is when a question calling for an incriminating answer is propounded.
defendant is to be excluded from any interest on a property located in the Philippines (B) This has to be so, because before a question is asked there would be no way of telling
when the action against the nonresident defendant affects the personal status of the whether the information to be elicited from the witness is self-incriminating or not. As
plaintiff and the defendant is temporarily outside the Philippines (C) when the action is stated in Jones on Evidence (Vol. 6, pp. 4926-4927), a person who has been summoned
against a nonresident defendant who is formerly a Philippine resident and the action to testify “cannot decline to appear, nor can he decline to be sworn as a witness” and
affects the personal status of the plaintiff (D) when the action against the nonresident “no claim of privilege can be made until a question calling for a criminating answer is
defendant relates to property within the Philippines in which the defendant has a claim asked; at that time, and generally speaking, at that time only, the claim of privilege may
or lien (E) All of the above. SUGGESTED ANSWER: There is no correct answer. Under properly be imposed‟ (Bagadiong vs. Gonzales, G.R. No. L-25966, December 28, 1979, De
Section 15 of Rule 14 of the Rules of Court, extraterritorial service of summons is Castro, J.). ALTERNATIVE ANSWER: (B), The right against self-incrimination may be
applicable, when the defendant does not reside and is not found in the Philippines, and waived expressly or impliedly. Thus, when Maria took the witness stand, she is deemed
the action affects the personal status of the plaintiff or relates to, or the subject of which to have waived her right against self-incrimination. XIX. Danny filed a complaint for
is, property within the Philippines, in which the defendant has or claims a lien or interest, damages against Peter. In the course of the trial, Peter introduced evidence on a matter
actual or Remedial Law Q&As (2007-2013) hectorchristopher@yahoo.com JayArhSals not raised in the pleadings. Danny promptly objected on the ground that the evidence
“Never Let The Odds Keep You From Pursuing What You Know In Your Heart You Were relates to a matter not in issue. How should the court rule on the objection? (1%) (A) The
Meant To Do.”-Leroy Satchel Paige Page 116 of 198 contingent, or in which the relief court must sustain the objection. (B) The court must overrule the objection. (C) The
demanded consists, wholly or in part, in excluding the defendant from any interest court, in its discretion, may allow amendment of the pleading if doing so would serve the
therein, or the property of the defendant has been in the Philippines. In Spouses ends of substantial justice. (D) The court, in its discretion, may order that the allegation
Domingo M. Belen vs. Hon. Pablo R. Chavez, G.R. No.175334, march 26, 2008, the in the pleadings which do not conform to the evidence presented be stricken out.
Supreme Court held that if the resident defendant is temporarily out of the country, any Remedial Law Q&As (2007-2013) hectorchristopher@yahoo.com JayArhSals “Never Let
of the following modes of service may be resorted to: (1) Substituted service set forth in The Odds Keep You From Pursuing What You Know In Your Heart You Were Meant To
Section 8; (2) personal service outside the country, with leave of court; (3) service by Do.”-Leroy Satchel Paige Page 119 of 198 (E) The matter is subject to the complete
publication, also with leave of court; or (4) any other manner the court may deem discretion of the court. SUGGESTED ANSWER: (C), (B), or (A), Under Section 5 of Rule 10
sufficient. Hence, extra-territorial service of summons is applicable to all choices given of the Rules of Civil Procedure, when issues not raised by the pleadings are tried with the
above. ALTERNATIVE ANSWER: (B), Under Section 16, Rule 14 of the Rules of Civil express or implied consent of the parties they shall be treated in all respects as if they
Procedure, when any action is commenced against a defendant who ordinarily resides had been raised in the pleadings. Such amendment of the pleadings as may be necessary
within the Philippines, but who is temporarily out of it, service may, by leave of court, be to cause them to conform to the evidence and to raise these issues may be made upon
also effected out of Philippines, as under the preceding section (Section 15, Rule 14). motion of any party at any time, even after judgment; but failure to amend does not
Clearly, a non-resident defendant cannot be considered temporarily outside the effect the result of the trial of these issues. If evidence is objected to at the trial on the
Philip[pines because Section 14, Rule 14 refers to a resident defendant who is only ground that it is not within the issues made by the pleadings, the court may allow the
temporarily outside the Philippines. XVII. When is attachment improper in criminal pleadings to be amended and shall do so with liberality if the presentation of the merits
cases? (1%) (A) When the accused is about to abscond from the Philippines. (B) When of the action and the ends of substantial justice will be served thereby. The court may
the criminal action is based on a claim for money or property embezzled or fraudulently grant a continuance to enable the amendment to be made. The Court may sustain the
misapplied or converted to the use of the accused who is a broker, in the course of his objection because the evidence introduced by Danny is immaterial, being a matter which
employment as such. (C) When the accused is about to conceal, remove, or dispose of his was not raised as an issue in the pleading. On the other hand, the Court also overrule the
property. (D) When the accused resides outside the jurisdiction of the trial court. objection and allow an amendment of the pleading if doing so would serve the ends of
SUGGESTED ANSWER: (D), Under Section 2 of Rule 127, when the civil action is properly justice. XX. The Labor Arbiter, ruling on a purely legal question, ordered a worker’s
instituted in the criminal action as provided in Rule 111, the offended party may have the reinstatement and this ruling was affirmed on appeal by the NLRC whose decision, under
property of the accused attached as security for the satisfaction of any judgment that the Labor Code, is final. The company’s recourse under the circumstances is to
may be recovered from the accused in the following cases: Remedial Law Q&As (2007- __________. (1%) (A) file a motion for reconsideration and if denied, file a petition for
2013) hectorchristopher@yahoo.com JayArhSals “Never Let The Odds Keep You From review with the Court of Appeals on the pure legal question the case presents. (B) file a
Pursuing What You Know In Your Heart You Were Meant To Do.”-Leroy Satchel Paige motion for reconsideration and if denied, appeal to the Secretary of Labor since a labor
Page 117 of 198 (a) When the accused is about to abscond from the Philippines; (b) policy issue is involved. (C) file a motion for reconsideration and if denied, file a petition
When the criminal action is based on a claim for money or property embezzled or for certiorari with the Court of Appeals on the ground of grave abuse of discretion by the
fraudulently misapplied or converted to the use of the accused who is a public officer, NLRC. (D) file a motion for reconsideration and if denied, file a petition for review on
officer of a corporation, attorney, factor, broker, agent, or clerk, in the course of his certiorari with the Supreme Court since a pure question of law is involved. Remedial Law
employment as such, or by any other person in a fiduciary capacity, or for a wilful Q&As (2007-2013) hectorchristopher@yahoo.com JayArhSals “Never Let The Odds Keep
violation of duty; (c) When the accused has concealed, removed, or disposed of his You From Pursuing What You Know In Your Heart You Were Meant To Do.”-Leroy Satchel
property, or is about to do so; and (d) When the accused resides outside the Philippines. Paige Page 120 of 198 (E) directly file a petition for certiorari with the Court of Appeals
XVIII. Maria was accused of libel. While Maria was on the witness stand, the prosecution since a motion for reconsideration would serve no purpose when a pure question of law
asked her to write her name and to sign on a piece of paper, apparently to prove that she is involved. SUGGESTED ANSWER: (C), In Nemia Castro vs. Rosalyn and Jamir Guevarra,
authored the libelous material. Maria objected as writing and signing her name would G.R. No. 192737, April 25, 2012, the Supreme Court held that a motion for
violate her right against self-incrimination. Was Maria’s objection proper? (1%) (A) No, reconsideration is a condition precedent for the filing of a petition for certiorari. Its
she can be cross examined just like any other witness and her sample signature may be purpose is to grant an opportunity for the court to correct any actual or perceived error
taken to verify her alleged authorship of the libelous statements. (B) No, her right against attributed to it by the re-examination of the legal and factual circumstances of the case.
selfincrimination is waived as soon as she became a witness. (C) No, this privilege may be In Saint Martin Funeral Homes vs. NLRC, G.R. No. 130866, September 16, 1998, the
invoked only by an ordinary witness and not by the accused when she opts to take the Supreme Court ruled that the petitions for certiorari under Rule 65 against decisions of
witness stand. (D) The objection was improper under all of A, B, and C. (E) The objection final order of the NLRC should be initially filed in the Court of Appeals in strict
observance of the doctrine on the hierarchy of courts as the appropriate forum for the (b), In Siapno vs. Manalo, G.R. No. 132260, August 30, 2005, the Court disregarded the
relief desired. ALTERNATIVE ANSWER: (E), In Beatriz Siok Ping Tang vs. Subic bay title/denomination of the plaintiff Manalo‟s amended petition as one for Mandamus
Distribution, G.R> No. 162575, December 15, 2010, the Supreme Court held that a with Revocation of Title and Damages; and adjudged the same to be a real action, the
motion for reconsideration is a condition sine qua non for the filing of am petition for filing fees for which should have been computed based on the assessed value of the
certiorari. The rule is, however, circumscribed by well-defined exceptions, such as (a) subject property or, if there was none, the estimated value thereof. 6. X filed a motion
where the order is a patent nullity, as where the court a quo had no jurisdiction; (b) for Bill of Particulars, after being served with summons and a copy of the complaint
where the questions raised in the certiorari proceeding have been duly raised and passed However, X's motion did not contain a notice of hearing. The court may therefore: a.
upon in the lower court; (c) where there is an urgent necessity for the resolution of the require the clerk of court to calendar the motion. b. motu proprio dismiss the motion for
question and any further delay would prejudice the interests of the Government or of not complying with Rule 15. c. allow the parties the opportunity to be heard. d. return
the petitioner or the subject matter of the action is perishable; (d) where, under the the motion to X's counsel for amendment. SUGGESTED ANSWER: (b), A motion for bill of
circumstances, a motion for reconsideration would be useless; (e) where petitioner was particulars which does not contain a notice of hearing is considered pro forma. As such,
deprived of due process and there is extreme urgency for relief; (f) where, in a criminal the motion is a useless piece of paper without force and effect which must not be taken
case, relief from an order of arrest is urgent and the granting of such relief by the trial cognizance by the Court. (Preysler, Jr. Vs. Manila Southcoast Development Corporation,
court is improbable; (g) where the proceedings in the lower court are a nullity for lack of G.R. No. 171872, June 28, 2010). ALTERNATIVE ANSWER: (c), Under Section 2, Rule 12 of
due process; (h) where the proceedings were ex parte, or in which the petitioner had o the Rules of Court, upon filing of a Motion for Bill of particulars, the Clerk of Court must
opportunity to object; and (i) where the issue raised is one purely of law or where public immediately bring it to the attention of the court which may either deny or grant it
interest is involved. Remedial Law Q&As (2007-2013) hectorchristopher@yahoo.com outright, or allow the parties the opportunity to be heard. Remedial Law Q&As (2007-
JayArhSals “Never Let The Odds Keep You From Pursuing What You Know In Your Heart 2013) hectorchristopher@yahoo.com JayArhSals “Never Let The Odds Keep You From
You Were Meant To Do.”-Leroy Satchel Paige Page 121 of 198 2012 Remedial Law Exam Pursuing What You Know In Your Heart You Were Meant To Do.”-Leroy Satchel Paige
MCQ (October 28, 2012) 1. In settlement proceedings, appeal may be taken from an: a. Page 124 of 198 7. A wants to file a Petition for Writ of Habeas Data against the AFP in
order appointing a special administrator; b. order appointing an administrator; c. order connection with threats to his life allegedly made by AFP intelligence officers. A needs
of an administrator to recover property of the estate; d. order to include or exclude copies of AFP highly classified intelligence reports collected by Sgt. Santos who is from
property from the estate. SUGGESTED ANSWER: (b) an order appointing a regular AFP. A can file his petition with: a. RTC where AFP is located; b. RTC where Sgt. Santos
administrator is appealable (See Sy Hong Eng vs. Sy Liac Suy, 8 Phil., 594). An order of a resides; c. Supreme Court; d. Court of Appeals. SUGGESTED ANSWER: (d), In accordance
CFI appointing an administrator of a deceased person‟s estate has been held to be a final with the principle of judicial hierarchy of the courts, A should file with the Court of
determination of the rights of the parties thereunder, and is appealable. (Intestate Estate Appeals. ALTERNATIVE ANSWER: (b), The petition may be filed with the Regional Trial
of Luis Morales et. Al. Vs. SIcat, L-5236, May 5, 1953). On the other hadn, an order Court where the petitioner or respondent resides, or that which has jurisdiction over the
appointing a special administrator is interlocutory in nature and a mere incident in the place where the data or information is gathered, collected or stored, at the option of the
judicial proceedings, hence not appealable. (Rule 109, Sec. 1, Rules of Court) (Samson vs. petitioner. (c), The petition may also be filed with the Supreme Court or the Court of
Samson, 102 Phil. 735; Tan vs. Gedorio, Jr. G.R. No. 166520, March 14, 2008). 2. Under Appeals or the Sandiganbayan when the action concerns public data files of government
the Rules on the Writ of Amparo, interim relief orders may be issued by the Court offices. (Sec.3, A.M. No. 08- 1-16-SC, The Rule on the Writ of Habeas Data, January 22,
except: a. production order; b. witness protection order; c. hold departure order; d. 2008). 8. W was arrested in the act of committing a crime on October 1, 2011. After an
temporary protection order. SUGGESTED ANSWER: (c), Under the Rules on the Writ of inquest hearing, an information was filed against W and his lawyer learned of the same
Amparo, upon filing of the petition or at any time before final judgment, the court, on October 5, 2011. W wants to file a motion for preliminary investigation and therefore
justice or judge may grant any of the following interim relief orders; (a) Temporary he has only up to _____ to file the same. a. October 20, 2011; b. October 10, 2011; c.
Protection Order; (b) Inspection Order; (c) Production Order; and (c) Witness Protection November 15, 2011; d. October 16, 2011. SUGGESTED ANSWER: (b), When a person is
Order. It does not include Hold Departure Order. (Sec. 12 (a) (b) (c) (d), A.M. No.07-9-12- lawfully arrested without a warrant involving an offense which requires a preliminary
SC) 3. A narrative testimony is usually objected to but the court may allow such investigation, he may ask a preliminary investigation with the same right to adduce
testimony if: a. it would expedite trial and give the court a clearer understanding of the evidence in his defense within five (5) days from the time he learns of the filing of the
matters related; b. the witness is of advanced age; c. the testimony relates to family complaint or information in court. (Rule 112, Sec. 7, Rules of Court). 9. Preliminary
genealogy; d. the witness volunteers information not sought by the examiner. Remedial Prohibitive Injunction will not lie: a. to enjoin repeated trespass on land. b. in petitions
Law Q&As (2007-2013) hectorchristopher@yahoo.com JayArhSals “Never Let The Odds for certiorari and mandamus. Remedial Law Q&As (2007-2013)
Keep You From Pursuing What You Know In Your Heart You Were Meant To Do.”-Leroy hectorchristopher@yahoo.com JayArhSals “Never Let The Odds Keep You From Pursuing
Satchel Paige Page 122 of 198 SUGGESTED ANSWER: (a), There is no legal principle which What You Know In Your Heart You Were Meant To Do.”-Leroy Satchel Paige Page 125 of
prevents a witness from giving his testimony in a narrative form if he is requested to do 198 c. to restrain implementation of national government infrastructure project. d. to
so by counsel. A witness may be allowed to testify by narration if it would be the best restrain voting of disputed shares of stock. SUGGESTED ANSWER: (c), No court in the
way of getting at what he knew or could state concerning the matter at issue. It would Philippines shall have jurisdiction to issue any restraining order, preliminary injunction,
expedite the trial and would perhaps furnish the court a clearer understanding of or preliminary mandatory injunction in any case, dispute, or controversy involving an
matters related as they occurred. (People vs. Calixto, G.R. No. 92355, January 24, 1991). infrastructure project, and natural resource development projects and public utilities
ALTERNATIVE ANSWER: (b), The Rules allow persons of tender age to testify in a operated by the Government (Section 1, P.D. 1818). 10. A defendant who fails to file a
narrative form because they cannot cope with the technicalities of examination of timely Answer or responsive pleading will not be declared in default in: a. probate
witnesses. The same rule should be applied to witnesses of advance age. 4. In default of proceedings where the estate is valued at P 1 00,000; b. forcible entry cases; c. collection
parents, the court may appoint a guardian for a minor giving first preference to: a. an case not exceeding P 100,000; d. violation of rental law. SUGGESTED ANSWERS: (b),
older brother or sister who is over 18 years old. b. the actual custodian over 21 years old. Under the Rules on Summary Procedure, if the defendant fails to file an Answer to the
c. a paternal grandparent d. an uncle or aunt over 21 years old. SUGGESTED ANSWER: complaint within a period of Ten (10) days from receipt thereof, the court may motu
(c), In default of parents or a courtappointed guardian, the court may appoint a guardian propio, or on motion of the plaintiff, render judgment as may be warranted by the facts
of the person or property, or both of a minor, observing as far as practicable, the alleged in the complaint and limited to what is prayed for therein. (Sec.6, Revised Rules
following order of preference: (a) the surviving grandparent. In case several of Summary Procedure). There is no declaration of default under the Rules on Summary
grandparents survive, the court shall select any of them taking into account all relevant Procedure. (c), A collection case not exceeding P100,000.00 is governed by the Law on
considerations; (b) the oldest brother or sister of the minor over twenty-one years of Small Claims which does not vest the Court the power and authority to declare a
age, unless unfit or disqualified; (c) the actual custodian of the minor over twenty-one defendant in default. 11. The validity of a search warrant is days: a. 15; b. 30; c. 60; d.
years of age, unless unfit or disqualified; and (d) any other person, who in the sound 120. SUGGESTED ANSWER: NO CORRECT ANSWER. The Committee recommends that the
discretion of the court, would serve the best interests of the minor. (SEC. 6, A.M. No. 03- examinee be given a full credit for any answer to the question. Validity of a Search
02-05-SC 2003-05-01, Rule on Guardianship of Minors). 5. In real actions, the docket and Warrant.- A search warrant shall be valid for ten (10) days from its date. Thereafter, it
filing fees are based on: a. fair market value of the property. b. assessed value of the shall be void. (Rule 126, Sec. 10, Rules of Court). 12. An accused may move for the
property. c. BIR zonal value of the property. Remedial Law Q&As (2007-2013) suspension of his arraignment if: Remedial Law Q&As (2007-2013)
hectorchristopher@yahoo.com JayArhSals “Never Let The Odds Keep You From Pursuing hectorchristopher@yahoo.com JayArhSals “Never Let The Odds Keep You From Pursuing
What You Know In Your Heart You Were Meant To Do.”-Leroy Satchel Paige Page 123 of What You Know In Your Heart You Were Meant To Do.”-Leroy Satchel Paige Page 126 of
198 d. fair market value of the property and amount of damages claimed. SUGGESTED 198 a. a motion for reconsideration is pending before the investigating prosecutor. b.
ANSWER: (c), Under Section 7, Rule 141 of the Rules of Court, in cases involving property, accused is bonded and his bondsman failed to notify him of his scheduled arraignment. c.
the fair market value of the real property in litigation stated in the current tax a prejudicial question exists. d. there is no available public attorney. SUGGESTED
declaration or current zonal valuation of the bureau of internal revenue, whichever is ANSWER: (c), Under Section 11, Rule 16 of the Rules of Criminal Procedure, upon motion
higher, or if there is none, the stated value of the property in litigation or the value of the of the proper party, the arraignment shall be suspended in the following cases: (a) The
personal property in litigation as alleged by the claimant shall be basis of the docket and accused appears to be suffering from an unsound mental condition which effectively
filing fees. ( As amended by A.M. 04-2- 04-SC, August 16, 2004). ALTERNATIVE ANSWER: renders him unable to fully understand the charge against him and to plead intelligently
thereto. In such case, the court shall order his mental examination and, if necessary, his the Sandiganbayan; d. Court martial cases. SUGGESTED ANSWER: (c), Territorial
confinement for such purpose; (b) There exists a prejudicial question; and (c) A petition jurisdiction is immaterial in cases falling under the Sandiganbayan‟s jurisdiction. All
for review of the resolution of the prosecutor is pending at either the Department of public officials who committed an offense which is cognizable by the Sandiganbayan shall
Justice, or the Office of the President; provided that the period of suspension shall not be tried before it regardless of the place of commission of the offense. In addition, the
exceed sixty (60) days counted from the filing of the petition with the reviewing office. court martial is not a criminal court. 17. X was charged for murder and was issued a
(Rule 116, Sec. 11, Rules of Court). 13. P failed to appear at the promulgation of warrant of arrest. X remains at large but wants to post bail. X's option is to: a. file a
judgment without justifiable cause. The judgment convicted P for slight physical injuries. motion to recall warrant of arrest; b. surrender and file a bail petition; c. file a motion for
Judgment may therefore be promulgated in the following manner: a. By the reading of reinvestigation; d. file a petition for review with the OOJ. SUGGESTED ANSWER: (b), Bail
the judgment in the presence of only the judge. b. By the clerk of court in the presence is the security given for the release of a person in the custody of the law (Rule 114, Sec.
of P's counsel. c. By the clerk of court in the presence of a representative of P. d. By 1, Rules of Court). The Rules use of word, “custody” to signify that bail is only available
entering the judgment into the criminal docket of the court. SUGGESTED ANSWER: (d), If for someone who is under the custody of the law. Hence, X should first surrender before
P fails to appear at the promulgation of judgment without justifiable cause, the he could be allowed to post bail. 18. The Energy Regulatory Commission (ERC)
promulgation shall be made by recording the judgment in the criminal docket and promulgates a decision increasing electricity rates by 3%. KMU appeals the decision by
serving him a copy thereof at his last known address or thru his counsel. (Rule 120, Sec. way of petition for review. The appeal will therefore: a. stay the execution of ERC
6, Rules of Court). 14. Being declared in default does not constitute a waiver of all rights. decision. b. shall not stay the ERC decision unless the Court of Appeals directs otherwise.
Remedial Law Q&As (2007-2013) hectorchristopher@yahoo.com JayArhSals “Never Let c. stay the execution of the ERC decision conditioned on KMU posting a bond. d. shall not
The Odds Keep You From Pursuing What You Know In Your Heart You Were Meant To stay the ERC decision. SUGGESTED ANSWER: (b), KMU‟s appeal of the decision of the
Do.”-Leroy Satchel Paige Page 127 of 198 However, the following right is considered Energy Regulations Commission shall not stay the decision increasing the electricity rates
waived: a. be cited and called to testify as a witness b. file a motion for new trial c. by 3%, unless the Court of Appeals shall direct otherwise upon such terms as it may
participate in deposition taking of witnesses of adverse party d. file a petition for deem just. (Rule 43, Sec. 12, Rules of Court). Remedial Law Q&As (2007-2013)
certiorari SUGGESTED ANSWER: (b), A party declared in default cannot take part in the hectorchristopher@yahoo.com JayArhSals “Never Let The Odds Keep You From Pursuing
trial but is nonetheless entitled to notices of subsequent proceedings. Thus, a party What You Know In Your Heart You Were Meant To Do.”-Leroy Satchel Paige Page 130 of
declared in default is deemed to have waived his right to file a motion for new trial since 198 19. RTC decides an appeal from the MTC involving a simple collection case. The
he had no right to an old trial on the first place. ALTERNATIVE ANSWER: NO CORRECT decision consists of only one page because it adopted by direct reference the findings of
ANSWER. The Committee may recommend that the examinee be given full credit for any fact and conclusions of law set forth in the MTC decision. Which statement is most
answer because the question is very tricky. A party declared in default is not deemed to accurate? a. The RTC decision is valid because it was issued by a court of competent
have waived any of the abovementioned rights. A party declared in default loses his jurisdiction. b. The RTC decision is valid because it expedited the resolution of the
standing in Court. He cannot take part in the trial but he is entitled to notices of appeal. c. The RTC decision is valid because it is a memorandum decision recognized by
subsequent proceedings. (Section 3(a), Rule 9, Rules of Court). When a defendant is law. d. The RTC decision is valid because it is practical and convenient to the judge and
declared in default, he does not waive any of the above-mentioned rights. A defendant the parties. SUGGESTED ANSWER: (c), A Memorandum decision can be welcomed as an
may still be cited and called to testify as a witness since he will participate in the trial, not acceptable method of dealing expeditiously with the case load of the courts of justice.
as a party but merely as a witness. In fact, it is not a right but rather an obligation of a The phrase Memorandum Decision appears to have been introduced in this jurisdiction
defendant cited and called to testify as a witness to so appear in court. He may also not by that law but by Section 24 of the Interim Rules and Guidelines of BP Blg. 129,
participate in the deposition taking of witnesses of the adverse party because the same is reading as follows: Section 24. Memorandum decisions – The judgment or final
at the instance of the said adverse party and may not yet be considered as part of the resolution of a court in appealed cases may adopt by reference the findings of fact and
trial. The defendant cannot also be said to have waived his right to file a motion for new conclusions of law contained in the decision or final order appealed from. (Francisco vs.
trial since this is a remedy available before finality of a judgment declaring a party in Perm Skul, G.R. No. 81006, May 12, 1989.) 20. The filing of a complaint with the Punong
default (BD Long Span Builders vs. R.S. Ampeloquio Realty Development, Inc., G.R. Barangay involving cases covered by the Katarungang Pambarangay Rules shall: a. not
No.169919, September 11, 2009). Moreover, a petition for certiorari under Rule 65 is not interrupt any prescriptive period. b. interrupt the prescriptive period for 90 days. c.
considered waived because it is still an available remedy, if the declaration of default was interrupt the prescriptive period for 60 days. d. interrupt the prescriptive period not
tainted with grave abuse of discretion. In Martinez vs. Republic, G.R. No. 160895, exceeding 60 days. SUGGESTED ANSWER: (d), The filing of a complaint with the Punong
October 30, 2006, 506 SCRA 134, the Supreme Court has clearly discussed the remedies Barangay involving cases covered by the Katarungang Pambarangay Rules shall interrupt
of a party Remedial Law Q&As (2007-2013) hectorchristopher@yahoo.com JayArhSals the prescriptive periods for offenses and cause of action under existing laws for a period
“Never Let The Odds Keep You From Pursuing What You Know In Your Heart You Were not exceeding Sixty (60) days from the filing of the complaint with the Punong barangay.
Meant To Do.”-Leroy Satchel Paige Page 128 of 198 declared in default in light of the (Sec.410, Local Government Code). Remedial Law Q&As (2007-2013)
1964 and 1997 Rules of Court and a number of jurisprudence applying and interpreting hectorchristopher@yahoo.com JayArhSals “Never Let The Odds Keep You From Pursuing
said rules. Citing Lina vs. Court of Appeals, No. L-63397, April 9, 1985, 135 SCRA 637, the What You Know In Your Heart You Were Meant To Do.”-Leroy Satchel Paige Page 131 of
High Court enumerated the following remedies, to wit: (a) The defendant in default may, 198 21. In a declaratory relief action, the court may refuse to exercise its power to
at any time after discovery thereof and before judgment, file a motion, under oath, to declare rights and construe instruments in what instance/s? a. When a decision would
set aside the order of default on the ground that his failure to answer was due to fraud, not terminate the controversy which gave rise to the action. b. In an action to
accident, mistake, or excusable neglect, and that he has meritorious defenses; (Sec.3, consolidate ownership under Art. 1607 of the Civil Code. c. To establish legitimate
Rule 18, Rules of Court); (b) If the judgment has already been rendered when the filiation and determine hereditary rights. d. (a) and (c) above SUGGESTED ANSWER: (a),
defendant discovered the default, but before the same has become final and executor, The court, may motu propio or upon motion, refuse to exercise the power to declare
he may file a motion for new trial under Section 1(a) of Rule 37, Rules of Court; (c) If the rights and to construe instruments in any case where a decision would not terminate the
defendant discovered the default after the judgment has become final and executor, he uncertainty or controversy which gave rise to the action, or in any case where the
may file a petition for relief under Section 2 of Rule 38, Rules of Court; and (d) He may declaration or construction is not necessary and proper under the circumstances (Rule
also appeal from the judgment rendered against him as contrary to the evidence or to 63, Sec.5, Rules of Court). 22. In election cases involving an act or omission of an MTC or
the law, even if no petition to set aside the order of default has been presented by him. RTC, a certiorari petition shall be filed with: a. The Court of Appeals b. The Supreme
(Rule 41, Sec.2, Rules of Court) (Rebecca T. Arquero vs. Court of Appeals, G.R. No. Court c. The COMELEC d. The Court of Appeals or the COMELEC both having concurrent
168053, Sept. 21, 2011, Peralta, J.). 15. At arraignment, X pleads not guilty to a Robbery jurisdiction SUGGESTED ANSWER: (c), Section 4, Rule 65 of the Rules of Court, as
charge. At the pretrial, he changes his mind and agrees to a plea bargaining, with the amended by A.M. No. 07-7-12- SC (Amendments to Rules 41, 45, 58, and 65 of the Rules
conformity of the prosecution and offended party, which downgraded the offense to of Court) provides that in election cases involving an act or omission of a municipal or a
theft. The Court should therefore: a. render judgment based on the change of plea. b. regional trial court, the petition shall be filed exclusively with the Commission on
allow the withdrawal of the earlier plea and arraign X for theft and render judgment. c. Elections, in aid of its appellate jurisdiction. (Galang vs. Hon. Geronimo, G.R. No. 192793,
receive evidence on the civil liability and render judgment. d. require the prosecution to February 22, 2011). 23. A charge for indirect contempt committed against an RTC judge
amend the information. SUGGESTED ANSWERS: (b) and (c), The Court should allow the may be commenced through: a. A written charge requiring respondent to show cause
withdrawal of the earlier plea and arraign X for theft and render judgment without need filed with the Court of Appeals. b. An order of the RTC Judge requiring respondent to
of an amendment of complaint or information. (Rule 116, Sec. 2, Rules of Court). Be that show cause in the same RTC. c. Verified petition filed with another branch of the RTC. d.
as it may, the Court has to receive evidence on the civil liability which is impliedly Verified petition filed with a court of higher or equal rank with the RTC. SUGGESTED
instituted with the criminal action before it renders a judgment against X. (Rule 111, ANSWER: Remedial Law Q&As (2007-2013) hectorchristopher@yahoo.com JayArhSals
Sec.1, Rules of Court). 16. A criminal case should be instituted and tried in the place “Never Let The Odds Keep You From Pursuing What You Know In Your Heart You Were
where the Remedial Law Q&As (2007-2013) hectorchristopher@yahoo.com JayArhSals Meant To Do.”-Leroy Satchel Paige Page 132 of 198 (b), The proceedings for indirect
“Never Let The Odds Keep You From Pursuing What You Know In Your Heart You Were contempt may be initiated motu propio by the court against which the contempt was
Meant To Do.”-Leroy Satchel Paige Page 129 of 198 offense or any of the essential committed by an order or any other formal charge requiring the respondent to show
elements took place, except in: a. Estafa cases; b. Complex crimes; c. Cases cognizable by cause why he should not be punished for contempt. It may also be commenced by a
verified petition with supporting particulars and certified true copies of documents or part of the res gestae in the crime of robbery. Remedial Law Q&As (2007-2013)
papers involved therein, and upon full compliance with the requirements for filing hectorchristopher@yahoo.com JayArhSals “Never Let The Odds Keep You From Pursuing
initiatory pleadings for civil actions in the court concerned (Rule 71, Sec.4, Rules of What You Know In Your Heart You Were Meant To Do.”-Leroy Satchel Paige Page 135 of
Court). 24. The statute of "non-claims" requires that: a. claims against the estate be 198 ALTERNATIVE ANSWER: (c), The former rule was that dying declaration was
published by the creditors. b. money claims be filed with the clerk of court within the inadmissible only in criminal prosecutions for homicide, murder or parricide wherein the
time prescribed by the rules. c. claims of an executor or administrator against the estate declarant victim (People vs. Lara, 54 Phil. 96). As amended, the Rule now provides for
be filed with the special administrator. d. within two (2) years after settlement and such admissibility in any case as long as the requisites concur. (Regalado, Remedial Law
distribution of the estate, an heir unduly deprived of participation in the estate may Compendium, Vol.II, 2008 Edition, Page 781). 30. Which of the following is not a Special
compel the re-settlement of the estate. SUGGESTED ANSWER: (b), After the Court has Proceeding? a. Absentees; b. Escheat; c. Change of First Name; d. Constitution of Family
granted letters testamentary or administration, it shall immediately issue a notice Home; SUGGESTED ANSWERS: (c), Under R.A. 9048, as amended by R.A. 10172, the
requiring all persons having money claims against the decedent to file them in the office correction of First Name can now be done administratively before the Local Civil
of the clerk of court. (Rule 86, Sec.1, Rules of Court). The Notice shall state the time for Registrar where the record sought to be corrected is kept or the nearest Philippine
the filing of claims against the estate, which shall not be more than twelve (12) nor less Consulate. Hence, it is no longer considered a special proceeding since the provisions of
than six (6) months after the date of the first publication of the notice. (Rule 86, Sec.2, Rules 103 and 108 do not apply anymore in the change of First name of a person. (d), the
Rules of Court). 25. A judicial compromise has the effect of _______ and is immediately rules on Constitution of the Family Home have already been repealed by Articles 152-162
executory and is not appealable. a. Estoppel; b. Conclusiveness of judgment; c. Res of the Family Code. Under Article 153 of the Family Code, a family home is deemed
Judicata; d. Stare decisis. SUGGESTED ANSWER: (c), A compromise agreement that has constituted on a house and lot from the time it is occupied as a family residence.
been made and duly approved by the court attains the effect and authority of res Consequently, there is no need to constitute a family home either judicially or
judicata, although no execution may be issued unless the agreement receives the extrajudicially. Hence, it is no longer considered a special proceeding. ALTERNATIVE
approval of the court where the litigation is pending and compliance with the terms of ANSWER: All the above-mentioned actions are considered Special Proceedings because
agreement is decreed.” (Ranola vs. Ranola, G.R. No. 185095, July 31, 2009). Remedial they are remedies which seek to establish a status, right or a particular fact. (Rule 1, Sec.
Law Q&As (2007-2013) hectorchristopher@yahoo.com JayArhSals “Never Let The Odds 2(c), Rules of Court). 31. Atty. X fails to serve personally a copy of his motion to Atty. Y
Keep You From Pursuing What You Know In Your Heart You Were Meant To Do.”-Leroy because the office and residence of Atty. Y and the latter's client changed and no
Satchel Paige Page 133 of 198 26. When a party or counsel willfully or deliberately forwarding addresses were given. Atty. X's remedy is to: a. Serve by registered mail; b.
commits forum shopping, the initiatory pleading may: a. be cured by amendment of the Serve by publication; c. Deliver copy of the motion to the clerk of court with proof of
complaint. b. upon motion, be dismissed with prejudice. c. be summarily dismissed with failure to serve; d. Certify in the motion that personal service and through mail was
prejudice as it may constitute direct contempt. d. be stricken from the record. impossible. SUGGESTED ANSWER: Remedial Law Q&As (2007-2013)
SUGGESTED ANSWER: (c), If the acts of the party or his counsel clearly constitute wilful hectorchristopher@yahoo.com JayArhSals “Never Let The Odds Keep You From Pursuing
and deliberate forum shopping, the same shall be ground for summary dismissal with What You Know In Your Heart You Were Meant To Do.”-Leroy Satchel Paige Page 136 of
prejudice and shall constitute direct contempt, as well as a cause for administrative 198 (c), Since the office and place of residence of the Atty. X and the latter‟s clinet
sanctions (Rule 7, Sec.5, Rules of Court). 27. Equity of Redemption is the right of the changed and no forwarding address were given, Atty. X can deliver a copy of the motion
mortgagor to redeem the mortgaged property after default in the performance of the by way of substituted service, to the clerk of court with proof of failure to serve the
conditions of the mortgage, before the sale or the confirmation of sale in a(n): a. motion, both by way of personal service or service by mail. (Rule 13, Sec. 8, Rules of
extrajudicial foreclosure of mortgage. b. judicial foreclosure of mortgage. c. execution Court). 32. When caught, X readily admitted to the Forestry Ranger that he cut the trees.
sale. d. foreclosure by a bank. SUGGESTED ANSWER: (b), Equity of redemption exists in Such a statement may be admitted and is not necessarily hearsay because: a. it is a
case of judicial foreclosure of a mortgage. This is simply the right of the defendant judicial admission of guilt. b. it shows the statement was true. c. it will form part of the
mortgagor to extinguish the mortgage and retain ownership of the property by paying circumstantial evidence to convict. d. it proves that such a statement was made.
the secured debt within a period of not less than ninety (90) days nor more than one SUGGESTED ANSWER: (d), The statement of X may be admitted under the concept of
hundred twenty (120) days from the entry of judgment, in accordance with Rule 68, or independently relevant statement, or statements which are on the very facts in issue or
even after the foreclosure sale but prior to its confirmation. (Spouses Rosales vs. Spouses those which are circumstantial evidence thereof. It is offered in evidence only to prove
Alfonso, G.R. No. 137792, August 12, 2003). 28. X and Y, both residents of Bgy. II, the tenor thereof, or the fact that such a statement was made, and not to prove the
Sampaloc, Manila entered into a P 100,000 loan agreement. Because Y defaulted, X sued truth of the facts asserted therein. Hence, the hearsay rule does not apply. (People vs.
Y for collection and the complainant prayed for issuance of preliminary attachment. Y Gaddi, 170 SCRA 649). 33. A complaint may be dismissed by the plaintiff by filing a notice
moved to dismiss the complaint because there was no Barangay conciliation. The court of dismissal: a. At anytime after service of the answer. b. At anytime before a motion of
should therefore: a. dismiss X's complaint for prematurity. b. dismiss X's complaint for summary judgment is filed. c. At the pre-trial. d. Before the complaint is amended.
lack of cause of action. c. deny Y's motion because it is exempt from Barangay SUGGESTED ANSWER: (b), A complaint may be dismissed by the plaintiff by filing a notice
conciliation. Remedial Law Q&As (2007-2013) hectorchristopher@yahoo.com JayArhSals of dismissal at any time before service of the answer or of a motion for summary
“Never Let The Odds Keep You From Pursuing What You Know In Your Heart You Were judgment. Upon such notice being filed, the court shall issue an order confirming the
Meant To Do.”-Leroy Satchel Paige Page 134 of 198 d. deny Y's motion because of the dismissal. (Rule 17, Sec.1, Rules of Court). 34. In a criminal case for violation of a city
amount of the loan. SUGGESTED ANSWER: (c), As a general rule, no complaint, petition, ordinance, the court may issue a warrant of arrest: a. for failure of the accused to submit
action or proceeding involving any matter within the authority of the Lupon shall be filed his counter-affidavit. b. after finding probable cause against the accused. c. for failure of
or instituted in court or any other government office for adjudication unless there has the accused to post bail. Remedial Law Q&As (2007-2013)
been a confrontation of the parties before the Lupon Chairman or the Pangkat and no hectorchristopher@yahoo.com JayArhSals “Never Let The Odds Keep You From Pursuing
conciliation or settlement has been reached as certified by the Lupon Secretary or the What You Know In Your Heart You Were Meant To Do.”-Leroy Satchel Paige Page 137 of
Pangkat Secretary, attested by the Lupon or Pangkat Chairman, or unless the Settlement 198 d. for non-appearance in court whenever required. SUGGESTED ANSWER: (d), The
has been repudiated. However, the parties may go directly to court in actions coupled criminal case for violation of a city ordinance is governed by the Revised Rules on
with provisional remedies such as preliminary injunction, attachment, delivery of Summary Procedure. Under the said Rule, the court shall not order the arrest of the
personal property and support pendent lite. (Sec.6, P.D. 1508, Katarungang accused except for failure to appear whenever required. (Section 16, 1991 Revised Rules
Pambarangay Law). Since X‟s complaint against Y involves collection of sum of money on Summary Procedure). Accordingly, the court may issue warrant of arrest for non-
with prayer for issuance of preliminary attachment, there is no need for prior barangay appearance of the accused whenever required in a criminal case for infraction of a city
conciliation, and therefore the Court should deny Y‟s Motion to Dismiss. 29. X was shot ordinance. 35. Under the Katarungan Pambarangay rules, the execution of an amicable
by Y in the course of a robbery. On the brink of death, X told W, a barangay tanod, that it settlement or arbitration award is started by filing a motion for execution with the
was Y who shot and held him up. In the trial for robbery with homicide, X's declaration Punong Barangay, who may issue a notice of execution in the name of the Lupon
can be admitted only as a dying declaration: a. to prove robbery. b. to prove homicide. c. Tagapamayapa. Execution itself, however, will be done by: a. a court-appointed sheriff.
to prove robbery and homicide. d. to prove the "corpus delicti". SUGGESTED ANSWER: b. any Barangay Kagawad. c. Punong Barangay. d. any member of the Pangkat ng
(b), a dying declaration is admissible as evidence if the following circumstances are Tagapagsundo. SUGGESTED ANSWER: (c), The Punong Barangay shall issue a notice of
present: (a) it concerns the cause and the surrounding circumstances of the declarant‟s execution in the name of the Lupong Taga-pamayapa and that if the execution be for the
death; (b) it is made when death appears to be imminent and the declarant is under a payment of money, the party obliged is allowed a period of five (5) days to make a
consciousness of impending death; (c) the declarant would have been competent to voluntary payment, failing which, the Punong Barangay shall take possession of sufficient
testify had he or she survived; and (d) the dying declaration is offered in a case in which personal property located in the barangay. (Sections 5 and 6, Article VII, Implementing
the subject of inquiry involves the declarant‟s death. (People vs. Jay Mandy Maglian, G.R. Rules and Regulations of the Katarungang Pambarangay Rule). 36. If the judgment debtor
No. 189834, March 30, 2011, Velasco, Jr., J.). Clearly, the dying declaration can only be dies after entry of judgment, execution of a money judgment may be done by: a.
offered in a case in which the subject of inquiry involves the declarant‟s death, and presenting the judgment as a claim for payment against the estate in a special
necessarily the same can only be admitted to prove the cause and the surrounding proceeding. b. filing a claim for the money judgment with the special administrator of
circumstances of such death. Be that as it may, the dying declaration may be offered as the estate of the debtor. c. filing a claim for the money judgment with the debtor's
successor in interest. d. move for substitution of the heirs of the debtor and secure a writ issue. d. Subpoena expires after 30 days from issue. SUGGESTED ANSWER: NO CORRECT
of execution. SUGGESTED ANSWER: (a), If death occurs after judgment has already been ANSWER. The Committee recommends that the examinee be given Remedial Law Q&As
entered, the final judgment shall be enforced as money claim against Remedial Law (2007-2013) hectorchristopher@yahoo.com JayArhSals “Never Let The Odds Keep You
Q&As (2007-2013) hectorchristopher@yahoo.com JayArhSals “Never Let The Odds Keep From Pursuing What You Know In Your Heart You Were Meant To Do.”-Leroy Satchel
You From Pursuing What You Know In Your Heart You Were Meant To Do.”-Leroy Satchel Paige Page 141 of 198 full credit for any answer to the question. ALTERNATIVE ANSWER:
Paige Page 138 of 198 the estate of the deceased defendant without the necessity of (c), According to the Committee, this it the most logical answer because search warrant
proving the same. (Paredes vs. Moya, 61 SCRA 526, 1970). 37. The Director of the BFAR expires 10 days after its issuance. 45. A person may be charged with direct contempt of
launches an intensified campaign against illegal fishpen operators situated in Laguna de court when: a. A person re-enters a property he was previously ejected from. b. A person
Bay. The illegal fishpen operators file a Section 3 (e), R.A. 3019 (causing undue injury or refuses to attend a hearing after being summoned thereto. c. He attempts to rescue a
benefit) case against the BFAR Director before the Sandiganbayan. The Director's best property in custodia legis. d. She writes and submits a pleading containing derogatory,
remedy before Sandiganbayan is: a. file a Motion to Quash based on lack of jurisdiction offensive or malicious statements. SUGGESTED ANSWER: (d), A person guilty of
over the person. b. file a Motion to Quash for non-exhaustion of administrative misbehavior in the presence of or so near a court as to obstruct or interrupt the
remedies. c. file a Motion to Dismiss because the complaint is a SLAPP suit. d. move for proceedings before the same, including disrespect toward the court, offensive
suspension of proceedings because of a pre-judicial question. SUGGESTED ANSWER: (c), personalities toward others, or refusal to be sworn or to answer as a witness, or to
The Director of the BFAR may file an answer interposing as a defense that the case is a subscribe an affidavit or deposition when lawfully required to do so, may be summarily
Strategic Lawsuit Against Public Participation (SLAPP) and attach supporting documents, adjudged in contempt by such court. (Rule 71, Sec. 1, Rules of Court). In Surigao Mineral
affidavits, papers and other evidence; and, by way of counterclaim, pray for damages, Reservation Board vs. Cloribel, 31 SCRA 1, the Supreme Court held that disrespectful,
attorney‟s fees and costs of suit. The Director who is seeking the dismissal of the case abusive and abrasive language, offensive personalities, unfounded accusations or
must prove by substantial evidence that his acts for the enforcement of environmental intemperate words tending to obstruct, embarrass or influence the court in
law are legitimate action for the protection, preservation and rehabilitation of the administering justice or to bring it into disrepute have no place in a pleading. Their
government. The party filing the action assailed as a SLAPP shall prove by preponderance employment serves no useful purpose and on the contrary constitutes direct contempt
of evidence that the action is not a SLAPP and is a valid claim. (Rule 6, Sec. 2, A.M. No. or contempt in facie curiae. 46. Under the Rules of Electronic Evidence, "ephemeral
09-6-8- SC, Rules of Procedure for Environmental Cases). 38. A complaint may be refiled electronic conversation" refers to the following, except: a. text messages; b. telephone
if dismissed on which of the following grounds? a. unenforceable under the Statute of conversations; c. faxed document; d. online chatroom sessions; SUGGESTED ANSWER:
Frauds; b. Res Judicata; c. Litis Pendencia; d. Lack of jurisdiction. SUGGESTED ANSWERS: (c), An “ephemeral electronic communication” refers to telephone conversations, text
(c) and (d), An order granting a motion to dismiss shall bar the refilling of the same action messages, chatroom sessions, streaming audio, streaming video, and other electronic
or claim based on the following grounds, namely: res judicata, prescription, claim or forms of communications, the evidence of which is not recorded or retained (Sec.1(k),
demand is paid, waived, abandoned or otherwise extinguished, and the claim on which Rule 2). A facsimile transmission is not considered as an electronic evidence Remedial
the action is founded is unenforceable under the statute of frauds. (Rule 16, Remedial Law Q&As (2007-2013) hectorchristopher@yahoo.com JayArhSals “Never Let The Odds
Law Q&As (2007-2013) hectorchristopher@yahoo.com JayArhSals “Never Let The Odds Keep You From Pursuing What You Know In Your Heart You Were Meant To Do.”-Leroy
Keep You From Pursuing What You Know In Your Heart You Were Meant To Do.”-Leroy Satchel Paige Page 142 of 198 under the Electronic Commerce Act. In MCC Industrial
Satchel Paige Page 139 of 198 Sec.5, (f), (h), and (i), Rules of Court). The Rules do not Sales Corporation vs. Ssangyong Corporation, the Supreme Court concluded that the
include litis pendentia and lack of jurisdiction. 39. The following are accurate statements terms “electronic data message” and “electronic document,: as defined under the
on joinder of causes of action, except: a. joinder of actions avoids multiplicity of suits. b. Electronic Commerce Act of 2000, do not include facsimile transmission. Accordingly, a
joinder of actions may include special civil actions. c. joinder of causes of action is facsimile transmission cannot be considered as electronic evidence. It is not the
permissive. d. the test of jurisdiction in case of money claims in a joinder of causes of functional equivalent of an original under the Best Evidence Rule and is not admissible as
act1on, is the "totality rule". SUGGESTED ANSWER: (b), The rule on joinder of actions electronic evidence. (Torres vs. PAGCOR, G.R. No. 193531, December 14, 2011). 47. A
under Section 5, Rule 2 of the 1997 Rules of Civil Procedure, as amended, requires that private electronic document's authenticity may be received in evidence when it is proved
the joinder shall ot include special civil actions governed by special rules. (Roman by: a. evidence that it was electronically notarized. b. evidence that it was digitally signed
Catholic Archbishop of San Fernando Pampanga vs. Fernando Soriano Jr., et al., G.R. No. by the person who purportedly signed the same. c. evidence that it contains electronic
153829, August 17, 2011, VIllarama, Jr., J.). 40. W, a legal researcher in the RTC of data messages. d. evidence that a method or process was utilized to verify the same.
Makati, served summons on an amended complaint on Z at the latter's house on a SUGGESTED ANSWER: (b), Before any private electronic document is offered as authentic
Sunday. The service is invalid because: a. it was served on a Sunday. b. the legal is received in evidence, its authenticity must be proved by evidence that it had been
researcher is not a "proper court officer". c. (a) and (b) above d. there is no need to serve digitally signed by the person purported to have signed the same. (Rule 5, Sec. 2(a), Rules
summons on an amended complaint. SUGGESTED ANSWERS: (b), The Rules do not allow on Evidence). 48. Atty. A drafts a pleading for his client 8 wherein B admits certain facts
a legal researcher to serve summons on amended complaint. He is not the proper court prejudicial to his case. The pleading was never filed but was signed by Atty. A. Opposing
officer who is duly authorized to serve the summons to the defendants. The question is counsel got hold of the pleading and presents the same in court. Which statement is the
about validity and not superfluity. (d), Where the defendants have already appeared most accurate? a. The prejudicial statements are not admissible because the unfiled
before the trial court by virtue of a summons on the original complaint, the amended document is not considered a pleading. b. The prejudicial statements are not admissible
complaint may be served upon them without need of another summons, even if new because the client did not sign the pleading. c. The prejudicial statements are not
causes of action are alleged. (Vlason Enterprises Corporation vs. Court of Appeals, G.R. admissible because these were not made by the client in open court. d. The prejudicial
Nos. 121662-64, July 6, 1999). 41. After a plea of not guilty is entered, the accused shall statements are not admissible because these were made outside the proceedings.
have _____ days to prepare for trial. a. 15; b. 10; c. 30; d. None of the above. Remedial SUGGESTED ANSWER: (a), Pleadings are defined as written statements of the respective
Law Q&As (2007-2013) hectorchristopher@yahoo.com JayArhSals “Never Let The Odds claims and defenses of the parties submitted to the court for appropriate judgment.
Keep You From Pursuing What You Know In Your Heart You Were Meant To Do.”-Leroy (Rule 6, Remedial Law Q&As (2007-2013) hectorchristopher@yahoo.com JayArhSals
Satchel Paige Page 140 of 198 SUGGESTED ANSWER: (a), After a plea of not guilty is “Never Let The Odds Keep You From Pursuing What You Know In Your Heart You Were
entered, the accused shall have at least fifteen (15) days to prepare for trial. The trial Meant To Do.”-Leroy Satchel Paige Page 143 of 198 Sec.1, Rules of Court). Filing is the act
shall commence within (30) days from receipt of the pre-trial order. (Rule 119, Sec. 1, of presenting the pleading or other paper to the clerk of court. (Rule 13, Sec.2, Rules of
Rules of Court). 42. The following motions require a notice of hearing served on the Court). Since Atty. A and his client B did not file the pleading, and it was merely the
opposite party, except: a. Motion to Set Case for Pretrial; b. Motion to take deposition; c. opposing counsel which presented the same in court, it should not be considered to have
Motion to correct TSN; d. Motion to postpone hearing. SUGGESTED ANSWER: (a), After been filed at all, and shall not prejudice Atty. A and his client B. After all, no person may
the last pleading has been served and filed, it shall be the duty of the plaintiff to be prejudiced by the acts of unauthorized strangers. ALTERNATIVE ANSWER: (d), The
promptly move ex parte that the case be set for pre-trial. (Rule 18, Sec.1, Rules of Court). Committee considers this as an alternative answer for a more liberal view. 49. Under the
43. Which of the following statements is incorrect? a. A Motion to Quash which is Rules on Examination of a child witness, a child witness is one: a. who is 18 years of age
granted is a bar to the prosecution for the same offense if the criminal action or liability or below at the time of testifying. b. who is below 18 years of age at the time of the
has been extinguished. b. In the Court of Appeals, the accused may file a motion for new incident/crime to be testified on. c. who is below 18 years of age at the time of the giving
trial based only on newly discovered evidence. c. A demurrer to evidence may be filed of testimony. d. who is 18 years of age in child abuse cases. SUGGESTED ANSWER: (c), A
without leave of court in a criminal case. d. None of the above. SUGGESTED ANSWER: “child witness” is any person who at the time of giving testimony is below the age of
(d), A Motion to Quash which is granted is a bar to the prosecution for the same offense eighteen (18) years. (Sec.4, Rules on Examination of a Child Witness). 50. In which of the
if the criminal action or liability has been extinguished. (Rule 117, Sec.6 in relation to following is Interpleader improper? a. in an action where defendants' respective claims
Section3). In the Court of Appeals, the accused may file a motion for new trial based only are separate and distinct from each other. b. in an action by a bank where the purchaser
on newly discovered evidence. (Rule 53, Sec. 1, Rules of Court). A demurrer to evidence of a cashier's check claims it was lost and another person has presented it for payment.
may be filed without leave of court in criminal case. (Rule 119, Sec. 23, Rules of Court). c. in an action by a lessee who does not know where to pay rentals due to conflicting
44. Which of the following is true? a. Summons expires after 5 days from issue. b. Writ of claims on the property. d. in an action by a sheriff against claimants who have conflicting
Execution expires after 10 days from issue. c. Search Warrant expires after 20 days from claims to a property seized by the sheriff in foreclosure of a chattel mortgage.
SUGGESTED ANSWER: (a), Under the Rules, whenever conflicting claims upon the same gotten wealth". SUGGESTED ANSWER: (b), The Sandiganbayan shall have exclusive
subject matter are or may be made against a person who claims no interest whatever in original jurisdiction over petitions for the issuance of the writs of mandamus, prohibition,
the subject matter, or an interest which in whole or in part is not disputed Remedial Law certiorari, habeas corpus, injunctions, and other ancillary writs and processes in aid of its
Q&As (2007-2013) hectorchristopher@yahoo.com JayArhSals “Never Let The Odds Keep appellate jurisdiction and over petitions of similar nature, including quo warranto, arising
You From Pursuing What You Know In Your Heart You Were Meant To Do.”-Leroy Satchel or that may arise in cases filed or which may be filed under Executive Order Nos. 1, 2, 14
Paige Page 144 of 198 by the claimants, he may bring an action against the conflicting and 14-A, issued in 1986: Provided, that the Remedial Law Q&As (2007-2013)
claimants to compel them to interplead and litigate their several claims among hectorchristopher@yahoo.com JayArhSals “Never Let The Odds Keep You From Pursuing
themselves. (Rule 62, Sec.1, Rules of Court). Undoubtedly, if the defendants‟ respective What You Know In Your Heart You Were Meant To Do.”-Leroy Satchel Paige Page 147 of
claims are separate and distinct from each other, an action for interpleader is not proper. 198 jurisdiction over these petitions shall not be exclusive of the Supreme Court. (Sec. 4,
51. The Parole Evidence Rule applies to: a. subsequent agreements placed on issue. b. R.A. 8249, Act amending P.D. 1606). 58. The judgment in a criminal case may be
written agreements or contractual documents. c. judgment on a compromise agreement. promulgated by the following, except by: a. a Sandiganbayan justice in cases involving
d. will and testaments. SUGGESTED ANSWER: (b), The parol evidence rule, embodied in anti-graft laws. b. a Clerk of Court of the court which rendered judgment. c. an Executive
Section 9, Rule 130 of the Rules of Court holds that when the terms of an agreement Judge of a City Court if the accused is detained in another city. d. any judge of the court
have been reduced into writing, it is considered as containing all the terms agreed upon in which it was rendered. SUGGESTED ANSWER: (a), The Sandiganbayan is a special court
and there can be, between the parties and their successors-in-interest, no evidence of of the same level as the Court of Appeals (CA), and possessing all the inherent powers of
such terms other than the contents of the written agreement. (Leighton Contractors a court of justice, with functions of a trial court. It is a collegial court. x x x The members
Phils. Inc., vs. CNP industries, Inc., G.R. No. 160972, March 9, 2010). Evidently, parol of the graft court act on the basis of consensus or majority rule. The three Justices of a
evidence only applies to written agreements or contractual documents. ALTERNATIVE division, rather than a single judge, are naturally expected to exert keener judiciousness
ANSWER: (d), Parol Evidence Rule applies because the term “Agreement” includes wills. and to apply broader circumspection in trying and deciding cases. (Edgar Payumo et al.
(Rule 130, Sec. 9(e), Rules of Court). 52. PDEA agents conducted a search on a house Vs. Hon. Sandiganbayan et al., G.R. No. 151911, July 25, 2011, Mendoza, J.). Thus, a
abandoned by its owners in Quezon City. The search, in order to be valid, must be made Sandiganbayan Justice alone may not promulgate judgment in a criminal case involving
in the presence of: a. any relative of the owner of the house. b. the Director of the PDEA anti-graft laws. On the other hand, a judgment in the regular court is promulgated by
and a member of the media. c. the Barangay Chairman and a Barangay Tanod. d. any reading it in the presence of the accused and any judge of the court in which it was
elected Quezon City official. SUGGESTED ANSWER: (d), Under the “chain of custody” rendered. When the judge is absent or outside the province or city, the judgment may be
principle, the apprehending team having initial custody and control of the drugs shall, promulgated by the clerk of court. if the accused is confined or detained in another
immediately after seizure and confiscation, physically inventory and photograph the province or city, the judgment may be promulgated by the executive judge of the
same in the presence of the accused or the person/s from whom such items were Regional Trial Court having jurisdiction over the place of confinement or detention upon
confiscated and/or seized or his/her representative or counsel, a representative from request of the court which rendered the judgment. (Rule 120, Sec. 6, Rules of Court). 59.
media and the DOJ, and any elected public official who shall be required to sign the Leave of court is always necessary in: a. a demurrer to evidence in a civil case. b. a
copies of the inventory and be given a copy thereof. (Sec. 21(1), RA 9165). Remedial Law demurrer to evidence in a criminal case. c. motion to amend a complaint. d. third party
Q&As (2007-2013) hectorchristopher@yahoo.com JayArhSals “Never Let The Odds Keep complaint. SUGGESTED ANSWER: (d), A third party complaint is a claim that a defending
You From Pursuing What You Know In Your Heart You Were Meant To Do.”-Leroy Satchel party may, with leave of court, file against a person not a party to Remedial Law Q&As
Paige Page 145 of 198 53. A judge of an MTC can hear and decide petitions for habeas (2007-2013) hectorchristopher@yahoo.com JayArhSals “Never Let The Odds Keep You
corpus or applications for bail where: a. the Supreme Court authorizes the MTC. b. the From Pursuing What You Know In Your Heart You Were Meant To Do.”-Leroy Satchel
judge is the Executive Judge of the MTC. c. the judge of the RTC where the case is raffled Paige Page 148 of 198 the action, called the third party defendant, for contribution,
has retired, was dismissed or had died. d. in the absence of all the RTC Judges in the indemnity, subrogation or any other relief, in respect of his opponent‟s claim. (Rule 6,
province or city. SUGGESTED ANSWER: (d), In the absence of all the Regional Trial Judges Sec. 11, Rules of Court). in a third party complaint, leave of court is always necessary. 60.
in a province or city, any Metropolitan Trial Judge, Municipal Trial Judge, Municipal Correctly complete the sentence: A lone witness --- a. is credible only if corroborated. b.
Circuit Trial Judge may hear and decide petitions for a writ of habeas corpus or is never credible. c. may be believed even if not corroborated. d. is always credible.
applications for bail in criminal cases in the province or city where the absent Regional SUGGESTED ANSWER: (c), The testimony of a lone prosecution witness, as long as it is
Trial Judges sit. (Section 35, Batas Pambansa Blg. 129). 54. Proof of service of summons credible and positive, can prove the guilt of the accused beyond reasonable doubt.
shall be through the following, except : a. written return of the sheriff; b. affidavit of the (People vs. Layson, G.R. No. 105689, February 23, 1994). Thus, a lone witness may be
person serving summons; c. affidavit of the printer of the publication; d. written believed even if not corroborated. 61. A judgment of conviction in a criminal case
admission of the party served. SUGGESTED ANSWER: (d), Proof of service of summons becomes final when: a. accused orally waived his right to appeal. b. accused was tried in
shall be made in writing by the server and shall be sworn to when made by a person absentia and failed to appear at the promulgation. c. accused files an application for
other than a sheriff or his deputy. (Rule 14, Sec. 18, Rules of Court). If the service has probation. d. reclusion perpetua is imposed and the accused fails to appeal. SUGGESTED
been made by publication, it may be proved by the affidavit of the printer to which a ANSWER: (c), A judgment of conviction in a criminal case becomes final when the
copy of the publication shall be attached, and directed to the defendant by registered accused after the lapse of the period for perfecting an appeal, or when the sentence has
mail to his last known address. (Rule 14, Sec. 19, Rules of Court). 55. As a mode of been partially or totally satisfied or served, or when the accused has waived in writing his
discovery, the best way to obtain an admission from any party regarding the genuineness right to appeal, or has applied for probation (Rule 120, Sec. 7, Rules of Court). 62. After a
of any material and relevant document is through a: a. motion for production of hearing on a Motion to Dismiss, the court may either dismiss the case or deny the same
documents. b. written interrogatories. c. request for admission under Rule 26. d. request or: a. defer resolution because the ground relied upon 1s not indubitable. b. order
for subpoena duces tecum. SUGGESTED ANSWER: (c), At any time after issues have been amendment of the pleading c. conduct a preliminary hearing d. None of the above.
joined, a party may file and serve upon any other party a written request for the SUGGESTED ANSWER: (b), After the hearing of a motion to dismiss, the court may
Remedial Law Q&As (2007-2013) hectorchristopher@yahoo.com JayArhSals “Never Let dismiss the action or claim, deny the motion, or Remedial Law Q&As (2007-2013)
The Odds Keep You From Pursuing What You Know In Your Heart You Were Meant To hectorchristopher@yahoo.com JayArhSals “Never Let The Odds Keep You From Pursuing
Do.”-Leroy Satchel Paige Page 146 of 198 admission by the latter of the genuineness of What You Know In Your Heart You Were Meant To Do.”-Leroy Satchel Paige Page 149 of
any material and relevant document described in and exhibited with the request or of 198 order the amendment of the pleading. The court shall not defer the resolution of the
the truth of any material and relevant matter of fact set forth in the request. (Rule 26, motion for the reason that the ground relied upon is not indubitable. (Rule 16, Sec.3,
Sec.1, Rules of Court). A request for admission is not intended to merely reproduce or Rules of Court). 63. Under Rule 52, a Second Motion for Reconsideration is a prohibited
reiterate the allegations of the evidentiary matters of fact described in the request, pleading. However,· where may such Motion be allowed? a. the Sandiganbayan; b. the
whose purpose is to establish said party‟s cause of action or defense. Unless it serves Office of the President; c. the Supreme Court; d. None of the above. SUGGESTED
that purpose, it is pointless, useless, and a mere redundancy. (Limos vs. Spouses Odones, ANSWER: (c), Under Rule 52, a second Motion for Reconsideration is a prohibited
G.R. No. 186979, August 11, 2010). 56. A judgment "non pro tunc" is one which: a. pleading. However, the Supreme Court en banc may entertain the same in the higher
dismisses a case without prejudice to it being re-filed. b. clarifies an ambiguous judgment interest of justice upon a vote of at least two-thirds of its actual membership. There is
or a judgment which is difficult to comply with. c. one intended to enter into the record reconsideration “in the highest interest of justice” when the assailed decision is not only
the acts which already have been done, but which do not appear in the records. d. is a legally erroneous but is likewise patently unjust and potentially capable of causing
memorandum decision. SUGGESTED ANSWER: (c), A nunc pro tunc entry in practice is an unwarranted and irremediable injury or damage to the parties. A second motion for
entry made now of something which was actually previously done, to have effect as the reconsideration can only be entertained before the ruling sought to be reconsidered
court, but to supply an omission in the record of action really had, but omitted through becomes final by operation of law or by the Court‟s declaration. (Sec.3, Rule 15, Internal
inadvertence or mistake. (Wilmerding vs. Corbin Banking Co., 28 South., 640, 641; 126 Rules of the Supreme Court). In the Division, a vote of three Members shall be required
Ala., 268). (Perkins vs. Haywood, 31 N. E., 670, 672 cited in Aliviado vs. Proctor and to elevate to a second motion for reconsideration to the Court En Banc. (Aliviado vs.
Gamble, G.R. No. 160506, June 6, 2011). 57. The Sandiganbayan can entertain a quo Proctor and Gamble Phils., Inc., et al, G.R. No. 160506, June 6, 2011, Del Castillo, J.). 64.
warranto petition only in: a. cases involving public officers with salary grade 27 or higher. The mortgage contract between X, who resides in Manila, and Y, who resides in Naga,
b. only in aid of its appellate jurisdiction. c. as a provisional remedy. d. cases involving "ill covering land in Quezon provides that any suit arising from the agreement may be filed
"nowhere else but in a Makati court". Y must thus sue only in: a. Makati; b. Makati out all or any part of any pleading of the party, or dismiss the action or proceeding or any
and/or Naga; c. Quezon and/or Makati; d. Naga. SUGGESTED ANSWER: (a), The rules on part thereof, or enter a judgment by default against the party, and in its discretion, order
venue of actions are merely procedural in character and can be a subject of stipulation. him to pay reasonable expenses incurred by the other, including attorney‟s fees. (Rule
Where the parties have validly agreed in writing before the filing of the action on the 29, Sec.5, Rules of Court). hence, even if an Answer was filed by a defendant, a judgment
exclusive venue of the action, the suit cannot be filed anywhere other than the by default can still be issued where a party wilfully fails to appear before the officer who
stipulated venue. (Rule 4, Sec. 4, Rules of Court). Since the stipulation between X and Y in is to take his deposition. In Arellano vs. Court of First Instance of Sorsogon, Branch I, 65
the mortgage contract is mandatory and restrictive in character, the venue of the action SCRA 46, the Supreme Court sustained the order of dismissal for failure of respondent to
is only in Makati City. ALTERNATIVE ANSWER: Remedial Law Q&As (2007-2013) serve any answer to petitioner Arellano‟s Interrogatories. The dismissal was based on
hectorchristopher@yahoo.com JayArhSals “Never Let The Odds Keep You From Pursuing Section 5 of Rule 29 which provides that if a party fails to serve answers to
What You Know In Your Heart You Were Meant To Do.”-Leroy Satchel Paige Page 150 of interrogatories submitted under Rule 25, after proper service of such interrogatories, the
198 None of the above. The venue of the action should only be Quezon City, the place Court on motion and notice may dismiss the action or render judgment by default even
where the real property is located. The rules on venue do not apply to actions involving a without prior order to serve answer. 70. Which of the following statements is not
mortgage. In Ochoa vs. Chinabank, G.R. No. 192877, March 23, 2011, the Supreme Court accurate? Remedial Law Q&As (2007-2013) hectorchristopher@yahoo.com JayArhSals
held that the exclusive venue of Makati City, as stipulated by the parties and sanctioned “Never Let The Odds Keep You From Pursuing What You Know In Your Heart You Were
by Section 4, Rule 4 of the Rules of Court, cannot be made to apply to the Petition for Meant To Do.”-Leroy Satchel Paige Page 153 of 198 a. A plea of guilty later withdrawn is
Extrajudicial Foreclosure filed by respondent bank because the provisions of Rule 4 admissible in evidence against the accused who made the plea. b. An unaccepted offer of
pertain to venue of actions, which an extrajudicial foreclosure is not. There is no reason a plea of guilty to a lesser offense is inadmissible in evidence against the accused. c. An
to depart from the doctrinal pronouncement of the Supreme Court. 65. Immediately offer to pay or payment of medical expenses arising from injury is not evidence or proof
after the witness had been sworn in to testify, without any formal offer of his testimony, of civil/criminal liability for the Injury. d. In civil cases, an offer of compromise by the
Atty. A started asking questions on direct examination to the witness. The court may still accused is admissible as an implied admission of guilt. SUGGESTED ANSWERS: (a), A plea
consider his testimony if: a. the formal offer is done after the direct testimony. b. the of guilty later withdrawn is not admissible in evidence against the accused who made the
opposing counsel did not object. c. the witness is an expert witness. d. the opposing plea (Rule 130, Sec. 27, Rules of Court). (d), In civil cases, an offer of compromise is not
counsel offered to stipulate on the testimony given. SUGGESTED ANSWER: (b), While it is an admission of any liability, and is not admissible in evidence against the offeror. (Rule
true that Atty. A failed to offer the questioned testimony when he called the witness on 130, Sec.27, Rules of Court). 71. Under the Rules on Evidence, the following is a
the stand, the opposing counsel waived this procedural error by failing to object at the conclusive presumption and therefore cannot be contradicted by evidence. a. A person
appropriate time i.e., when the ground for objection became reasonably apparent the intends the ordinary consequences of his voluntary act. b. Official duty has been
moment the witness was called to testify without any prior offer having been made by regularly performed. c. A tenant cannot deny his landlord's title during the tenancy
the proponent. (Catuira vs. Court of Appeals, G.R. No. 105813, September 12, 1994). 66. period. d. A writing is truly dated. SUGGESTED ANSWER: (c), The tenant is not permitted
A private document may be considered as evidence when it is sequentially: a. marked, to deny the title of his landlord at the time of the commencement of the Relation of
identified, authenticated. b. identified, marked and offered in evidence. c. marked, landlord and tenant between them (Rule 131, Sec.2, Rules of Court). 72. Cesar, age 16, a
identified, authenticated and offered in evidence. d. marked, authenticated and offered habitual offender, was caught in possession of .001 grams of marijuana. He was charged
in evidence. SUGGESTED ANSWER: (c), Before any private document is offered as for violation of Sec. 16 of R.A. 9165, The Comprehensive Dangerous Drugs Law. The court
authentic is received in Remedial Law Q&As (2007-2013) hectorchristopher@yahoo.com which has jurisdiction is: a. the MTC; b. the RTC; c. Special Drugs Court; d. Family Court.
JayArhSals “Never Let The Odds Keep You From Pursuing What You Know In Your Heart SUGGESTED ANSWER: (d), The State is mandated to safeguard the well-being of its
You Were Meant To Do.”-Leroy Satchel Paige Page 151 of 198 evidence, its due citizenry, particularly children from harmful effects of dangerous drugs on their Remedial
execution and authenticity must be proved. (Rule 132, Sec. 20). The private document Law Q&As (2007-2013) hectorchristopher@yahoo.com JayArhSals “Never Let The Odds
must be marked during the pre-marking of exhibits. It must be identified and Keep You From Pursuing What You Know In Your Heart You Were Meant To Do.”-Leroy
authenticated by a witness, and thereafter offered, as the court shall not consider any Satchel Paige Page 154 of 198 physical and mental well-being and to defend them
evidence which has not been formally offered. (Rule 132, Sec. 34). In addition, the against acts or omissions detrimental to their development and preservation. Pursuant
private document must also be admitted by the court in order to be considered as to this policy and the mandate Republic Act No. 8369, also known as The Family Courts
evidence. 67. The Court of Appeals cannot issue a temporary restraining order in the Act of 1997, the Family Courts are vested with exclusive jurisdiction to hear and decide
following cases·, except: a. bidding and awarding of a project of the national cases against minors charged with drugrelated offenses (A.M. NO. 07-8-2-SC-2, SEC.2).
government. b. against any freeze order issued by the AMLC under the antimoney The objective is to ensure that rights of children charged with violation of any of the
laundering law. c. against infrastructure projects like the SLEX extension. d. against the offenses under The Comprehensive Dangerous Drugs Act of 2002 are well protected, and
DAR in the implementation of the CARL Law. SUGGESTED ANSWER: (a), There is no law that their interests and those of their family and the community are adequately
which prohibits the Court of Appeals from issuing a temporary restraining order on the balanced. (A.M. NO. 07-8-2-SC-2, SEC.2). 73. A court can motu proprio dismiss a case on
bidding and awarding of a project of the national government. On the contrary, there are the following grounds, except : a. failure to prosecute; b. lack of jurisdiction over the
laws which expressly prohibit the Court of Appeals from issuing a temporary restraining parties; c. litis pendentia; d. prescription. SUGGESTED ANSWER: (b), A court cannot motu
order against any of the following: (i) freeze order issued by the AMLC under the anti- propio dismiss a case on the ground of lack of jurisdiction over the parties because the
money laundering law, except the Supreme Court. (R.A. 10167, Sec.10); (ii) infrastructure objection on the said ground can be waived by the failure of the defendant to raise the
projects like the SLEX extension because only the Supreme Court can issue the same. same in his motion to dismiss or in his answer as affirmative defense. (Rule 9, Sec.1,
(Sec.10, R.A. No. 10167 and R.A. No. 8975); and (iii) DAR in the implementation of the Rules of Court). 74. A person entitled to the estate of a deceased person escheated in
CARL Law. (Sec.55, R.A. No. 6657). 68. Choose the most accurate phrase to complete the favor of the State has: a. 5 years from date of judgment to file a claim. b. 2 years from
statement: Mandamus will lie --- a. to compel a judge to consolidate trial of two cases date of judgment to file a claim. c. 5 years from date of registration of the judgment to
pending before different branches of the court. b. to compel a judge to reduce his file a claim. d. 2 years from date of registration of the judgment to file a claim.
decision in writing. c. to direct a probate court to appoint a particular person as regular SUGGESTED ANSWER: (a), A person entitled to the estate of a deceased person
administrator. d. to compel a judge to grant or deny an application for preliminary escheated in favour of the State has a period of five (5) years from the date of such
injunction. SUGGESTED ANSWER: (b), The 1987 Constitution no less commands that “No judgment within which to file a claim thereto with the court. A claim not made within
decision shall be Remedial Law Q&As (2007-2013) hectorchristopher@yahoo.com said time shall be forever barred. If the claim is meritorious, such person shall have
JayArhSals “Never Let The Odds Keep You From Pursuing What You Know In Your Heart possession of and title to the same, or if sold, the municipality or city shall be
You Were Meant To Do.”-Leroy Satchel Paige Page 152 of 198 rendered by any court accountable to, him for the proceeds, after deducting reasonable charges for the care of
without expressing therein clearly and distinctly the facts and the law on which it is the estate. (Rule 91, Sec. 4, Rules of Court). 75. The MTC, acting as an Environmental
based.” (Art. VIII, Sec. 14, 1987 Constitution). Relative thereto, the Rules of Court also Court, has original Remedial Law Q&As (2007-2013) hectorchristopher@yahoo.com
require a judgment or final order to be in writing, personally and directly prepared by the JayArhSals “Never Let The Odds Keep You From Pursuing What You Know In Your Heart
judge stating clearly and distinctly the facts and the law on which it is based, signed by You Were Meant To Do.”-Leroy Satchel Paige Page 155 of 198 and exclusive jurisdiction
him, and filed with the clerk of court. (Rule 36, Sec.1, Rules of Court). (Lenido Lumanog over the following, except: a. criminal offenses punishable under the Chain Saw Act (R.A.
and Augusto Santos vs. People, G.R. No. 182555, September 7, 2010, Villarama, Jr., J.). 9175) b. violation of the NIPAS Law (R.A. 7586) c. violation of the Mining Laws d.
Evidently, mandamus will lie to compel a judge to perform his ministerial duty to reduce violation of Anti-Pollution Laws SUGGESTED ANSWER: (a), The Metropolitan Trial Court
his decision in writing. 69. A judgment by default can be issued despite an Answer being (MTC) exercises exclusive original jurisdiction over all offenses punishable with
filed in: a. annulment of marriage. b. legal separation. c. cases where a party willfully fails imprisonment not exceeding six (6) years irrespective of the amount of fine. (BP 129, Sec.
to appear before the officer who is to take his deposition. d. declaration of nullity of 32). Relative thereto, R.A. 9175 or otherwise known as the Chain Saw Act of 2002,
marriage. SUGGESTED ANSWER: (c), If a party or an officer or managing agent of a party penalizes any person who found to be in possession of a chain saw and uses the same to
wilfully fails to appear before the officer who is to take his deposition, after being served cut trees and timber in forest land or elsewhere except as authorized by the Department
with a proper notice, or fails to serve answers to interrogatories submitted under Rule 25 with imprisonment of six (6) years and one (1) day to eight (8) years or a fine of not less
after proper service of such interrogatories, the court on motion and notice, may strike than Thirty thousand pesos (P30,000.00) but not more than fifty thousand pesos
(P50,000.00) or both at the discretion of the court. Clearly, the court which has private prosecutor, if any. c. trial prosecutor of the pairing court. d. the same trial
jurisdiction over violations of the Chain Saw Act is the Regional Trial Court, and not the prosecutor who manifested his inability to prosecute the case. SUGGESTED ANSWER: (d),
MTC, acting as an Environmental Court. 76. A special administrator may be appointed by All criminal actions either commenced by complaint or information shall be prosecuted
a court when: a. the executor cannot post a bond. b. the executor fails to render an under the direction and control of a public prosecutor. (Rule 110, Sec. 5, Rules of Court).
account. c. regular administrator has a claim against estate he represents. d. a Motion The trial prosecutor assumes full discretion and control over a case. Accordingly, the
for Reconsideration is filed with respect to a decision disallowing probate of a will. same trial prosecutor who manifested his inability should prosecute the case. 83. A
SUGGESTED ANSWER: (c), If the executor or administrator has a claim against estate he decision or resolution of a division of the Supreme Court when concurred in by members
represents, he shall give notice thereof, in writing, to the court, and the court shall who actually took part in the deliberation on the issues in a case and voted thereon, is a
appoint a special administrator (Rule 86, Sec. 8, Rules of Court). 77. A defendant declared decision or resolution of the Supreme Court. a. three (3); b. five(S); c. eight (8); d. ten
in default may, after judgment but before finality, file a: a. Petition for Relief from (10). SUGGESTED ANSWER: (a), Cases or matters heard by a division shall be decided or
Judgment; b. Petition for Certiorari; c. Motion for Reconsideration; d. Motion to Set resolved with the concurrence of a majority of the Members who actually took part in
Aside Order of Default. SUGGESTED ANSWER: Remedial Law Q&As (2007-2013) the deliberations on the issues in the case and voted thereon, and in no case without the
hectorchristopher@yahoo.com JayArhSals “Never Let The Odds Keep You From Pursuing concurrence of at least three of such Members. When the required number is not
What You Know In Your Heart You Were Meant To Do.”-Leroy Satchel Paige Page 156 of obtained, the case shall be decided en banc: Provided, that no doctrine or principle of
198 (c), A defendant declared in default may after judgment but before finality file a law laid down by the court in a decision rendered en banc Remedial Law Q&As (2007-
Motion for Reconsideration in order to give the Court an opportunity to rectify its 2013) hectorchristopher@yahoo.com JayArhSals “Never Let The Odds Keep You From
mistakes and set aside the previous judgment by default before it attains finality. Pursuing What You Know In Your Heart You Were Meant To Do.”-Leroy Satchel Paige
ALTERNATIVE ANSWER: A defendant declared in default may, after judgment but before Page 159 of 198 or in division may be modified or reversed except by the court sitting en
finality, file a Motion for New Trial. It is well-settled that a defendant who has been banc. (Article VIII, Sec. 4, 1987 Constitution). 84. A and B adopted their nephew. They
declared in default has the following remedies, to wit: (1) he may, at any time after filed an action for revocation of the adoption on May 1, 1998 on the ground that their
discovery of the default but before judgment, file a motion, under oath, to set aside the nephew neglected them. Based on the Rules of Domestic Adoption, the judge must: a.
order of default on the ground that his failure to answer was due to fraud, accident, advise A and B to just disinherit the nephew. b. disallow the revocation. c. refer the
mistake or excusable neglect, and that he has a meritorious defense; (2) if judgment has petition to the DSWD. d. grant the petition after hearing. SUGGESTED ANSWERS: (a) and
already been rendered when he discovered the default, but before the same has become (b), Adoption being in the best interest of the child, shall not be subject to rescission by
final and executor, he may file a motion for new trial under Section 1(a) of Rule 37; (3) if the adopter(s). However, the adopter(s) may disinherit the adoptee for causes provided
he discovered the default after the judgment has become final and executor, he may file in Article 919 of the Civil Code. (Sec.19, R.A. 8552 Rules of Domestic Adoption). 85.
a petition for relief under Section 2 of Rule 38; and (4) he may also appeal from the Sandiganbayan exercises concurrent jurisdiction with the Supreme Court and the Court
judgment rendered against him as contrary to the evidence or to the law, even if no of Appeals over: a. Petitions for Writ of Certiorari and Prohibition; b. Petitions for Writ of
petition to set aside the order of default has been presented by him. (B.D. long Span Habeas Corpus; c. Petitions for Quo Warranto; d. Petitions for Writ of Amparo and
Builders vs. R.S. Ampeloquio Realty Development, Inc., G.R. No. 169919, September 11, Habeas Corpus. SUGGESTED ANSWER: (d), The Sandiganbayan shall have exclusive
2009). 78. With leave of court, a party may amend his pleading if: a. there is yet no original jurisdiction over petitions for the issuance of the writs of mandamus, prohibition,
responsive pleading served. b. the amendment is unsubstantial. c. the amendment certiorari, habeas corpus, injunction, and other ancillary writs and processes in aid of its
involves clerical errors of defect in the designation of a party. d. the amendment is to appellate jurisdiction: Provided, that the jurisdiction over these petitions shall not be
conform to the evidence. SUGGESTED ANSWER: (d), When issues not raised by the exclusive of the Supreme Court. (Sec.2, R.A. 7975-An Act to Strengthen the Functional
pleadings are tried with the express or implied consent of the parties, they shall be and Structural Organization of the Sandiganbayan, amending for that purpose
treated in all respects as if they had been raised in the pleadings. Such amendment of Presidential Decree No. 1606, as amended). 86. C, a convict, was able to get favorable
the pleadings as may be necessary to cause them to conform to the evidence and to results of a post-conviction DNA testing showing that C could not have committed the
raise these issues may be made upon motion of any party at any time, even after crime. To gain freedom, C may: a. file a petition for Writ of Habeas Corpus before the
judgment; but failure to amend does not affect the result of the trial of these issues. court of origin. b. apply for full pardon. c. file a Motion to annul judgment of conviction
(Rule 10, Sec. 5, Rules of Court). Remedial Law Q&As (2007-2013) on the ground of fraud. Remedial Law Q&As (2007-2013) hectorchristopher@yahoo.com
hectorchristopher@yahoo.com JayArhSals “Never Let The Odds Keep You From Pursuing JayArhSals “Never Let The Odds Keep You From Pursuing What You Know In Your Heart
What You Know In Your Heart You Were Meant To Do.”-Leroy Satchel Paige Page 157 of You Were Meant To Do.”-Leroy Satchel Paige Page 160 of 198 d. file a Motion for new
198 79. When a Motion to Quash search warrant is denied, the best remedy is: a. appeal trial under Rule 121. SUGGESTED ANSWER: (a), The convict or the prosecution may file a
the denial order. b. file a motion to suppress evidence. c. file an injunction suit. d. file a petition for a writ of habeas corpus in the court of origin if the results of the post-
certiorari petition. SUGGESTED ANSWER: (b), When a motion to quash search warrant is conviction DNA testing are favourable to the convict. In case the court, after due hearing,
denied, the best remedy is to file a motion to suppress evidence since they are finds the petition to be meritorious, it shall reverse or modify the judgment of conviction
alternative and not cumulative remedies. (Regalado, Remedial law Compendium, 2004 and order the release of the convict, unless continued detention is justified for a lawful
Edition, Tenth Edition, page 662). ALTERNATIVE ANSWER: (d), In Santos vs. Pryce gases cause. A similar petition may be filed either in the Court of Appeals or the Supreme
Inc. G.R. No. 165122, November 23, 2007, the Supreme Court held that the special civil Court, or with any member of said courts, which may conduct a hearing thereon or
action for certiorari is the proper recourse in assailing the quashal of the search warrant. remand the petition to the court of origin and issue the appropriate orders. (Sec.10, Rule
The Trial court‟s unwarranted reversal of its earlier finding of probable cause constituted on DNA Evidence). 87. X filed a complaint with the RTC through ABC, a private letter
grave abuse of discretion. Hence, the Supreme Court had allowed direct recourse to it or forwarding agency. The date of filing of the complaint shall be: a. the date stamped by
even to the Court of Appeals via a special civil action for certiorari from a trial court‟s ABC on the envelope containing the complaint. b. the date of receipt by the Clerk of
quashal of search warrant. 80. A court may take judicial notice of: a. the Twitter account Court. c. the date indicated by the receiving clerk of ABC. d. the date when the case is
of President Aquino. b. a Committee Report issued by the Congressional Committee on officially raffled. SUGGESTED ANSWER: (b), Under the Riles, the manner of filing of
Labor Relations. c. the effects of taking aspirin everyday. d. the arbitral award issued by pleadings, appearances, motions, notices, judgments and all other papers shall only be
International Court of Arbitration. SUGGESTED ANSWER: (b), A court shall take judicial made by presenting the original copies thereof, plainly indicated as such, personally to
notice, without the introduction of evidence, of the existence and territorial extent of the clerk of court or bny sending them by registered mail. (Rule 13, Sec.3). Nonetheless,
states, their political history, forms of government and symbols of nationality, the law of if the complaint was filed with the court through a private letter-forwarding agency, the
nations, the admiralty and maritime courts of the world and their seals, the political established rule is that the date of delivery of pleadings to a private letter-forwarding
constitution and history of the Philippines, the official acts of the legislative, executive, agency is not to be considered as the date of filing in court, but rather the date of actual
and judicial departments of the Philippines, the laws of nature, the measure of time, and receipt by the court, is deemed to be the date of filing of the pleading. (Benguet Electric
the geographical divisions. (Rule 129, Sec. 1, Rules of Court). 81. The case of R, who is Cooperative, Inc. vs. National Labor Relations Commission, G.R. No. 89070, May 18,
under detention, was raffled to the RTC on March 1. His arraignment should be set not 1992). Hence, the date of the actual receipt by the court is considered as the date of
later than: a. March 4; b. March 16; Remedial Law Q&As (2007-2013) filing of the complaint. 88. An objection to any interrogatories may be presented within_
hectorchristopher@yahoo.com JayArhSals “Never Let The Odds Keep You From Pursuing days after service thereof: a. 15; b. 10; c. 5; d. 20. Remedial Law Q&As (2007-2013)
What You Know In Your Heart You Were Meant To Do.”-Leroy Satchel Paige Page 158 of hectorchristopher@yahoo.com JayArhSals “Never Let The Odds Keep You From Pursuing
198 c. March 30; d. March 11. SUGGESTED ANSWER: (d), The arraignment of R should be What You Know In Your Heart You Were Meant To Do.”-Leroy Satchel Paige Page 161 of
set not later than March 11. Under Section 1, Rule 116 of the Rules of Court, the accused 198 SUGGESTED ANSWER: (b), Objections to any interrogatories may be presented to the
shall be arraigned within ten (10) days from the date of the raffle. 82. After the DOJ court within ten (10) days after service thereof, with notice as in case of motion. Upon
Secretary granted accused's Petition for Review, the prosecution filed a motion to filing of the aforementioned objections, the answer to such written interrogatories shall
withdraw the Information before the trial court. The judge therein denied the same. The be deferred until the objections are resolved, which shall be at as early a time as is
trial prosecutor manifested before the judge that he can no longer prosecute the case practicable. (Rule 25, Sec.3, Rules of Court). 89. The deposition of a witness, whether or
because he is only an alter ego of the DOJ Secretary who ordered him to withdraw the not a party, may be used for any purpose if the Court finds the following circumstances
Information. The case should therefore be prosecuted by: a. a DOJ state prosecutor. b. are attendant, EXCEPT: a. when the witness is dead. b. when the witness is incarcerated.
c. when the witness is outside the Philippines and absence is procured by the party Court en banc is equally divided in opinion, or the necessary majority cannot be had, the
offering deposition. d. when the witness is 89 years old and bed-ridden. SUGGESTED case shall again be deliberated on, and if after such deliberation no decision is reached,
ANSWER: (c), The deposition of a witness, whether or not a party, may be used by any the original action commenced in the court shall be dismissed; in appealed cases, the
party for any purpose if the court finds: (1) that the witness is dead; (2) that the witness judgment or order appealed from shall stand affirmed; and on all incidental matters, the
resides at a distance more than one hundred (100) kilometres from the place of trial or petition or motion shall be denied. (Rule 56, Sec. 7, Rules of Court). 94. An example of a
hearing, or is out of the Philippines, unless it appears that his absence was procured by special judgment is one which orders: a. the defendant to deliver and reconvey personal
the party offering the deposition; (3) that the witness is unable to attend or testify property to the plaintiff. b. defendant to execute a Deed of Sale in favor of plaintiff. c.
because of age, sickness, infirmity, or imprisonment; (4) that the party offering the defendant to paint a mural for the plaintiff. d. Defendant to vacate the leased premises.
deposition has been unable to procure the attendance of the witness by subpoena; or (5) SUGGESTED ANSWER: (c), A special judgment is one which requires the performance of
upon application and notice, that such exceptional circumstances exist to make it any act other than the payment of money, or the sale or delivery of a real or personal
desirable, in the interest of justice and with due regard to the importance of presenting property. A disobedience to such judgment is an indirect contempt, and the judgment is
the testimony of the witnesses orally in open court, to allow the deposition to be used. executed by contempt proceeding. (Sura vs. Martin, 26, SCRA 286; Barrete vs. Amila, 230
(Rule 23, Sec. 4 (c), Rules of Court). 90. One of the exemptions to the general rule that SCRA 219; Magallanes vs. Sarita, 18 SCRA 575; Moslem vs. Soriano, 124 SCRA 190; People
evidence not formally offered shall not be considered is: a. in judgment on the pleadings. vs. Pascual, 12326-CR, February 14, 1974). A judgment ordering the defendant to paint a
b. evidence in land registration proceedings. c. evidence lost/destroyed due to force mural for the plaintiff is considered a special judgment. 95. At the promulgation of
majeure after being marked, identified and described in the record. d. documentary judgment, P, who is bonded, failed to appear without justifiable cause. In order for P not
evidence proving a foreign judgment. SUGGESTED ANSWER: Remedial Law Q&As (2007- to lose his remedies under the Rules, he must: a. within 15 days from receipt of a copy of
2013) hectorchristopher@yahoo.com JayArhSals “Never Let The Odds Keep You From the decision, file a Motion for Reconsideration. b. within 15 days from the promulgation,
Pursuing What You Know In Your Heart You Were Meant To Do.”-Leroy Satchel Paige surrender to the court and file a motion for leave to avail of remedies. c. notify his
Page 162 of 198 (a), Where an answer fails to tender an issue, or otherwise admits the bondsman within 15 days so that his bail will not be confiscated. d. file a petition for
material allegations of the adverse party‟s pleading, the court may, on motion of that certiorari. SUGGESTED ANSWER: (b), If the judgment is for conviction and the failure of
party, direct judgment on such pleading. (Rule 34, Sec. 1, Rules of Court). Judgment on the accused to appear was without justifiable cause, he shall lose the remedies available
the pleadings is, therefore, based exclusively upon the allegations appearing in the in these rules against the judgment and the court shall Remedial Law Q&As (2007-2013)
pleadings of the parties and the annexes, if any, without consideration of any evidence hectorchristopher@yahoo.com JayArhSals “Never Let The Odds Keep You From Pursuing
aliunde. (Philippine National Bank vs. Merelo B. Aznar, et. al, G.R. No. 171805, May 30, What You Know In Your Heart You Were Meant To Do.”-Leroy Satchel Paige Page 165 of
2011, Leonardo-De Castro, J.). The court therefore may be allowed to render judgment 198 order his arrest. Within fifteen (15) days from promulgation of judgment, however,
based merely on the pleadings without need of trial and formal offer of evidence. the accused may surrender and file a motion for leave of court to avail of these
ALTERNATIVE ANSWER: (b), The Rules of Court shall not apply to election cases, land remedies. He shall state the reasons for his absence at the scheduled promulgation and if
registration, cadastral, naturalization and insolvency proceedings, and other cases not he proves that his absence was for a justifiable cause, he shall be allowed to avail of said
herein provided for, except by analogy or in suppletory character and whenever remedies within fifteen 915) days from notice. (Rule 120, Sec. 6, Rules of Court) (Pascua
practicable and convenient (Rule 1, Sec. 4, Rules of Court). (Government Insurance vs. Court of Appeals, 348 SCRA 197; People vs. De Grano, G.R. No. 167710, June 5, 2009,
System (GSIS) vs. Dinnah Villaviza et. al., G.R. No. 180291, July 27, 2010, Mendoza, J.). In Peralta, J.). 96. X, the designated executor of a will, files a petition for probate of the
one case, the Supreme Court sustained the Court of Appeals when it denied an same. X and his counsel failed to appear without justifiable cause at the hearing on the
application for naturalization in the basis of documents not formally offered in evidence presentation of evidence and the court therefore dismissed, motu proprio, his petition
during the trial. The High Court noted that the procedure in Sec.34 of Rule 132 providing for failure to prosecute. The effect of the dismissal is: a. not an adjudication upon the
that the Court shall consider no evidence which has not been formally offered, does not merits. b. the will can no longer be probated. c. it is a dismissal with prejudice. d. a bar to
apply to naturalization proceeding conformably to Section 4, Rule 1 of the Rules of Court. a subsequent action on the same cause. SUGGESTED ANSWER: (c), The dismissal of a
(Ong Chia vs. Republic, 328 SCRA 9 (2001). Applying the same principle, we should not case for failure to prosecute has the effect of adjudication on the merits, and is
also apply the said rule on evidence in land registration proceedings. After all, in one necessarily understood to be with prejudice to the filing of another action, unless
case, the Supreme Court already made it clear that the liberal construction principle does otherwise provided in the order of dismissal. Stated differently, the general rule is that
not apply in land registration cases because it is not governed by the Rules of Court. dismissal of a case for failure to prosecute is to be regarded as an adjudication on the
(Bienvenido Castillo vs. Republic of the Philippines, G.R. No., 182980, June 22, 2011, merits and with prejudice to the filing of another action, and the only exception is when
Carpio, J.). 91. In Petition for Certiorari, the Court of Appeals issues a Writ of Preliminary the order of dismissal expressly contains a qualification that the dismissal is without
Injunction against the RTC restraining the latter from trying a crucial case. The Court of prejudice. (See Rule 17, Sec. 3, Rules of Court; Gomez vs. Alcantara, G.R. No. 179556,
Appeals should therefore: a. decide the main case within 60 days. b. decide the certiorari February 13, 2009). 97. The Rule on Small Claims is applicable to: a. claims for unpaid
petition within 6 months. c. decide the main case or the petition within 60 days. d. rentals of P 100,000 or less, with prayer for ejectment. b. enforcement of a barangay
decide the main case or the petition within 6 Remedial Law Q&As (2007-2013) amicable settlement involving a money claim of P 50,000 after one (1) year from date of
hectorchristopher@yahoo.com JayArhSals “Never Let The Odds Keep You From Pursuing settlement. c. action for damages arising from a quasi-delict amounting to P 100,000. d.
What You Know In Your Heart You Were Meant To Do.”-Leroy Satchel Paige Page 163 of action to collect on a promissory note amounting to P 105,000 where plaintiff Remedial
198 months from issue of the preliminary injunction. SUGGESTED ANSWER: (d), The trial Law Q&As (2007-2013) hectorchristopher@yahoo.com JayArhSals “Never Let The Odds
court, the Court of Appeals, the Sandiganbayan or the Court of Tax appeals that issued a Keep You From Pursuing What You Know In Your Heart You Were Meant To Do.”-Leroy
writ of preliminary injunction against a lower court, board, officer, or quasi-judicial Satchel Paige Page 166 of 198 expressly insists in recovering only P 1 00,000. SUGGESTED
agency shall decide the main case or petition within six (6) months from the issuance of ANSWER: (c), The Rule on Small Claims shall be applied in all actions which are: (a) purely
the writ. (Rule 58, Sec. 5, as amended by A>M. No. 07-7-12-SC). 92. Witness A was civil in nature where the claim or relief prayed for by the plaintiff is solely for payment or
examined on direct examination by the prosecutor. The defense counsel however reimbursement of sum of money, and (b) the civil aspect of criminal actions, either filed
employed dilatory tactics and was able to secure numerous postponements of A's cross before the institution of the criminal action, or reserved upon the filing of the criminal
examination. A suffered a stroke and became incapacitated. His uncompleted testimony action in court, pursuant to Rule 111 of the Revised Rules of Criminal Procedure. These
may therefore be: a. ordered stricken from the record. b. allowed to remain in the claims or demands may be for damages arising from fault or negligence. (Sec. 4, A.M. No.
record. c. held in abeyance until he recovers. d. not be given any probative weight. 08-8-7-SC, The Rule of Procedure for Small Claims Cases). 98. When directed by the
SUGGESTED ANSWER: (a), The uncompleted testimony of A should be ordered stricken judge, a clerk of court can receive evidence addressed by the parties in: a. case where
from the record because A has not been crossexamined by the defense. Consequently, it the judge is on leave. b. small claims proceedings. c. cases where the parties agree in
stands to reason that the striking out of the A‟s testimony altogether wiped out the writing. d. land registration proceedings. SUGGESTED ANSWER: (c), The Rules provide
required authentication for the prosecution‟s exhibits. They become inadmissible unless that the judge of the court where the case is pending shall personally receive the
the court, in its discretion, reopens the trial upon a valid ground and permits the evidence to be adduced by the parties. However, in default or exparte hearings, and in
rectification of the mistakes. (Spouse Dela Cruz vs. Papa, G.R. No. 185899, December 8, any case where the parties agree in writing, the court may delegate the reception of
2010). ALTERNATIVE ANSWER: (b), The uncompleted testimony of A should be allowed evidence to its clerk of court who is a member of the bar. (Rule 30, Sec. 9, Rules of
to remain on the record since it was due to the fault of the defense that they were not Court). 99. A certificate against ForumShopping is not required in: a. petitions for
able to exercise their right to cross-examine the witness. The defense should be probate of will. b. application for search warrant. c. complaint-in-intervention. d. petition
penalized for employing dilatory tactics which resulted in the witness‟ eventual for Writ of Kalikasan. SUGGESTED ANSWER: (b), A certification against forum shopping is
incapacity to testify. 93. If the Supreme Court en bane is equally divided in opinion not required in an application for search warrant. The Rules of Court, require only
covering an original action, the case shall be: a. re-raffled to a division. b. original action initiatory pleading to be accompanied with a certificate of non-forum shopping omitting
shall be dismissed. c. The judgment appealed from shall be official. d. again deliberated any mention of “applications” as in Supreme Court No. 04-94. Hence, the absence of
upon. Remedial Law Q&As (2007-2013) hectorchristopher@yahoo.com JayArhSals such certification will not result in the dismissal of the application for search warrant.
“Never Let The Odds Keep You From Pursuing What You Know In Your Heart You Were (Savage vs. Judge A.B. Taypin, G.R. No. 134217, May 11, 2000). Remedial Law Q&As
Meant To Do.”-Leroy Satchel Paige Page 164 of 198 SUGGESTED ANSWER: (b) Where the (2007-2013) hectorchristopher@yahoo.com JayArhSals “Never Let The Odds Keep You
From Pursuing What You Know In Your Heart You Were Meant To Do.”-Leroy Satchel expropriation proceedings? (A) The court shall declare the defendant who fails to answer
Paige Page 167 of 198 100. An accused's custodial rights, e.g., right to counsel and right the complaint in default and render judgment against him. Remedial Law Q&As (2007-
to remain silent, is available: a. at preliminary investigation. b. at police line-up for 2013) hectorchristopher@yahoo.com JayArhSals “Never Let The Odds Keep You From
identification purposes. c. at ultra-violet examination to determine presence of ultra Pursuing What You Know In Your Heart You Were Meant To Do.”-Leroy Satchel Paige
violet powder on accused's hands. d. at one-on-one confrontation with eyewitness. Page 170 of 198 (B) The court shall refer the case to the Board of Commissioners to
SUGGESTED ANSWER: (a), Any person under investigation for the commission of an determine the amount of just compensation. (C) The plaintiff shall make the required
offense shall have the right to be informed of his right to remain silent and to have deposit and forthwith take immediate possession of the property sought to be
competent and independent counsel preferably of his own choice. If the person cannot expropriated. (D) The plaintiff may appropriate the property for public use after
afford the services of counsel, he must be provided with one. These rights cannot be judgment and payment of the compensation fixed in it, despite defendant’s appeal. (5)
waived except in writing and in the presence of counsel. (Article III, Sec. 12 (1), 1987 Which of the following is a correct statement of the rule on amendment of the
Constitution). These guaranteed rights are available in all kinds of investigation including information in a criminal proceeding? (A) An amendment that downgrades the offense
a preliminary investigation. In a preliminary investigation, a public prosecutor determines requires leave of court even before the accused pleads. (B) Substantial amendments are
whether a crime has been committed and whether there is probable cause that the allowed with leave of court before the accused pleads. (C) Only formal amendments are
accused is guilty thereof. (Rules of Court, Rule 112, Section 1). (Metropolitan Bank and permissible before the accused pleads. (D) After the plea, a formal amendment may be
Trust Company vs. Rogelio Reynaldo, et.al., G.R. No. 164538, August 9, 2010, Del Castillo, made without leave of court. (6) Gary who lived in Taguig borrowed P1 million from Rey
J.). The right to have a preliminary investigation conducted before being bound over to who lived in Makati under a contract of loan that fixed Makati as the venue of any action
trial for a criminal offense and hence formally at risk of incarceration or some other arising from the contract. Gary had already paid the loan but Rey kept on sending him
penalty, is not a mere formal or technical right: it is a substantive right. To deny the letters of demand for some balance. Where is the venue of the action for harassment
accused‟s claim to a preliminary investigation would be to deprive him of the full that Gary wants to file against Rey? (A) In Makati since the intent of the party is to make
measure of his right to due process.” (Sales vs. Sandiganbayan, G.R. No. 143802, it the venue of any action between them whether based on the contract or not. (B) In
November 16, 2001). Applying the foregoing constitutional and procedural precepts, Taguig or Makati at the option of Gary since it is a personal injury action. (C) In Taguig
there is no doubt that the custodial rights are available during the preliminary since Rey received the letters of demand there. (D) In Makati since it is the venue fixed in
investigation. ALTERNATIVE ANSWER: There are some authorities however, who believe their contract. (7) Which of the following is NOT within the power of a judicial receiver to
that the custodial rights do not apply during the preliminary investigation is a summary perform? (A) Bring an action in his name. (B) Compromise a claim. (C) Divide the residual
proceeding and merely inquisitorial in nature. Hence, the accused cannot yet invoke the money in his hands among the persons legally entitled to the same. Remedial Law Q&As
full exercise of his rights including the right to counsel. Moreover, a preliminary (2007-2013) hectorchristopher@yahoo.com JayArhSals “Never Let The Odds Keep You
investigation is not part of a trial and it is only in a trial where an accused can demand From Pursuing What You Know In Your Heart You Were Meant To Do.”-Leroy Satchel
the full exercise of his rights, such as the right to confront and cross-examine his accusers Paige Page 171 of 198 (D) Invest the funds in his hands without court approval. (8) Which
to Remedial Law Q&As (2007-2013) hectorchristopher@yahoo.com JayArhSals “Never of the following precepts forms part of the rules governing small claims? (A) Permissive
Let The Odds Keep You From Pursuing What You Know In Your Heart You Were Meant To counterclaim is not allowed. (B) The court shall render its decision within 3 days after
Do.”-Leroy Satchel Paige Page 168 of 198 establish his innocence (Albana vs. Belo, G.R. hearing. (C) Joinder of separate claims is not allowed. (D) Motion to declare defendant in
No. 158734, October 2, 2009, Leonardo-De Castro, J.). In a preliminary investigation, a default is allowed. (9) The Metropolitan Trial Court convicted Virgilio and Dina of
full and exhaustive presentation of the parties‟ evidence is not even required, but only concubinage. Pending appeal, they applied for bail, claiming they are entitled to it as a
such as may engender a well-grounded belief that an offense has been committed and matter of right. Is their claim correct? (A) No, bail is not a matter of right after conviction.
that the accused is probably guilty thereof. (George Miller vs. Secretary Hernando B. (B) Yes, bail is a matter of right in all cases not involving moral turpitude. (C) No, bail is
Perez, G.R. No. 165412, May 30, 2011, Villarama, Jr.). Ergo, the custodial rights of the dependent on the risk of flight. (D) Yes, bail is a matter of right in the Metropolitan Trial
accused are not available during the preliminary investigation. (c), At ultra-violet Court before and after conviction. (10) As a rule, the judge shall receive the evidence
examination to determine presence of ultra violet powder on accused‟s hands. The personally. In which of the following circumstances may the court delegate the reception
custodial rights of an accused are already available at the time an ultraviolet examination of evidence to the clerk of court? (A) When a question of fact arises upon a motion. (B)
to determine presence of ultra-violet powder on his hands is being conducted. There is a When the trial of an issue of fact requires the examination of a long account. (C) In
custodial investigation when a person is taken under the custody of the law or otherwise default or ex-parte hearings. (D) Upon motion of a party on reasonable grounds. (11)
deprived of his freedom of action in any significant way. “Custodial investigation is in the Which of the following is in accord with the applicable rules on receivership? (A) The
stage “where the police investigation is no longer a general inquiry into an unsolved court may appoint the plaintiff as receiver of the property in litigation over the
crime but has begun to focus on a particular suspect taken into custody by the police defendant’s objection. (B) A receiver may be appointed after judgment if the judgment
who carry out a process of interrogation that leads itself to elicit incriminating obligor refuses to apply his property to satisfy the judgment. (C) The trial court cannot
statements.” (People vs. Sunga, G.R. No. 126029, March 27, 2003). Otherwise stated, a appoint a receiver when the case is on appeal. (D) The filing of bond on appointment of a
custodial investigation begins when the investigation starts to focus on a particular receiver is mainly optional. Remedial Law Q&As (2007-2013)
suspect. Among the rights guaranteed to a suspect is that he must continuously have a hectorchristopher@yahoo.com JayArhSals “Never Let The Odds Keep You From Pursuing
counsel assisting him from the very start of that interrogation (Poeple vs. Morial, et. al., What You Know In Your Heart You Were Meant To Do.”-Leroy Satchel Paige Page 172 of
G.R. No. 129295, April 15, 2001). Clearly, when an accused is compelled to undergo ultra- 198 (12) Bearing in mind the distinction between private and public document, which of
violet examination to determine the presence of the ultraviolet powder on his hands, it is the following is admissible in evidence without further proof of due execution or
no longer a mere general inquiry but rather a custodial investigation which focuses on genuineness? (A) Baptismal certificates. (B) Official record of the Philippine Embassy in
him as a suspect in the commission of the crime. Therefore, for all intents and purposes, Singapore certified by the Vice- Consul with official seal. (C) Documents acknowledged
he is entitled to exercise his Constitutional safeguard and guaranteed rights to counsel before a Notary Public in Hong Kong. (D) Unblemished receipt dated December 20, 1985
and to remain silent. Remedial Law Q&As (2007-2013) hectorchristopher@yahoo.com signed by the promisee, showing payment of a loan, found among the well-kept file of
JayArhSals “Never Let The Odds Keep You From Pursuing What You Know In Your Heart the promissor. (13) Ramon witnessed the commission of a crime but he refuses to testify
You Were Meant To Do.”-Leroy Satchel Paige Page 169 of 198 2011 Remedial Law Exam for fear of his life despite a subpoena being served on him. Can the court punish him for
MCQ (November 27, 2011) (1) Anna filed a petition for appointment as regular contempt? (A) No, since no person can be compelled to be a witness against another. (B)
administratrix of her fathers' estate. Her sister Sophia moved to dismiss the petition on Yes, since public interest in justice requires his testimony. (C) No, since Ramon has a valid
the ground that the parties, as members of the same family, have not exerted earnest reason for not testifying. (D) Yes, since litigants need help in presenting their cases. (14)
effort toward a compromise prior to the filing of the petition. Should the petition be The right to intervene is not absolute. In general, it CANNOT be allowed where (A) the
dismissed? (A) Yes, since such earnest effort is jurisdictional in all estate cases. (B) No, intervenor has a common interest with any of the parties. (B) it would enlarge the issues
since such earnest effort is not required in special proceedings. (C) Yes, since such and expand the scope of the remedies. (C) the intervenor fails to put up a bond for the
earnest effort is required prior to the filing of the case. (D) No, since such earnest effort protection of the other parties. (D) the intervenor has a stake in the property subject of
toward a compromise is not required in summary proceedings. (2) A pending criminal the suit. (15) Which of the following grounds for dismissal invoked by the court will NOT
case, dismissed provisionally, shall be deemed permanently dismissed if not revived after PRECLUDE the plaintiff from refiling his action? (A) Res judicata. (B) Lack of jurisdiction
2 years with respect to offenses punishable by imprisonment (A) of more than 12 years. over the subject matter. (C) Unenforceability under the Statutes of Fraud. (D)
(B) not exceeding 6 years or a fine not exceeding P1,000.00. (C) of more than 6 years or a Prescription. (16) When may a co-owner NOT demand the partition of the thing owned in
fine in excess of P1,000.00. (D) of more than 6 years. (3) Angie was convicted of false common? Remedial Law Q&As (2007-2013) hectorchristopher@yahoo.com JayArhSals
testimony and served sentence. Five years later, she was convicted of homicide. On “Never Let The Odds Keep You From Pursuing What You Know In Your Heart You Were
appeal, she applied for bail. May the Court of Appeals deny her application for bail on Meant To Do.”-Leroy Satchel Paige Page 173 of 198 (A) When the creditor of one of the
ground of habitual delinquency? (A) Yes, the felonies are both punishable under the co-owners has attached the property. (B) When the property is essentially indivisible. (C)
Revised Penal Code. (B) Yes, her twin convictions indicated her criminal inclinations. (C) When related co-owners agreed to keep the property within the family. (D) When a co-
No, the felonies fall under different titles in the Revised Penal Code. (D) No, the charges owner uses the property as his residence. (17) The city prosecutor of Manila filed, upon
are both bailable. (4) Which of the following is NOT CONSISTENT with the rules governing Soledad’s complaint, a criminal action for estafa against her sister, Wella, before the RTC
of Manila for selling to Victor a land that she previously sold to Soledad. At the same Yes, his previous conviction requires posting of bail for the present charge. (B) Yes, since
time Soledad filed a civil action to annul the second sale before the RTC of Quezon City. he may be deemed to have violated the terms of his pardon. (C) No, because he is
May the Manila RTC motu proprio suspend the criminal action on ground of prejudicial presumed innocent until proven otherwise. (D) No, one charged with the violation of a
question? (A) Yes, if it may be clearly inferred that complainant will not object to the city ordinance is not required to post bail, notwithstanding a previous pardon. (29)
suspension of the criminal case. (B) No, the accused must file a motion to suspend the Which of the following claims survive the death of the defendant and need not be
action based on prejudicial question. (C) Yes, if it finds from the record that such presented as a claim against the estate? (A) Contingent money claims arising from
prejudicial question exists. (D) Yes, if it is convinced that due process and fair trial will be contract. (B) Unenforced money judgment against the decedent, with death occurring
better served if the criminal case is suspended. (18) Which of the following conforms to before levy on execution of the property. (C) Claims for damages arising from quasi-
the applicable rule on replevin? (A) The applicant must file a bond executed to the delict. (D) Claims for funeral expenses. (30) In a case, the prosecutor asked the medical
adverse party in an amount equal to the value of the property as determined by the expert the question, "Assuming that the assailant was behind the deceased Remedial
court. (B) The property has been wrongfully detained by the adverse party. (C) The Law Q&As (2007-2013) hectorchristopher@yahoo.com JayArhSals “Never Let The Odds
applicant has a contingent claim over the property object of the writ. (D) The plaintiff Keep You From Pursuing What You Know In Your Heart You Were Meant To Do.”-Leroy
may apply for the writ at any time before judgment. (19) Gerry sued XYZ Bus Co. and Satchel Paige Page 177 of 198 before he attacked him, would you say that treachery
Rico, its bus driver, for injuries Gerry suffered when their bus ran off the road and hit attended the killing?" Is this hypothetical question permissible? (A) No, since it asks for
him. Of the two defendants, only XYZ Bus Co. filed an answer, alleging that its bus ran off his legal opinion. (B) Yes, but conditionally, subject to subsequent proof that the
the road because one of its wheels got caught in an open manhole, causing the bus to assailant was indeed behind the deceased at that time. (C) Yes, since hypothetical
swerve without the driver’s fault. Someone had stolen the manhole cover and the road questions may be asked of an expert witness. (D) No, since the medical expert has no
gave no warning of the danger it posed. On Gerry’s motion and over the objection of XYZ personal knowledge of the fact. (31) The city prosecutor charged Ben with serious
Bus Co., the court declared Rico, the Remedial Law Q&As (2007-2013) physical injuries for stabbing Terence. He was tried and convicted as charged. A few days
hectorchristopher@yahoo.com JayArhSals “Never Let The Odds Keep You From Pursuing later, Terence died due to severe infection of his stab wounds. Can the prosecution file
What You Know In Your Heart You Were Meant To Do.”-Leroy Satchel Paige Page 174 of another information against Ben for homicide? (A) Yes, since Terence’s death shows
198 bus driver, in default and rendered judgment ordering him to pay P50,000 in irregularity in the filing of the earlier charge against him. (B) No, double jeopardy is
damages to Gerry. Did the court act correctly? (A) No, since the court should have tried present since Ben had already been convicted of the first offense. (C) No, there is double
the case against both defendants upon the bus company‟s answer. (B) No, the court jeopardy since serious physical injuries is necessarily included in the charge of homicide.
should have dropped Rico as defendant since the moneyed defendant is the bus (D) Yes, since supervening event altered the kind of crime the accused committed. (32)
company. (C) Yes, the court can, under the rules, render judgment against the defendant Arvin was caught in flagrante delicto selling drugs for P200,000.00. The police officers
declared in default. (D) Yes, since, in failing to answer, Rico may be deemed to have confiscated the drugs and the money and brought them to the police station where they
admitted the allegations in the complaint. (20) Which of the following has NO PLACE in prepared the inventory duly signed by police officer Oscar Moreno. They were, however,
an application for a replevin order? A statement (A) that the property is wrongfully unable to take pictures of the items. Will this deficiency destroy the chain of custody rule
detained by the adverse party. (B) that the property has not been distrained for a tax in the drug case? (A) No, a breach of the chain of custody rule in drug cases, if
assessment or placed under custodia legis. (C) of the assessed value of the property. (D) satisfactorily explained, will not negate conviction. (B) No, a breach of the chain of
that the applicant owns or has a right to the possession of the property. (21) 008-997- custody rule may be offset by presentation in court of the drugs. (C) Yes, chain of custody
0001 In which of the following instances is the quantum of evidence ERRONEOUSLY in drug cases must be strictly observed at all times to preserve the integrity of the
applied? (A) in Writ of Amparo cases, substantial evidence. (B) to satisfy the burden of confiscated items. (D) Yes, compliance with the chain of custody rule in drug cases is the
proof in civil cases, preponderance of evidence. (C) to overcome a disputable only way to prove the accused’s guilt beyond reasonable doubt. Remedial Law Q&As
presumption, clear and convincing evidence. (D) to rebut the presumptive validity of a (2007-2013) hectorchristopher@yahoo.com JayArhSals “Never Let The Odds Keep You
notarial document, substantial evidence. (22) The accused jumps bail and fails to appear From Pursuing What You Know In Your Heart You Were Meant To Do.”-Leroy Satchel
on promulgation of judgment where he is found guilty. What is the consequence of his Paige Page 178 of 198 (33) A sued B in the RTC of Quezon City, joining two causes of
absence? (A) Counsel may appeal the judgment in the absence of the accused. (B) The action: for partition of real property and breach of contract with damages. Both parties
judgment shall be promulgated in his absence and he loses his right of appeal. Remedial reside in Quezon City but the real property is in Manila. May the case be dismissed for
Law Q&As (2007-2013) hectorchristopher@yahoo.com JayArhSals “Never Let The Odds improper venue? (A) Yes, since causes of action pertaining to different venues may not
Keep You From Pursuing What You Know In Your Heart You Were Meant To Do.”-Leroy be joined in one action. (B) No, since causes of action pertaining to different venues may
Satchel Paige Page 175 of 198 (C) The promulgation of the judgment shall be suspended be joined in the RTC if one of the causes of action falls within its jurisdiction. (C) Yes,
until he is brought to the jurisdiction of the court. (D) The judgment shall be void. (23) because special civil action may not be joined with an ordinary civil action. (D) No, since
What should the court sheriff do if a third party serves on him an affidavit of claim plaintiff may unqualifiedly join in one complaint as many causes of action as he has
covering the property he had levied? (A) Ask the judgment obligee to file a court- against opposing party. (34) What is the doctrine of judicial stability or non interference?
approved indemnity bond in favor of the third-party claimant or the sheriff will release (A) Once jurisdiction has attached to a court, it can not be deprived of it by subsequent
the levied property. (B) Ask the judgment obligee to file a court-approved bond for the happenings or events. (B) Courts will not hear and decide cases involving issues that
sheriff’s protection in case he proceeds with the execution. (C) Immediately lift the levy come within the jurisdiction of administrative tribunals. (C) No court has the authority to
and release the levied property. (D) Ask the third-party claimant to support his claim with interfere by injunction with the judgment of another court of coordinate jurisdiction. (D)
an indemnity bond in favor of the judgment obligee and release the levied property if A higher court will not entertain direct resort to it unless the redress sought cannot be
such bond is filed. (24) Which of the following is NOT REGARDED as a sufficient proof of obtained from the appropriate court. (35) Which of the following admissions made by a
personal service of pleadings? (A) Official return of the server. (B) Registered mail party in the course of judicial proceedings is a judicial admission? (A) Admissions made in
receipt. (C) Written admission of the party served. (D) Affidavit of the server with a a pleading signed by the party and his counsel intended to be filed. (B) An admission
statement of the date, place and manner of service. (25) A sued B for ejectment. Pending made in a pleading in another case between the same parties. (C) Admission made by
trial, B died, survived by his son, C. No substitution of party defendant was made. Upon counsel in open court. (D) Admissions made in a complaint superseded by an amended
finality of the judgment against B, may the same be enforced against C? (A) Yes, because complaint. (36) What defenses may be raised in a suit to enforce a foreign judgment? (A)
the case survived B‟s death and the effect of final judgment in an ejectment case binds That the judgment is contrary to Philippine procedural rules. Remedial Law Q&As (2007-
his successors in-interest. (B) No, because C was denied due process. (C) Yes, because the 2013) hectorchristopher@yahoo.com JayArhSals “Never Let The Odds Keep You From
negligence of B’s counsel in failing to ask for substitution, should not prejudice A. (D) No, Pursuing What You Know In Your Heart You Were Meant To Do.”-Leroy Satchel Paige
because the action did not survive B’s death. (26) What is the proper remedy to secure Page 179 of 198 (B) None, the judgment being entitled to full faith and credit as a matter
relief from the final resolutions of the Commission On Audit? (A) Petition for review on of general comity among nations. (C) That the foreign court erred in the appreciation of
certiorari with the Supreme Court. Remedial Law Q&As (2007-2013) the evidence. (D) That extrinsic fraud afflicted the judgment. (37) Cindy charged her
hectorchristopher@yahoo.com JayArhSals “Never Let The Odds Keep You From Pursuing husband, George, with bigamy for a prior subsisting marriage with Teresa. Cindy
What You Know In Your Heart You Were Meant To Do.”-Leroy Satchel Paige Page 176 of presented Ric and Pat, neighbors of George and Teresa in Cebu City, to prove, first, that
198 (B) Special civil action of certiorari with the Court of Appeals. (C) Special civil action George and Teresa cohabited there and, second, that they established a reputation as
of certiorari with the Supreme Court. (D) Appeal to the Court of Appeals. (27) Which of husband and wife. Can Cindy prove the bigamy by such evidence? (A) Yes, the
the following is a duty enjoined on the guardian and covered by his bond? (A) Provide for circumstantial evidence is enough to support a conviction for bigamy. (B) No, at least one
the proper care, custody, and education of the ward. (B) Ensure the wise and profitable direct evidence and two circumstantial evidence are required to support a conviction for
investment of the ward’s financial resources. (C) Collect compensation for his services to bigamy. (C) No, the circumstantial evidence is not enough to support a conviction for
the ward. (D) Raise the ward to become a responsible member of society. (28) Berto was bigamy. (D) No, the circumstantial evidence cannot overcome the lack of direct evidence
charged with and convicted of violating a city ordinance against littering in public places in any criminal case. (38) To prove payment of a debt, Bong testified that he heard Ambo
punishable by imprisonment of one month or a fine of P1,000.00. But the city mayor say, as the latter was handing over money to Tessie, that it was in payment of debt. Is
pardoned him. A year later, he was charged with violating a city ordinance against Bong’s testimony admissible in evidence? (A) Yes, since what Ambo said and did is an
jaywalking which carried the same penalty. Need Berto post bail for such offense? (A) independently relevant statement. (B) No, since what Ambo said and did was not in
response to a startling occurrence. (C) No, since Bong’s testimony of what Ambo said and bond. (D) When the accused changes his address without notice to the bondsman.
did is hearsay. (D) Yes, since Ambo‟s statement and action, subject of Bong‟s testimony, Remedial Law Q&As (2007-2013) hectorchristopher@yahoo.com JayArhSals “Never Let
constitutes a verbal act. (39) Considering the qualifications required of a would-be The Odds Keep You From Pursuing What You Know In Your Heart You Were Meant To
witness, who among the following is INCOMPETENT to testify? (A) A person under the Do.”-Leroy Satchel Paige Page 183 of 198 (50) Which of the following MISSTATES a
influence of drugs when the event he is asked to testify on took place. (B) A person requisite for the issuance of a search warrant? (A) The warrant specifically describes the
convicted of perjury who will testify as an attesting witness to a will. Remedial Law Q&As place to be searched and the things to be seized. (B) Presence of probable cause. (C) The
(2007-2013) hectorchristopher@yahoo.com JayArhSals “Never Let The Odds Keep You warrant issues in connection with one specific offense. (D) Judge determines probable
From Pursuing What You Know In Your Heart You Were Meant To Do.”-Leroy Satchel cause upon the affidavits of the complainant and his witnesses. (51) Ranger Motors filed
Paige Page 180 of 198 (C) A deaf and dumb. (D) A mental retardate. (40) Arthur, a a replevin suit against Bart to recover possession of a car that he mortgaged to it. Bart
resident foreigner sold his car to Bren. After being paid but before delivering the car, disputed the claim. Meantime, the court allowed, with no opposition from the parties,
Arthur replaced its original sound system with an inferior one. Bren discovered the Midway Repair Shop to intervene with its claim against Bart for unpaid repair bills. On
change, rejected the car, and demanded the return of his money. Arthur did not comply. subsequent motion of Ranger Motors and Bart, the court dismissed the complaint as well
Meantime, his company reassigned Arthur to Singapore. Bren filed a civil action against as Midway Repair Shop’s intervention. Did the court act correctly? (A) No, since the
Arthur for contractual fraud and damages. Upon his application, the court issued a writ dismissal of the intervention bars the right of Bart to file a separate action. (B) Yes,
of preliminary attachment on the grounds that (a) Arthur is a foreigner; (b) he departed intervention is merely collateral to the principal action and not an independent
from the Philippines; and (c) he was guilty of fraud in contracting with Bren. Is the writ of proceeding. (C) Yes, the right of the intervenor is merely in aid of the right of the original
preliminary attachment proper? (A) No, Arthur is a foreigner living abroad; he is outside party, which in this case had ceased to exist. (D) No, since having been allowed to
the court’s jurisdiction. (B) Yes, Arthur committed fraud in changing the sound system intervene, the intervenor became a party to the action, entitled to have the issue it
and its components before delivering the car bought from him. (C) Yes the timing of his raised tried and decided. (52) The accused was convicted for estafa thru falsification of
departure is presumptive evidence of intent to defraud. (D) No, since it was not shown public document filed by one of two offended parties. Can the other offended party
that Arthur left the country with intent to defraud Bren. (41) What is the movant’s charge him again with the same crime? (A) Yes, since the wrong done the second
remedy if the trial court incorrectly denies his motion to dismiss and related motion for offended party is a separate crime. (B) No, since the offense refers to the same series of
reconsideration? (A) Answer the complaint. (B) File an administrative action for gross act, prompted by one criminal intent. (C) Yes, since the second offended party is entitled
ignorance of the law against the trial judge. (C) File a special civil action of certiorari on to the vindication of the wrong done him as well. (D) No, since the second offended
ground of grave abuse of discretion. (D) Appeal the orders of denial. (42) During trial, party is in estoppel, not having joined the first criminal action. (53) Henry testified that a
plaintiff offered evidence that appeared irrelevant at that time but he said he was month after the robbery Asiong, one of the accused, told him that Carlos was one of
eventually going to relate to the issue in the case by some future evidence. The those who Remedial Law Q&As (2007-2013) hectorchristopher@yahoo.com JayArhSals
defendant objected. Should the trial court reject the evidence in question on ground of “Never Let The Odds Keep You From Pursuing What You Know In Your Heart You Were
irrelevance? (A) No, it should reserve its ruling until the relevance is shown. (B) Yes, since Meant To Do.”-Leroy Satchel Paige Page 184 of 198 committed the crime with him. Is
the plaintiff could anyway subsequently present the evidence anew. Remedial Law Q&As Henry’s testimony regarding what Asiong told him admissible in evidence against Carlos?
(2007-2013) hectorchristopher@yahoo.com JayArhSals “Never Let The Odds Keep You (A) No, since it is hearsay. (B) No, since Asiong did not make the statement during the
From Pursuing What You Know In Your Heart You Were Meant To Do.”-Leroy Satchel conspiracy. (C) Yes, since it constitutes admission against a co-conspirator. (D) Yes, since
Paige Page 181 of 198 (C) Yes, since irrelevant evidence is not admissible. (D) No, it it part of the res gestae. (54) Dorothy filed a petition for writ of habeas corpus against
should admit it conditionally until its relevance is shown. (43) Ben testified that Jaime, her husband, Roy, to get from him custody of their 5 year old son, Jeff. The court granted
charged with robbery, has committed bag-snatching three times on the same street in the petition and required Roy to turn over Jeff to his mother. Roy sought reconsideration
the last six months. Can the court admit this testimony as evidence against Jaime? (A) but the court denied it. He filed a notice of appeal five days from receipt of the order
No, since there is no showing that Ben witnessed the past three robberies. (B) Yes, as denying his motion for reconsideration. Did he file a timely notice of appeal? (A) No,
evidence of his past propensity for committing robbery. (C) Yes, as evidence of a pattern since he filed it more than 2 days after receipt of the decision granting the petition. (B)
of criminal behavior proving his guilt of the present offense. (D) No, since evidence of No, since he filed it more than 2 days after receipt of the order denying his motion for
guilt of a past crime is not evidence of guilt of a present crime. (44) What is the right reconsideration. (C) Yes, since he filed it within 15 days from receipt of the denial of his
correlation between a criminal action and a petition for Writ of Amparo both arising motion for reconsideration. (D) Yes, since he filed it within 7 days from receipt of the
from the same set of facts? (A) When the criminal action is filed after the Amparo denial of his motion for reconsideration. (55) Angel Kubeta filed a petition to change his
petition, the latter shall be dismissed. (B) The proceeding in an Amparo petition is first name "Angel." After the required publication but before any opposition could be
criminal in nature. (C) No separate criminal action may be instituted after an Amparo received, he filed a notice of dismissal. The court confirmed the dismissal without
petition is filed. (D) When the criminal action is filed after the Amparo petition, the latter prejudice. Five days later, he filed another petition, this time to change his surname
shall be consolidated with the first. (45) Alex filed a petition for writ of amparo against "Kubeta." Again, Angel filed a notice of dismissal after the publication. This time,
Melba relative to his daughter Toni's involuntary disappearance. Alex said that Melba however, the court issued an order, confirming the dismissal of the case with prejudice.
was Toni's employer, who, days before Toni disappeared, threatened to get rid of her at Is the dismissal with prejudice correct? (A) Yes, since such dismissal with prejudice is
all costs. On the other hand, Melba countered that she had nothing to do with Toni's mandatory. (B) No, since the rule on dismissal of action upon the plaintiff’s notice does
disappearance and that she took steps to ascertain Toni's whereabouts. What is the not apply to special proceedings. (C) No, since change of name does not involve public
quantum of evidence required to establish the parties' respective claims? (A) For Alex, interest and the rules should be liberally construed. (D) Yes, since the rule on dismissal of
probable cause; for Melba, substantial evidence. (B) For Alex, preponderance of action upon the Remedial Law Q&As (2007-2013) hectorchristopher@yahoo.com
evidence; for Melba, substantial evidence. (C) For Alex, proof beyond reasonable doubt; JayArhSals “Never Let The Odds Keep You From Pursuing What You Know In Your Heart
for Melba, ordinary diligence. (D) For both, substantial evidence. Remedial Law Q&As You Were Meant To Do.”-Leroy Satchel Paige Page 185 of 198 plaintiff‟s notice applies
(2007-2013) hectorchristopher@yahoo.com JayArhSals “Never Let The Odds Keep You and the two cases involve a change in name. (56) A complaint without the required
From Pursuing What You Know In Your Heart You Were Meant To Do.”-Leroy Satchel "verification" (A) shall be treated as unsigned. (B) lacks a jurisdictional requirement. (C) is
Paige Page 182 of 198 (46) In which of the following situations is the declaration of a a sham pleading. (D) is considered not filed and should be expunged. (57) The decisions
deceased person against his interest NOT ADMISSIBLE against him or his successors and of the Commission on Elections or the Commission on Audit may be challenged by (A)
against third persons? (A) Declaration of a joint debtor while the debt subsisted. (B) petition for review on certiorari filed with the Supreme Court under Rule 45. (B) petition
Declaration of a joint owner in the course of ownership. (C) Declaration of a former for review on certiorari filed with the Court of Appeals under Rule 42. (C) appeal to the
copartner after the partnership has been dissolved. (D) Declaration of an agent within Supreme Court under Rule 54. (D) special civil action of certiorari under Rule 65 filed with
the scope of his authority. (47) Defendant Dante said in his answer: "1. Plaintiff Perla the Supreme Court. (58) Which of the following states a correct guideline in hearing
claims that defendant Dante owes her P4,000 on the mobile phone that she sold him; 2. applications for bail in capital offenses? (A) The hearing for bail in capital offenses is
But Perla owes Dante P6,000 for the dent on his car that she borrowed." How should the summary; the court does not sit to try the merits of the case. (B) The prosecution’s
court treat the second statement? (A) A cross claim (B) A compulsory counterclaim (C) A conformity to the accused’s motion for bail is proof that its evidence of his guilt is not
third party complaint (D) A permissive counterclaim (48) How will the court sheriff strong. (C) The accused, as applicant for bail, carries the burden of showing that the
enforce the demolition of improvements? (A) He will give a 5-day notice to the judgment prosecution’s evidence of his guilt is not strong. (D) The prosecution must have full
obligor and, if the latter does not comply, the sheriff will have the improvements forcibly opportunity to prove the guilt of the accused. (59) Apart from the case for the settlement
demolished. (B) He will report to the court the judgment obligor’s refusal to comply and of her parents' estate, Betty filed an action against her sister, Sigma, for reconveyance of
have the latter cited in contempt of court. (C) He will demolish the improvements on title to a piece of land. Betty claimed that Sigma forged the signatures of their late
special order of the court, obtained at the judgment obligee‟s motion. (D) He will inform parents to make it appear that they sold the land to her when they did not, thus
the court of the judgment obligor’s noncompliance and proceed to demolish the prejudicing Betty’s legitime. Sigma moved to dismiss the action on the ground that the
improvements. (49) When may the bail of the accused be cancelled at the instance of the dispute should be resolved in the estate proceedings. Is Sigma correct? Remedial Law
bondsman? (A) When the accused jumps bail. (B) When the bondsman surrenders the Q&As (2007-2013) hectorchristopher@yahoo.com JayArhSals “Never Let The Odds Keep
accused to the court. (C) When the accused fails to pay his annual premium on the bail You From Pursuing What You Know In Your Heart You Were Meant To Do.”-Leroy Satchel
Paige Page 186 of 198 (A) Yes, questions of collation should be resolved in the estate party. (72) Which of the following CANNOT be disputably presumed under the rules of
proceedings, not in a separate civil case. (B) No, since questions of ownership of property evidence? (A) That the thing once proved to exist continues as long as is usual with things
cannot be resolved in the estate proceedings. (C) Yes, in the sense that Betty needs to of that nature. (B) That the law has been obeyed. (C) That a writing is truly dated. (D)
wait until the estate case has been terminated. (D) No, the filing of the separate action is That a young person, absent for 5 years, it being unknown whether he still lives, is
proper; but the estate proceeding must be suspended meantime. (60) What is the considered dead for purposes of succession. (73) Which of the following is NOT
consequence of the unjustified absence of the defendant at the pre-trial? (A) The trial REQUIRED in a petition for mandamus? (A) The act to be performed is not discretionary.
court shall declare him as in default. (B) The trial court shall immediately render (B) There is no other adequate remedy in the ordinary course of law. (C) The respondent
judgment against him. (C) The trial court shall allow the plaintiff to present evidence neglects to perform a clear duty under a contract. (D) The petitioner has a clear legal
exparte. (D) The trial court shall expunge his answer from the record. (61) What is the right to the act demanded. Remedial Law Q&As (2007-2013)
remedy of the accused if the trial court erroneously denies his motion for preliminary hectorchristopher@yahoo.com JayArhSals “Never Let The Odds Keep You From Pursuing
investigation of the charge against him? (A) Wait for judgment and, on appeal from it, What You Know In Your Heart You Were Meant To Do.”-Leroy Satchel Paige Page 190 of
assign such denial as error. (B) None since such order is final and executory. (C) Ask for 198 (74) When is the defendant entitled to the return of the property taken under a writ
reconsideration; if denied, file petition for certiorari and prohibition. (D) Appeal the of replevin? (A) When the plaintiff‟s bond is found insufficient or defective and is not
order denying the motion for preliminary investigation. (62) Which of the following replaced. (B) When the defendant posts a redelivery bond equal to the value of the
renders a complaint for unlawful detainer deficient? (A) The defendant claims that he property seized. (C) When the plaintiff takes the property and disposes of it without the
owns the subject property. (B) The plaintiff has tolerated defendant’s possession for 2 sheriff’s approval. (D) When a third party claims the property taken yet the applicant
years before demanding that he vacate it. (C) The plaintiff‟s demand is for the lessee to does not file a bond in favor of the sheriff. (75) Character evidence is admissible (A) in
pay back rentals or vacate. (D) The lessor institutes the action against a lessee who has criminal cases, the accused may prove his good moral character if pertinent to the moral
not paid the stipulated rents. (63) In a judicial foreclosure proceeding, under which of the trait involved in the offense charged. (B) in criminal cases, the prosecution may prove the
following instances is Remedial Law Q&As (2007-2013) hectorchristopher@yahoo.com bad moral character of the accused to prove his criminal predisposition. (C) in criminal
JayArhSals “Never Let The Odds Keep You From Pursuing What You Know In Your Heart cases, the bad moral character of the offended party may not be proved. (D) when it is
You Were Meant To Do.”-Leroy Satchel Paige Page 187 of 198 the court NOT ALLOWED evidence of the good character of a witness even prior to impeachment. (76) X’s action
to render deficiency judgment for the plaintiff? (A) If the mortgagee is a banking for sum of money against Y amounting to P80,000.00 accrued before the effectivity of
institution. (B) if upon the mortgagor’s death during the proceeding, the mortgagee the rule providing for shortened procedure in adjudicating claims that do not exceed
submits his claim in the estate proceeding. (C) If the mortgagor is a third party who is not P100,000.00. X filed his action after the rule took effect. Will the new rule apply to his
solidarily liable with the debtor. (D) If the mortgagor is a nonresident person and cannot case? (A) No since what applies is the rule in force at the time the cause of action
be found in the Philippines. (64) In which of the following cases is the plaintiff the real accrued. (B) No, since new procedural rules cover only cases where the issues have
party in interest? (A) A creditor of one of the co-owners of a parcel of land, suing for already been joined. (C) Yes, since procedural rules have retroactive effect. (D) Yes, since
partition (B) An agent acting in his own name suing for the benefit of a disclosed principal procedural rules generally apply prospectively to pending cases. (77) A motion for
(C) Assignee of the lessor in an action for unlawful detainer (D) An administrator suing for reconsideration of a decision is pro forma when (A) it does not specify the defects in the
damages arising from the death of the decedent (65) The defendant in an action for sum judgment. (B) it is a second motion for reconsideration with an alternative prayer for
of money filed a motion to dismiss the complaint on the ground of improper venue. After new trial. Remedial Law Q&As (2007-2013) hectorchristopher@yahoo.com JayArhSals
hearing, the court denied the motion. In his answer, the defendant claimed prescription “Never Let The Odds Keep You From Pursuing What You Know In Your Heart You Were
of action as affirmative defense, citing the date alleged in the complaint when the cause Meant To Do.”-Leroy Satchel Paige Page 191 of 198 (C) it reiterates the issues already
of action accrued. May the court, after hearing, dismiss the action on ground of passed upon but invites a second look at the evidence and the arguments. (D) its
prescription? (A) Yes, because prescription is an exception to the rule on Omnibus arguments in support of the alleged errors are grossly erroneous. (78) Which of the
Motion. (B) No, because affirmative defenses are barred by the earlier motion to dismiss. following correctly states the rule on foreclosure of mortgages? (A) The rule on
(C) Yes, because the defense of prescription of action can be raised at anytime before the foreclosure of real estate mortgage is suppletorily applicable to extrajudicial
finality of judgment. (D) No, because of the rule on Omnibus Motion. (66) What is the foreclosures. (B) In judicial foreclosure, an order of confirmation is necessary to vest all
effect of the failure of the accused to file a motion to quash an information that charges rights in the purchaser. (C) There is equity of redemption in extra-judicial foreclosure. (D)
two offenses? (A) He may be convicted only of the more serious offense. Remedial Law A right of redemption by the judgment obligor exists in judicial foreclosure. (79) The
Q&As (2007-2013) hectorchristopher@yahoo.com JayArhSals “Never Let The Odds Keep information charges PNP Chief Luis Santos, (Salary Grade 28), with "taking advantage of
You From Pursuing What You Know In Your Heart You Were Meant To Do.”-Leroy Satchel his public position as PNP Head by feloniously shooting JOSE ONA, inflicting on the latter
Paige Page 188 of 198 (B) He may in general be convicted of both offenses. (C) The trial mortal wounds which caused his death." Based solely on this allegation, which court has
shall be void. (D) He may be convicted only of the lesser offense. (67) Which of the jurisdiction over the case? (A) Sandiganbayan only (B) Sandiganbayan or Regional Trial
following is a correct application of the rules involved in consolidation of cases? (A) Court (C) Sandiganbayan or Court Martial (D) Regional Trial Court only (80) Distinguish
Consolidation of cases pending in different divisions of an appellate court is not allowed. between conclusiveness of judgment and bar by prior judgment. (A) Conclusiveness of
(B) The court in which several cases are pending involving common questions of law and judgment bars another action based on the same cause; bar by prior judgment precludes
facts may hear initially the principal case and suspend the hearing in the other cases. (C) another action based on the same issue. (B) Conclusiveness of judgment bars only the
Consolidation of cases pending in different branches or different courts is not defendant from questioning it; bar by prior judgment bars both plaintiff and defendant.
permissible. (D) The consolidation of cases is done only for trial purposes and not for (C) Conclusiveness of judgment bars all matters directly adjudged; bar by prior judgment
appeal. (68) Summons was served on "MCM Theater," a business entity with no juridical precludes all matters that might have been adjudged. (D) Conclusiveness of judgment
personality, through its office manager at its place of business. Did the court acquire precludes the filing of an action to annul such judgment; bar by prior judgment allows
jurisdiction over MCM Theater’s owners? (A) Yes, an unregistered entity like MCM the filing of such an action. (81) Which of the following matters is NOT A PROPER
Theater may be served with summons through its office manager. (B) No, because MCM SUBJECT of judicial notice? Remedial Law Q&As (2007-2013)
has no juridical personality and cannot be sued. (C) No, since the real parties in interest, hectorchristopher@yahoo.com JayArhSals “Never Let The Odds Keep You From Pursuing
the owners of MCM Theater, have not been served with summons. (D) Yes since MCM, What You Know In Your Heart You Were Meant To Do.”-Leroy Satchel Paige Page 192 of
as business entity, is a de facto partnership with juridical personality. (69) Fraud as a 198 (A) Persons have killed even without motive. (B) Municipal ordinances in the
ground for new trial must be extrinsic as distinguished from intrinsic. Which of the municipalities where the MCTC sits. (C) Teleconferencing is now a way of conducting
following constitutes extrinsic fraud? (A) Collusive suppression by plaintiff‟s counsel of a business transactions. (D) British law on succession personally known to the presiding
material evidence vital to his cause of action. (B) Use of perjured testimony at the trial. judge. (82) The RTC of Malolos, Branch 1, issued a writ of execution against Rene for P20
(C) The defendant’s fraudulent representation that caused damage to the plaintiff. (D) million. The sheriff levied on a school building that appeared to be owned by Rene.
Use of falsified documents during the trial. Remedial Law Q&As (2007-2013) Marie, however, filed a third party claim with the sheriff, despite which, the latter
hectorchristopher@yahoo.com JayArhSals “Never Let The Odds Keep You From Pursuing scheduled the execution sale. Marie then filed a separate action before the RTC of
What You Know In Your Heart You Were Meant To Do.”-Leroy Satchel Paige Page 189 of Malolos, Branch 2, which issued a writ of preliminary injunction enjoining the sheriff
198 (70) Upon review, the Secretary of Justice ordered the public prosecutor to file a from taking possession and proceeding with the sale of the levied property. Did Branch 2
motion to withdraw the information for estafa against Sagun for lack of probable cause. correctly act in issuing the injunction? (A) Yes, since the rules allow the filing of the
The public prosecutor complied. Is the trial court bound to grant the withdrawal? (A) Yes, independent suit to check the sheriff‟s wrongful act in levying on a third party‟s
since the prosecution of an action is a prerogative of the public prosecutor. (B) No, since property. (B) Yes, since Branch 2, like Branch 1, is part of the RTC of Malolos. (C) No,
the complainant has already acquired a vested right in the information. (C) No, since the because the proper remedy is to seek relief from the same court which rendered the
court has the power after the case is filed to itself determine probable cause. (D) Yes, judgment. (D) No, since it constitutes interference with the judgment of a co-equal court
since the decision of the Secretary of Justice in criminal matters is binding on courts. (71) with concurrent jurisdiction. (83) What is the effect and ramification of an order allowing
Unexplained or unjustified non-joinder in the Complaint of a necessary party despite new trial? (A) The court’s decision shall be held in suspension until the defendant could
court order results in (A) the dismissal of the Complaint. (B) suspension of proceedings. show at the reopening of trial that it has to be abandoned. (B) The court shall maintain
(C) contempt of court. (D) waiver of plaintiff‟s right against the unpleaded necessary the part of its judgment that is unaffected and void the rest. (C) The evidence taken upon
the former trial, if material and competent, shall remain in use. (D) The court shall vacate Page 196 of 198 (A) No, since the surety‟s undertaking is not annual but lasts up to
the judgment as well as the entire proceedings had in the case. (84) Which of the judgment. (B) Yes, since surety companies would fold up otherwise. (C) No, since the
following is sufficient to disallow a will on the ground of mistake? (A) An error in the surety company technically takes the place of the accused with respect to court
description of the land devised in the will. Remedial Law Q&As (2007-2013) attendance. (D) Yes, since the accused has breached its agreement with the surety
hectorchristopher@yahoo.com JayArhSals “Never Let The Odds Keep You From Pursuing company. (96) To prove that Susan stabbed her husband Elmer, Rico testified that he
What You Know In Your Heart You Were Meant To Do.”-Leroy Satchel Paige Page 193 of heard Leon running down the street, shouting excitedly, "Sinasaksak daw ni Susan ang
198 (B) The inclusion for distribution among the heirs of properties not belonging to the asawa niya! (I heard that Susan is stabbing her husband!)" Is Leon's statement as
testator. (C) The testator intended a donation intervivos but unwittingly executed a will. narrated by Rico admissible? (A) No, since the startling event had passed. (B) Yes, as part
(D) An error in the name of the person nominated as executor. (85) As a rule, the estate of the res gestae. (C) No, since the excited statement is itself hearsay. (D) Yes, as an
shall not be distributed prior to the payment of all charges to the estate. What will justify independently relevant statement. (97) Which of the following NOT TRUE regarding the
advance distribution as an exception? (A) The estate has sufficient residual assets and doctrine of judicial hierarchy? (A) It derives from a specific and mandatory provision of
the distributees file sufficient bond. (B) The specific property sought to be distributed substantive law. (B) The Supreme Court may disregard the doctrine in cases of national
might suffer in value. (C) An agreement among the heirs regarding such distribution. (D) interest and matters of serious implications. (C) A higher court will not entertain direct
The conformity of the majority of the creditors to such distribution. (86) A party recourse to it if redress can be obtained in the appropriate courts. (D) The reason for it is
aggrieved by an interlocutory order of the Civil Service Commission (CSC) filed a petition the need for higher courts to devote more time to matters within their exclusive
for certiorari and prohibition with the Court of Appeals. May the Court of Appeals take jurisdiction. (98) Plaintiff Manny said in his complaint: "3. On March 1, 2001 defendant
cognizance of the petition? (A) Yes, provided it raises both questions of facts and law. (B) Letty borrowed P1 million from plaintiff Manny and made a promise to pay the loan
No, since the CSC Chairman and Commissioners have the rank of Justices of the Court of within six months." In her answer, Letty alleged: "Defendant Letty specifically denies the
Appeals. (C) No, since the CSC is a Constitutional Commission. (D) Yes, since the Court of allegations in paragraph 3 of the complaint that she borrowed P1 million from plaintiff
Appeals has jurisdiction over the petition concurrent with the Supreme Court. (87) Which Manny on March 1, 2001 and made a promise to pay the loan within six months." Is
of the following is appealable? (A) An order of default against the defendant. (B) The Letty’s denial sufficient? (A) Yes, since it constitutes specific denial of the loan. (B) Yes,
denial of a motion to dismiss based on improper venue. (C) The dismissal of an action since it constitutes positive denial of the existence of the loan. Remedial Law Q&As
with prejudice. (D) The disallowance of an appeal. (88) Which of the following is NOT (2007-2013) hectorchristopher@yahoo.com JayArhSals “Never Let The Odds Keep You
REQUIRED of a declaration against interest as an exception to the hearsay rule? (A) The From Pursuing What You Know In Your Heart You Were Meant To Do.”-Leroy Satchel
declarant had no motive to falsify and believed such declaration to be true. (B) The Paige Page 197 of 198 (C) No, since it fails to set forth the matters defendant relied upon
declarant is dead or unable to testify. Remedial Law Q&As (2007-2013) in support of her denial. (D) No, since she fails to set out in par. 2 of her answer her
hectorchristopher@yahoo.com JayArhSals “Never Let The Odds Keep You From Pursuing special and affirmative defenses. (99) When may an information be filed in court without
What You Know In Your Heart You Were Meant To Do.”-Leroy Satchel Paige Page 194 of the preliminary investigation required in the particular case being first conducted? (A)
198 (C) The declaration relates to a fact against the interest of the declarant. (D) At the Following an inquest, in cases of those lawfully arrested without a warrant. (B) When the
time he made said declaration he was unaware that the same was contrary to his accused, while under custodial investigation, informs the arresting officers that he is
aforesaid interest. (89) To prove the identity of the assailant in a crime of homicide, a waiving his right to preliminary investigation. (C) When the accused fails to challenge the
police officer testified that, Andy, who did not testify in court, pointed a finger at the validity of the warrantless arrest at his arraignment. (D) When the arresting officers take
accused in a police lineup. Is the police officer’s testimony regarding Andy's identification the suspect before the judge who issues a detention order against him. (100) In a civil
of the accused admissible evidence? (A) Yes, since it is based on his personal knowledge action involving three separate causes of action, the court rendered summary judgment
of Andy’s identification of the accused. (B) Yes, since it constitutes an independently on the first two causes of action and tried the third. After the period to appeal from the
relevant statement. (C) No, since the police had the accused identified without warning summary judgment expired, the court issued a writ of execution to enforce the same. Is
him of his rights. (D) No, since the testimony is hearsay. (90) In which of the following the writ of execution proper? (A) No, being partial, the summary judgment is
cases is the testimony in a case involving a deceased barred by the Survivorship interlocutory and any appeal from it still has to reckon with the final judgment. (B) Yes
Disqualification Rule or Dead Man Statute? (A) Testimony against the heirs of the since, assuming the judgment was not appealable, the defendant should have
deceased defendant who are substituted for the latter. (B) The testimony of a mere questioned it by special civil action of certiorari. (C) No, since the rules do not allow a
witness who is neither a party to the case nor is in privity with the latter. (C) The partial summary judgment. (D) No, since special reason is required for execution pending
testimony of an oppositor in a land registration case filed by the decedent’s heirs. (D) rendition of a final decision in the case. Remedial Law Q&As (2007-2013)
The testimony is offered to prove a claim less than what is established under a written hectorchristopher@yahoo.com JayArhSals “Never Let The Odds Keep You From Pursuing
document signed by the decedent. (91) The prosecution moved for the discharge of What You Know In Your Heart You Were Meant To Do.”-Leroy Satchel Paige Page 198 of
Romy as state witness in a robbery case it filed against Zoilo, Amado, and him. Romy 198 References:  Answers to Bar Examination Questions by the UP LAW COMPLEX
testified, consistent with the sworn statement that he gave the prosecution. After (2007, 2009, 2010)  PHILIPPINE ASSOCIATION OF LAW SCHOOLS (2008)  lawphil.net
hearing Romy, the court denied the motion for his discharge. How will denial affect
Romy? (A) His testimony shall remain on record. (B) Romy will be prosecuted along with SUGGESTED ANSWERS TO 2014 REMEDIAL LAW BAR EXAM I. Ludong, Balatong, and
Zoilo and Amado. (C) His liability, if any, will be mitigated. (D) The court can convict him Labong were charged with murder. After trial, the court announced that the case was
based on his testimony. Remedial Law Q&As (2007-2013) hectorchristopher@yahoo.com considered submitted for decision. Subsequently, the Clerk of Court issued the notices of
JayArhSals “Never Let The Odds Keep You From Pursuing What You Know In Your Heart promulgation of judgment which were duly received. On promulgation day, Ludong and
You Were Meant To Do.”-Leroy Satchel Paige Page 195 of 198 (92) In proceedings for the his lawyer appeared. The lawyers of Balatong and Labong appeared but without their
settlement of the estate of deceased persons, the court in which the action is pending clients and failed to satisfactorily explain their absence when queried by the court. Thus,
may properly (A) pass upon question of ownership of a real property in the name of the the judge ordered the Clerk of Court to proceed with the reading of the judgment
deceased but claimed by a stranger. (B) pass upon with the consent of all the heirs the convicting all the accused. With respect to Balatong and Labong, the judge ordered that
issue of ownership of estate asset, contested by an heir if no third person is affected. (C) the judgment be entered in the criminal docket and copies be furnished their lawyers.
rule on a claim by one of the heirs that an estate asset was held in trust for him by the The lawyers of Ludong, Balatong, and Labong filed within the reglementary period a Joint
deceased. (D) rescind a contract of lease entered into by the deceased before death on Motion for Reconsideration. The court favorably granted the motion of Ludong
the ground of contractual breach by the lessee. (93) Which of the following stipulations downgrading his conviction from murder to homicide but denied the motion as regards
in a contract will supersede the venue for actions that the rules of civil procedure fix? (A) Balatong and Labong. (4%) (A) Was the court correct in taking cognizance of the Joint
In case of litigation arising from this contract of sale, the preferred venue shall be in the Motion for Reconsideration? (B) Can Balatong and Labong appeal their conviction in case
proper courts of Makati. (B) Should the real owner succeed in recovering his stolen car Ludong accepts his conviction for homicide? ANSWERS: (A) No, the court was not correct
from buyer X, the latter shall have recourse under this contract to seller Y exclusively in taking cognizance of the Joint Motion for Reconsideration insofar as Balatong and
before the proper Cebu City court. (C) Venue in case of dispute between the parties to Labong were concerned. Under Section 6 Rule 120, if the judgment was for conviction
this contract shall solely be in the proper courts of Quezon City. (D) Any dispute arising and the failure of the accused to appear was without justifiable cause, he shall lose the
from this contract of sale may be filed in Makati or Quezon City. (94) Allan was riding a remedies available under the Rules of Court and the court shall order his arrest. The
passenger jeepney driven by Ben that collided with a car driven by Cesar, causing Allan accused may regain the remedies only if he surrenders and files a motion for leave to
injury. Not knowing who was at fault, what is the best that Allan can do? (A) File a tort avail of the remedies under the Rules of Court. Here the failure of Balatong and Labong
action against Cesar. (B) Await a judicial finding regarding who was at fault. (C) Sue Ben to appear was without justifiable cause as even their lawyers were not aware of the
for breach of contract of carriage. (D) Sue both Ben and Cesar as alternative defendants. reason for their absence. Hence they lost their remedies under the Rules. Since Balatong
(95) A surety company, which provided the bail bond for the release of the accused, filed and Labong did not surrender and file a motion for leave to avail of remedies, it was
a motion to withdraw as surety on the ground of the accused’s non-payment of the incorrect for the trial court to take cognizance of the joint motion for reconsideration
renewal premium. Can the trial court grant the withdrawal? Remedial Law Q&As (2007- insofar as Balatong and Labong were concerned. The trial court should instead have
2013) hectorchristopher@yahoo.com JayArhSals “Never Let The Odds Keep You From ordered their arrest. (People v. De Grano, 5 June 2009, Peralta, J.). On the other hand, it
Pursuing What You Know In Your Heart You Were Meant To Do.”-Leroy Satchel Paige was correct for the trial court to take cognizance of the joint motion for reconsideration
insofar as Ludong was concerned since he and his lawyer were present during the a residential house in Las Piñas City. The lease contract provided, among others, for a
promulgation. (B) No, Balatong and Labong cannot appeal their conviction in case monthly rental of P25,000.00, plus ten percent (10%) interest rate in case of non-
Ludong accepts his conviction for homicide. Since Balatong and Labong failed to appear payment on its due date. Subsequently, Landlord migrated to the United States of
during the promulgation of the conviction without justifiable cause, they lost the America (USA) but granted in favor of his sister Maria, a special power of attorney to
remedies under the Rules of Court including the remedy of an appeal. II. McJolly is a manage the property and file and defend suits over the property rented out to Tenant.
trouble-maker of sorts, always getting into brushes with the law. In one incident, he Tenant failed to pay the rentals due for five (5) months. Maria asks your legal advice on
drove his Humvee recklessly, hitting a pedicab which sent its driver and passengers in how she can expeditiously collect from Tenant the unpaid rentals plus interests due. (6%)
different directions. The pedicab driver died, while two (2) of the passengers suffered (A) What judicial remedy would you recommend to Maria? (B) Where is the proper
slight physical injuries. Two (2) Informations were then filed against McJolly. One, for venue of the judicial remedy which you recommended? (C) If Maria insists on filing an
Reckless Imprudence Resulting in Homicide and Damage to Property, and two, for ejectment suit against Tenant, when do you reckon the one (1)-year period within which
Reckless Imprudence Resulting in Slight Physical Injuries. The latter case was scheduled to file the action? ANSWERS: (A) The judicial remedy that I would recommend to Maria is
for arraignment earlier, on which occasion McJolly immediately pleaded guilty. He was to file a collection suit for the P125,000 rentals in arrears and the P12,500 interest due.
meted out the penalty of public censure. A month later, the case for reckless imprudence The remedy would be expeditious since it would be governed by the Rules on Summary
resulting in homicide was also set for arraignment. Instead of pleading, Procedure as the amount of the demand, excluding interest, does not exceed P200,000.
McJollyinterposed the defense of double jeopardy. Resolve. (4%) ANSWER: The defense (B) The proper venue of the collection suit would be in Marikina City, where Tenant
of double jeopardy is meritorious and the second information for reckless imprudence resides. Under the Rules of Civil Procedure, venue in personal actions is with the
resulting in homicide should be quashed on the ground of double jeopardy. The Supreme residence of either the plaintiff or the defendant, at the plaintiff’s election. Since the
Court has held that reckless imprudence is a single crime and that its consequences on Plaintiff does not reside in the Philippines, venue may be laid only in Marikina City where
persons and property are material only to determine the penalty. Here there was only the defendant Tenant resides. (C) If Maria insists on filing an ejectment suit against
one act and crime of reckless imprudence. The death, the physical injuries, and the Tenant, the one-year period within which to file the action shall be reckoned from the
damage to the tricycle are only consequences of the same reckless act of McJolly. Hence expiration of 5- days from notice of the last demand to pay and vacate. (Cruz v. Atencio,
there was double jeopardy when a second information arising from the same reckless act 28 February 1959; Sy Oh v. Garcia, 30 June 1969). VI. As a rule, courts may not grant an
was brought against the accused. (Ivler v. Modesto-San Pedro, 17 November 2010). III. application for provisional remedy without complying with the requirements of notice
While passing by a dark uninhabited part of their barangay, PO2 Asintado observed and hearing. These requirements, however, may be dispensed with in an application for:
shadows and heard screams from a distance. PO2 Asintado hid himself behind the (1%) (A) writ of preliminary injunction (B) writ for preliminary attachment (C) an order
bushes and saw a man beating a woman whom he recognized as his neighbor, Kulasa. granting support pendente lite (D) a writ of replevin ANSWER: (B) VII. Co Batong, a
When Kulasa was already in agony, the man stabbed her and she fell on the ground. The Taipan, filed a civil action for damages with the Regional Trial Court (RTC) of Parañaque
man hurriedly left thereafter. PO2 Asintado immediately went to Kulasa’s rescue. Kulasa, City against Jose Penduko, a news reporter of the Philippine Times, a newspaper of
who was then in a state of hysteria, kept mentioning to PO2 Asintado “Si Rene, gusto general circulation printed and published in Parañaque City. The complaint alleged,
akong patayin! Sinaksak niya ako!” When PO2 Asintado was about to carry her, Kulasa among others, that Jose Penduko wrote malicious and defamatory imputations against
refused and said “Kaya ko. Mababaw lang to. Habulin mo si Rene.” The following day, Co Batong; that Co Batong’s business address is in Makati City; and that the libelous
Rene learned of Kulasa’s death and, bothered by his conscience, surrendered to the article was first printed and published in Parañaque City. The complaint prayed that Jose
authorities with his counsel. As his surrender was broadcasted all over media, Rene Penduko be held liable to pay P200,000.00, as moral damages; P150,000.00, as
opted to release his statement to the press which goes: “I believe that I am entitled to exemplary damages; and P50,000.00, as attorney’s fees. Jose Penduko filed a Motion to
the presumption of innocence until my guilt is proven beyond reasonable doubt. Dismiss on the following grounds: 1. The RTC is without jurisdiction because under the
Although I admit that I performed acts that may take one’s life away, I hope and pray Totality Rule, the claim for damages in the amount of P350,000.00 fall within the
that justice will be served the right way. God bless us all. (Sgd.) Rene” The trial court exclusive original jurisdiction of the Metropolitan Trial Court (MeTC) of Parañaque City.
convicted Rene of homicide on the basis of PO2 Asintado’s testimony, Kulasa’s 2. The venue is improperly laid because what the complaint alleged is Co Batong’s
statements, and Rene’s statement to the press. On appeal, Rene raises the following business address and not his residence address. Are the grounds invoked in the Motion
errors: 1. The trial court erred in giving weight to PO2 Asintado’s testimony, as the latter to Dismiss proper? (4%) ANSWER: No, the grounds invoked in the motion to dismiss
did not have any personal knowledge of the facts in issue, and violated Rene’s right to improper. 1. The invocation of the Totality Rule is misplaced. Under Art. 360 of the
due process when it considered Kulasa’s statements despite lack of opportunity for her Revised Penal Code, jurisdiction over a civil action for damages in case of libel is with the
cross-examination. 2. The trial court erred in holding that Rene’s statement to the press Court of First Instance, now the Regional Trial Court. (Nocum v. Tan, 23 September
was a confession which, standing alone, would be sufficient to warrant conviction. 2005). The said provision does not mention any jurisdictional amount over such action;
Resolve. (4%) ANSWER: Rene’s appeal is denied for lack of merit. 1. The contention that hence the Totality Rule is inapplicable. 2. The ground that the complaint mentioned the
the trial court erred in giving weight to PO2 Asintado’s testimony since he did not have complainant’s office address rather than his residence is of no moment since the
personal knowledge of the facts in issue is without merit. The contention in effect complaint also stated that the libelous article was printed and first published in
challenges Kulasa’s statement for being hearsay. Under the Rules of Evidence, a Paranaque City. Under Article 360 of the Revised Penal Code, venue in a civil action for
statement made immediately subsequent to a startling occurrence is excepted from the libel also lies in the place where the libelous article was printed and first published. VIII.
hearsay rule as part of the res gestae. Here Kulasa’s statement was made immediately Johnny, a naturalized citizen of the United States of America (USA) but formerly a Filipino
subsequent to a starling occurrence, that is, her stabbing by Rene, and was made in a citizen, executed a notarial will in accordance with the laws of the State of California,
state of hysteria, showing that she was under the influence of the startling occurrence. USA. Johnny, at the time of his death, was survived by his niece Anastacia, an American
Hence testimony regarding the statement is excepted from the hearsay rule. Since citizen residing at the condominium unit of Johnny located at Fort Bonifacio, Taguig City;
Kulasa’s statement is an exception to the hearsay rule, Rene cannot complain that his a younger brother, Bartolome, who manages Johnny’s fish pond in Lingayen, Pangasinan;
right to due process was violated when the trial court considered Kulasa’s statement and a younger sister, Christina, who manages Johnny’s rental condominium units in
despite lack of opportunity to cross-examine her. There should be no serious question Makati City. Johnny’s entire estate which he inherited from his parents is valued at P200
about the admissibility against an accused of hearsay where this hearsay falls under an million. Johnny appointed Anastacia as executrix of his will. (4%) (A) Can Johnny’s
exception to the hearsay rule, especially here where the declarant is dead and thus notarial will be probated before the proper court in the Philippines? (B) Is Anastacia
unavailable to testify. (ANTONIO R. BAUTISTA, BASIC EVIDENCE 214-215 [2004 ed.]). In qualified to be the executrix of Johnny’s notarial will? ANSWERS: (A) Yes, the formal
U.S. v. Gil, 13 Phil. 530 (1909), the Supreme Court upheld dying declarations as an validity of a will is governed also by the national law of the decedent. (Article 817, Civil
exception to the confrontation clause since “such declarations have always been Code). A will proved and allowed in a foreign country, according to the laws of such
regarded as an exception to the general rule regarding hearsay evidence.” 2. The country, may be allowed, filed, and recorded by the proper Regional Trial Court in the
argument that the trial court erred in holding that Rene’s statement to the press was a Philippines. (S1 R77). (B) Yes, assuming that Anastacia is of legal age, she is qualified to
confession which, standing alone, would be sufficient to warrant conviction is be an executor although an alien because she is a resident of the Philippines. (S1 R78). IX.
meritorious. Firstly, Rene’s statement is not a confession but an admission. A confession Bayani, an overseas worker based in Dubai, issued in favor of Agente, a special power of
is one wherein a person acknowledges his guilt of a crime, which Rene did not do. attorney to sell his house and lot. Agente was able to sell the property but failed to remit
Secondly, even assuming it is a confession, standing alone it would not be sufficient to the proceeds to Bayani, as agreed upon. On his return to the Philippines, Bayani, by way
warrant conviction since it is an extrajudicial confession which is not sufficient ground for of a demand letter duly received by Agente, sought to recover the amount due him.
conviction unless corroborated by evidence of corpus delicti. (S3 R133). Nonetheless this Agente failed to return the amount as he had used it for the construction of his own
was a harmless error since the admission of Rene was corroborated by the testimony of house. Thus, Bayani filed an action against Agente for sum of money with damages.
PO2 Asintado on Kulasa’s statement. IV. An order of the court requiring a retroactive re- Bayani subsequently filed an ex-parte motion for the issuance of a writ of preliminary
dating of an order, judgment or document filing be entered or recorded in a judgment is: attachment duly supported by an affidavit. The court granted the ex-parte motion and
(1%) (A) pro hac vice (B) non pro tunc (C) confession relicta verificatione (D) nolle issued a writ of preliminary attachment upon Bayani’s posting of the required bond.
prosequi ANSWER: (B) (Note: Should be “nunc pro tunc.”). V. Landlord, a resident of Bayani prayed that the court’s sheriff be deputized to serve and implement the writ of
Quezon City, entered into a lease contract with Tenant, a resident of Marikina City, over attachment. On November 19, 2013, the Sheriff served upon Agente the writ of
attachment and levied on the latter’s house and lot. On November 20, 2013, the Sheriff erroneous and invalid as she never contracted marriage with anybody, much less John
served on Agente summons and a copy of the complaint. On November 22, 2013, Agente Starr. There is no need to file a petition for declaration of nullity of marriage since there
filed an Answer with Motion to Discharge the Writ of Attachment alleging that at the was no marriage to speak of in the first place, the marriage contract being a sham
time the writ of preliminary attachment was issued, he has not been served with contract. (Republic v. Olaybar, 10 February 2014, Peralta, J.). XIII. A foreign dog trained to
summons and, therefore, it was improperly issued. (4%) (A) Is Agente correct? (B) Was sniff dangerous drugs from packages, was hired by FDP Corporation, a door to door
the writ of preliminary attachment properly executed? ANSWERS: (A) No, Agente is not forwarder company, to sniff packages in their depot at the international airport. In one
correct. Under the Rules of Civil Procedure, a writ of attachment may issue even before of the routinary inspections of packages waiting to be sent to the United States of
service of summons upon the defendant. (S2 R57). (B) No, the writ of preliminary America (USA), the dog sat beside one of the packages, a signal that the package
attachment not properly executed. Under S5 R57, no levy on preliminary attachment contained dangerous drugs. Thereafter, the guards opened the package and found two
shall be enforced unless there is prior or simultaneous service of the summons and the (2) kilograms of cocaine. The owner of the package was arrested and charges were filed
accompanying papers. (S5 R The Supreme Court has held that subsequent service of against him. During the trial, the prosecution, through the trainer who was present
summons will not cure the irregularity that attended the enforcement of the writ (Onate during the incident and an expert in this kind of field, testified that the dog was highly
v. Abrogar, 23 February 1995). Here the sheriff levied upon the house and lot prior to the trained to sniff packages to determine if the contents were dangerous drugs and the
service of the summons and the complaint upon Agente. Hence the writ of preliminary sniffing technique of these highly trained dogs was accepted worldwide and had been
attachment was not properly executed. The subsequent service of summons and the successful in dangerous drugs operations. The prosecution moved to admit this evidence
complaint did not cure the irregularity in the enforcement of the writ. X. Prince Chong to justify the opening of the package. The accused objected on the grounds that: (i) the
entered into a lease contract with King Kong over a commercial building where the guards had no personal knowledge of the contents of the package before it was opened;
former conducted his hardware business. The lease contract stipulated, among others, a (ii) the testimony of the trainer of the dog is hearsay; and (iii) the accused could not
monthly rental of P50,000.00 for a four (4)-year period commencing on January 1, 2010. cross-examine the dog. Decide. (4%) ANSWER: The accused’s objections are overruled.
On January 1, 2013, Prince Chong died. Kin Il Chong was appointed administrator of the The objection that the guards had no personal knowledge of the contents of the package
estate of Prince Chong, but the former failed to pay the rentals for the months of January before it was opened is misplaced. The one testifying is the trainer not the guards and he
to June 2013 despite King Kong’s written demands. Thus, on July 1, 2013, King Kong filed had personal knowledge of the circumstances since he was present during the incident.
with the Regional Trial Court (RTC) an action for rescission of contract with damages and Besides there is no rule of evidence that one cannot testify about the contents of a
payment of accrued rentals as of June 30, 2013. (4%) (A) Can Kin Il Chong move to package if he did not have prior personal knowledge of its contents before opening it.
dismiss the complaint on the ground that the RTC is without jurisdiction since the The objection that the testimony of the trainer of the dog is hearsay is not valid. Hearsay
amount claimed is only P300,000.00? (B) If the rentals accrued during the lifetime of is an out-of-court declaration made by a person which is offered for the truth of the
Prince Chong, and King Kong also filed the complaint for sum of money during that time, matter asserted. Here what is involved is a dog who is not a person who can make an
will the action be dismissible upon Prince Chong’s death during the pendency of the outof-court declaration. (Lempert & Saltzburg, A MODERN APPROACH TO EVIDENCE 370-
case? ANSWERS: (A) No, Kin II Chong cannot move to dismiss the complaint on the 371 [1982]). A dog is not treated as a declarant or witness who can be cross-examined.
ground that the RTC is without jurisdiction since the amount claimed is only P300,000. (People v. Centolella, 305 N.Y.S.2d 279). Hence testimony that the dog sat beside the
Under B.P. Blg. 129, the RTC has original and exclusive jurisdiction over actions incapable package is not testimony about an out-of-court declaration and thus not hearsay. The
of pecuniary estimation. Here the action is for rescission which is incapable of pecuniary objection that the accused could not cross-examine the dog is without merit. Under the
estimation. The P300,000 accrued rentals is only incidental to the main purpose of the Constitution, the accused’s right of confrontation refers to witnesses. As previously
action which is to rescind the lease contract. (B) No, the action will not be dismissible discussed, a dog is not a witness who can be cross-examined. Note: It is urged that
upon Prince Chong’s death during the pendency of the case. Under S20 R3, when the utmost liberality be exercised in grading this number. The answer is not found in
action is on a contractual money claim and the defendant dies before entry of final Philippine law and jurisprudence and even in commentaries by writers on evidence. XIV.
judgment, the action shall not be dismissed but shall instead be allowed to continue until When a Municipal Trial Court (MTC), pursuant to its delegated jurisdiction, renders an
entry of final judgment. Here the action is on a contractual money claim, that is, a claim adverse judgment in an application for land registration, the aggrieved party’s remedy is:
for rentals based on a lease contract. Hence it shall be allowed to continue until final (1%) (A) ordinary appeal to the Regional Trial Court (B) petition for review on certiorari to
judgment. (S20 R3, S5 R86). XI. A search warrant was issued for the purpose of looking the Supreme Court (C) ordinary appeal to the Court of Appeals (D) petition for review to
for unlicensed firearms in the house of Ass-asin, a notorious gun for hire. When the the Court of Appeals ANSWER: (C) (See Sec. 34, B.P. Blg. 129) XV. The Ombudsman, after
police served the warrant, they also sought the assistance of barangay tanods who were conducting the requisite preliminary investigation, found probable cause to charge Gov.
assigned to look at other portions of the premises around the house. In a nipa hut thirty Matigas in conspiracy with Carpintero, a private individual, for violating Section 3(e) of
(30) meters away from the house of Ass-asin, a barangay tanod came upon a kilo of Republic Act (RA) No. 3019 (Anti-Graft and Corrupt Practices Act, as amended). Before
marijuana that was wrapped in newsprint. He took it and this was later used by the the information could be filed with the Sandiganbayan, Gov. Matigas was killed in an
authorities to charge Ass-asin with illegal possession of marijuana. Ass-asin objected to ambush. This, notwithstanding, an information was filed against Gov. Matigas and
the introduction of such evidence claiming that it was illegally seized. Is the objection of Carpintero. At the Sandiganbayan, Carpintero through counsel, filed a Motion to Quash
Assasin valid? (4%) ANSWER: Yes, the objection of Ass-asin is valid. Under the the Information, on the ground of lack of jurisdiction of the Sandiganbayan, arguing that
Constitution, the right of the people against unlawful search is inviolable except in cases with the death of Gov. Matigas, there is no public officer charged in the information. Is
where a valid search warrant was issued or in exceptional cases where the law provides the motion to quash legally tenable? (4%) ANSWER: No, the motion to quash is not
for a warrantless search. (Sec. 2, Art. III, Constitution). Under the fruit of the poisonous legally tenable. In a case involving similar facts, the Supreme Court held that the death of
tree doctrine, items seized by virtue of an unlawful search are inadmissible in evidence. the public officer did not mean that the allegation of conspiracy between the public
(Sec. 3[2], Art. III, Constitution). Here the the seizure of the marijuana was illegal since it officer and the private person can no longer be proved or that their alleged conspiracy is
was not pursuant to a search warrant. The search warrant was for the search and seizure already expunged. The only thing extinguished by the death of the public officer was his
of unlicensed firearms not marijuana. Nor would the exception regarding items seized criminal liability. His death did not extinguish the crime nor did it remove the basis of the
under plain view apply. The marijuana was wrapped in newsprint and clearly not in plain charge of conspiracy between him and the private person. Hence the Sandiganbayan had
sight. Hence the marijuana may not be introduced in evidence over Ass-asin’s objection. jurisdiction over the offense charged. (People v. Go, 25 March 2014, Peralta, J.) XVI.
XII. Mary Jane met Shiela May at the recruitment agency where they both applied for Plaintif filed a complaint denominated as accion publiciana, against defendant. In his
overseas employment. They exchanged pleasantries, including details of their personal answer, defendant alleged that he had no interest over the land in question, except as
circumstances. Fortunately, Mary Jane was deployed to work as front desk receptionist lessee of Z. Plaintif subsequently filed an affidavit of Z, the lessor of defendant, stating
at a hotel in Abu Dhabi where she met Sultan Ahmedwho proposed marriage, to which that Z had sold to plaintif all his rights and interests in the property as shown by a deed
she readily accepted. Unfortunately for Shiela May, she was not deployed to work of transfer attached to the affidavit. Thus, plaintif may ask the court to render: (1%) (A)
abroad, and this made her envious of Mary Jane. Mary Jane returned to the Philippines summary judgment (B) judgment on the pleadings (C) partial judgment (D) judgment by
to prepare for her wedding. She secured from the National Statistics Office (NSO) a default ANSWER: (A) (S1 & 3, R35) XVII. A was charged before the Sandiganbayan with a
Certificate of No Marriage. It turned out from the NSO records that Mary Jane had crime of plunder, a non-bailable offense, where the court had already issued a warrant
previously contracted marriage with John Starr, a British citizen, which she never did. The for his arrest. Without A being arrested, his lawyer filed a Motion to Quash Arrest
purported marriage between Mary Jane and John Starr contained all the required Warrant and to Fix Bail, arguing that the allegations in the information did not charge the
pertinent details on Mary Jane. Mary Jane later on learned that Shiela May is the best crime of plunder but a crime of malversation, a bailable offense. The court denied the
friend of John Starr. As a lawyer, Mary Jane seeks your advice on her predicament. What motion on the ground that it had not yet acquired jurisdiction over the person of the
legal remedy will you avail to enable Mary Jane to contract marriage with Sultan Ahmed? accused and that the accused should be under the custody of the court since the crime
(4%) ANSWER: The legal remedy I would avail to enable Mary Jane to contract marriage charged was nonbailable. The accused’s lawyer counter-argued that the court can rule
with Sultan Ahmed is to file a petition under Rule 108 to cancel entries in the marriage on the motion even if the accused was at-large because it had jurisdiction over the
contract between John Starr and Mary Jane, particularly the portion and entries thereon subject matter of the case. According to said lawyer, there was no need for the accused
relating to the wife. Rule 108 may be availed of to cancel erroneous or invalid entries in to be under the custody of the court because what was filed was a Motion to Quash
the Civil Registry. Here the entry of Mary Jane as the wife of John Starr is clearly Arrest and to Fix Bail, not a Petition for Bail. (A) If you are the Sandiganbayan, how will
you rule on the motion? (3%) (B) If the Sandiganbayan denies the motion, what judicial court did not acquire jurisdiction over the person of Debi since there was no valid
remedy should the accused undertake? (2%) ANSWERS: (A) If I were the Sandiganbayan, I substituted service of summons. Substituted service of summons should have been
would deny the Motion to Quash Arrest Warrant and to Fix Bail. The motion to quash made at Debi’s residence. (S7 R14). Hence the judgment of the RTC was void. Since the
warrant of arrest may be considered since only jurisdiction over the person not custody judgment is void, the petition for annulment thereof is imprescriptible. (S3 R47).
of the law is required. Jurisdiction over the person of A was obtained by his voluntary Furthermore, default judgments are not allowed in declaration of nullity of marriage.
appearance made through the filing of the motion seeking affirmative relief. (See (S3[e] R9). Hence the trial court’s rendition of a default judgment was made with grave
Miranda v. Tuliao, 31 March 2006). Nonetheless I would still deny the motion to quash abuse of discretion amounting to lack of jurisdiction. XXI. Goodfeather Corporation,
arrest warrant. The ground that the offense charged is malversation not plunder is not a through its President, Al Pakino, filed with the Regional Trial Court (RTC) a complaint for
valid ground to quash the arrest warrant. A should simply file an application for bail and specific performance against Robert White. Instead of filing an answer to the complaint,
contend that he is entitled thereto as a matter of right. The motion to fix amount of bail, Robert White filed a motion to dismiss the complaint on the ground of lack of the
which is in effect an application for bail cannot be granted unless the accused is in appropriate board resolution from the Board of Directors of Goodfeather Corporation to
custody of the law. (Miranda v. Tuliao, 31 March 2006). Here A was not in custody of the show the authority of Al Pakino to represent the corporation and file the complaint in its
law but still at large. Hence the motion to fix the amount of bail should be denied. (B) If behalf. The RTC granted the motion to dismiss and, accordingly, it ordered the dismissal
the Sandiganbayan denies the motion, the judicial remedy that the accused should of the complaint. Al Pakino filed a motion for reconsideration which the RTC denied. As
undertake is to file a petition for certiorari under Rule 65 with the Supreme Court. nothing more could be done by Al Pakino before the RTC, he filed an appeal before the
Certiorari is available to challenge interlocutory orders rendered with grave abuse of Court of Appeals (CA). Robert White moved for dismissal of the appeal on the ground
discretion since appeal is unavailable. Here the order denying the Motion to Quash that the same involved purely a question of law and should have been filed with the
Arrest Warrant and to Fix Bail is interlocutory since it does not completely dispose of the Supreme Court (SC). However, Al Pakino claimed that the appeal involved mixed
case. Hence certiorari is available. A should aver that the Sandiganbayan acted with questions of fact and law because there must be a factual determination if, indeed, Al
grave abuse of discretion amounting to lack of or excess of jurisdiction in denying his Pakino was duly authorized by Goodfeather Corporation to file the complaint. Whose
motion. XVIII. A was charged with murder in the lower court. His Petition for Bail was position is correct? Explain. (4%) ANSWER: Robert White’s position is correct. In a case
denied after a summary hearing on the ground that the prosecution had established a involving similar facts, the Supreme Court held that the issue of whether or not the trial
strong evidence of guilt. No Motion for Reconsideration was filed from the denial of the court erred in dismissing the complaint on the ground that the person who filed the
Petition for Bail. During the reception of the evidence of the accused, the accused complaint in behalf of the plaintiff corporation was not authorized to do so is a legal
reiterated his petition for bail on the ground that the witnesses so far presented by the issue, reviewable only by the Supreme Court in a petition for review on certiorari under
accused had shown that no qualifying aggravating circumstance attended the killing. The Rule 45. (Tamondong v. Court of Appeals, 26 November 2004). (Note: An alternative
court denied the petition on the grounds that it had already ruled that: (i) the evidence answer would be that the appeal raises a factual question of whether or not Al Pakino
of guilt is strong; (ii) the resolution for the Petition for Bail is solely based on the was indeed authorized to file the complaint in behalf of Goodfeather Corporation. A
evidence presented by the prosecution; and (iii) no Motion for Reconsideration was filed reading of Tamondong would show that the appellant only raised a legal question of
from the denial of the Petition for Bail. (6%) (A) If you are the Judge, how will you resolve whether it was proper to dismiss the complaint for failure to state a cause of action but
the incident? (B) Suppose the accused is convicted of the crime of homicide and the did not raise a factual issue as to whether the filer was in fact authorized by the
accused filed a Notice of Appeal, is he entitled to bail? ANSWERS: (A) If I were the judge, I corporation.). XXII. Which of the following decisions may be appealed directly to the
will grant the Petition for Bail if the evidence does not show any qualifying aggravating Supreme Court (SC)? (Assume that the issues to be raised on appeal involve purely
circumstance. In such a case the offense would be only homicide which is bailable. (i) The questions of law) (1%) (A) Decision of the Regional Trial Court (RTC) rendered in the
ground that the court had already ruled that the evidence of guilt is strong is improper. exercise of its appellate jurisdiction. (B) Decision of the RTC rendered in the exercise of
An order denying an application for bail is interlocutory and remains at the control of the its original jurisdiction. (C) Decision of the Civil Service Commission. (D) Decision of the
court until final judgment. Hence the court is not bound by its earlier ruling and may Office of the President. ANSWER: (B) Note: In an appeal from RTC judgment in the
reconsider the same if the evidence or law warrants the same. (ii) The ground that the exercise of its appellate jurisdiction, the appeal should be to the CA even if the questions
resolution for the Petition for Bail is solely based on the evidence presented by the are only legal. Hence A should be excluded. (S2[c] R42). XXIII. Mr. Humpty filed with the
prosecution is improper. While S8 R114 provides that the prosecution has the burden of Regional Trial Court (RTC) a complaint against Ms. Dumpty for damages. The RTC, after
proof to show that the evidence of guilt is strong, it should not be taken to mean that the due proceedings, rendered a decision granting the complaint and ordering Ms. Dumpty
resolution of the bail application is based solely on the prosecution evidence. At the to pay damages to Mr. Humpty. Ms. Dumpty timely filed an appeal before the Court of
hearing for the bail application, both the prosecution and the accused must be given Appeals (CA), questioning the RTC decision. Meanwhile, the RTC granted Mr. Humpty’s
reasonable opportunity to prove or to disprove, respectively, that the evidence of guilt is motion for execution pending appeal. Upon receipt of the RTC’s order granting execution
strong. (Santos v. Ofilada, 245 SCRA 56). (iii) The ground that no motion for pending appeal, Ms. Dumpty filed with the CA another case, this time a special civil
reconsideration was filed from the order denying the petition for bail is improper. As action for certiorari assailing said RTC order. Is there a violation of the rule against forum
previously discussed, an order denying bail is merely interlocutory. Hence the failure to shopping considering that two (2) actions emanating from the same case with the RTC
move for reconsideration thereof during the trial will not render the order final and were filed by Ms. Dumpty with the CA? Explain. (4%) ANSWER: No, there is no violation
conclusive. (B) No, after conviction by the RTC of an offense not punishable by death, of the rule against forum shopping. Forum shopping applies where two or more initiatory
reclusion perpetua, or life imprisonment, admission to bail is discretionary. (S5 R114). pleadings were filed by the same party. This is discernible from the use of the phrase
XIX. A vicarious admission is considered an exception to the hearsay rule. It, however, “commenced any action or filed any claim” in S5 R7. Here the first case involves the filing
does not cover: (1%) (A) admission by a conspirator (B) admission by a privy (C) judicial by Ms. Dumpty of a notice of appeal which is not an initiatory pleading. Hence there is no
admission (D) adoptive admission (C) Note: a vicarious admission is an extrajudicial forum shopping. XXIV. Solomon and Faith got married in 2005. In 2010, Solomon
admission. Hence C is not covered by the rule regarding vicarious admissions. XX. Tom contracted a second marriage with Hope. When Faith found out about the second
Wallis filed with the Regional Trial Court (RTC) a Petition for Declaration of Nullity of his marriage of Solomon and Hope, she filed a criminal case for bigamy before the Regional
marriage with Debi Wallis on the ground of psychological incapacity of the latter. Before Trial Court (RTC) of Manila sometime in 2011. Meanwhile, Solomon filed a petition for
filing the petition, Tom Wallis had told Debi Wallis that he wanted the annulment of their declaration of nullity of his first marriage with Faith in 2012, while the case for bigamy
marriage because he was already fed up with her irrational and eccentric behaviour. before the RTC of Manila is ongoing. Subsequently, Solomon filed a motion to suspend
However, in the petition for declaration of nullity of marriage, the correct residential the proceedings in the bigamy case on the ground of prejudicial question. He asserts that
address of Debi Wallis was deliberately not alleged and instead, the residential address the proceedings in the criminal case should be suspended because if his first marriage
of their married son was stated. Summons was served by substituted service at the with Faith will be declared null and void, it will have the effect of exculpating him from
address stated in the petition. For failure to file an answer, Debi Wallis was declared in the crime of bigamy. Decide. (4%) ANSWER: Motion to suspend proceedings denied.
default and Tom Wallis presented evidence ex-parte. The RTC rendered judgment Under the Rules of Criminal Procedure, a prejudicial question arises if there has been a
declaring the marriage null and void on the ground of psychological incapacity of Debi previously filed civil action. Here the civil action was filed after the criminal action. Hence
Wallis. Three (3) years after the RTC judgment was rendered, Debi Wallis got hold of a no prejudicial question will arise. Moreover the Supreme Court has held that a pending
copy thereof and wanted to have the RTC judgment reversed and set aside. If you are the case for declaration of nullity of marriage does not raise a prejudicial question to a
lawyer of Debi Wallis, what judicial remedy or remedies will you take? Discuss and charge of bigamy since a person who contracts a second marriage without first awaiting
specify the ground or grounds for said remedy or remedies. (5%) ANSWER: If I were the a judicial declaration of nullity of his first marriage has already committed bigamy.
lawyer of Debi Wallis, the judicial remedy I would take is to file with the Court of Appeals (People v. Odtuhan, 17 July 2013, Peralta, J.). XXV. Mr. Boaz filed an action for ejectment
an action for annulment of the RTC judgment under Rule 47. An action for annulment of against Mr. Jachin before the Metropolitan Trial Court (MeTC). Mr. Jachin actively
judgment may be resorted to since the remedies of appeal and petition for relief are no participated in every stage of the proceedings knowing fully well that the MeTC had no
longer available through no fault of Debi Wallis. (S1 R47). The ground for annulment of jurisdiction over the action. In his mind, Mr. Jachin was thinking that if the MeTC
judgment would be lack of jurisdiction. Lack of jurisdiction also covers lack of jurisdiction rendered judgment against him, he could always raise the issue on the jurisdiction of the
over the person of the defendant since the judgment would be void. (1 FLORENZ D. MeTC. After trial, the MeTC rendered judgment against Mr. Jachin. What is the remedy
REGALADO, REMEDIAL LAW COMPENDIUM 558 [7th rev. ed., 3rd printing]). Here the of Mr. Jachin? (1%) (A) File an appeal (B) File an action for nullification of judgment (C)
File a motion for reconsideration (D) File a petition for certiorari under Rule 65 ANSWER: RTC was not correct in ruling that the case was within its original jurisdiction and that
(A) See S8 R40. R47 is not available since appeal is still available. Not C since a prohibited hence it may conduct a full-blown trial of the appealed case as if it were originally filed
pleading. XXVI. Parole evidence is an: (1%) (A) agreement not included in the document with it. Under S8 R40, if an appeal is taken from an MTC order dismissing a case for lack
(B) oral agreement not included in the document (C) agreement included in the of jurisdiction without a trial on the merits, the RTC on appeal may affirm the dismissal
document (D) oral agreement included in the document ANSWER: (A) Note: It is order and if it has jurisdiction thereover, try the case on the merits as if the case was
suggested that either A or B be considered as correct. Strictly speaking parol evidence originally filed with it. Here the RTC did not have jurisdiction over the case since it is an
does not have to be an agreement; it is simply any evidence, whether written or oral, ejectment suit cognizable exclusively by the MTC. The assessed value of the land is
which is not contained in a written agreement subject of a case and which seeks to irrelevant for the purpose of determining jurisdiction in ejectment suits and would not
modify, alter, or explain the terms of the written agreement. XXVII. Mr. Avenger filed oust the MTC of jurisdiction in the same manner as allegations of ownership would not
with the Regional Trial Court (RTC) a complaint against Ms. Bright for annulment of deed oust the MTC of jurisdiction. The RTC should have reversed the dismissal order and
of sale and other documents. Ms. Bright filed a motion to dismiss the complaint on the remanded the case to the MTC for further proceedings. (S8 R40). Note: Utmost liberality
ground of lack of cause of action. Mr. Avenger filed an opposition to the motion to should be given to the examinee on this question as it does not appear to be within the
dismiss. State and discuss the appropriate remedy/remedies under each of the following coverage of the remedial law examination per the bar examination syllabus given by the
situations: (6%) (A) If the RTC grants Ms. Bright’s motion to dismiss and dismisses the Supreme Court. SUGGESTED ANSWERS TO THE 2015 REMEDIAL LAW BAR EXAMINATION
complaint on the ground of lack of cause of action, what will be the remedy/remedies of By Prof. Manuel R. Riguera I. Lender extended to Borrower a P100,000.00 loan covered
Mr. Avenger? (B) If the RTC denies Ms. Bright’s motion to dismiss, what will be her by a promissory note. Later, Borrower obtained another P100,000.00 loan again covered
remedy/remedies? (C) If the RTC denies Ms. Bright’s motion to dismiss and, further by a promissory note. Still later, Borrower obtained a P300,000.00 loan secured by a real
proceedings, including trial on the merits, are conducted until the RTC renders a decision estate mortgage on his land valued at P500,000.00. Borrower defaulted on his payments
in favor of Mr. Avenger, what will be the remedy/remedies of Ms. Bright? ANSWERS: (A) when the loans matured. Despite demand to pay the P500,000.00 loan, Borrower
If the RTC grants Ms. Brights’s motion to dismiss, the remedies of Mr. Avenger are: (a) refused to pay. Lender, applying the totality rule, filed against Borrower with the
File a motion for reconsideration under Rule 37. (b) Re-file the complaint. The dismissal Regional Trial Court (RTC) of Manila, a collection suit for P500,000.00. a.) Did Lender
does not bar the re-filing of the case (S5 R16). (c) Appeal from the order of dismissal. The correctly apply the totality rule and the rule on joinder of causes of action? (2%) At the
dismissal order is a final order as it completely disposes of the case; hence it is trial, Borrower's lawyer, while cross-examining Lender, successfully elicited an admission
appealable. (d) File an amended complaint as a matter of right curing the defect of lack from the latter that the two promissory notes have been paid. Thereafter, Borrower's
of cause of action before the dismissal order becomes final. This is because a motion to lawyer filed a motion to dismiss the case on the ground that as proven only P300,000.00
dismiss is not a responsive pleading; hence Mr. Avenger can amend the complaint as a was the amount due to Lender and which claim is within the exclusive original
matter of right. (S2 R10). (B) If the RTC denies Ms. Bright’s motion to dismiss, her jurisdiction of the Metropolitan Trial Court. He further argued that lack of jurisdiction
remedies are: (a) File a motion for reconsideration. (b) Proceed to trial and if she loses, over the subject matter can be raised at any stage of the proceedings. b.) Should the
appeal and assign the failure to dismiss as a reversible error. (c) File a special civil action court dismiss the case? (3%) ANSWERS: a) Yes Lender correctly applied the totality rule
for certiorari and/or mandamus if the denial of the order to dismiss is made with grave and the rule on joinder of causes of action. Under the rule on joinder of causes of action,
abuse of discretion amounting to lack of or excess of jurisdiction. (C) If the RTC renders a a party may in one pleading assert as many causes of action as he may have against an
decision in favor of Mr. Avenger, Ms. Bright’s remedies are: (a) File a motion for opposing party. Under the totality rule, where the claims in all the causes of action are
reconsideration or new trial under Rule 37. (b) File an appeal to the Court of Appeals principally for recovery of money, the aggregate amount claimed shall be the test of
under Rule 41. (c) File an appeal to the Supreme Court under Rule 45 if the appeal will jurisdiction. Here the causes of action by Lender are all against borrower and all the
raise only questions of law. (d) File a petition for relief from judgment under Rule 38. (e) claims are principally for recovery of money. Hence the aggregate amount claimed,
File an action for annulment of judgment under Rule 47 on the ground of extrinsic fraud which is P500,000 shall be the test of jurisdiction and thus it is the RTC of Manila which
or lack of jurisdiction. XXVIII. A was adopted by B and C when A was only a toddler. Later has jurisdiction. Although the rules on joinder of causes of action state that the joinder
on in life, A filed with the Regional Trial Court (RTC) a petition for change of name under shall not include special civil actions, the remedy resorted to with respect to the third
Rule 103 of the Rules of Court, as he wanted to reassume the surname of his natural loan was not foreclosure but collection. Hence joinder of causes of action would still be
parents because the surname of his adoptive parents sounded offensive and was proper. b) No, the court should not dismiss the case. The Supreme Court has held that
seriously affecting his business and social life. The adoptive parents gave their consent to subject-matter jurisdiction is determined by the amount of the claim alleged in the
the petition for change of name. May A file a petition for change of name? If the RTC complaint and not the amount substantiated during the trial. (Dionisio v Sioson Puerto,
grants the petition for change of name, what, if any, will be the effect on the respective 31 October 1974). Here the amount claimed was P500,000. Even if the claim
relations of A with his adoptive parents and with his natural parents? Discuss. (4%) substantiated during the trial was only P300,000 that is not determinative of subject-
ANSWER: Yes, A may file a petition for change of name. Changing name on the ground matter jurisdiction. Hence the argument that lack of subject-matter jurisdiction can be
that it is offensive and seriously affects the petitioner’s business and social life is a valid raised at any time is misplaced since in the first place the RTC has jurisdiction. II. Circe
ground especially where the adoptive parents had given their consent. The grant of the filed with the RTC a complaint for the foreclosure of real estate mortgage against siblings
petition will not change A’s relations with his adoptive and natural parents. The Supreme Scylla and Charybdis, co-owners of the property and cosignatories to the mortgage deed.
Court has held that change of name under Rule 103 affects only the name and not the The siblings permanently reside in Athens, Greece. Circe tipped off Sheriff Pluto that
status of the petitioner. (Republic v. CA, 21 May 1992). XXIX. Estrella was the registered Scylla is on a balikbayan trip and is billeted at the Century Plaza Hotel in Pasay City.
owner of a huge parcel of land located in a remote part of their barrio in Benguet. Sheriff Pluto went to the hotel and personally served Scylla the summons, but the latter
However, when she visited the property after she took a long vacation abroad, she was refused to receive summons for Charybdis as she was not authorized to do so. Sheriff
surprised to see that her childhood friend, John, had established a vacation house on her Pluto requested Scylla for the email address and fax number of Charybdis which the
property. Both Estrella and John were residents of the same barangay. To recover latter readily gave. Sheriff Pluto, in his return of the summons, stated that "Summons for
possession, Estrella filed a complaint for ejectment with the Municipal Trial Court (MTC), Scylla was served personally as shown by her signature on the receiving copy of the
alleging that she is the true owner of the land as evidenced by her certificate of title and summons. Summons on Charybdis was served pursuant to the amendment of Rule 14 by
tax declaration which showed the assessed value of the property as P21,000.00. On the facsimile transmittal of the summons and complaint on defendant's fax number as
other hand, John refuted Estrella’s claim of ownership and submitted in evidence a Deed evidenced by transmission verification report automatically generated by the fax
of Absolute Sale between him and Estrella. After the filing of John’s answer, the MTC machine indicating that it was received by the fax number to which it was sent on the
observed that the real issue was one of ownership and not of possession. Hence, the date and time indicated therein." Circe, sixty (60) days after her receipt of Sheriff Pluto's
MTC dismissed the complaint for lack of jurisdiction. On appeal by Estrella to the return, filed a Motion to Declare Charybdis in default as Charybdis did not file any
Regional Trial Court (RTC), a full-blown trial was conducted as if the case was originally responsive pleading. a.) Should the court declare Charybdis in default? (2%) Scylla
filed with it. The RTC reasoned that based on the assessed value of the property, it was seasonably filed her answer setting forth therein as a defense that Charybdis had paid
the court of proper jurisdiction. Eventually, the RTC rendered a judgment declaring John the mortgage debt. b.) On the premise that Charybdis was properly declared in default,
as the owner of the land and, hence, entitled to the possession thereof. (4%) (A) Was the what is the effect of Scylla's answer to the complaint? (2%) ANSWERS: a) No, the court
MTC correct in dismissing the complaint for lack of jurisdiction? Why or why not? (B) should not declare Charybdis in default. Under the Rules of Court, the amendment of
Was the RTC correct in ruling that based on the assessed value of the property, the case Rule 14 allowing service of summons by facsimile transmittal refers only to service of
was within its original jurisdiction and, hence, it may conduct a full-blown trial of the summons upon a foreign private juridical entity under Section 12 of Rule 14, not to a
appealed case as if it was originally filed with it? Why or why not? ANSWERS: (A) No, the non-resident defendant under Section 15 of Rule 14. Service of summons by facsimile
MTC was not correct in dismissing the case for lack of jurisdiction. The Supreme Court cannot be effected under Section 15 unless leave of court was obtained specifically
has held that an allegation of ownership as a defense in the answer will not oust the MTC permitting service by facsimile transmittal. Here the defendant is not a foreign private
of jurisdiction in an ejectment case. (Subano v. Vallecer, 24 March 1959). What juridical entity but a non-resident defendant and no leave of court was obtained to serve
determines subject-matter jurisdiction is the allegations in the complaint and not those summons by facsimile. Hence there was no valid service of summons and thus the court
in the answer. Furthermore, the MTC is empowered under S16 R70 to resolve the issue could not declare Charybdis in default. b) The effect of Scylla’s answer to the complaint is
of ownership, albeit for the purpose only of resolving the issue of possession. (B) No the that the court shall try the case against both Scylla and Charybdis upon the answer filed
by Scylla. Under Section 3(c) of Rule 9, when a pleading asserting a claim states a did not dismiss the petition, the RD-DENR-EMB in his Comment moved to dismiss the
common cause of action against several defending parties, some of whom answer and petition on the ground that petitioners failed to appeal the issuance of the ECC and to
the others fail to do so, the court shall try the case against all upon the answers thus filed exhaust administrative remedies provided in the DENR Rules and Regulations. b.) Should
and render judgment upon the evidence presented. Here there was a common cause of the court dismiss the petition? (3%) ANSWERS: a) No, the court was not correct in motu
action against Scylla and Charybdis since both were co-signatories to the mortgage deed. proprio dismissing the petition for lack of jurisdiction. In a case involving similar facts, the
Hence the court should not render judgment by default against Charybdis but should Supreme Court held that the requirement that the petition be filed in the area where the
proceed to try the case upon the answer filed and the evidence presented by Scylla. III. actionable neglect or omission took place relates to venue and not to subject-matter
Juliet invoking the provisions of the Rule on Violence Against Women and their Children jurisdiction. Since what is involved is improper venue and not subject-matter jurisdiction,
filed with the RTC designated as a Family Court a petition for . issuance of a Temporary it was wrong for the court to dismiss outright the petition since venue may be waived.
Protection Order (TPO) against her husband, Romeo. The Family Court issued a 30-day (Dolot v. Paje, 27 August 2013). b) No, the court should not dismiss the petition. The
TPO against Romeo. A day before the expiration of the TPO, Juliet filed a motion for Supreme Court has held that in environmental cases, the defense of failure to exhaust
extension. Romeo in his opposition raised, among others, the constitutionality of R.A. administrative remedies by appealing the ECC issuance would apply only if the defect in
No. 9262 (The VAWC Law) arguing that the law authorizing the issuance of a TPO violates the issuance of the ECC does not have any causal relation to the environmental damage.
the equal protection and due process clauses of the 1987 Constitution. The Family Court Here the issuance of the ECC has a direct causal relation to the environmental damage
judge, in granting the motion for extension of the TPO, declined to rule on the since it permitted the bulldozing of a portion of the mountain and the cutting down and
constitutionality of R.A. No. 9262. The Family Court judge reasoned that Family Courts buring of several trees and plants. (See Paje v. Casiño, 3 February 2015). VII. Plaintiff
are without jurisdiction to pass upon constitutional issues, being a special court of sued defendant for collection of P1 million based on the latter's promissory note. The
limited jurisdiction and R.A. No. 8369, the law creating the Family Courts, does not complaint alleges, among others: 1) Defendant borrowed P1 million from plaintiff as
provide for such jurisdiction. Is the Family Court judge correct when he declined to evidenced by a duly executed promissory note; 2) The promissory note reads: "Makati,
resolve the constitutionality of R.A. No. 9262? (3%) ANSWER: No, the Family Court judge Philippines Dec. 30, 2014 For value received from plaintiff, defendant promises to pay
was not correct when he declined to resolve the constitutionality of R.A. No. 9262. The plaintiff P1 million, twelve (12) months from the above indicated date without necessity
Supreme Court has held that despite its designation as a Family Court, a Regional Trial of demand. Signed Defendant" A copy of the promissory note is attached as Annex "A."
Court remains possessed of authority as a court of general jurisdiction to resolve the Defendant, in his verified answer, alleged among others: 1) Defendant specifically denies
constitutionality of a statute. (Garcia v. Drilon, 25 June 2013) IV. Strauss filed a complaint the allegation in paragraphs 1 and 2 of the complaint, the truth being defendant did not
against Wagner for cancellation of title. Wagner moved to dismiss the complaint because execute any promissory note in favor of plaintiff, or 2) Defendant has paid the P1 million
Grieg, to whom he mortgaged the property as duly annotated in the TCT, was not claimed in the promissory note (Annex "A" of the Complaint) as evidenced by an
impleaded as defendant. a.) Should the complaint be dismissed? (3%) b.) If the case "Acknowledgment Receipt" duly executed by plaintiff on January 30, 2015 in Manila with
should proceed to trial without Grieg being impleaded as a party to the case, what is his his spouse signing as witness. A copy of the "Acknowledgment Receipt" is attached as
remedy to protect his interest? (2%) ANSWERS: a) No, the complaint should not be Annex "1" hereof. Plaintiff filed a motion for judgment on the pleadings on the ground
dismissed. The Supreme Court has held that non-joinder of an indispensable party is not that defendant's answer failed to tender an issue as the allegations therein on his
a ground of a motion to dismiss. (Vesagas v. CA, 371 SCRA 508). Here although Grieg, the defenses are sham for being inconsistent; hence, no defense at all. Defendant filed an
registered mortgagee, is an indispensable party (Metrobank v. Alejo, 364 SCRA 813 opposition claiming his answer tendered an issue. a.) Is judgment on the pleadings
[2001]), his non-joinder does not warrant the dismissal of the complaint. b) The remedy proper? (3%) Defendant filed a motion for summary judgment on the ground that there
of Grieg is to file a motion for leave to intervene. Under Rule 19, a person who has a legal are no longer any triable genuine issues of facts. b.) Should the court grant defendant's
interest in the matter in litigation may intervene in the action. Here Grieg is a mortgagee motion for summary judgment? (3%) ANSWERS: a) No, judgment on the pleadings is not
and such fact was annotated in the title. Hence he has a legal interest in the title subject- proper. Under Section 2 of Rule 8, a party may set forth two or more statements of a
matter of the litigation and may thus intervene in the case. V. Ernie filed a petition for defense alternatively or hypothetically. The Supreme Court has held that inconsistent
guardianship over the person and properties of his father, Ernesto. Upon receipt of the defenses may be pleaded alternatively or hypothetically provided that each defense is
notice of hearing, Ernesto filed an opposition to the petition. Ernie, before the hearing of consistent with itself. (Baclayon v. Court of Appeals, 26 February 1990). Hence Plaintiff’s
the petition, filed a motion to order Ernesto to submit himself for mental and physical contention that defendant’s answer failed to tender an issue as his defenses are sham
examination which the court granted. After Ernie's lawyer completed the presentation of for being inconsistent is without merit. b) Yes, the court should grant Defendant’s
evidence in support of the petition and the court's ruling on the formal offer of evidence, motion for summary judgment. Under Section 2 of Rule 35, a defendant may at any time,
Ernesto's lawyer filed a demurrer to evidence. Ernie's lawyer objected on the ground that move with supporting admissions for a summary judgment in his favor. Here the Plaintiff
a demurrer to evidence is not proper in a special proceeding. a.) Was Ernie's counsel's had impliedly admitted the genuineness and due execution of the acknowledgment
objection proper? (2%) b.) If Ernesto defies the court's order directing him to submit to receipt, which was the basis of Defendant’s defense, by failing to specifically deny it
physical and mental examinations, can the court order his arrest? (2%) ANSWERS: a) No, under oath. Hence the Defendant may move for a summary judgment on the basis that
Ernie’s counsel’s objection was not proper. Under the Rule on Special Proceedings, in the Plaintiff had admitted that Defendant had already paid the P1 million obligation. VIII.
absence of special provisions, the rules provided for in ordinary actions, shall be, as far as Aldrin entered into a contract to sell with Neil over a parcel of land. The contract
practicable, applicable in special proceedings. Here there are no special provisions on stipulated a P500,000.00 down payment upon signing and the balance payable in twelve
demurrer to evidence in the rules on guardianship. Hence the provisions on demurrer to (12) monthly installments of P100,000.00. Aldrin paid the down payment and had paid
evidence in ordinary actions are applicable to special proceedings. Such application is three (3) monthly installments when he found out that Neil had sold the same property
practicable since it would be a waste of time to continue hearing the case if upon the to Yuri for P1.5 million paid in cash. Aldrin sued Neil for specific performance with
facts and the law, guardianship would not be proper. b) No, the court cannot order damages with the RTC. Yuri, with leave of court, filed an answer-in-intervention as he
Ernesto’s arrest. Under Section 3(d) of Rule 29, a court cannot direct the arrest of a party had already obtained a TCT in his name. After trial, the court rendered judgment
for disobeying an order to submit to a physical or mental examination. The court may ordering Aldrin to pay all the installments due, the cancellation of Yuri's title, and Neil to
impose other penalties such as rendering judgment by default or issuing an order that execute a deed of sale in favor of Aldrin. When the judgment became final and
the physical or mental condition of the disobedient party shall be taken as established in executory, Aldrin paid Neil all the installments but the latter refused to execute the deed
accordance with the claim of the party obtaining the order. VI. A law was passed of sale in favor of the former. Aldrin filed a "Petition for the Issuance of a Writ of
declaring Mt. Karbungko as a protected area since it was a major watershed. The Execution" with proper notice of hearing. The petition alleged, among others, that the
protected area covered a portion located in Municipality A of the Province I and a decision had become final and executory and he is entitled to the issuance of the writ of
portion located in the City of Z of Province II. Maingat is the leader of Samahan ng execution as a matter of right. Neil filed a motion to dismiss the petition on the ground
Tagapag-ingat ng Karbungko (STK), a people's organization. He learned that a portion of that it lacked the required certification against forum shopping. a.) Should the court
the mountain located in the City of Z of Province II was extremely damaged when it was grant Neil's Motion to Dismiss? (3%) Despite the issuance of the writ of execution
bulldozed and leveled to the ground, and several trees and plants were cut down and directing Neil to execute the deed of sale in favor of Aldrin, the former obstinately
burned by workers of World Pleasure Resorts, Inc. (WPRI) for the construction of a hotel refused to execute the deed. b.) What is Aldrin's remedy? (2%) ANSWERS: a) No, the
and golf course. Upon inquiry with the project site engineer if they had a permit for the court should not grant Neil’s Motion to Dismiss. Under Section 5 of Rule 7, a certification
project, Maingat was shown a copy of the Environmental Compliance Certificate (ECC) against forum shopping is required only for initiatory pleadings or petitions. Here the
issued by the DENR-EMB, Regional Director (RD-DENR-EMB). Immediately, Maingat and “Petition for the Issuance of a Writ of Execution,” although erroneously denominated as
STK filed a petition for the issuance of a writ of continuing mandamus against RD-DENR- a petition is actually a motion for issuance of a writ of execution under Rule 39. Hence
EMB and WPRI with the RTC of Province I, a designated environmental court, as the RD- the motion to dismiss on the ground of lack of a certification against forum shopping
DENR-EMB negligently issued the ECC to WPRI. On scrutiny of the petition, the court should be denied. b) Aldrin’s remedy is to file a motion for judgment for specific act
determined that the area where the alleged actionable neglect or omission subject of the under Section 10(a) of Rule 39. Under Section 10(a) of Rule 39, if a judgment directs a
petition took place in the City of Z of Province II, and therefore cognizable by the RTC of party to execute a conveyance of land and the party fails to comply, the court may direct
Province II. Thus, the court dismissed outright the petition for lack of jurisdiction. a.) Was the act to be done at the disobedient party’s cost by some other person appointed by
the court correct in motu proprio dismissing the petition? (3%) Assuming that the court the court or the court may by an order divest the title of the party and vest it in the
movant or other person. IX. Hades, an American citizen, through a dating website, got and P02 Romulus proceeded to the condo unit identified by Paz. PO 1 Remus knocked at
acquainted with Persephone, a Filipina. Hades came to the Philippines and proceeded to the door and when a man opened the door, POI Remus and his companions introduced
Baguio City where Persephone resides. Hades and Persephone contracted marriage, themselves as police officers. The man readily identified himself as Oasis Jung and
solemnized by the Metropolitan Trial Court judge of Makati City. After the wedding, gestured to them to come in. Inside, the police officers saw a young lady with her nose
Hades flew back to California, United States of America, to wind up his business affairs. bleeding and face swollen. Asked by P02 Romulus what happened, the lady responded
On his return to the Philippines, Hades discovered that Persephone had an illicit affair that she was beaten up by Oasis Jung. The police officers arrested Oasis Jung and
with Phanes. Immediately, Hades returned to the United States and was able to obtain a brought him and the young lady back to the police station. PO 1 Remus took the young
valid divorce decree from the Superior Court of the County of San Mateo, California, a lady's statement who identified herself as AA. She narrated that she is a sixteen-year-old
court of competent jurisdiction against Persephone. Hades desires to marry Hestia, also a high school student; that previous to the incident, she had sexual intercourse with Oasis
Filipina, whom he met at Baccus Grill in Pasay City. a.) As Hades' lawyer, what petition Jung at least five times on different occasions and she was paid P5,000.00 each time and
should you file in order that your client can avoid prosecution for bigamy if he desires to it was the first time that Oasis Jung physically hurt her. P02 Romulus detained Oasis Jung
marry Hestia? (2%) b.) In what court should you file the petition? (1 %) c.) What is the at the station's jail. After the inquest proceeding, the public prosecutor filed an
essential requisite that you must comply with for the purpose of establishing information for Violation of R.A. No. 9262 (The VAWC Law) for physical violence and five
jurisdictional facts before the court can hear the petition? (3%) ANSWERS: a) As Hade’s separate informations for violation of R.A. No. 7610 (The Child Abuse Law). Oasis Jung's
lawyer, I would file a petition for cancellation of entry of marriage under Rule 108 with lawyer filed a motion to be admitted to bail but the court issued an order that approval
prayer for recognition of foreign divorce judgment. In a case involving similar facts, the of his bail bond shall be made only after his arraignment. a.) Did the court properly
Supreme Court held that a foreign divorce decree must first be recognized before it can impose that bail condition? (3%) Before arraignment, Oasis Jung's lawyer moved to
be given effect. The Supreme Court stated that the recognition may be prayed for in the quash the other four separate informations for violation of the child abuse law invoking
petition for cancellation of the marriage entry under Rule 108. (Corpuz v. Sto. Tomas, the single larceny rule. b.) Should the motion to quash be granted? (2%) c.) After his
628 SCRA 266). b) I would file the petition in the regional trial court of Makati City, where release from detention on bail, can Oasis Jung still question the validity of his arrest?
the corresponding civil registry is located. (Section 1 of Rule 108). c) For the Rule 108 (2%) ANSWERS: a) No, the court did not properly impose the condition that the approval
petition, the jurisdictional facts are the following: 1. Joinder of the local civil registrar and of the bail bond shall be made only after the arraignment. In a case involving similar
all persons who have or claim any interest which would be affected by petition. 2. Notice facts, the Supreme Court held that in cases where it is authorized, bail should be granted
of the order of hearing to the persons named in the petition. 3. Publication of the order before arraignment, otherwise the accused may be hindered from filing a motion to
of hearing in a newspaper of general circulation in the province. X. An information for quash since his arraignment would necessarily be deferred pending the resolution of the
murder was filed against Rapido. The RTC judge, after personally evaluating the motion to quash. This would amount to a substantial dilution of his right to file a motion
prosecutor's resolution, documents and parties' affidavits submitted by the prosecutor, to quash. (Lavides v. Court of Appeals, 1 February 2000). b) No, the motion to quash
found probable cause and issued a warrant of arrest. Rapido's lawyer examined the rollo should not be granted. In a case involving similar facts, the Supreme Court held that each
of the case and found that it only contained the copy of the information, the submissions act of sexual intercourse with a minor is a separate and distinct offense under R.A. No.
of the prosecutor and a copy of the warrant of arrest. Immediately, Rapido's counsel 7610. Hence the single larceny or single offense rule is not applicable. (Id.). c) Yes, Oasis
filed a motion to quash the arrest warrant for being void, citing as grounds: a.) The judge Jung can still question the validity of his arrest after his release from detention on bail.
before issuing the warrant did not personally conduct a searching examination of the Under the Rules on Criminal Procedure, admission to bail shall not bar the accused from
prosecution witnesses in violation of his client's constitutionally-mandated rights; b.) challenging the validity of his arrest provided that he does so before entering his plea.
There was no prior order finding probable cause before the judge issued the arrest (Sec. 26, Rule 114). XIII. Jaime was convicted for murder by the Regional Trial Court of
warrant. May the warrant of arrest be quashed on the grounds cited by Rapido' s Davao City in a decision promulgated on September 30, 2015. On October 5, 2015, Jaime
counsel? State your reason for each ground. (4%) ANSWER: No, the warrant of arrest filed a Motion for New Trial on the ground that errors of law and irregularities prejudicial
may not be quashed on the grounds cited by Rapido’s counsel. a) The Supreme Court has to his rights were committed during his trial. On October 7, 2015, the private prosecutor,
held in Soliven v. Makasiar, 167 SCRA 393 (1988) that Section 2 of Art. III of the with the conformity of the public prosecutor, filed an Opposition to Jaime's motion. On
Constitution does not mandatorily require the judge to personally examine the October 9, 2015, the court granted Jaime's motion. On October 12, 2015, the public
complainant and his witnesses. The judge may opt to personally evaluate the report and prosecutor filed a motion for reconsideration. The court issued an Order dated October
supporting documents submitted by the regarding the existence of probable cause and 16, 2015 denying the public prosecutor's motion for reconsideration. The public
on the basis thereof issue a warrant of arrest. b) There is no requirement of a prior order prosecutor received his copy of the order of denial on October 20, 2015 while the private
by the judge finding probable cause. The SC has held that the judge may rely upon the prosecutor received his copy on October 26, 2015. a.) What is the remedy available to
resolution of the investigating prosecutor provided that he personally evaluates the the prosecution from the court's order granting Jaime's motion for new trial? (3%) b.) In
same and the affidavits and supporting documents, which he did. (People v. Grey, 26 July what court and within what period should a remedy be availed of? (1%) c.) Who should
2010). XI. The Ombudsman found probable cause to charge with plunder the provincial pursue the remedy? (2%) ANSWERS: a) The remedy available to the prosecution from the
governor, vice governor, treasurer, budget officer, and accountant. An Information for court's order granting Jaime's motion for new trial is a special civil action for certiorari
plunder was filed with the Sandiganbayan against the provincial officials except for the under Rule 65. Under Section 1(b) of Rule 41, no appeal may be taken from an
treasurer who was granted immunity when he agreed to cooperate with the interlocutory order and the aggrieved party may file an appropriate special civil action as
Ombudsman in the prosecution of the case. Immediately, the governor filed with the provided in Rule 65. Here the order granting the motion for new trial is an interlocutory
Sandiganbayan a petition for certiorari against the Ombudsman claiming there was grave order since it does not completely dispose of the case but still leaves something to be
abuse of discretion in excluding the treasurer from the Information. a.) Was the remedy done, that is, conducting the new trial. Hence the available remedy is the special civil
taken by the governor correct? (2%) b.) Will the writ of mandamus lie to compel the action for certiorari under Rule 65. b) The special civil action for certiorari should be filed
Ombudsman to include the treasurer in the Information? (3%) c.) Can the Special with the Court of Appeals. It should be filed within 60 days from receipt by the public
Prosecutor move for the discharge of the budget officer to corroborate the testimony of prosecutor of the order denying the motion for reconsideration pursuant to Section 4 of
the treasurer in the course of presenting its evidence? (2%) ANSWERS: a) No, the remedy Rule 65. The 60-day period should be reckoned from the receipt by the public prosecutor
taken by the governor was not correct. The SC has held that the proper remedy from the who has the direction and control of the prosecution pursuant to Section 5 of Rule 110.
Ombudsman’s orders or resolutions in criminal cases is a petition for certiorari under c) The remedy should be pursued by the Office of the Solicitor General. Under Section
Rule 65 filed with the Supreme Court. (Quarto v OMB, 5 Oct 2011; Cortes v. OMB, 10 35(1), Chapter 12, Title III of Book IV of the 1987 Administrative Code, the authority to
June 2013). Here the petition for certiorari was filed not with the Supreme Court but the represent the government in criminal cases before the Court of Appeals and Supreme
Sandiganbayan. Hence the remedy taken was not correct. b) No, the writ of mandamus Court is vested solely in the Office of the Solicitor General. (Cario v. De Castro, 30 April
will not lie to compel the Ombudsman to include the Treasurer in the information. The 2008). XIV. Pedro was charged with theft for stealing Juan's cellphone worth P10,000.00.
Supreme Court has held that mandamus will lie only if the exclusion of a person from the Prosecutor Marilag at the pre-trial submitted the judicial affidavit of Juan attaching the
information was arbitrary. Here the exclusion was not arbitrary but based on Sec. 17 of receipt for the purchase of the cellphone to prove civil liability. She also submitted the
RA 6770 which empowers the Ombudsman to grant immunity to witnesses. (Id.). c) No, judicial affidavit of Mario, an eyewitness who narrated therein how Pedro stole Juan's
the Special Prosecutor cannot move for the discharge of the budget officer to cellphone. At the trial, Pedro's lawyer objected to the prosecution's use of judicial
corroborate the testimony of the treasurer. Under Section 17 of Rule 119, a requirement affidavits of her witnesses considering the imposable penalty on the offense with which
for discharge is that there is no other direct evidence available for the prosecution of the his client was charged. a.) Is Pedro's lawyer correct in objecting to the judicial affidavit of
offense and that there is absolute necessity for the testimony of the accused whose Mario? (2%) b.) Is Pedro's lawyer correct in objecting to the judicial affidavit of Juan?
discharge is requested. Here since the budget officer’s testimony is merely corroborative, (2%) At the conclusion of the prosecution's presentation of evidence, Prosecutor Marilag
there is no absolute necessity for it. Necessity is not there when the testimony would orally offered the receipt attached to Juan's judicial affidavit, which the court admitted
simply corroborate or otherwise strengthen the prosecution’s evidence. (Jimenez v over the objection of Pedro's lawyer. After Pedro's presentation of his evidence, the
People, 17 September 2014). Hence the Special Prosecutor cannot move for the court rendered judgment finding him guilty as charged and holding him civilly liable for
discharge of the budget officer. XII. Paz was awakened by a commotion coming from a P20,000.00. Pedro's lawyer seasonably filed a motion for reconsideration of the decision
condo unit next to hers. Alarmed, she called up the nearby police station. PO 1 Remus asserting that the court erred in awarding the civil liability on the basis of Juan's judicial
affidavit, a documentary evidence which Prosecutor Marilag failed to orally offer. c.) Is officer, will he have a cause of action? (3%) ANSWERS: a) The remedy available to
the motion for reconsideration meritorious? (2%) ANSWERS: a) No, Pedro’s lawyer is not Hercules to secure his immediate release from detention is a petition for writ of habeas
correct in objecting to the judicial affidavit of Mario. The Judicial Affidavit Rule applies to corpus. Under Rule 102, the writ of habeas corpus is available in cases of illegal
criminal actions where the maximum of the imposable penalty does not exceed six years. detention. Section 5 of Rule 102 provides that a court or judge authorized to grant the
Here the penalty for theft of property not exceeding P12,000 does not exceed 6 years. writ must, when the petition therefor is presented and it appears that the writ ought to
Hence the Judicial Affidavit Rule applies. b) No, Pedro's lawyer is not correct in objecting issue, grant the same forthwith, and immediately thereupon the clerk of court shall issue
to the judicial affidavit of Juan. The Judicial Affidavit Rule applies with respect to the civil the writ or in case of emergency, the judge may issue the writ under his own hand and
aspect of the criminal actions, whatever the penalties involved are. Here the purpose of may depute any officer or person to serve it. The court or judge before whom the writ is
introducing the judicial affidavit of Juan was to prove his civil liability. c) No, the motion returned must immediately proceed to hear and examine the return. (Section 12, Rule
for reconsideration is not meritorious. A judicial affidavit is not a documentary evidence 102). b) I will raise the defense that the warrantless search was authorized as a “stop and
but is testimonial evidence. It is simply a witness’s testimony reduced to writing in frisk.” “Stop and frisk” is the right of a police officer to stop a citizen on the street,
affidavit form. This is shown by Section 6 of the Judicial Affidavit Rule which states that interrogate him and pat him for weapons and contraband whenever he observes unusual
the offer of testimony in judicial affidavit shall be made at the start of the presentation conduct which leads him to conclude that criminal activity may be afoot. (Terry v. Ohio,
of the witness. Hence the motion for reconsideration on the ground that Juan’s judicial 392 U.S. 1). c) Yes Hercules will have a cause of action. Under Article 32(4) of the Civil
affidavit was a documentary evidence which was not orally offered is without merit. XV. Code, any public officer who violates the right of a person to freedom from arbitrary or
Water Builders, a construction company based in Makati City, entered into a illegal detention shall be liable to the latter for damages. The action to recover damages
construction agreement with Super Powers, Inc., an energy company based in Manila, for is an independent civil action. Here Hercules was illegally detained as there was no
the construction of a mini hydro electric plant. Water Builders failed to complete the probable cause to arrest him without warrant. XVIII. The residents of Mt. Ahohoy,
project within the stipulated duration. Super Powers cancelled the contract. Water headed by Masigasig, formed a nongovernmental organization - Alyansa Laban sa
Builders filed a request for arbitration with the Construction Industry Arbitration Minahan sa Ahohoy (ALMA) to protest the mining operations of Oro Negro Mining in the
Commission (CIAC). After due proceedings, CIAC rendered judgment in favor of Super mountain. ALMA members picketed daily at the entrance of the mining site blocking the
Powers, Inc. ordering Water Builders to pay the former P 10 million, the full amount of ingress and egress of trucks and equipment of Oro Negro, hampering its operations.
the down payment paid, and P2 million by way of liquidated damages. Dissatisfied with Masigasig had an altercation with Mapusok arising from the complaint of the mining
the CIAC's judgment, Water Builders, pursuant to the Special Rules of Court on engineer of Oro Negro that one of their trucks was destroyed by ALMA members.
Alternative Dispute Resolution (ADR Rules) filed with the RTC of Pasay City a petition to Mapusok is the leader of the Association of Peace Keepers of Ahohoy (APKA), a civilian
vacate the arbitral award. Super Powers, Inc., in its opposition, moved to dismiss the volunteer organization serving as auxiliary force of the local police to maintain peace and
petition, invoking the ADR Rules, on the ground of improper venue as neither of the order in the area. Subsequently, Masigasig disappeared. Mayumi, the wife of Masigasig,
parties were doing business in Pasay City. Should Water Builders' petition be dismissed? and the members of ALMA searched for Masigasig, but all their efforts proved futile.
(3%) ANSWER: Yes Water Builders’ petition should be dismissed. Under Rule 11.3 of the Mapagmatyag, a member of ALMA, learned from Maingay, a member of APKA, during
Special ADR Rules, the petition for vacation of a domestic arbitral award may be filed their binge drinking that Masigasig was abducted by other members of APKA, on order of
with the Regional Trial Court having jurisdiction over the place in which one of the Mapusok. Mayumi and ALMA sought the assistance of the local police to search for
parties is doing business, where any of the parties reside or where arbitration Masigasig, but they refused to extend their cooperation. Immediately, Mayumi filed with
proceedings were conducted. Here neither of the parties were doing business in Pasay the RTC, a petition for the issuance of the writ of amparo against Mapusok and APKA.
City nor was there a showing that arbitration proceedings were conducted in Pasay City. ALMA also filed a petition for the issuance of the writ of amparo with the Court of
XVI. AA, a twelve-year-old girl, while walking alone met BB, a teenage boy who Appeals against Mapusok and APKA. Respondents Mapusok and APKA, in their Return
befriended her. Later, BB brought AA to a nearby shanty where he raped her. The filed with the RTC, raised among their defenses that they are not agents of the State;
Information for rape filed against BB states: "On or about October 30, 2015, in the City of hence, cannot be impleaded as respondents in an amparo petition. a.) Is their defense
S.P. and within the jurisdiction of this Honorable Court, the accused, a minor, fifteen (15) tenable? (3%) Respondents Mapusok and APKA, in their Return filed with the Court of
years old with lewd design and by means of force, violence and intimidation, did then Appeals, raised as their defense that the petition should be dismissed on the ground that
and there, willfully, unlawfully and feloniously had sexual intercourse with AA, a minor, ALMA cannot file the petition because of the earlier petition filed by Mayumi with the
twelve (12) years old against the latter's will and consent." At the trial, the prosecutor RTC. b.) Are respondents correct in raising their defense? (3%) c.) Mayumi later filed
called to the witness stand AA as his first witness and manifested that he be allowed to separate criminal and civil actions against Mapusok. How will the cases affect the
ask leading questions in conducting his direct examination pursuant to the Rule on the amparo petition she earlier filed? (1 %) ANSWERS: a) No, the defense of Mapusok and
Examination of a Child Witness. BB's counsel objected on the ground that the prosecutor APKA that they are not agents of the State and hence cannot be impleaded as
has not conducted a competency examination on the witness, a requirement before the respondents in an amparo petition is not tenable. The writ of amparo is available in cases
rule cited can be applied in the case. a.) Is BB's counsel correct? (3%) In order to obviate where the enforced or involuntary disappearance of a persons is with the authorization,
the counsel's argument on the competency of AA as prosecution witness, the judge motu support or acquiescence of the State. (See Sec. 3[g] of R.A. No. 9851 and Navia v.
proprio conducted his voir dire examination on AA. b.) Was the action taken by the judge Pardico, 19 June 2012, e.b.). Here Mapusok and APKA may be considered as acting with
proper? (2%) After the prosecution had rested its case, BB' s counsel filed with leave a the support or at least the acquiescence of the State since APKA serves as an auxiliary
demurrer to evidence, seeking the dismissal of the case on the ground that the force of the police and the police refused to assist in the search for Masigasig. b) Yes
prosecutor failed to present any evidence on BB' s minority as alleged in the Information. respondents are correct in raising their defense. Under Section 2(c) of the Rule on the
c.) Should the court grant the demurrer? (3%) ANSWERS: a) No, BB’s counsel is not Writ of Amparo, the filing of a petition by an authorized party on behalf of the aggrieved
correct. Under the Rules on Examination of a Child Witness, there is no requirement that party suspends the right of all others, observing the order in Section 2 of the Rule on the
a competency examination of the child witness be conducted before leading questions Writ of Amparo. Here the petition for writ of amparo had earlier been filed by the spouse
may be asked of her. A competency examination may be conducted by the court (not the of the aggrieved party Masigasig. Thus it suspends the right of all others, including ALMA,
prosecutor) only if substantial doubt exists as to the child’s competency to testify. to file the petition. c) The amparo petition shall be consolidated with the criminal action.
(Section 6, RECW). Here there is no showing of any substantial doubt as to the (Section 23, Rule on the Writ of Amparo).
competency of AA to testify. Hence BB’s counsel is not correct. b) No, the action taken by
the judge was improper. Under the Rules on Examination of a Child Witness, a
competency examination may be conducted by the court only if substantial doubt exists
as to the child’s competency to testify. (Section 6, RECW). Here the judge’s voir dire is in
effect a competency examination. However there is no showing of any substantial doubt
as to the competency of AA to testify. Hence the judge’s action was improper. c) No the
court may not grant the demurrer. Under the Rules of Criminal Procedure, a demurrer to
evidence may be granted on the ground of insufficiency of evidence. Here even assuming
that minority was not proved, BB may still be convicted of rape since minority is not an
element of rape. XVII. Hercules was walking near a police station when a police officer
signaled for him to approach. As soon as Hercules came near, the police officer frisked
him but the latter found no contraband. The police officer told Hercules to get inside the
police station. Inside the police station, Hercules asked the police officer, "Sir, may
problema po ba?" Instead of replying, the police officer locked up Hercules inside the
police station jail. a.) What is the remedy available to Hercules to secure his immediate
release from detention? (2%) b.) If Hercules filed with the Ombudsman a complaint for
warrantless search, as counsel for the police officer, what defense will you raise for the
dismissal of the complaint? (3%) c.) If Hercules opts to file a civil action against the police

You might also like